Você está na página 1de 251

CRIMINAL LAW REVIEW

DIGESTS
ATTY. ROMEO CALLEJO, SR.
CRIMINAL LAW REVIEW DIGESTS
JUSTICE ROMEO CALLEJO NOTE: = Callejo Ponente

BAYAN MUNA VS. ALBERTO ROMULO, IN HIS impair the value of a treaty, e.g., the Rome Statute--has to
CAPACITY AS EXECUTIVE SECRETARY be rejected outright. For nothing in the provisions of
the Agreement, in relation to the Rome Statute, tends to
FACTS: diminish the efficacy of the Statute, let alone defeats the
1. The Rome Statute established the International Criminal purpose of the ICC. Lest it be overlooked, the Rome
Court (ICC) with "the power to exercise its jurisdiction over Statute contains a proviso that enjoins the ICC from
persons for the most serious crimes (including genocide) seeking the surrender of an erring person, should the
of international concern x x x and shall be complementary process require the requested state to perform an act
to the national criminal jurisdictions." The RP is already a that would violate some international agreement it has
signatory but pending ratification by Senate. entered into. We refer to Art. 98(2) of the Rome Statute,
which reads:
2. The RP entered into a Non-Surrender Agreement with 2. The Court may not proceed with a
the US which provides that before a person (current or request for surrender which would require the requested
former Government official, employee, or military State to act inconsistently with its obligations under
personnel or national of one party) is surrendered or international agreements pursuant to which the consent of
transferred to any international tribunal, express consent a sending State is required to surrender a person of that
of the other party is required. State to the Court, unless the Court can first obtain the
cooperation of the sending State for the giving of consent
3. Petitioner assails the validity of the Agreement because for the surrender.
it, among other things, (1) contravenes the Rome Statute;
and (2) amends and is repugnant to Sec. 17 of RA 9851 RA 9851
("Philippine Act on Crimes Against International Sec. 17 of RA 9851 provides: In the interest of justice, the
Humanitarian Law, Genocide, and Other Crimes Against relevant Philippine authorities may dispense with the
Humanity"). investigation or prosecution of a crime punishable under
this Act if another court or international tribunal is already
ISSUE: W/n the Agreement is valid. conducting the investigation or undertaking the
prosecution of such crime. Instead, the
HELD/RATIO: YES. authorities may surrender or extradite suspected or
ROME STATUTE accused persons in the Philippines to the appropriate
Art. 1 of the Rome Statute pertinently provides that the international court, if any, or to another State pursuant
ICC x x x shall have the power to exercise its to the applicable extradition laws and treaties.
jurisdiction over persons for the most serious crimes of
international concern, as referred to in this Statute, Petitioners view (na mali naman):
and shall be complementary to national criminal - That the Agreement amends existing municipal
jurisdictions. The jurisdiction and functioning of the laws on the State's obligation in relation to grave
Court shall be governed by the provisions of this Statute. crimes against the law of nations, i.e., genocide,
Under par. 3 of Art. 20, Rome Statute, which again crimes against humanity and war crimes.
underscores the primacy of the jurisdiction of a state vis-a- - That the Philippines is required to surrender to
vis that of the ICC. As far as relevant, the provision states the proper international tribunal those persons
that "no person who has been tried by another court for accused of the grave crimes defined under RA
conduct x x x [constituting crimes within its jurisdiction] 9851, if it does not exercise its primary
shall be tried by the [International Criminal] Court with jurisdiction to prosecute them.
respect to the same conduct x x x." - That the Philippines has only two options, to wit:
The foregoing provisions of the Rome Statute, (1) surrender the accused to the proper
taken collectively, show that the Rome Statute expressly international tribunal; or (2) surrender the
recognizes the primary jurisdiction of states, like the RP, accused to another State if such surrender is
over serious crimes committed within their respective "pursuant to the applicable extradition laws and
borders, the complementary jurisdiction of the ICC coming treaties." But the Philippines may exercise these
into play only when the signatory states are unwilling or options only in cases where "another court or
unable to prosecute. international tribunal is already conducting the
Given the above consideration, petitioner's investigation or undertaking the prosecution of
suggestion--that the RP, by entering into the Agreement, such crime;" otherwise, the Philippines must
violated its duty required by the imperatives of good faith prosecute the crime before its own courts
and breached its commitment under the Vienna pursuant to RA 9851.
Convention to refrain from performing any act tending to

1
CRIMINAL LAW REVIEW DIGESTS
JUSTICE ROMEO CALLEJO NOTE: = Callejo Ponente

- That the Agreement prevents the Philippines Circular supported by R.A. No. 7638 since the said law
without the consent of the US from surrendering does not pertain to LPG traders.
to any international tribunal US nationals accused RTC denied MR. Hence this petition for review on
of crimes covered by RA 9851, and, thus, in certiorari to SC.
effect amends Sec. 17 of RA 9851.
Issue: W/N the DOE Circular is void on the ground that it
According to the SC: introduced new offences not punished under
- The Agreement merely reinforces the primacy of B.P. Blg. 33?
the national jurisdiction of the US and the
Philippines in prosecuting criminal offenses Held & Ratio: DOE Circular is valid.
committed by their respective citizens and
military personnel, among others. The For an administrative regulation to have the force
jurisdiction of the ICC pursuant to the Rome of penal law (1) the violation of the administrative
Statute over high crimes indicated thereat is regulation must be made a crime by the delegating statute
clearly and unmistakably complementary to itself; and (2) the penalty for such violation must be
the national criminal jurisdiction of the provided by the statute itself.
signatory states. The Circular satisfies the first requirement. B.P.
- RA 9851 clearly: (1) defines and establishes the Blg. 33, criminalizes illegal trading, adulteration,
crimes against international humanitarian law, underfilling, hoarding, and overpricing of petroleum
genocide and other crimes against humanity; (2) products. Under this general description of what
provides penal sanctions and criminal liability for constitutes criminal acts involving petroleum products, the
their commission; and (3) establishes special Circular merely lists the various modes by which the said
courts for the prosecution of these crimes and for criminal acts may be perpetrated, namely: no price display
the State to exercise primary criminal jurisdiction. board, no weighing scale, no tare weight or incorrect tare
Nowhere in RA 9851 is there a proviso that goes weight markings, no authorized LPG seal, no trade name,
against the tenor of the Agreement. unbranded LPG cylinders, no serial number, no
distinguishing color, no embossed identifying markings on
cylinder, underfilling LPG cylinders, tampering LPG
HON. SEC. PEREZ (AS DOE SECRETARY) V. LPG cylinders, and unauthorized decanting of LPG
REFILLERS ASSOCIATION OF THE PHILS. cylinders. These specific acts and omissions are
obviously within the contemplation of the law, which seeks
Facts: B.P. Blg. 33 penalizes illegal trading, hoarding, to curb the pernicious practices of some petroleum
overpricing, adulteration, underdelivery, and underfilling of merchants.
petroleum products, as well as possession for trade of As for the second requirement: B.P. Blg. 33,
adulterated petroleum products and of underfilled LPG provides that the monetary penalty for any person who
cylinders. The law set the monetary penalty for violators to commits any of the acts aforestated is limited to a minimum
a minimum of P20,000 and a maximum of P50,000. of P20,000 and a maximum of P50,000. Under the Circular,
To implement the law the DOE issued Circular the maximum pecuniary penalty for retail outlets is P20,000,
No. 2000-06-010. Respondent LPG Refillers Association an amount within the range allowed by law. However, the
of the Philippines, Inc. (LPG Refillers) asked the DOE to Circular is silent as to any maximum penalty for
set aside the Circular for being contrary to law. DOE the refillers, marketers, and dealers. This mere silence,
denied the request. does not amount to violation of the statutory maximum
LPG Refillers then filed a petition for prohibition limit. The mere fact that the Circular provides penalties on a
and annulment of the Circular with the RTC. RTC nullified per cylinder basis does not in itself run counter to the law
the Circular on the ground that it introduced new offenses since all that B.P. Blg. 33 prescribes are the minimum and
not included in the law (per RTC: the Circular, in providing the maximum limits of penalties.
penalties on a per cylinder basis for each violation, might It is B.P. Blg. 33, which defines what constitute
exceed the maximum penalty under the law). punishable acts involving petroleum products and which
DOE argued: penalties for the acts and set the minimum and maximum limits for the
omissions enumerated in the Circular are sanctioned by corresponding penalties. The Circular merely implements
B.P. Blg. 33 and R.A. No. 8479. LPG Refillers countered: the said law, albeit it is silent on the maximum pecuniary
enabling laws do not expressly penalize the acts and penalty for refillers, marketers, and dealers.
omissions enumerated in the Circular. Neither is the Noteworthy, the enabling laws on which the
Circular is based were specifically intended to provide the
DOE with increased administrative and penal measures

2
CRIMINAL LAW REVIEW DIGESTS
JUSTICE ROMEO CALLEJO NOTE: = Callejo Ponente

with which to effectively curtail rampant adulteration and Issues:


shortselling, as well as other acts involving petroleum 1. Should the Writ for Habeas Corpus be granted?
products, which are inimical to public interest. To nullify 2. Is the SC Admin. Circular 12-2000 a penal law?
the Circular would be to render inutile government efforts What does it really order?
to protect the general consuming public against the 3. What is the positivist theory of criminal law?
nefarious practices of some unscrupulous LPG traders.
Held: 1. No!
Note: LPG Refillers filed an MR with the SC on Aug. 28, 2. No!
2007 (G.R. NO. 159149) contending that the Circular, in 3. See Doctrine
providing penalties on a per cylinder basis, is no longer
regulatory, but already confiscatory in nature. MR denied. Doctrine:
Circular is not confiscatory. The penalties do not exceed 1. Section 4, Rule 102 of the Rules of Court, as amended,
the ceiling prescribed in B.P. Blg. 33, which penalizes any provides that the writ of habeas corpus is not allowed if the
person who commits any act [t]herein prohibited. Violation person alleged to be restrained of his liberty is in the
on a per cylinder basis falls within the phrase any custody of an officer under process issued by a court or
act. To provide the same penalty for one who violates a judge or by virtue of a judgment or order of a court of
prohibited act in B.P. Blg. 33, regardless of the number of record. In this case De Joya was imprisoned by virtue of a
cylinders involved would result in an indiscriminate, court judgment.
oppressive and impractical operation of B.P. Blg. 33. The
equal protection clause demands that all persons subject 2. SC Admin. Circular No. 12-2000 is not a penal law;
to such legislation shall be treated alike, under like hence, Article 22 of the RPC is not applicable. The
circumstances and conditions, both in the privileges circular applies only to those cases pending as of the date
conferred and in the liabilities imposed. of its effectivity and not to cases already terminated by
final judgment. Also, it did not abolish imprisonment. It
merely lays down a rule of preference in the application of
DE JOYA V. JAIL WARDEN the penalties for violation of B.P. Blg. 22. It says that when
imposing a fine would better serve the interest of justice,
Doctrine: Positivist Theory of Criminal Law the guilty party may just be fined instead of being
imprisoned.
Facts: Norma de Joya was charged with two counts of BP
22 or the bouncing checks law in a Batangas MTC. 3. In imposing penalties for crimes, the courts must bear in
Crim Case 25484 was for issuing a Solid Bank mind that Philippine penal law is based on the Spanish
check to Flor catapang de Tenorio worth 150,000 which penal code and has adopted features of the positivist
was dishonoured because the account was closed. theory of criminal law. The positivist theory states that the
Crim Case 25773 was for issuing a Scurity Bank basis for criminal liability is the sum total of the social and
and Trust Company check to Resurreccion Castillo for economic phenomena to which the offense is expressed.
225,000 which was also dishonoured because the account The adoption of the aspects of the theory is exemplified by
was closed. the indeterminate sentence law.
De Joya pleaded not guilty and then jumped bail Philippine penal law looks at the convict as a
during trial. She was unable to present evidence and she member of society. Among the important factors to be
lost both cases. Both decisions were promulgated without considered in determining the penalty to be imposed on
her being there despite due notice. Aside from being him are (1) his relationship towards his dependents, family
ordered to pay the amounts, she was also given the and their relationship with him; and (2) his relationship
penalty of one year imprisonment. towards society at large and the State. The State is
In the meantime SC Admin Circular 12-2000 was concerned not only in the imperative necessity of
passed concerning punishments for BP 22 violations. Two protecting the social organization against the criminal acts
years later, de Joya was apprehended while applying for of destructive individuals but also in redeeming the
an NBI clearance. She was jailed in Batangas and then individual for economic usefulness and other social ends.
she asked to be released by virtue of the circular. She The purpose of penalties is to secure justice.
thought that it could be applied retroactively and that it The penalties imposed must not only be retributive but
meant imprisonment was no longer a punishment for bp must also be reformative, to give the convict an
22 violations. opportunity to live a new life and rejoin society as a
The RTC denied her motion hence this habeas productive and civic-spirited member of the community.
corpus petition.

3
CRIMINAL LAW REVIEW DIGESTS
JUSTICE ROMEO CALLEJO NOTE: = Callejo Ponente

LIANG VS. PEOPLE Special Agencies Convention as well as in the ADB


Charter and Headquarters Agreement. These
FACTS: Jeffrey Liang is an economist working in the organizations were accorded privileges and immunities in
Asian Development Bank (ADB). Sometime in 1994, he their charters by language similar to that applicable to the
was charged before the Metropolitan Trial Court (MTC) of United Nations. It is clear therefore that these
Mandaluyong City with two counts of grave oral organizations were intended to have similar privileges and
defamation for allegedly uttering defamatory words against immunities. From this, it can be easily deduced that
fellow ADB worker Joyce Cabal. Because of this, he was international organizations enjoy absolute immunity similar
arrested, but then, he was able to post bail, and so he was to the diplomatic prerogatives granted to diplomatic
released from custody. envoys.
The next day, the MTC judge received an office On the other hand, international officials are
of protocol from the Department of Foreign Affairs (DFA) governed by a different rule. Section 18(a) of the General
stating that Liang is covered by immunity from legal Convention on Privileges and Immunities of the United
process under Section 45 of the Agreement between the Nations states that officials of the United Nations shall be
ADB and the Philippine Government. Because of this, the immune from legal process in respect of words spoken or
MTC judge dismissed the two criminal cases without written and all acts performed by them in their official
notice to the prosecution. capacity. The Convention on Specialized Agencies
The prosecution filed a Motion for carries exactly the same provision. The Charter of the
Reconsideration but it was denied. It then filed a Petition ADB provides under Article 55(i) that officers and
for Certiorari and Mandamus with the Regional Trial Court employees of the bank shall be immune from legal
(RTC) of Pasig City. The latter set aside the MTC ruling process with respect to acts performed by them in their
and ordered for an enforcement of a warrant of arrest. official capacity except when the Bank waives
Liang filed a MR but it was denied. Hence, this Petition for immunity. Section 45 (a) of the ADB Headquarters
Review. Agreement accords the same immunity to the officers and
staff of the bank. There can be no dispute that
ISSUE: Whether or not Liang is covered by immunity international officials are entitled to immunity only with
under the Agreement??? NO. respect to acts performed in their official capacity, unlike
international organizations which enjoy absolute immunity.
HELD: The immunity mentioned in Section 45 of the Clearly, the most important immunity to an international
Agreement is not absolute, but subject to the exception official, in the discharge of his international functions, is
that the act was done in an official capacity. Slandering a immunity from local jurisdiction. There is no argument in
person could not possibly be covered by the immunity doctrine or practice with the principle that an international
agreement because our laws do not allow the commission official is independent of the jurisdiction of the local
of a crime, such as defamation, in the name of official authorities for his official acts. Those acts are not his, but
duty. It is well-settled principle of law that a public official are imputed to the organization, and without waiver the
may be liable in his personal private capacity for whatever local courts cannot hold him liable for them. In strict law, it
damage he may have caused by his act done with malice would seem that even the organization itself could have no
or in bad faith or beyond the scope of his authority or right to waive an officials immunity for his official
jurisdiction. acts. This permits local authorities to assume jurisdiction
Under the Vienna Convention on Diplomatic over and individual for an act which is not, in the wider
Relations, a diplomatic agent enjoys immunity from sense of the term, his act at all. It is the organization itself,
criminal jurisdiction of the receiving state except in the as a juristic person, which should waive its own immunity
case of an action relating to any professional or and appear in court, not the individual, except insofar as
commercial activity exercised by the diplomatic agent in he appears in the name of the organization. Provisions for
the receiving state outside his official functions. The immunity from jurisdiction for official acts appear, aside
commission of a crime is not part of an official duty. from the aforementioned treatises, in the constitution of
most modern international organizations. The acceptance
CONCURRING OPINION, PUNO, J: of the principle is sufficiently widespread to be regarded as
The phrase immunity from every form of legal process as declaratory of international law.
used in the UN General Convention has been interpreted
to mean absolute immunity from a states jurisdiction to
adjudicate or enforce its law by legal process, and it is KHOSROW MINUCHER v. CA and ARTHUR SCALZO
said that states have not sought to restrict that immunity of
the United Nations by interpretation or Facts: Khosrow Minucher is an Iranian national who came
amendment. Similar provisions are contained in the to study in the RP in 1974 and was appointed Labor

4
CRIMINAL LAW REVIEW DIGESTS
JUSTICE ROMEO CALLEJO NOTE: = Callejo Ponente

Attach for the Iranian Embassies in Tokyo, Japan and mission" are the diplomatic staff, the administrative staff
Manila. When the Shah (monarch title) of Iran was and the technical and service staff. Only the heads of
deposed, he became a refugee and continued to stay as missions, as well as members of the diplomatic staff,
head of the Iranian National Resistance Movement. excluding the members of the administrative, technical
Scalzo, on the other hand, was a special agent of the US and service staff of the mission, are accorded diplomatic
Drugs Enforcement Agency. He conducts surveillance rank.
operations on suspected drug dealers in the Philippines Scalzo was an Assistant Attach of the US
believed to be the source of prohibited drugs shipped to diplomatic mission. Attaches assist a chief of mission in
the US and make the actual arrest. his duties and are administratively under him, but
In May 1986, Minucher (and one Abbas their main function is to observe, analyze and interpret
Torabian) was charged with for the violation of RA 6425 trends and developments in their respective fields in
(Dangerous Drugs Act of 1972). The criminal charge was the host country and submit reports to their own ministries
followed by a buy-bust operation conducted by the or departments in the home government. These
Philippine police narcotic agents in his house where a officials are not generally regarded as members of the
quantity of heroin was said to have been seized. The diplomatic mission, nor are they normally designated as
narcotic agents were accompanied by private respondent having diplomatic rank.
Arthur Scalzo who became one of the principal witnesses While the diplomatic immunity of Scalzo might
for the prosecution. They were acquitted. thus remain contentious, it was sufficiently established
On 03 August 1988, Minucher filed a case before that, indeed, he worked for the United States Drug
the RTC for damages on account of what he claimed to Enforcement Agency and was tasked to conduct
have been trumped-up charges of drug trafficking made by surveillance of suspected drugactivities within the country
Arthur Scalzo. According to Minucher, he and Scalzo on the dates pertinent to this case. If it should be
conducted some business. Minucher expressed his desire ascertained that Arthur Scalzo was acting well within his
to obtain a US Visa for him and his Abbass wife. Scalzo assigned functions when he committed the acts alleged in
told him that he could help him for a $2,000 fee per visa. the complaint, the present controversy could then be
After a series of business transactions between the two, resolved under the related doctrine of State Immunity
when Scalzo came to deliver the visas to Minuchers from Suit.
house, he told the latter that he would be leaving the The precept that a State cannot be sued in the
Philippines soon and requested him to come out of the courts of a foreign state is a long-standing rule of
house so he can introduce him to his cousin waiting in the customary international law. If the acts giving rise to a suit
cab. To his surprise, 30-40 armed Filipino soldiers came to are those of a foreign government done by its foreign
arrest him. agent, although not necessarily a diplomatic personage,
In his defense, Scalzo asserted his diplomatic but acting in his official capacity, the complaint could be
immunity as evidenced by a Diplomatic Note. He barred by the immunity of the foreign sovereign from suit
contended that the US Government, pursuant to the without its consent. Suing a representative of a state is
Vienna Convention, recognized it on Diplomatic Relations believed to be, in effect, suing the state itself. The
and the Philippine government itself thru its Executive proscription is not accorded for the benefit of an individual
Department and DFA.The courts ruled in favor of Scalzo but for the State, in whose service he is, under the maxim
on the ground that as a special agent of the US Drug par in parem, non habet imperium that all states are
Enforcement Administration, he was entitled to diplomatic sovereign equals and cannot assert jurisdiction over one
immunity. another. The implication is that if the judgment against an
RTC: decision in favor of plaintiff. official would require the state itself to perform an
CA: Reversed. Scalzo was sufficiently clothed affirmative act to satisfy the award, such as the
with diplomatic immunity pursuant to the terms of the appropriation of the amount needed to pay the damages
Vienna Convention. decreed against him, the suit must be regarded as being
against the state itself, although it has not been formally
Issue: WON Scalzo is entitled to diplomatic immunity - impleaded.
Yes. A foreign agent, operating within a territory, can
be cloaked with immunity from suit but only as long as it
Ratio: The Vienna Convention lists the classes of heads of can be established that he is acting within the directives of
diplomatic missions to include: (a) ambassadors or the sending state. The consent of the host state is an
nuncios accredited to the heads of state, (b) envoys, indispensable requirement of basic courtesy between the
ministers or internuncios accredited to the heads of states; two sovereigns. The buy-bust operation and other such
and (c) charges d' affairs accredited to the ministers of acts are indication that the Philippine government has
foreign affairs. Comprising the "staff of the (diplomatic) given its imprimatur, if not consent, to the activities within

5
CRIMINAL LAW REVIEW DIGESTS
JUSTICE ROMEO CALLEJO NOTE: = Callejo Ponente

Philippine territory of agent Scalzo of the United States Nothing in the Constitution prohibits such
Drug Enforcement Agency. In conducting surveillance agreements recognizing immunity from jurisdiction or
activities on Minucher, later acting as the poseur-buyer some aspects of jurisdiction (such as custody), in relation
during the buy-bust operation, and then becoming a to long-recognized subjects of such immunity like Heads
principal witness in the criminal case against Minucher, of State, diplomats and members of the armed forces
Scalzo hardly can be said to have acted beyond the scope contingents of a foreign State allowed to enter another
of his official function or duties. States territory. The receiving State can exercise
jurisdiction over the forces of the sending State only to the
extent agreed upon by the parties. This is due to the
SUZETTE NICOLAS v. ALBERTO ROMULO, in his recognition of extraterritorial immunity given to bodies
capacity as Secretary of Foreign Affairs such as visiting foreign armed forces.

Facts: Respondent Lance Corporal (L/CPL) Daniel Smith Who has custody of Smith after conviction?
is a member of the US Armed Forces. He was charged
(along with 3 others) with the crime of rape committed Applying the provisions of VFA, the Court finds that there
against a Filipina, Suzette Nicolas, inside the Subic Bay is a different treatment when it comes to detention as
Freeport Zone, Olongapo City (inside a Starex Van). against custody. The moment the accused has to be
Pursuant to the Visiting Forces Agreement (VFA) between detained after conviction, the rule that governs is that: the
the Philippines and the US, the US, at its request, was confinement or detention by Philippine authorities of US
granted custody of defendant Smith pending the personnel shall be carried out in facilities agreed on by
proceedings. During the trial, the US Government faithfully appropriate Philippines and US authorities.
complied with its undertaking to bring defendant Smith to It is clear that the parties to the VFA recognized
the trial court every time his presence was required. The the difference between CUSTODY DURING TRIAL and
RTC of Makati found Smith guilty (others were acquitted DETENTION AFTER CONVICTION, because they
due to insufficiency of evidence) and sentenced him to provided for a specific arrangement to cover detention.
suffer the penalty of reclusion perpetua. Pending And this specific arrangement clearly states that the
agreement of the parties as to where Smith shall be detention shall be carried out in facilities AGREED ON by
detained, he was temporarily committed to the Makati City authorities of both parties AND that the detention shall be
Jail. However, defendant Smith was taken out of the by Philippine authorities.
Makati jail and brought to a facility for detention under the Therefore, the Romulo-Kenney Agreements, which are
control of the US government due to new agreements agreements on the detention of the accused in the US
between the Philippines and the US, referred to as the Embassy, are not in accord with the VFA itself because
Romulo-Kenney Agreement. Under such agreement, the such detention is not by Philippine authorities.
DFA of the Philippines and the Embassy of the US agreed The petitions were partly granted.
that, in accordance with the VFA, Smith shall be detained
in a room at the U.S. Embassy Compound and guarded by Side issue: WON the VFA was constitutional -YES!
U.S. military personnel. The matter was brought before It is constitutional as ruled by the court in the case of
the CA which dismissed the motion for having become Bayan v. Zamora. The VFA was duly concurred in by the
moot. Philippine Senate and has been recognized as a treaty by
the United States as attested and certified by the duly
Issue: Does the US have custody over Smith from the authorized representative of the United States
commission of the offense until completion of all government.
judicial proceedings? YES!

The VFA provides that in cases of offenses committed by NAVALES V ABAYA


the members of the US Armed Forces in the Philippines,
the custody of any US personnel over whom the Facts: Last July 27, 2003 more than 300 junior officers
Philippines is to exercise jurisdiction shall immediately and enlisted men mostly from the elite units of the AFP
reside with US military authorities, if they so request, from quietly entered the premises of the Ayala Center in Makati
the commission of the offense until completion of all City. They disarmed the security guards and took over the
judicial proceedings. US military authorities shall make Oakwood Premier Apartments (Oakwood). The soldiers
such personnel available to those authorities in time for then made a statement through ABS-CBN News network
any investigative or judicial proceedings relating to the that they went to Oakwood to air their grievances against
offense with which the person has been charged. the administration of President Gloria Macapagal Arroyo
such as graft and corruption in the military, sale of arms

6
CRIMINAL LAW REVIEW DIGESTS
JUSTICE ROMEO CALLEJO NOTE: = Callejo Ponente

and ammunition to the enemies of the State, etc. They academic when the RTC accepted the Amended
declared the withdrawal of support from the chain of Information under which only 31 of the accused were
command and demanded the resignation of key civilian charged and dismissing the case as against the other 290.
and military leaders of the Arroyo administration. It has become moot against those charges that were
After a series of negotiations between the dismissed.
soldiers and the Government team led by Ambassador However in said order it further declared that all
Cimatu an agreement was forged between the two groups. the charges before the court-martial against the accused
Subsequently DOJ charged the 3221 soldiers who took and former accused are not service-connected, believing
part in the Oakwood incident with violation of Article 134- that the crimes defined in and penalized by the Articles of
A coup d etat of the RPC. War were committed in furtherance of coup detat and thus
Thereafter several of the accused filed in the absorbed by the said crime.
RTC (branch 61) an Omnibus Motion praying that the RTC Thus, insofar as those whose case against them
assume jurisdiction over all charges filed before the was dismissed, there was nothing left to be resolved after
military tribunal. the Omnibus Motion was considered moot and academic.
While such motion was pending, DOJ issued a This dismissal made the petitioners no longer parties to
Resolution finding probable cause for coup d etat against the case and no further relief could be granted to them.
only 31 of the original 321 accused and the charges 1Lt Navales, et al. since they are strangers to the
against them were dismissed. proceedings in the criminal case are not bound by any
RTC (branch 61) admitted the Amended judgment rendered by the court, thus they cannot find
Information charging only 31 of the original accused with solace in the declaration of the RTC that the charges
the crime of coup d etat defined under Article 134-A of the filed against them before the General Court-Martial
RPC. were not service connected.
However, 1Lt. Navales, et. al who were earlier In view of the clear mandate of RA 7055 that
dropped as accused in the crime of coup d etat were military courts have jurisdiction to try cases involving
charged before the General Court Martial with violations of violations of Articles 54 to 70, Articles 72 to 92 and Articles
the Articles of War. 95 to 97 of the Articles of War as these are considered
At this point the RTC acted on the Omnibus service connected crimes. It even mandates that it
Motion filed by the 243 of the original accused declaring should be tried by the court martial. The RTC thus has
the petition for the court assume jurisdiction over all no legal basis to rule that the violation of the following
charges filed before the military court and requiring the Articles of War were committed in furtherance of coup
prosecution to produce evidence to establish probable d etat and as such absorbed by the latter crime. In
cause as MOOT AND ACADEMIC. Furthermore, it making such a declaration the RTC acted without or in
declared that all the charges before the court-martial excess of jurisdiction and is NULL AND VOID.
against the accused are hereby declared NOT SERVICE The writs of prohibition and habeas corpus
CONNECTED BUT IS ABSORBED AND IN prayed for by the petitioners must fail.
FURTHERANCE TO THE ALLEGED CRIME OF COUP D As a general rule, the writ of habeas corpus will
ETAT. not issue where the person alleged to be restrained of his
March 1, 2004, the General Court-martial has set liberty is in custody of an officer under a process issued by
the arraignment/trial of those charged with violations of the a court with jurisdiction and that the writ should not be
Articles of War. allowed after the party sought to be released had been
Petitions for the issuance of temporary charged before any court or quasi-judicial body.
restraining order were filed and the court directed that Thus, the rules apply to petitioners who were
parties to observe the status quo prevail before the filing of detained under Commitment Order issued by the Chief of
the petition. Staff of the AFP.
On the other hand, the office of the writ of
Issue: Whether or not the petitioners are entitled to the prohibition is to prevent inferior courts, corporations,
writs of prohibition and habeas corpus. boards or persons from usurping or exercising a
jurisdiction or power with which they have not been vested
Ruling: No. The Order of the RTC declaring that all the by law.
charges before the court-martial against accused were not In this case, the General Court Martial has
service-connected but absorbed and in furtherance of the jurisdiction over the charges filed against 1Lt.
crime of coup d etat, cannot be given effect. Navales, et. al under RA 7055. A writ of prohibition
When RTC resolved the Omnibus Motion to cannot be issued to prevent it from exercising its
assume jurisdiction over all the charges filed before the jurisdiction.
military tribunal had already been rendered moot and

7
CRIMINAL LAW REVIEW DIGESTS
JUSTICE ROMEO CALLEJO NOTE: = Callejo Ponente

HELD: NO. There is no dispute that Gonzales et al, being


GONZALES v. ABAYA officers of the AFP, are subject to military law according to
Commonwealth Act 408 (AKA Articles of War). Section 1
*READ: concurring opinion of Justice Callejo of R.A. 7055 provides that as a general rule, members of
the AFP and other persons subject to military law,
FACTS: This is about the Oakwood Mutiny (July 26, 2003) including members of the Citizens Armed Forces
where members of the AFP aimed to destabilize the Geographical Units, who commit crimes or offenses
government with use of high-powered weapons and penalized under the Revised Penal Code (like coup
explosive devices. Navy Lt. Trillanes IV & the troops detat), other special penal laws, or local ordinances shall
sported red armbands with the emblem Magdalo. They be tried by the proper civil court. It also provides the
broadcasted their grievances against GMA, such as the exception to the general rule, i.e., where the civil court,
graft and corruption in the military, the illegal sale of arms before arraignment, has determined the offense to be
& ammunition to the "enemies" of the State, and the service-connected, then the offending soldier shall be tried
bombings in Davao City intended to acquire more military by a court martial. Lastly, the law states an exception to
assistance from the US government. They declared the exception, i.e., where the President of the Philippines,
withdrawal of support from the GMA & demanded her in the interest of justice, directs before arraignment that
resignation. any such crimes or offenses be tried by the proper civil
After several hours of negotiation, they eventually court.
surrendered. DOJ charged them with coup detat (defined The same provision also identifies "service-
under Art. 134-A of the RPC) in RTC. Respondent Gen. connected crimes or offenses" as "limited to those defined
Narciso Abaya, then Chief of Staff, filed with the military in Articles 54 to 70, Articles 72 to 92, and Articles 95 to 97"
tribunal for violations of the Articles of War (Art. 63: of the Articles of War. Violations of these Articles are
disrespect toward the Pres., Art. 64: disrespect toward a within the jurisdiction of the court martial.
superior officer, Art. 67: mutiny/sedition, Art. 96: for SC held that the offense for violation of Article 96
conduct unbecoming an officer and a gentleman and of the Articles of War is service-connected. This is
Art. 97: conduct prejudicial to good order & military expressly provided in Section 1 (second paragraph) of
discipline). R.A. No. 7055. The charge against Gonzales et al
Following the doctrine of absorption, Gen. Abaya concerns the alleged violation of their solemn oath as
recommended that those charged with coup detat with officers to defend the Constitution and the duly-constituted
RTC should not be charged before the military tribunal for authorities. Such violation allegedly caused dishonor and
violations of Articles of War. The RTC decided that all disrespect to the military profession. In short, the charge
charges before the court martial against the accusedare has a bearing on their professional conduct or behavior as
hereby declared not service-connected, but rather military officers. Equally indicative of the "service-
absorbed & in furtherance of the alleged crime of coup. connected" nature of the offense is the penalty prescribed
However, Judge Advocate Generals Office of the AFP for the same dismissal from the service imposable only
(JAGO)s Colonel recommended that 29 of the officers by the military court. Such penalty is purely disciplinary in
(out of 321 coz DOJ dropped the case against the others) character, evidently intended to cleanse the military
be prosecuted before a general court martial for violation profession of misfits and to preserve the stringent
of Art. 96. The AFP Judge Advocate General then directed standard of military discipline.
accused to answer the charge. Instead of complying, they There is no merit in Gonzales et als argument
filed with the SC a Petition for Prohibition praying that the that they can no longer be charged before the court
respondents (JAGO) be ordered to desist from charging martial because the same has been declared by the RTC
them with violation of Art. 96. as "not service-connected, but rather absorbed and in
Gonzales et al maintain that since the RTC has furtherance of the alleged crime of coup detat," hence,
made a determination in its Order of February 11, 2004 triable by said court (RTC). The RTC, in making such
that the offense for violation of Article 96 (conduct declaration, practically amended the law which expressly
unbecoming an officer and a gentleman) of the Articles of vests in the court martial the jurisdiction over "service-
War is not service-connected, but is absorbed in the crime connected crimes or offenses." What the law has
of coup detat, the military tribunal cannot compel them to conferred the court should not take away. Evidently, such
submit to its jurisdiction. declaration by the RTC constitutes grave abuse of
discretion tantamount to lack or excess of jurisdiction and
ISSUE: Whether the petitioners (rebels) are entitled to the is, therefore, void.
writ of prohibition. The trial court aggravated its error when it
justified its ruling by holding that the charge of Conduct
Unbecoming an Officer and a Gentleman is absorbed and

8
CRIMINAL LAW REVIEW DIGESTS
JUSTICE ROMEO CALLEJO NOTE: = Callejo Ponente

in furtherance to the alleged crime of coup detat. Firstly, competent tribunal of any country where the
the doctrine of absorption of crimes is peculiar to criminal offender may be found or into which he may be
law and generally applies to crimes punished by the same carried. The jurisdiction of piracy, unlike all other
statute, unlike here where different statutes are involved. crimes, has no territorial limits. As it is against all
Secondly, the doctrine applies only if the trial court has so may it be punished by all.
jurisdiction over both offenses. Here, Section 1 of R.A.
7055 deprives civil courts of jurisdiction over service-
connected offenses, including Article 96 of the Articles of
War. Thus, the doctrine of absorption of crimes is not PEOPLE V. ROGER TULIN
applicable to this case.
Military law is sui generis, applicable only to MT Tabangao is a cargo vessel owned by PNOC. It was
military personnel because the military constitutes an sailing near the coast of Mindoro loaded with barrels of
armed organization requiring a system of discipline kerosene, gasoline, and diesel oil with a total value of
separate from that of civilians. 40.4M. The vessel was suddenly boarded by 7 fully armed
Clearly, the instant petition for prohibition must pirates (accused in the case Emilio Changco, Cecilio
fail. The office of prohibition is to prevent the unlawful and Changco, Tulin, Loyola, Infante, etc.). they detained and
oppressive exercise of authority and is directed against took control of the vessel. The name MT Tabangao and
proceedings that are done without or in excess of the PNOC logo were painted over with black. Then it was
jurisdiction, or with grave abuse of discretion, there being painted with the name Galilee. The ship crew was forced
no appeal or other plain, speedy, and adequate remedy in to sail to Singapore.
the ordinary course of law. Stated differently, prohibition is In Singapore, the ship was awaiting another
the remedy to prevent inferior courts, corporations, vessel that did not arrive. Instead, the ship went back to
boards, or persons from usurping or exercising a Batangas Philippines and remained at sea. Days later, it
jurisdiction or power with which they have not been vested went back to Singapore. This time, another vessel called
by law. the Navi Pride anchored beside it. Another accused,
In fine, SC holds that herein respondents have Cheong San Hiong, supervised the Navis crew and
the authority in convening a court martial and in charging received the cargo on board MT Tabangao/Galilee.
petitioners with violation of Article 96 of the Articles of After the transfer of goods were completed, MT
War. Tabangao/Galilee went back to the Philippines and the
original crew members were released by the pirates in
batches. The crew was ordered not to tell authorities of
PEOPLE VS. LOL-LO what happened.
The chief engineer of the crew, however,
While a boat of Dutch possession was in the high seas, reported the incident to the coast guard. Afterwards, a
Moros surrounded it with small boats. They robbed them series of arrests were effected in different places. An
of food and cargo, attacked some of the men on board, information charging the accused with qualified piracy or
and brutally violated 2 women. Lo-lo and Saraw were two violation of the PD 532 Piracy in the Philippine Waters
of the moros responsible. Lo-lo and Saraw later returned was filed against the accused.
to Tawi-tawi where they were arrested and charged with As it turns out, Navi Pride captain, Hiong, was
the crime of mutiny. They were claiming that the employed with Navi Marine Services ( a Singaporean firm,
Philippine courts does not have jurisdiction since the I think). Before the seizure of the MT Tabangon, Navi
incident happened in the high seas. CFI still found them Marine was dealing for the first time with Paul Gan, a
guilty. Singaporean broker who offered to sell bunker oil to the
former. When the transaction pushed through, Hiong was
Issue: Can a piracy committed outside of the PH triable assigned to supervise a ship to ship transfer. He was told
here? YES. that the Galilee would be making the transfer, so Navi
Pride ship-sided with Galilee and the transfer was
Held: YES. effected. Paul Gan received the payment. Upon arrival in
- All of the elements of the crime of piracy are Singapore, Hiong was asked again to transact another
present. Piracy is robbery or forcible depredation transfer of oil. The same procedure was followed. Hiong
on the high seas, without lawful authority and then went to the Philippines to arrange another transfer
done animo furandi, and in the spirit and intention with Changco the pirates head. This was how Hiong was
of universal hostility. arrested by the NBI agents.
- Piracy is a crime not against any particular state
but against all mankind. It may be punished in the

9
CRIMINAL LAW REVIEW DIGESTS
JUSTICE ROMEO CALLEJO NOTE: = Callejo Ponente

All the accused put up denials and alibis. The trial ordered the Fiscal to file a case against Guevarra for
court, with ROMEO CALLEJO deciding, ruled that the Homicide through reckless Imprudence.
accused were all guilty. Guevarra filed a motion to quash stating that the
information contains averments which if true would
ISSUE: w/n the accused are guilty of qualified piracy constitute an excuse or justification. His primary argument
YES! was that the term discernment connotes intent under
the exempting circumstance found under Art. 12 sec. 3 of
RULING: [only the important part for crim] the RPC (9<x<15 exempting except if acting with
Hiong argues that he can not be convicted under PD 534 discernment). If this were true, then no minor between the
or Art 122 of the RPC as amended, since both laws punish age of 9 to 15 may be convicted of a quasi offense under
piracy committed in Philippine waters. Hiong also Art. 365 (Criminal Negligence).
contends that the court never acquired jurisdiction over
him since the crime was committed outside Philippine Issue: Is discernment the same as intent? NO.
waters.
Art. 122 of the RPC (piracy in general and mutiny Held: Intent is defined as a determination to do certain
in the high seas) provided that piracy must be committed things. On the other hand, discernment is the mental
in the high seas by any person not a member of its capacity to understand the difference between right and
complement nor a passenger thereof. It was amended by wrong. They convey two distinct thoughts. It is therefore
RA 7659, which broadened the law to include offenses incorrect to say that since a minor above nine but below
committed in Philippine waters. PD 532 on the other hand, fifteen years of age acted with discernment, then he
embraces any person, including a passenger or member intended such act to be done.
of the complement of said vessel in the Philippine waters. The second element of dolo (deceit) is
Passenger or not, member of the complement or not, any intelligence; without this power, necessary to determine
person is covered by the law. No conflict exists among the the morality of human acts to distinguish a licit from an
mentioned laws, they exist harmoniously as separate illicit act, no crime can exist. Thats why we have article
laws. 12.
The attack on and the seizure of MT Tabangao In evaluating felonies committed by means of
and its cargo were committed in Philippine waters, culpa (fault), three (3) elements are indispensable,
although the captive vessel was later brought by the namely, intelligence, freedom of action, and negligence.
pirates to Singapore, where its cargo was off-loaded, Obviously, intent is wanting in such felonies. However,
transferred and sold. Such transfer was done under intelligence remains as an essential element, hence, it is
Hiongs supervision. Although the disposition by the necessary that a minor above nine but below fifteen years
pirates of the vessel and its cargo was not done in of age be possessed with intelligence in committing a
Philippine waters, it is still deemed part of the same negligent act which results in a quasi-offense. For him to
act. Piracy falls under Title 1 of Book 2 of the RPC. It be criminally liable, he must discern the rightness or
is an exception to the rule on territoriality in criminal wrongness of the effects of his negligent act.
law. The same principle applies to the case, even if As such, Guevarra was not exempted and the
Hiong is charged with violation of a special penal law, case was remanded to the lower court.
instead of the RPC. Regardless of the law penalizing
piracy, it remains to be a reprehensible crime against
the whole world. PEOPLE vs OJEDA
PONENTE: Corona

GUEVARRA V. ALMODOVAR FACTS: This is a case for estafa and violation of BP 22.
Cora Ojeda used to buy fabrics from Ruby Chua. All in all
Facts: John Philip Guevarra, an 11 year old, was playing Ojeda 228,306 pesos using 22 postdated checks. When
with his best friend Teodoro Almine, Jr. and three other the checks were presented for payment, they were
children in their backyard. They were target-shooting a dishonored due to account closed. Criminal charges
tansan using an air rifle borrowed from a neighbor. In the were lodged against Ojeda.
course of their game, Almine was hit by a pellet on his left In defense Ojeda claims good faith, absence of
collar bone which caused his unfortunate death. deceit, lack of notice of dishonor and full payment of the
After the preliminary investigation, the examining amount of the checks. Also, Ojeda claims she advised
Fiscal exculpated Guevarra due to his age and because Chua not to cash the checks because they were not yet
the unfortunate occurrence appeared to be an accident. sufficiently funded. Finally, she claims she made partial
Almines parents appealed to the Ministry of Justice, which

10
CRIMINAL LAW REVIEW DIGESTS
JUSTICE ROMEO CALLEJO NOTE: = Callejo Ponente

payments worth 50,000 pesos in the form of finished against the door. In the darkness and confusion Ah Chong
garments. thought that the blow had been inflicted by the person who
The trial court convicted her but only for 14 had forced the door open. Seizing a common kitchen knife
counts out of 22 bouncing checks issued. This was which he kept under his pillow, Ah Chong struck out wildly
because some checks were not covered by the indictment at the intruder who, it afterwards turned out, was his
and others were not signed by her but by her husband. roommate, Pascual.
Pascual ran out upon the porch and fell down on
ISSUE: whether or not the defense of Ojeda for absence the steps. Seeing that Pascual was wounded, Ah Chong
of deceit is tenable. YES. called to his employers who slept in the next house, No.
28, and ran back to his room to secure bandages to bind
HELD: Under Art. 315 of the RPC the following are the up Pascual's wounds.
requisites for estafa: first, a check is postdated or issued in The deceased and the accused had an
payment of an obligation contracted at the time it is understanding to knock at the door and acquaint his
issued; second, lack or insufficiency of funds to cover the companion with his identity.
check; third, damage to the payee thereof. Deceit and Ah Chong alleged that it was because of
damage are essential elements of the offense and must repeated robberies that he kept a knife under his pillow for
be established by satisfactory proof to warrant conviction. his personal protection. He admitted that he had stabbed
Deceit was not proven by the prosecution. In fact, Ojeda his roommate, but said that he did it under the impression
not only made arrangements for the payment of the debts that Pascual was "a ladron" (burglar) because he forced
but in fact paid (because during the pendency of the open the door of their sleeping room, despite warnings.
appeal an affidavit of desistance was introduced by TC: Ah Chong was guilty of simple homicide, with
Ojeda). This is a sign of good faith and absence of malice extenuating circumstances. Ah Chong admitted that he
an essential element of estafa and crimes under the killed his roommate but insisted that he struck the fatal
RPC which are mala in se. blow without any intent to do a wrongful act, in the
exercise of his lawful right of self-defense.
Minor ruling: there was also lack of notice of dishonor.
Prosecution merely presented a copy of the demand letter ISSUE: WON one can be held criminally responsible who,
and the registry receipt. However, the registry receipt does by reason of a mistake as to the facts, does an act for
not prove itself. It needs to be authenticated and identified. which he would be exempt from criminal liability if the facts
In this case, it was not. were as he supposed them to be, but which would
constitute the crime of homicide or assassination if the
actor had known the true state of the facts at the time
UNITED STATES vs. AH CHONG when he committed the act

Ah Chong was employed as a cook at Officers' quarters RULING: there is no criminal liability, provided always that
No. 27 at Fort Mc Kinley and at the same place Pascual the alleged ignorance or mistake or fact was not due to
Gualberto was employed as a house boy or muchacho. negligence or bad faith. Ah Chong acquiited.
No one slept in the house except the two servants, There can be no doubt that defendant would be
ocuppying a small room toward the rear of the building. entitle to complete exception from criminal liability for the
The door of the room was not furnished with a permanent death of the victim of his fatal blow, if the intruder who
bolt or lock, and occupants, as a measure of security, had forced open the door of his room had been in fact a
attached a small hook or catch on the inside of the door, dangerous thief or "ladron," as the defendant believed him
and were in the habit of reinforcing this somewhat to be. No one, under such circumstances, would doubt the
insecure means of fastening the door by placing against it right of the defendant to resist and repel such an intrusion,
a chair. and the thief having forced open the door notwithstanding
Ah Chong was suddenly awakened by someone defendant's thrice-repeated warning to desist, and his
who was forcing to open the door. He sat up in bed and threat that he would kill the intruder if he persisted in his
called out twice, "Who is there?" He heard no answer and attempt, it will not be questioned that in the darkness of
was convinced by the noise at the door that it was being the night, in a small room, with no means of escape, with
pushed open by someone bent upon forcing his way into the thief advancing upon him despite his warnings
the room. Fearing that the intruder was a robber or a thief, defendant would have been wholly justified in using any
Ah Chong leaped to his feet and called out. "If you enter available weapon to defend himself from such an assault,
the room, I will kill you." and in striking promptly, without waiting for the thief to
At that moment he was struck just above the discover his whereabouts and deliver the first blow.
knee by the edge of the chair which had been placed

11
CRIMINAL LAW REVIEW DIGESTS
JUSTICE ROMEO CALLEJO NOTE: = Callejo Ponente

But the evidence clearly discloses that the


intruder was not a thief or a "ladron." That neither the TORRES, J., dissenting:
defendant nor his property nor any of the property under the crime of homicide by reckless negligence, defined and
his charge was in real danger at the time when he struck punishes in article 568 of the Penal Code, was committed,
the fatal blow. That there was no such "unlawful inasmuch as the victim was wilfully killed, and while the
aggression" on the part of a thief or "ladron" as defendant act was done without malice or criminal intent it was,
believed he was repelling and resisting, and that there was however, executed with real negligence, for the acts
no real "necessity" for the use of the knife to defend his committed by the deceased could not warrant the
person or his property or the property under his charge. aggression by the defendant under the erroneous belief on
ignorance or mistake of fact, if such ignorance or the part of the accused that the person who assaulted him
mistake of fact is sufficient to negative a particular intent was a malefactor; the defendant therefore incurred
which under the law is a necessary ingredient of the responsibility in attacking with a knife the person who was
offense charged (e.g., in larcerny, animus furendi; in accustomed to enter said room, without any justifiable
murder, malice; in crimes intent) "cancels the presumption motive.
of intent," and works an acquittal; except in those cases
where the circumstances demand a conviction under the
penal provisions touching criminal negligence; and in PEOPLE V. DELIM
cases where, under the provisions of article 1 of the Penal
Code one voluntarily committing a crime or misdeamor FACTS: Marlon, Manuel and Robert Delim are brothers.
incurs criminal liability for any wrongful act committed by They are the uncles of Leon and Ronald Delim. Modesto
him, even though it be different from that which he Delim, the victim (deceased), was adopted by the father of
intended to commit. the brothers.
Ignorantia facti excusat ("Ignorance or mistake in On January 23, 1999, Modesto, Rita (wife),
point of fact is, in all cases of supposed offense, a Randy (son) and their 2 grandchildren were about to eat
sufficient excuse"). their dinner when Marlon, Robert and Ronald barged into
Since evil intent is in general an inseparable element in the house. They were armed with a short handgun. Marlon
every crime, any such mistake of fact as shows the act poked his gun at Modesto while Robert and Ronald
committed to have proceeded from no sort of evil in the simultaneously grabbed and hog-tied the victim. A piece of
mind necessarily relieves the actor from criminal liability cloth was placed in the mouth of Modesto. They then
provided always there is no fault or negligence on his part; herded Modesto out of the house on their way towards the
That is to say, the question as to whether he honestly, in direction of Paldit, Sison, Pangasinan. Leon and Manuel,
good faith, and without fault or negligence fell into the also armed with short handguns, stayed put by the door to
mistake is to be determined by the circumstances as they the house of Modesto and ordered Rita and Randy to stay
appeared to him at the time when the mistake was made, where they were. Leon and Manuel left the house at
and the effect which the surrounding circumstances might around 7am the following day.
reasonably be expected to have on his mind, in forming On January 27, 1999, Randy, in the company of
the intent, criminal or other wise, upon which he acted. his relatives, found Modesto under thick bushes in a
Ah Chong struck the fatal blow alleged in the grassy area. He was already dead. The cadaver was
information in the firm belief that the intruder who forced bloated and in the state of decomposition. It exuded a bad
open the door of his sleeping room was a thief, from odor. Tiny white worms swarmed over and feasted on the
whose assault he was in imminent peril, both of his life cadaver. Randy and his relatives immediately rushed to
and of his property and of the property committed to his the police station to report the incident and to seek
charge; that in view of all the circumstances, as they must assistance.
have presented themselves to the defendant at the time, According to the autopsy, the cause of death was
he acted in good faith, without malice, or criminal intent, in a gunshot wound at the head and the stab wounds
the belief that he was doing no more than exercising his sustained by the victim on his left and forearm were
legitimate right of self-defense; that had the facts been as defensive wounds. The investigators confirmed that the
he believed them to be he would have been wholly exempt accused had no licenses for their firearms.
from criminal liability on account of his act; and that he can Only Marlon, Ronald and Leon were arrested.
not be said to have been guilty of negligence or Manuel and Robert were not found.
recklessness or even carelessness in falling into his To exculpate themselves, Marlon, Ronald and
mistake as to the facts, or in the means adopted by him to Leon interposed denial and alibi.
defend himself from the imminent danger which he believe The trial court rendered judgment finding
threatened his person and his property and the property accused-appellants guilty of aggravated murder (The trial
under his charge. court appreciated treachery as a qualifying circumstance

12
CRIMINAL LAW REVIEW DIGESTS
JUSTICE ROMEO CALLEJO NOTE: = Callejo Ponente

and of taking advantage of superior strength, nighttime prompts the accused to engage in a particular criminal
and use of unlicensed firearms as separate of aggravating activity. Motive is not an essential element of a crime and
circumstances in the commission of the crime) and hence the prosecution need not prove the same. As a
sentenced to suffer the penalty of death. The amount of general rule, proof of motive for the commission of the
P75,000 for moral damages and P25,000 for exemplary offense charged does not show guilt and absence of proof
damages was awarded. of such motive does not establish the innocence of
accused for the crime charged such as murder. The
ISSUE: history of crimes shows that murders are generally
1) WON the crime charged in the information is murder or committed from motives comparatively trivial. Crime is
kidnapping Murder rarely rational. In murder, the specific intent is to kill the
2) WON the prosecution mustered the requisite quantum victim. In kidnapping, the specific intent is to deprive the
of evidence to prove that accused are guilty of murder victim of his/her liberty. If there is no motive for the crime,
3) WON the qualifying circumstances should be the accused cannot be convicted for kidnapping. In
considered kidnapping for ransom, the motive is ransom. Where
accused kills the victim to avenge the death of a loved
HELD/ RATIO: one, the motive is revenge.
1) The crime charged is murder. In this case, it is evident on the face of the
In determining what crime is charged in an information, the Information that the specific intent of the malefactors in
material inculpatory facts recited therein describing the barging into the house of Modesto was to kill him and that
crime charged in relation to the penal law violated are he was seized precisely to kill him with the attendant
controlling. Where the specific intent of the malefactor is modifying circumstances. The act of the malefactors of
determinative of the crime charged such specific intent abducting Modesto was merely incidental to their primary
must be alleged in the information and proved by the purpose of killing him. Moreover, there is no specific
prosecution. A decade ago, this Court held in People v. allegation in the information that the primary intent of the
Isabelo Puno, et al., that for kidnapping to exist, there malefactors was to deprive Modesto of his freedom or
must be indubitable proof that the actual specific intent of liberty and that killing him was merely incidental to
the malefactor is to deprive the offended party of his liberty kidnapping. Irrefragably then, the crime charged in the
and not where such restraint of his freedom of action is Information is Murder under Article 248 of the Revised
merely an incident in the commission of another offense Penal Code and not Kidnapping under Article 268 thereof.
primarily intended by the malefactor.
If the primary and ultimate purpose of the 2) YES
accused is to kill the victim, the incidental deprivation of In this case, the prosecution was burdened to prove the
the victim's liberty does not constitute the felony of corpus delicti which consists of two things: 1) the criminal
kidnapping but is merely a preparatory act to the killing, act and 2) defendant's agency in the commission of the
and hence, is merged into, or absorbed by, the killing of act. Wharton says that corpus delicti includes two things:
the victim. The crime committed would either be homicide 1) the objective; 2) the subjective element of crimes. In
or murder. homicide (by dolo) and in murder cases, the prosecution is
What is primordial then is the specific intent of burdened to prove: (a) the death of the party alleged to be
the malefactors as disclosed in the information or criminal dead; (b) that the death was produced by the criminal act
complaint that is determinative of what crime the accused of some other than the deceased and was not the result of
is charged with that of murder or kidnapping. accident, natural cause or suicide; and (c) that defendant
Specific intent is used to describe a state of mind committed the criminal act or was in some way criminally
which exists where circumstances indicate that an responsible for the act which produced the death. To prove
offender actively desired certain criminal consequences or the felony of homicide or murder, there must be
objectively desired a specific result to follow his act or incontrovertible evidence, direct or circumstantial, that the
failure to act. Specific intent must be alleged in the victim was deliberately killed (with malice); in other words,
Information and proved by the state in a prosecution for a that there was intent to kill. Such evidence may consist
crime requiring specific intent. Kidnapping and murder are inter alia in the use of weapons by the malefactors, the
specific intent crimes. nature, location and number of wounds sustained by the
Specific intent may be proved by direct evidence victim and the words uttered by the malefactors before, at
or by circumstantial evidence. It may be inferred from the the time or immediately after the killing of the victim. If the
circumstances of the actions of the accused as victim dies because of a deliberate act of the malefactor,
established by the evidence on record. intent to kill is conclusively presumed.
Specific intent is not synonymous with motive. In the case at bar, the prosecution adduced the
Motive generally is referred to as the reason which requisite quantum of proof of corpus delicti. Modesto

13
CRIMINAL LAW REVIEW DIGESTS
JUSTICE ROMEO CALLEJO NOTE: = Callejo Ponente

sustained 5 gunshot wounds. He also sustained 7 stab later, or that his violent end was the consequence of the
wounds, defensive in nature. The use by the malefactors abduction, and nothing more, would be to unduly put to
of deadly weapons, more specifically handguns and risk our standard of moral certainty required for all
knives, in the killing of the victim as well as the nature, convictions.
number and location of the wounds sustained by said The evidence would indeed point out that Marlon,
victim are evidence of the intent by the malefactors to kill Ronald and Robert seized Modesto Delim from his house
the victim with all the consequences flowing therefrom. while Leon and Manuel stood guard and stayed at the
door of the victim's house. Randy Manalo Bantas and Rita
3) NO Manalo Bantas, however, could only testify on the
Qualifying circumstances such as treachery and abuse of participation of each of the malefactors in the abduction of
superior strength must be alleged and proved clearly and Modesto Delim but not on what might have happened to
conclusively as the crime itself. Article 14, paragraph 16 of him thereafter. In arriving at its verdict convicting
the Revised Penal Code provides that there is treachery appellants for "aggravated murder," the trial court
when the offender commits any of the crimes against the considered the act of the accused of forcibly taking
person, employing means, methods or forms in the Modesto Delim from his house as being likewise enough
execution thereof which tend directly and especially to to substantiate the killing by them of the victim. The
insure its execution, without risk to himself arising from the conclusion could rightly be assailed. The accounts of
defense which the offended party might make. For Randy and his mother Rita would indicate that the forcible
treachery to be appreciated as a qualifying circumstance, taking of Modesto was carried out in absolute silence, with
the prosecution is burdened to prove the following not one of the five intruders uttering any word which could
elements: (a) the employment of means of execution give a clue on the reason for the abduction and, more
which gives the person attacked no opportunity to defend particularly, whether the same was carried out for the
himself or retaliate; (b) the means of execution is purpose of killing Modesto. The two witnesses were
deliberately or consciously adopted. In this case, the unaware of any existing grudge between the malefactors
victim was defenseless when seized by Marlon and and the victim that could have prompted them to violently
Ronald. However, the prosecution failed to present any snuff out the life of the latter. While the motive of an
witness or conclusive evidence that Modesto was accused in a criminal case might generally be immaterial,
defenseless immediately before and when he was not being an element of the crime, motive could be
attacked and killed. It cannot be presumed that although important and consequential when the evidence on the
he was defenseless when he was seized the victim was in commission of the crime would be short of moral certainty.
the same situation when he was attacked, shot and The facts point to only one established fact, i.e., that the
stabbed by the malefactors. As to superior strength, what accused forcibly took Modesto Delim from his residence to
is primordial is that the assailants deliberately took an unknown destination on the night of 23 January 1999,
advantage of their combined strength in order to would be scanty to support a conclusion that the five,
consummate the crime. In this case, the prosecution failed aside from abducting the victim, likewise killed him. There
to adduce evidence that Marlon and Ronald deliberately was an unexplained gap in what ought to have been a
took advantage of their numerical superiority when continuous chain of events. The body bore several
Modesto was killed. The barefaced facts that the defensive wounds, which could give rise to the not too
malefactors outnumbered Modesto and were armed while unlikely scenario that Modesto might have ultimately been
Modesto was not does not constitute proof that the three released by his abductors sometime before he was killed.
took advantage of their numerical superiority and their
handguns when Modesto was shot and stabbed.
The aggravating circumstance of unlicensed RAFAEL REYES TRUCKING CORPORATION vs.
firearm and dwelling was not alleged in the information PEOPLE OF THE PHILIPPINES
thus, cannot be considered.
In sum, Marlon, Ronald and Leon are guilty only FACTS: Romeo Dunca, driver of a trailer truck registered
of Homicide. under the name of Rafael Reyes Trucking Corp (RRTC),
was charged with reckless imprudence resulting in double
Separate Opinion: VITUG homicide and damage to property. The private offended
The crime charged should be modified to parties also instituted a separate civil action against RRTC
kidnapping and serious illegal detention. as employer of Romeo based on quasi delict. From the
The evidence would show that Modesto Delim records, it was shown that Romeo, while driving along the
was forcibly abducted from his residence by appellants, all national road in Isabela, approached a damaged portion of
armed, on the night of 23 January 1999. But to say that the road, where the surface of the road was uneven.
the same group was also responsible for his death, days However, the left lane parallel to this portion was smooth.

14
CRIMINAL LAW REVIEW DIGESTS
JUSTICE ROMEO CALLEJO NOTE: = Callejo Ponente

Thus, they used the parallel lane to avoid taking the damages under Article 2176 of the Civil Code, arising from
damage road. However, at that moment, there was an the same act or omission of the accused.
incoming Nissan vehicle causing the truck to ram with the When private respondents, as complainants in
former. As a result, 2 of the passengers of the Nissan died the criminal action, reserved the right to file the separate
instantly. The RTC rendered a decision in the criminal civil action, they waived other available civil actions
case, finding Romeo guilty of the crime of Double predicated on the same act or omission of the accused-
Homicide through Reckless Imprudence with violation of driver. Such civil action includes the recovery of indemnity
the Motor Vehicle Law (RA 4136) and ordering him to under the Revised Penal Code, and damages under
indemnify the heirs of the victims. With regard to the civil Articles 32, 33, and 34 of the Civil Code of the Philippines
action filed against RRTC, the RTC ordered the company arising from the same act or omission of the accused.
to pay the offended parties actual damages. Both Romeo The intention of offended parties to proceed
and RRTC appealed, while the offended parties moved primarily and directly against RRTC as employer of
that the dispositive portion be changed so as to hold accused truck driver, Romeo, became clearer when they
RRTC solidarily liable for the damages awarded in the did not ask for the dismissal of the civil action against the
criminal case in case Romeo becomes insolvent. The trial latter based on quasi delict. Thus, the Court of Appeals
court granted the motion of the offended parties and and the trial court erred in holding the accused civilly
issued a supplemental decision declaring RRTC as liable, and petitioner-employer of the accused subsidiarily
subsidiarily liable for the damages awarded in the criminal liable for damages arising from crime (ex delicto) in the
case in the event of Romeos insolvency. RRTC appealed criminal action as the offended parties in fact filed a
the supplemental decision. However, during the pendency separate civil action against the employer based on quasi
of the appeal, Romeo jumped bail and fled to a foreign delict resulting in the waiver of the civil action ex delicto.
country. The CA affirmed the trial courts supplemental
decision.
People v Carmen
ISSUE: W/N RRTC may be held subsidiarily liable for
damages awarded to the offended parties in the Facts: A boy named Randy Luntayao was believed by his
criminal action against Romeo despite the filing of a father (Eddie) to have a nervous breakdown manifested
separate civil action by the offended parties against it. by him talking and laughing by himself. He thinks that the
NO. breakdown was caused by skipping meals whenever he
took the boy with him to the farm.
HELD/RATIO: In negligence cases, the aggrieved party Upon the suggestion of one of the accused in this case,
has the choice between (1) an action to enforce civil Eddie, wife Perlita and their three children went with said
liability arising from crime under Article 100 of the Revised accused to Cebu. Upon arriving in Cebu they went to the
Penal Code; and (2) a separate action for quasi delict house of another accused Carmen and diagnosed the boy
under Article 2176 of the Civil Code of the Philippines. to be possessed with a bad spirit and that she could
Once the choice is made, the injured party can not avail exorcise. Warning that in conducting exorcism, the bad
himself of any other remedy because he may not recover spirit might transfer to Eddie it was best to do the healing
damages twice for the same negligent act or omission of prayer without him. Eddie, wife and children were locked
the accused. This is the rule against double recovery. In inside a room in the house.
other words, "the same act or omission can create two The exorcism conducted by Carmen was witnessed by
kinds of liability on the part of the offender, that is, civil two children who were playing takyan when they heard a
liability ex delicto, and civil liability quasi delicto" either of shout asking for help from his mother. They ran to the
which "may be enforced against the culprit, subject to the direction of the house of Carmen and saw that Randy was
caveat under Article 2177 of the Civil Code that the being immersed in water head first by the 4 accused.
offended party cannot recover damages under both types They also saw him being tied on a bench while Carmen
of liability." poured water into the mouth of the boy. Each time the boy
In this case, RRTC, as employer of the accused struggled to raise his head, accused Alexander banged
Romeo, who has been adjudged guilty in the criminal case the boys head against the bench. She also witnessed
for reckless imprudence, cannot be held subsidiarily liable accused Celedonia dropped her weight on the body of the
because of the filing of the separate civil action based on boy. They also took turns in pounding the boys chest with
quasi delict against it. In view of the reservation to file, and their clenched fist. Then Carmen asked one of the
the subsequent filing of the civil action for recovery of civil accused to get a knife and after which the knife was slowly
liability, the same was not instituted with the criminal plunged into the left side of the boys body. Then the boy
action. Such separate civil action was for recovery of was carried into the house.

15
CRIMINAL LAW REVIEW DIGESTS
JUSTICE ROMEO CALLEJO NOTE: = Callejo Ponente

Around 5 oclock in the afternoon Randy was already convict Anselmo Balagtas with bailarina Irene. Get him
dead. dead or alive."
After Eddie and his family returned to Negros Occidental, Defendant corporal Galanta and chief of police
Eddie sought the assistance from the Bombo Radyo Oanis were 2 of the 5 who reported to the office of the
station in Bacolod City. As the incident took place in Cebu, Provincial Inspector where they were shown a copy of the
NBI in Cebu conducted the investigation and autopsy telegram and a picture of Balagtas. They were instructed
report of the exhumed body. to arrest Balagtas and, if overpowered, to follow the
The family filed a case in court against Carmen et.al of instruction in the telegram (dead or alive).
murder. The trial court found them guilty of murder The group of defendants Oanis and Galanta went
arguing that killing a person with treachery is murder. It to the house where Irene was supposedly living. There
cited a court decision stating that even if there is no intent Oanis approached one Brigida Mallare who pointed to
to kill, in inflicting physical injuries with treachery, the them Irenes room and said that Irene was sleeping with
accused in that case was convicted of murder. her paramour at that time.
Intent is presumed from the commission of an unlawful Defendants Oanis and Galanta then went to the
act. In the case at bar, there is enough evidence that the room of Irene, and seeing a man sleeping with his back
accused confederated with each other in inflicting physical towards the door where they were, simultaneously or
harm to the victim (illegal act). These acts were successively fired at him with their .32 and .45 caliber
intentional and thus they should be liable for all the direct revolvers. Awakened by the gunshots, Irene saw her
and natural consequences of their unlawful act. paramour already wounded, and looking at the door where
the shots came, she saw the defendants still firing at him.
Issue: Whether or not the accused is guilty of the crime of Later it was found that the person shot was not the
murder? NO. notorious criminal Anselmo Balagtas but one Serapio
Tecson, Irene's paramour.
Ruling: There was no criminal intent on the part of the Oanis and Galanta gave contradicting versions of
accused to kill the boy. It was shown that the accused the incident and it was made apparent from these
are members of a cult and the bizarre ritual was contradictions that when each of the appellants tries to
consented to by the parents of the boy. Their liability exculpate himself of the crime charged, he is at once
arises from their reckless imprudence because they ought belied by the other; but their mutual incriminating
to know their actions would not bring about the cure. They averments corroborated substantially, the testimony of
are guilty of reckless imprudence resulting in homicide and Irene. A careful examination of Irene's testimony showed
not murder. that it contained all indicia of veracity. In her cross-
The RPC states that reckless imprudence examination, even misleading questions had been put
consists in voluntarily, but without malice, doing or which were unsuccessful, the witness having stuck to the
failing to do an act from which material damage truth in every detail of the occurrence.
results by reason of inexcusable lack of precaution on LC: homicide through reckless imprudence
the person performing such an act. Intentional The LC took into consideration the fact that Oanis
felonies such as murder or homicide, what takes the and Galanta acted in innocent mistake of fact in the
place of the element of is the failure of the offender to honest performance of their official duties, both of them
take precautions due to lack if skill. believing that Tecson was Balagtas
The accused lack medical skills in treating the
victim of his ailment, resulted in the latters death. Issue: WON Oanis and Galanta may be held responsible
Treachery in this case cannot be appreciated for the death of Tecson
in the absence of intent to kill. The acts of the accused
therefore considered by the court as treachery are in fact Held: YES. Murder, not homicide through reckless
efforts by the accused to restrain the boy so that they can imprudence with qualifying circumstance of alevosia
cure him. Thus, the decision of the RTC is affirmed and Although an officer in making a lawful arrest is justified in
modified declaring the accused guilty of reckless using such force as is reasonably necessary to secure and
imprudence resulting in homicide. detain the offender, he is never justified in using
unnecessary force or in resorting to dangerous means
when the arrest could be effected otherwise.
PEOPLE vs. ANTONIO Z. OANIS and ALBERTO It may be true that Anselmo Balagtas was a
GALANTA notorious criminal, a life-termer, but these facts alone
constitute no justification for killing him when in effecting
Facts: The Provincial Inspector at Cabanatuan, Nueva his arrest, he offers no resistance or in fact no resistance
Ecija, received a telegram: "Information received escaped can be offered, as when he is asleep.

16
CRIMINAL LAW REVIEW DIGESTS
JUSTICE ROMEO CALLEJO NOTE: = Callejo Ponente

A mitigating circumstance of weight consisting kept insisting that he should or could go back to the
in the incomplete justifying circumstance defined in article restaurant while the latter prevented him from doing
11, No. 5, of the Revised Penal Code: a person incurs no so. Upon nearing their house, the appellant abruptly
criminal liability when he acts in the fulfillment of a duty or stopped the pick-up and the victim alighted. Holding a
in the lawful exercise of a right or office. bottle of beer in his right hand, the victim raised both of his
There are two requisites in order that the hands, stood in front of the pick-up and said, "sige kung
circumstance may be taken as a justifying one: gusto mo sagasaan mo ako, hindi ka makakaalis." The
a) that the offender acted in the performance of a duty or appellant slowly drove the pick-up forward threatening to
in the lawful exercise of a right; and run over the victim. At this juncture, the victim exclaimed,
(b) that the injury or offense committed be the necessary "papatayin mo ba ako?. The appellant backed-up almost
consequence of the due performance of such duty or the hitting an owner type jeep parked at the side of the road
lawful exercise of such right or office. and on board was prosecution eyewitness, Ma. Cecilia
In the instance case, only the first requisite is present Mariano. Then at high speed, the appellant drove the pick-
up forward hitting the victim in the process. Not satisfied
PARAS, DISSENTING: with what he had done, the appellant put the vehicle in
In my opinion, therefore, the appellants are not criminally reverse thereby running over the victim a second time.
liable if the person killed by them was in fact Anselmo The appellant then alighted from the vehicle and walked
Balagtas for the reason that they did so in the fulfillment of towards their house.
their duty and in obedience to an order issued by a Witnesses rushed the victim to the Dolorosa
superior for some lawful purpose (Revised Penal Code, Hospital at Norzagaray, Bulacan where the victim expired
art. 11, pars. 5 and 6). They also cannot be held criminally shortly thereafter.
liable even if the person killed by them was not Anselmo The appellant was not immediately prosecuted
Balagtas, but Serapio Tecson, because they did so under for the death of his father which he was able to pass off as
an honest mistake of fact not due to negligence or bad an accident. But when his older sister, Leslie C. Padilla,
faith. (U.S. vs. Ah Chong, 15 Phil., 488). arrived from the United States to attend her father's wake
and funeral, she made inquiries about the circumstances
HONTIVEROS, DISSENTING: surrounding his death and was given different versions of
Appellants found there asleep a man closely resembling the incident, some of which insinuated that her father did
the wanted criminal. Oanis said: If you are Balagtas stand not meet his demise accidentally. Later, a suspicion of foul
up," But the supposed criminal showed his intention to play moved her to engage the services of the NBI for a
attack the appellants, a conduct easily explained by the formal investigation into the matter. An information for
fact that he should have felt offended by the intrusion of parricide against appellant thereafter.
persons in the room where he was peacefully lying down
with his mistress. In such predicament, it was nothing but Issue: W/N Castillo Jr. is guilty of parricide. YES.
human on the part of the appellants to employ force and to
make use of their weapons in order to repel the imminent Held: The prosecution has successfully established the
attack by a person who, according to their belief, was elements of parricide: (1) the death of the deceased; (2)
Balagtas It was unfortunate, however that an innocent that he or she was killed by the accused; and (3) that the
man was actually killed. deceased was a legitimate ascendant or descendant, or
the legitimate spouse of the accused.

PEOPLE V. CASTILLO JR. Issue: W/N the parricide was committed thru reckless
imprudence as claimed by the appellant.
Facts: On Nov. 6, 1993, between 7:30 and 8:30 pm,
appellant Castillo, Jr. was in the D&G Restaurant in Held: Appellant claims that there was no intention on his
Bulacan, with his father Castillo, Sr., who is the victim in part to kill his father, and that he had accidentally stepped
this case. They were drinking. After 2 hours, a group of on the gas pedal forcefully, causing the vehicle to travel, at
noisy customers arrived in the restaurant. Castillo Sr., a fast speed.
aware of his sons propensity to get into fights, urged The details of the incident as narrated by Mariano
Castillo Jr. to go home with him. The 2 boarded a blue and Agaran bespeak of a crime committed with full intent.
pick-up truck and went. And we have held that a deliberate intent to do an unlawful
Castillo Jr. drove the vehicle home with Castillo act is essentially inconsistent with the idea of reckless
Sr. in the passengers seat. During the trip home, an imprudence. What qualifies an act as one of reckless or
argument ensued between the appellant and the victim simple negligence or imprudence is the lack of malice or
who were both a bit drunk already because the former criminal intent in the execution thereof. Otherwise stated,

17
CRIMINAL LAW REVIEW DIGESTS
JUSTICE ROMEO CALLEJO NOTE: = Callejo Ponente

in criminal negligence, the injury caused to another should at Zurbaran Mart as compared to her expenses. Accused-
be unintentional, it being simply the incident of another act appellant would force sex on Grace especially when he
done without malice but with lack of foresight, was drunk. Defense claimed that the accused was initially
carelessness, or negligence, and which has harmed thinking about ending his life by shooting himself because
society or an individual. of the financial woes and his wifes relentless pestering
Mariano testified that the blue pick-up truck and nagging, but in the process of both spouses struggle
suddenly rushed forward at a high speed then stopped. to take possession of the gun, it went off and hit Grace.
The victim alighted, went in front of the car and screamed
papatayin mo ba ako while the driver was trying to Issue:
intimidate him with the headlights. Then the truck backed 1) WON THE KILLING WAS ACCIDENTAL, AND THAT
up, almost hitting them and then rushed forward hitting the THE DECEASED WAS EXEMPT FROM CRIMINAL
old man. Then afterwards, it even backed up on the body. LIABILITY (NO!)
The records are bereft of any evidence that the 2) MAIN ISSUE AS TO ARTICLE 3: WON THE KILLING
appellant had tried to avoid hitting the victim who WAS DUE TO SIMPLE NEGLIGENCE (NO!)
positioned himself in front of the pick-up. On the contrary, 3) WON ACCUSED IS GUILTY OF PARRICIDE BEYOND
Mariano's testimony is to the effect that prior to actually REASONABLE DOUBT (YES!)
hitting the victim, the appellant was "intimidating" him by
moving the pick-up forward, thus prompting the victim to HELD: 1) No. First, accused-appellant cannot Paragraph
exclaim, "papatayin mo ba ako?". Worse, the appellant 4, Article 12 of the Revised Penal Code in order to be
backed-up to gain momentum, then accelerated at a very exempted from criminal liability. Said provision pertinently
fast speed knowing fully well that the vehicle would states:
definitely hit the victim who was still standing in front of the Art. 12. Circumstances which exempt from
same. criminal liability. The following are exempt from criminal
The appellant's actuations subsequent thereto liability: 4) Any person who, while performing a lawful act
also serve to refute his allegation that he did not intend to with due care, causes an injury by mere accident without
kill his father. Surely, the appellant must have felt the fault or intention of causing it.
impact upon hitting the victim. The normal reaction of any Accident to be exempting, presupposes that the
person who had accidentally ran over another would be to act done is lawful. Here, however, the act of accused-
immediately alight from the vehicle and render aid to the appellant of drawing a weapon in the course of a quarrel,
victim. But as if to ensure the victim's death, the appellant the same not being in self-defense, is unlawful -- it at least
instead backed-up, thereby running over the victim again. constitutes light threats (Article 285, par. 1). There is thus
no room for the invocation of accident as a ground for
exemption. The gun was not even licensed or registered
PEOPLE V. NEPOMUCENO, JR. hence, he could have been charged with illegal
possession of a firearm. Secondly, appellant's claim that
Article 3 of RPC- Felonies are committed either thru the shooting happened when he tried to prevent his wife
dolo (deceit) or culpa (fault) from killing herself and he and his wife grappled for the
possession of the gun is belied by the expert testimony of
Facts: Accused-appellant Guillermo Nepomuceno, Jr. has Dr. Arizala of the who conducted a second post mortem
appealed in regard to the decision finding him guilty of the examination. Moreover, the act of accused ordering Eden
crime of parricide as defined and penalized under Article Ontog to call a taxi in which he brought the wounded
246 of RPC for the death of Grace Nepomuceno. On May Grace to the hospital is "merely an indication or act of
2, 1994 in Manila, the said accused, did then and there repentance or contrition on the part of appellant. Accused-
willfully, unlawfully and feloniously, with intent to kill and appellant's voluntary surrender is not sufficient ground to
with treachery and evident premeditation, attack, assault exculpate him from criminal liability. The law merely
and use personal violence upon the person of one considers such act as a mitigating circumstance. Non-
GRACE NEPOMUCENO Y BENITEZ, his wife, with whom flight is not proof of innocence.
he was married in lawful wedlock, by then and there
shooting her with a gun of unknown caliber hitting her on 2) No. What qualifies an act of reckless or simple
the left hip, thereby inflicting upon the victim a fatal negligence or imprudence is the lack of malice or criminal
gunshot wound. intent in the execution thereof. Moreover, if the version of
The prosecution presented Monserrat de Leon, grappling for the gun were to be believed, there should
sister of the victim,who declared that Grace would confide have been nitrates on both hands of Grace, as examined
to her that accused-appellant was jobless and that Grace by the NBI doctor who conducted the post-mortem
had problems with the low income of the store she owned examination on the cadaver of the victim. Thus, these

18
CRIMINAL LAW REVIEW DIGESTS
JUSTICE ROMEO CALLEJO NOTE: = Callejo Ponente

physical evidence, the lack of powder burns or nitrates on incident was accidental. It is also clear that the accused
the hands of Grace and the trajectory of the bullet that Pugay and his group merely wanted to make fun of the
entered her left thigh being slightly upwards and from left deceased. Hence, the respective criminal responsibility of
to right instead of downwards, repudiate accused- Pugay and Samson arising from different acts directed
appellant's claim of simple negligence. against the deceased is individual and not collective, and
each of them is liable only for the act committed by him
3) Yes, but with mitigating circumstance of voluntary (U.S. vs. Magcomot, et. al. 13, Phil. 386; U.S. vs. Abiog,
surrender. The prosecution has sufficiently established the et. al. 37 Phil. 1371).
elements of parricide by its evidence. These elements are:
(1) the death of the deceased; (2) that she was killed by ISSUE: What is the criminal responsibility of Pugay?
the accused; and (3) that the deceased was a legitimate
ascendant or descendant, or the legitimate spouse of the HELD: Homicide through reckless imprudence. Having
accused (Article 246). The first and third elements were taken the can from under the engine of the ferris wheel
stipulated during the pre-trial stage of the case, thus: the and holding it before pouring its contents on the body of
victim and the accused are legally married, and that the deceased, this accused knew that the can contained
immediately after the shooting, the accused voluntarily gasoline. Clearly, he failed to exercise all the diligence
and bodily carried the victim into a taxicab and proceeded necessary to avoid every undesirable consequence arising
to the hospital where she died. Further, accused- from any act that may be committed by his companions
appellant having admitted that he shot his wife, he has the who at the time were making fun of the deceased.
burden of proof of establishing the presence of any A man must use common sense and exercise due
circumstance which may relieve him of responsibility. reflection in all his acts; it is his duty to be cautious,
careful, and prudent, if not from instinct, then through fear
of incurring punishment. He is responsible for such results
PEOPLE vs. PUGAY & SAMSON (may 2 or more as anyone might foresee and for acts which no one would
persons kill the same victim) have performed except through culpable abandon.
Otherwise his own person, rights and property, all those of
FACTS: The deceased victim Miranda, a 25-year old his fellow-beings, would ever be exposed to all manner of
retardate, and the accused Pugay were friends. During a danger and injury.
town fiesta, Gabion, the witness, was sitting in the ferris
wheel and reading a comic book. He then saw Pugay and ISSUE: What is the criminal responsibility of Samson?
Samson with several companions making fun of Miranda.
Pugay suddenly took a can of gasoline from under the HELD: homicide with ordinary mitigating circumstance of
engine of the ferris wheel and poured its contents on the no intention to commit so grave a wrong as that
body of the Miranda. The victim died due to the incident. committed. His conviction of murder because of the
Gabion, Pugay, Samson and 5 others were brought to the presence of treachery is improper. There is entire absence
municipal building for interrogation. Pugay and Samson of proof in the record that the accused Samson had some
gave statements to the police. Pugay admitted in his reason to kill the deceased before the incident. On the
statement that he poured a can of gasoline on the contrary, there is adequate evidence showing that his act
deceased believing that the contents thereof was water was merely a part of their fun-making that evening. For the
and Samson set the deceased on fire. Samson alleged in circumstance of treachery to exist, the attack must be
his statement that he saw Pugay pour gasoline on deliberate and the culprit employed means, methods, or
Miranda but did not see the person who set him on fire. forms in the execution thereof which tend directly and
Pugay and Samson were found guilty on the crime of specially to insure its execution, without risk to himself
murder but crediting in favor of the accused Pugay the arising from any defense which the offended party might
mitigating circumstance of lack of intention to commit so make.
grave a wrong. There can be no doubt that the accused Samson
knew very well that the liquid poured on the body of the
ISSUE: Whether or not there was a conspiracy? deceased was gasoline and a flammable substance for he
would not have committed the act of setting the latter on
HELD: None, there is nothing in the records showing that fire if it were otherwise. Giving him the benefit of doubt, it
there was previous conspiracy or unity of criminal purpose can be conceded that as part of their fun-making he
and intention between the two accused-appellants merely intended to set the deceased's clothes on fire. His
immediately before the commission of the crime. There act, however, does not relieve him of criminal
was no animosity between the deceased and the accused responsibility. Burning the clothes of the victim would
Pugay or Samson. Their meeting at the scene of the cause at the very least some kind of physical injuries on

19
CRIMINAL LAW REVIEW DIGESTS
JUSTICE ROMEO CALLEJO NOTE: = Callejo Ponente

his person, a felony defined in the Revised Penal Code. If because of the employment of terms without defining
his act resulted into a graver offense, as what took place them.
in the instant case, he must be held responsible therefor. A statute or act may be said to be vague when it
Article 4 of the aforesaid code provides, inter alia, that lacks comprehensible standards that men of common
criminal liability shall be incurred by any person intelligence most necessarily guess at its meaning and
committing a felony (delito) although the wrongful act done differ in its application. In such instance, the statute is
be different from that which he intended. repugnant to the Constitution in two (2) respects it
We are disposed to credit in his favor the violates due process for failure to accord persons,
ordinary mitigating circumstance of no intention to commit especially the parties targeted by it, fair notice of what
so grave a wrong as that committed as there is evidence conduct to avoid; and, it leaves law enforcers unbridled
of a fact from which such conclusion can be drawn. The discretion in carrying out its provisions and becomes an
eyewitness Gabion testified that the accused Pugay and arbitrary flexing of the Government muscle.
Samson were stunned when they noticed the deceased A facial challenge is allowed to be made to vague
burning. statute and to one which is overbroad because of possible
chilling effect upon protected speech. The possible harm
to society in permitting some unprotected speech to go
Estrada v Sandiganbayan (GR No. 148560. November unpunished is outweighed by the possibility that the
19, 2001) protected speech of other may be deterred and perceived
grievances left to fester because of possible inhibitory
Facts: Petitioner Estrada, President of the Phililippines is effects of overly broad statutes. But in criminal law, the law
being prosecuted under RA 7080 (An Act Defining and cannot take chances as in the area of free speech.
Penalizing the Crime of Plunder as amended by RA 7659.
He contends that the, Plunder law is unconstitutional for 2. WON the Plunder Law requires less evidence
being vague; second, The Plunder Law requires less for providing the predicate crimes of plunder and
evidence for proving the predicate crimes of plunder and therefore violates the rights of the accused to due
therefore violates the rights of the accused to due process; process
and third Plunder as defined in RA 7080 is a malum
prohibitum, and if so, whether it is within the power of No. Sec. 4 (Rule of Evidence) states that: For purposes of
Congress to so classify it. establishing the crime of plunder, it shall not be necessary
to prove each and every criminal act done by the accused
Issues: in furtherance of the scheme or conspiracy to amass,
1. WON Plunder Law is unconstitutional for being accumulate or acquire ill-gotten wealth, it being sufficient
vague to establish beyond reasonable doubt a pattern of overt or
criminal acts indicative of the overall unlawful scheme or
No. As long as the law affords some comprehensible conspiracy.
guide or rule that would inform those who are subject to it In a criminal prosecution for plunder, as in all
what conduct would render them liable to its penalties, its other crimes, the accused always has in his favor the
validity will be sustained. The amended information itself presumption of innocence guaranteed by the Bill of Rights,
closely tracks the language of law, indicating w/ and unless the State succeeds in demonstrating by proof
reasonable certainty the various elements of the offense beyond reasonable doubt that culpability lies, the accused
w/c the petitioner is alleged to have committed. is entitled to an acquittal.
We discern nothing in the foregoing that is vague The reasonable doubt standard has acquired
or ambiguous that will confuse petitioner in his defense. such exalted stature in the realm of constitutional law as it
Petitioner however bewails the failure of the law gives life to the Due Process Clause which protects the
to provide for the statutory definition of the terms accused against conviction except upon proof of
combination and series in the key phrase a reasonable doubt of every fact necessary to constitute the
combination or series of overt or criminal acts. These crime with which he is charged.
omissions, according to the petitioner, render the Plunder Not everything alleged in the information needs
Law unconstitutional for being impermissibly vague and to be proved beyond reasonable doubt. What is required
overbroad and deny him the right to be informed of the to be proved beyond reasonable doubt is every element of
nature and cause of the accusation against him, hence the crime chargedthe element of the offense.
violative of his fundamental right to due process. Relative to petitioners contentions on the
A statute is not rendered uncertain and void purported defect of Sec. 4 is his submission that pattern
merely because general terms are used herein, or is a very important element of the crime of plunder; and
that Sec. 4 is two-pronged, (as) it contains a rule of

20
CRIMINAL LAW REVIEW DIGESTS
JUSTICE ROMEO CALLEJO NOTE: = Callejo Ponente

evidence and a substantive element of the crime, such like an animal and utterly dehumanized as to completely
that without it the accused cannot be convicted of plunder. disrupt the normal course of his or her growth as a human
We do not subscribe to petitioners stand. being.
Primarily, all the essential elements of plunder can be There are crimes however in which the
culled and understood from its definition in Sec. 2, in abomination lies in the significance and implications of the
relation to sec. 1 par. (d). Sec. 4 purports to do no more subject criminal acts in the scheme of the larger socio-
than prescribe a rule of procedure for the prosecution of a political and economic context in which the state finds
criminal case for plunder. Being a purely procedural itself to be struggling to develop and provide for its poor
measure, Sec. 4 does not define or establish any and underprivileged masses.
substantive right in favor of the accused but only operated The legislative declaration in R.A. No.7659 that
in furtherance of a remedy. plunder is a heinous offense implies that it is a malum in
What is crucial for the prosecution is to present se. For when the acts punished are inherently immoral or
sufficient evidence to engender that moral certitude inherently wrong, they are mala in se and it does not
exacted by the fundamental law to prove the guilt of the matter that such acts are punished in a special law,
accused beyond reasonable doubt. especially since in the case of plunder the predicate
crimes are mainly mala in se.
3. WON Plunder as defined in RA 7080 is a malum
prohibitum, and if so, whether it is within the power of Held: PREMISES CONSIDERED, this Court holds that RA
Congress to so classify it. 7080 otherwise known as the Plunder Law, as amended
by RA 7659, is CONSTITUTIONAL. Consequently, the
No. It is malum in se which requires proof of criminal petition to declare the law unconstitutional is DISMISSED
intent. Precisely because the constitutive crimes are mala for lack of merit
in se the element of mens rea must be proven in a
prosecution for plunder. It is noteworthy that the amended
information alleges that the crime of plunder was FAJARDO V PEOPLE
committed willfully, unlawfully and criminally. It thus
alleges guilty knowledge on the part of petitioner. Art 3-Mala ProhibitaIllegal possession of part of firearm
In support of his contention In support of his & plain view doctrine
contention that the statute eliminates the requirement of
mens rea and that is the reason he claims the statute is Facts: Fajardo filed a Petition for certiorari on the decision
void, petitioner cites the following remarks of Senator of CA and RTC finding Fajardo guilty of violating PD 1866
Taada made during the deliberation on S.B. No.733 (illegal possession of firearms). The case stemmed from a
Senator Taada was only saying that where the complaint filed by citizens that armed men were drinking
charge is conspiracy to commit plunder, the prosecution liquor and firing guns at Fajardos residence. The police
need not prove each and every criminal act done to further and the PISOG arrived at their residence to find men
the scheme or conspiracy, it being enough if it proves scampering and then saw Valerio w/ 2 45 caliber pistols,
beyond reasonable doubt a pattern of overt or criminal engaging in a shootout w/ the police before running into
acts indicative of the overall unlawful scheme or Fajardos house. Fajardo was also seen tucking a 45
conspiracy. As far as the acts constituting the pattern are caliber pistol in her shorts before running into her home.
concerned, however, the elements of the crime must be The police opted not to enter and just cordoned the area.
proved and the requisite mens rea must be shown. At around 2 and 4 am, Valerio was seen tossing 2
The application of mitigating and extenuating receivers (part ng pistol) which was recovered and
circumstances in the Revised Penal Code to prosecutions surrendered to SPo1 Tan who used them to apply for a
under the Anti-Plunder Law indicates quite clearly that warrant. They found several ammos within the house and
mens rea is an element of plunder since the degree of filed for illegal possession of firearms against both parties
responsibility of the offender is determined by his criminal but Fajardo countered that the search warrant was
intent. defective in that when it was issued it wasnt based on
Finally, any doubt as to whether the crime of Tans personal knowledge and they didnt accompany the
plunder is a malum in se must be deemed to have been police while conducting the search. She also disowned the
resolved in the affirmative by the decision of Congress in ammos because they allegedly belonged to her brother
1993 to include it among the heinous crimes punishable and also denied having the pistol when the police arrived.
by reclusion perpetua to death. RTC: Fajardo and her bodyguard Valerio is guilty
The evil of a crime may take various forms. There of illegal possession of firearm.
are crimes that are, by their very nature, despicable, either CA: Affirmed the findings but said that the search
because life was callously taken or the victim is treated warrant is void because it wasnt based on Tans personal

21
CRIMINAL LAW REVIEW DIGESTS
JUSTICE ROMEO CALLEJO NOTE: = Callejo Ponente

knowledge and so the ammo recovered was declared


inadmissible. But the 2 receivers retrieved before the
warrant were valid under the plain view doctrine. GARCIA V. CA

Issues: FACTS: Based on the complaint of Aquilino Pimentel who


1. Do the 2 receivers come under plain view doctrine? ran in the senatorial elections, he charged elections officer
2. Is Fajardo guilty of illegal possession of part of firearm? Arsenia Garcia for willfully decreasing the votes received
by senatorial candidate Pimentel from 6,988 votes, as
Held: clearly disclosed in the total number of votes in the 159
1. YES, Under the plain view doctrine, objects falling in precincts of the statement of votes by precincts of said
the "plain view" of an officer, who has a right to be in municipality to 1921 votes with a difference of 5,077. The
the position to have that view, are subject to seizure RTC convicted accused and gave a prison sentence. On
and may be presented as evidence. appeal however it was contended that there was no
It applies when the following requisites concur: criminal intent and bad faith in his actions. Respondent on
(a) the law enforcement officer in search of the the other hand contends that a violation of an election law
evidence has a prior justification for an intrusion or is is a mala prohibita and good faith is not a defense.
in a position from which he can view a particular area-
Navas presence was justified because they saw ISSUE: WON a violation of section 27b of R.A. 6646 is a
armed persons mala in se or mala prohibita? And could good faith and
(b) the discovery of the evidence in plain view is lack of criminal intent be a valid defense?
inadvertent - Nava clearly saw the throwing of
suspicious objects HELD: mala in se
(c) it is immediately apparent to the officer that the
item he observes may be evidence of a crime, Generally, mala in se felonies are defined and penalized in
contraband, or otherwise subject to seizure. the Revised Penal Code. When the acts complained of are
Considering that the sighting of Valerio holding a inherently immoral, they are deemed mala in se, even if
pistol he had reasonable ground to believe Valerio they are punished by a special law. Accordingly, criminal
had thrown contraband items intent must be clearly established with the other elements
of the crime; otherwise, no crime is committed. On the
2. NO, illegal possession of firearms is committed when other hand, in crimes that are mala prohibita, the criminal
the holder (1) possesses a firearm or a part thereof acts are not inherently immoral but become punishable
and (2) lacks the authority or license to possess the only because the law says they are forbidden. With these
firearm. crimes, the sole issue is whether the law has been
The rule is that ownership is not an essential violated. Criminal intent is not necessary where the acts
element. What the law requires is merely possession are prohibited for reasons of public policy.
which includes not only actual physical possession An election offense is defined as:
but also constructive possession or the subjection of (b) Any member of the board of election inspectors or
the thing to one's control and management. The board of canvassers who tampers, increases, or
offense of illegal possession of firearms is a malum decreases the votes received by a candidate in any
prohibitum punished by a special law, in which case election or any member of the board who refuses, after
good faith and absence of criminal intent are not valid proper verification and hearing, to credit the correct votes
defenses.It is sufficient that the offender has the intent or deduct such tampered votes.
to perpetrate the act prohibited by the special law.
Fajardo was neither in physical nor Clearly, the acts prohibited in Section 27(b) are
constructive possession of the subject receivers. The mala in se. For otherwise, even errors and mistakes
testimony of Nava shows that he only saw Valerio on committed due to overwork and fatigue would be
top of the house when the receivers were thrown. punishable. Given the volume of votes to be counted and
None of the witnesses saw Fajardo holding the canvassed within a limited amount of time, errors and
receivers, before or during their disposal. At the very miscalculations are bound to happen. And it could not be
least, Fajardo's possession of the receivers was the intent of the law to punish unintentional election
merely incidental because Valerio, the one in actual canvass errors. However, intentionally increasing or
physical possession, was seen at the rooftop of her decreasing the number of votes received by a candidate is
house. inherently immoral, since it is done with malice and intent
to injure another.
*Only Valerio was found guilty.

22
CRIMINAL LAW REVIEW DIGESTS
JUSTICE ROMEO CALLEJO NOTE: = Callejo Ponente

Criminal intent is presumed to exist on the part of Senior Inspector Rico Tome (TOME) received a
the person who executes an act which the law punishes, dispatch informing him that 2 trucks did not stop at
unless the contrary shall appear. Thus, whoever invokes checkpoint. The trucks were then intercepted and the
good faith as a defense has the burden of proving its drivers were interrogated. Tigoy was asked why he didnt
existence. stop at the checkpoint and followed up by what was
During trial of this case, petitioner admitted that loaded in the truck. Tigoy replied that there is S.O.P
she was indeed the one who announced the figure of (grease money in street parlance), causing suspicion and
1,921, which was subsequently entered by then accused leading to the police finding piles of sawn lumber beneath
Viray in his capacity as secretary of the board. Petitioner cement bags. Since the drivers didnt have a permit, they
likewise admitted that she was the one who prepared the were turned over to an Investigator and was detained.
COC, though it was not her duty. To our mind, preparing TIGOY was then charged with qualified theft (possessing
the COC even if it was not her task, manifests an intention lumber without a permit in violation of the Revised
to perpetuate the erroneous entry in the COC. Forestry Code).
Neither can this Court accept petitioners
explanation that the Board of Canvassers had no idea how ISSUE: W/N Tigoy is guilty of conspiracy in possessing or
the SOV (Exhibit "6") and the COC reflected that private transporting lumber without the necessary permit in
complainant had only 1,921 votes instead of 6,921 votes. violation of the Revised Forestry Code of the Philippines. -
As chairman of the Municipal Board of Canvassers, YES
petitioners concern was to assure accurate, correct and
authentic entry of the votes. Her failure to exercise RATIO: In offenses considered as mala prohibita or
maximum efficiency and fidelity to her trust deserves not when the doing of an act is prohibited by a special
only censure but also the concomitant sanctions as a law, the commission of the prohibited act is the crime
matter of criminal responsibility pursuant to the dictates of itself. It is sufficient that the offender has the intent to
the law. perpetrate the act prohibited by the special law, and
Public policy dictates that extraordinary diligence that it is done knowingly and consciously.
should be exercised by the members of the board of Direct proof of previous agreement to commit an
canvassers in canvassing the results of the elections. Any offense is not necessary to prove conspiracy. Conspiracy
error on their part would result in the disenfranchisement may be proven by circumstantial evidence. It may be
of the voters. The Certificate of Canvass for senatorial deduced from the mode, method and manner by which the
candidates and its supporting statements of votes offense is perpetrated, or inferred from the acts of the
prepared by the municipal board of canvassers are accused when such acts point to a joint purpose and
sensitive election documents whose entries must be design, concerted action and community of interest. It is
thoroughly scrutinized. not even required that the participants have an agreement
for an appreciable period to commence it.
TIGOYs defense was that he could not have
TIGOY VS. PEOPLE conspired with Bertodazo for he did not know about the
unlicensed lumber in the trucks as he believed that he was
Doctrine: In offenses considered as mala prohibita or transporting bags of cement and he was not around when
when the doing of an act is prohibited by a special law, the the trucks were with the lumber hidden under the bags of
commission of the prohibited act is the crime itself. It is cement. But his actions (not stopping at checkpoint,
sufficient that the offender has the intent to perpetrate the offering S.O.P) adequately show that he intentionally
act prohibited by the special law, and that it is done participated in the commission of the offense for which he
knowingly and consciously. had been charged.

FACTS: Nestor Ong (ONG), engaged in the trucking Digest of Digest:


business, entered into a Contract to Transport with FACTS: TIGOY, as truck driver, didnt stop at a check
Lolong Bertodazo (BERTODAZO), to transport point, got intercepted by police, offered grease money
construction materials from Lanao del Norte to Dipolog when asked what was loaded on the truck, and was
City. Ong instructed Rodolfo Tigoy (TIGOY) and charged with qualified theft for illegal possession of lumber
SUMAGANG, the formers truck drivers, to bring his trucks in violation to the Revised Forestry Code when he was
to BERTODAZO in Lanao del Norte for loading of found transporting lumber underneath cement bags
materials, leave it there, then go back at dawn to pick it up without a permit. He claimed he couldnt have conspired
for the trip to Dipolog City. The following morning, the as he didnt have knowledge of the lumber loaded.
drivers arrived with the trucks loaded up with cement. After
a few checks of the trucks, they left. ISSUE: same

23
CRIMINAL LAW REVIEW DIGESTS
JUSTICE ROMEO CALLEJO NOTE: = Callejo Ponente

founded on the principle that every person is to be held to


HELD: In offenses considered as mala prohibita or contemplate and to be responsible for the natural
when the doing of an act is prohibited by a special consequences of his own acts. If a person inflicts a
law, the commission of the prohibited act is the crime wound with a deadly weapon in such manner as to put life
itself. It is sufficient that the offender has the intent to in jeopardy and death follows, it does not alter nor
perpetrate the act prohibited by the special law, and diminish its criminality to prove that other causes co-
that it is done knowingly and consciously. The fact that operated in producing the fatal result.
he didnt stop at check point, offered grease money
pointed to the obvious fact that he knew lumber was
loaded in his truck. Conspirator? Yezzir! MELBA QUINTO vs. DANTE ANDRES and
RANDYVER PACHECO

PEOPLE v. MOLDES FACTS:


- At around 7:30 a.m. on November 13, 1995, 11-year
Facts: Inocente Moldes was convicted of homicide by the old Edison Garcia, a Grade 4 elementary school pupil,
CFI of Leyte. On the night of April 3 in a barrio, there was and his playmate, Wilson Quinto saw respondents
a dance in a private house, and the deceased was the Dante Andres and Randyver Pacheco by the mouth of
master of ceremonies. Moldes insisted on dancing out of a drainage culvert.
turn and was reproved by the deceased. Moldes went to - Andres and Pacheco invited Wilson to go fishing with
the porch and began cutting down the decorations with his them inside the drainage culvert. Wilson assented.
bolo. He went into the yard and challenged everyone to a - When Garcia saw that it was dark inside, he opted to
fight. Not attracting enough attention, he began chopping remain seated in a grassy area about two meters from
bamboo trees. the entrance of the drainage system.
The deceased, unarmed, spoke to him in a - Respondent Pacheco had a flashlight. He, along with
friendly manner as he descended into the yard. But respondent Andres and Wilson, entered the drainage
Moldes struck him with his bolo, inflicting a wound on his system which was covered by concrete culvert about
left arm (long incised wound on the lower portion directed a meter high and a meter wide, with water about a
downwards). As the deceased fell, Moldes inflicted a foot deep.
slight wound on his back and ran away. The wound was - After a while, respondent Pacheco, who was holding
treated the next morning. However, it failed to stop the a fish, came out of the drainage system and left
hemorrhage and the deceased died after 12 days. without saying a word. Respondent Andres also
Moldess defense was that he was behaving at came out, went back inside, and emerged again, this
the dance, and it was the deceased who struck him on the time, carrying Wilson who was already dead.
dance floor with a cane. He attempted to run away but the - Respondent Andres laid the boys lifeless body down
deceased followed him with a cane and a bolo. He in the grassy area. Shocked at the sudden turn of
succeeded in wrenching the bolo away and inflicted the events, Garcia fled from the scene. For his part,
wounds in self-defense. respondent Andres went to the house of petitioner
The CFI convicted him. The attorney urged that Melba Quinto, Wilsons mother, and informed her that
Moldes did not intend to commit as serious a wound as her son had died. Melba Quinto rushed to the
was inflicted but struck only in the dark and in self- drainage culvert while respondent Andres followed
defense. It is also contended that had the deceased her.
secured proper surgical treatment, the wound would not - The police authorities of Tarlac, Tarlac, did not file
have been fatal (but such was not available in that barrio). any criminal complaint against the respondents for
Wilsons death.
Issue: W/N he is guilty of homicide YES
- Two weeks thereafter, investigators took the sworn
statements of respondent Pacheco, Garcia and
Ratio: The SC ruled that there was no element of self-
petitioner Quinto.
defense. When one resorts to the use of a lethal weapon
- Respondent Pacheco alleged that he had never been
and strikes another with the force that must have been
to the drainage system catching fish with respondent
used in this case, it is presumed that he realizes the
Andres and Wilson. He also declared that he saw
natural consequences of his act.
Wilson already dead when he passed by the drainage
The GR is that he who inflicts the injury is not
system while riding on his carabao.
relieved of responsibility if the wound inflicted is
dangerous, even though the immediate cause of the death
was erroneous or unskillful medical treatment. This rule is

24
CRIMINAL LAW REVIEW DIGESTS
JUSTICE ROMEO CALLEJO NOTE: = Callejo Ponente

- The cadaver of Wilson was exhumed. Postmortem intervening cause, produces the injury, and without
findings stated that the CAUSE OF DEATH: Asphyxia which the result would not have occurred. The
by drowning; traumatic head injuries, contributory. proximate legal cause is that acting first and
- The NBI filed a criminal complaint for homicide producing the injury, either immediately, or by setting
against respondents Andres and Pacheco in the other events in motion, all constituting a natural and
Office of the Provincial Prosecutor, which found continuous chain of events, each having a close
probable cause for homicide by dolo against the two. causal connection with its immediate predecessor
- The respondents filed a demurer to evidence which - There must be a relation of cause and effect, the
the trial court granted on the ground of insufficiency of cause being the felonious act of the offender, the
evidence, per its Order dated January 28, 1998. It effect being the resultant injuries and/or death of the
also held that it could not hold the respondents liable victim.
for damages because of the absence of preponderant - The felony committed is not the proximate cause of
evidence to prove their liability for Wilsons death. the resulting injury when:
- The petitioner appealed the order to the CA insofar as (a) there is an active force that intervened between
the civil aspect of the case was concerned. In her the felony committed and the resulting injury, and the
brief, she averred that THE TRIAL COURT ERRED active force is a distinct act or fact absolutely foreign
IN DISMISSING THE CASE AND IN RULING THAT from the felonious act of the accused; or
NO PREPONDERANT EVIDENCE EXISTS TO (b) the resulting injury is due to the intentional act
HOLD ACCUSED-APPELLEES CIVILLY LIABLE of the victim.
FOR THE DEATH OF THE VICTIM WILSON
QUINTO In the present case, the respondents were
- The CA rendered judgment affirming the assailed charged with homicide by dolo. The prosecution was
order saying that the acquittal in this case is not burdened to prove the corpus delicti which consists of two
merely based on reasonable doubt but rather on a things: first, the criminal act and second, defendants
finding that the accused-appellees did not commit the agency in the commission of the act.
criminal acts complained of. Thus, pursuant to the In homicide (by dolo) and in murder cases, the
above rule and settled jurisprudence, any civil prosecution is burdened to prove: (a) the death of the
action ex delicto cannot prosper. Acquittal in a party alleged to be dead; (b) that the death was produced
criminal action bars the civil action arising therefrom by the criminal act of some other than the deceased and
where the judgment of acquittal holds that the was not the result of accident, natural cause or suicide;
accused did not commit the criminal acts imputed to and (c) that defendant committed the criminal act or was in
them. Hence, this petition. some way criminally responsible for the act which
produced the death. To prove the felony of homicide or
ISSUE: W/N the respondents are still liable for damages murder, it must be proved that there was intent to kill.
upon extinction of criminal liability. Such evidence may consist inter alia in the use of
weapons by the malefactors, the nature, location and
HELD/RATIO: NO. number of wounds sustained by the victim and the words
- The extinction of the penal action does not carry with uttered by the malefactors before, at the time or
it the extinction of the civil action. However, the civil immediately after the killing of the victim. If the victim dies
action based on delict shall be deemed extinguished if because of a deliberate act of the malefactor, intent to kill
there is a finding in a final judgment in the civil action is conclusively presumed.
that the act or omission from where the civil liability The trial court took into account the following
[23] facts:
may arise does not exist.
- Moreover, a person committing a felony is criminally Again, it could be seen from the pictures
liable for all the natural and logical consequences presented by the prosecution that there were stones inside
resulting therefrom although the wrongful act done be the culvert. The stones could have caused the victim to
different from that which he intended. Natural refers slip and hit his head on the pavement. Since there was
to an occurrence in the ordinary course of human life water on the culvert, the portion soaked with water must
or events, while logical means that there is a rational be very slippery, aside from the fact that the culvert is
connection between the act of the accused and the round. If the victim hit his head and lost consciousness,
resulting injury or damage. The felony committed he will naturally take in some amount of water and drown.
must be the proximate cause of the resulting injury. That the deceased fell or slipped cannot be
- Proximate cause is that cause which in natural and totally foreclosed because even Garcia testified that the
continuous sequence, unbroken by an efficient drainage culvert was dark, and that he himself was so
afraid that he refused to join respondents Andres and

25
CRIMINAL LAW REVIEW DIGESTS
JUSTICE ROMEO CALLEJO NOTE: = Callejo Ponente

Pacheco inside. Respondent Andres had no flashlight; Issue:


only respondent Pacheco had one. 1.) W/N Court erred in convicting Opero for robbery with
The petitioner even failed to adduce homicide? [per Opero: he never intended to kill the
preponderance of evidence that either or both the deceased, his intention being merely to rob her, had he
respondents hit the deceased with a blunt object or the intention to kill her he could have easily done so with
instrument, and, consequently, any blunt object or the knife]
instrument that might have been used by any or both of
the respondents in hitting the deceased. 2.) W/N Art.49, par.1 of the RPC [which provides that in
In sum, the petitioner failed to adduce cases in which a felony committed is different from that
preponderance of evidence to prove a cause of action for which the offender intended to commit and the penalty
damages based on the deliberate acts alleged in the prescribed for the felony committed is higher than the
Information. offense to which the accused intended to commit, the
penalty corresponding to the offense which accused
intended to commit shall be imposed in its maximum
PEOPLE V. DIEGO OPERO period] should apply to Opero?

Facts: Case is an automatic review of the death sentence Held & Ratio:
imposed on Diego Opero for robbery with homicide. Opero 1.) Appellant's theory finds no basis in the law or in
was charged together with Reynaldo Lacsinto and jurisprudence. It was been repeatedly held that when
Milagros Villegas who were both convicted with a lesser direct and intimate connection exists between the robbery
penalty; while another accused, Asteria Avila, was and the killing, regardless of which of the two precedes the
acquitted. other, or whether they are committed at the same time, the
Liew Soon Ping, wife of Dr. Hong, was found crime committed is the special complex crime of robbery
dead in Room 314 of the House International Hotel in with homicide. If the circumstances would indicate no
Ongpin, Manila. Her body was discovered by Salvador intention to kill, as in the instant case were evidently, the
Oliver, a GSIS security guard assigned to the hotel when intention is to prevent the deceased from making an
he brought a little girl (found loitering by Barcing, another outcry, and so a "pandesal" was stuffed into her mouth,
nd
security guard at the 2 flr of the building) to Room 314 the mitigating circumstance of not having intended to
where she was said to be residing. Upon reaching the commit so grave a wrong may be appreciated. The
room, Oliver knocked at the door, and when nobody stuffing of the "pandesal" in the mouth would not have
answered, he pushed the door open but he smelled foul produced asphyxiation had it not slid into the neckline,
odor emanating from the room. Oliver covered his nose "caused by the victim's own movements, " according to Dr.
with a handkerchief and together with Barcing and the little Singian. The movements of the victim that caused the
girl, they entered the room where they saw prostrate on a "pandesal" to slide into the neckline were, however,
bed Liew Soon Ping with her face down and both feet and attributable to what appellant and his co-accused did to
hands tied. A towel was tied around her mouth. The room the victim, for if they did not hogtie her, she could have
was ransacked and personal belongings thrown all easily removed the "pandesal" from her mouth and
around. avoided death by asphyxiation.
Opero admitted that he robbed the victim, he also It may not avail appellant to contend that the
stated that he and his co-accused Lacsinto subdued the death was by mere accident for even if it were so, which is
victim by assaulting her, tying up her hands and feet not even beyond doubt for the sliding of the pandesal into
stabbing her and stuffing her mouth with a piece of the neckline to produce asphyxiation could reasonably
pandesal. have been anticipated, it is a settled doctrine that when
Dr. Singian, then Chief of the Medico Legal death supervenes by reason or on the occasion of the
Division of the Western Police District made the following robbery, it is immaterial that the occurrence of death was
findings (among others) in his necropsy report: superficial by mere accident. What is important and decisive is that
stab wound measuring 0.8 c.m. on the right side of the death results by reason or on the occasion of the
chin caused by a sharp bladed instrument; superficial stab robbery. These Spanish doctrines were cited by this Court
wound on the mid-axilliary line caused by a sharp bladed in People vs. Mangulabnan, et al., 99 Phil. 992.
instrument; stab wound on the left forearm. However he
found the cause of death to be due to asphyxiation by 2.) Article 49, par. 1 of the RPC applies only to cases
suffocation with an impacted bolus (read: pandesal) into when the crime committed befalls a different person from
the oropharynx and compression of the neck with a broad the one intended to be the victim. This was the explicit
clothing around the neck. ruling in the case of People vs. Albuquerque, 59 Phil. 150-
153, citing Supreme Court of decisions Spain.

26
CRIMINAL LAW REVIEW DIGESTS
JUSTICE ROMEO CALLEJO NOTE: = Callejo Ponente

In the instant case, the intended victim, not any survived it had he not twice removed the drainage which
other person, was the one killed, as a result of an intention Dr. Mendoza had placed to control or isolate the infection.
to rob. As stated earlier, what may be appreciated in
appellant's favor is only the mitigating circumstance of not ISSUE: Whether or not the removal of the drainage is the
having intended to commit so grave a wrong as that real cause of death of Aribuabo??? NO.
committed, under paragraph 3 of Article 13 of the Revised
Penal Code, an entirely different situation from that HELD: It was a wound in the abdomen which
contemplated under paragraph 1, Article 49 of the same occasionally results in traumatic peritonitis. The infection
Code, where as already explained, the different felony was caused by the fecal matter from the large intestine
from that intended, befalls someone different from the which has been perforated. The possibility, admitted by
intended victim, i.e. when the person intended to be killed said physician that the patient might have survived said
is a stranger to the offender, but the person actually killed wound had he not removed the drainage, does not mean
is the offender's father, thereby making the intended felony that the act of the patient was the real cause of his death.
which is homicide different from the crime actually Even without said act the fatal consequence could have
committed which is parricide. followed, and the fact that the patient had so acted in a
Notwithstanding the presence of the mitigating paroxysm of pain does not alter the juridical
circumstance of not having intended to commit so grave a consequences of the punishable act of the accused.
wrong as that comitted, there still remains one aggravating One who inflicts an injury on another is deemed
circumstance to consider, after either one of the two by the law to be guilty of homicide if the injury
aggravating circumstances present, that of superior contributes mediately or immediately to the death
strength and dwelling, is offset by the mitigating of such other. The fact that the other causes
circumstance aforesaid. The higher of the imposable contribute to the death does not relieve the actor
penalty for the crime committed, which is reclusion of responsibility. . . . (13 R. C.L., 748.)
perpetua to death, should therefore be the proper penalty Furthermore, it does not appear that the patient,
to be imposed on appellant. This is the penalty of death as in removing the drainage, had acted voluntarily and with
imposed by the lower court. the knowledge that he was performing an act prejudicial to
his health, inasmuch as self-preservation is the strongest
instinct in living beings. It much be assumed, therefore,
PEOPLE VS. QUIANZON that he unconsciously did so due to his pathological
condition and to his state of nervousness and restlessness
FACTS: A novena for the suffrage of the soul of the on account of the horrible physical pain caused by the
deceased person was being held in the house of Victoria wound, aggravated by the contract of the drainage tube
Cacpal in a barrio in Ilocos Norte. Andres Aribuabo, one of with the inflammed peritoneum.
the persons present, went to ask for food of Juan The question herein raised by Quianzon has
Quianzon, then in the kitchen, who had the victuals in his already been finally settled by jurisprudence. The
care. It was the second or third time that Aribuabo Supreme Court of Spain, in a decision of April 3, 1879,
approached Quianzon with the same purpose whereupon said in the case similar to the present, the following:
the latter, greatly peeved, took hold of a firebrand and "Inasmuch as a man is responsible for the consequences
applied to the neck of Aribuabo. The latter ran to the place of his act and in this case the physical condition and
where the people were gathered exclaiming that he is temperament of the offended party nowise lessen the evil,
wounded and was dying. Raising his shirt, he showed to the seriousness whereof is to be judged, not by the
those present a wound in his abdomen below the navel. violence of the means employed, but by the result actually
Aribuabo died as the result of the wound on the tenth day produced; and as the wound which the appellant inflicted
after the incident. upon the deceased was the cause which determined his
There is no conflict between the prosecution and death, without his being able to counteract its effects, it is
the defense as regards the foregoing facts. However, evident that the act in question should be qualified as
there is a question as to who wounded Aribuabo. The homicide, etc."
prosecution claims that it was Quianzon. According to its
witnesses, Quianzon confessed that he had wounded INTOD V CA
Aribuabo with a bamboo spit. On the other hand, it is
contended by the defense that granting that it was Facts: Petitioner, Sulpicio Intod, filed this petition for
Quianzon who inflicted the wound which resulted in review of the decision of the Court of Appeals affirming in
Aribuabos death, he should not be convicted of homicide toto the judgment of the RTC, finding him guilty of the
but only of serious physical injuries because said wound crime of attempted murder.
was not necessarily fatal and the deceased would have

27
CRIMINAL LAW REVIEW DIGESTS
JUSTICE ROMEO CALLEJO NOTE: = Callejo Ponente

Sulpicio Intod and 3 other men went to Salvador aimed at something quite impossible or carried out with
Mandayas house to ask him to go with them to the house means which prove inadequate would constitute a felony
of Bernardina Palangpangan. The group had a meeting against person or family. For this provision to apply, there
with Aniceto Dumalagan who told Mandaya that he must be either (1) legal responsibility, or (2) physical
wanted Palangpangan to be killed because of a land impossibility of accomplishing the intended act in order to
dispute between them and that Mandaya should qualify the act as an impossible crime.
accompany the 4 men otherwise he would also be killed. Legal impossibility occurs where the intended
At 10:00 p.m. of that same day, Intod and companions, all acts even if completed, would not amount to a crime.
armed with firearms arrived at Palangpangans house. Thus, legal impossibility would apply to those
Thereafter, petitioner fired at the said room. It turned out circumstances where: (1) the motive, desire and
the Palangpangan was in another city and her home was expectation is to perform an act in violation of the law; (2)
then occupied by her son-in-law and his family. No one there is no intention to perform the physical act; (3) there
was in the room when the accused fired. No one was hit is a performance of the intended physical act; and (4) the
by the gunfire. consequence resulting from the intended act does not
The RTC convicted Intod of attempted murder. amount to a crime. Factual impossibility occurs when
Intod seeks a modification of the judgment on the ground extraneous circumstances unknown to actor or beyond
that he is only liable for an impossible crime Art. 4(2). control prevent consummation of intended crime. Factual
Intod contends that, Palangpangan's absence from her impossibility of the commission of the crime is not a
room on the night he and his companions riddled it with defense. If the crime could have been committed had the
bullets made the crime inherently impossible. On the other circumstances been as the defendant believed them to be,
hand, Respondent (People of the Phil) argues that the it is no defense that in reality, the crime was impossible of
crime was not impossible instead the facts were sufficient commission. Legal impossibility on the other hand is a
to constitute an attempt and to convict Intod for attempted defense which can be invoked to avoid criminal liability for
murder. Respondent likewise alleged that there was intent. an attempt.
Further, in its Comment to the Petition, respondent pointed The factual situation in the case at bar presents a
out that the crime of murder was not consummated, not physical impossibility which rendered the intended crime
because of the inherent impossibility of its impossible of accomplishment and under Article 4,
accomplishment (Art 4 (2), RPC), but due to a cause of paragraph 2 of the Revised Penal Code, such is sufficient
accident other that petitioners and his co-accuseds own to make the act an impossible crime.
spontaneous desistance (Art. 3). Palangpangan did not To uphold the contention of respondent that the
sleep at her house at that time. Had it not been for this offense was Attempted Murder because the absence of
fact, the crime is possible. Palangpangan was a supervening cause independent of
the actor's will, will render useless the provision in Article
Issue: WON he is liable for attempted murder? NO. 4, which makes a person criminally liable for an act "which
would be an offense against persons or property, were it
Held: Only impossible crime. Article 4, paragraph 2 is an not for the inherent impossibility of its accomplishment . . .
innovation of the Revised Penal Code. This seeks to
remedy the void in the Old Penal Code where:
. . . it was necessary that the execution of the act has GEMMA JACINTO VS. PEOPLE OF THE PHILIPPINES
been commenced, that the person conceiving the
idea should have set about doing the deed, employing Facts: Baby Aquino handed petitioner Jacinto with a
appropriate means in order that his intent might postdated BDO Check in the amount of P10,000.00 as
become a reality, and finally, that the result or end payment for Aquino's purchases from Mega Foam Int'l.,
contemplated shall have been physically possible. So Inc. (petitioner was then the collector of Mega Foam).
long as these conditions were not present, the law Somehow, the check was deposited in the Land Bank
and the courts did not hold him criminally liable. account of Generoso Capitle, the husband of Jacqueline
This legal doctrine left social interests entirely Capitle, the sister of petitioner. Meanwhile, Rowena
unprotected. The RPC, inspired by the Positivist School, Ricablanca, another employee of Mega Foam, received a
recognizes in the offender his formidability, and now phone call from Land Bank looking for Generoso Capitle to
penalizes an act which were it not aimed at something inform him that the BDO check deposited in his account
quite impossible or carried out with means which prove had been dishonored. Ricablanca then phoned accused
inadequate, would constitute a felony against person or Anita Valencia asking her to inform Jacqueline Capitle
against property. The rationale of Article 4(2) is to punish about the bounced check. Valencia told Ricablanca of a
such criminal tendencies. Article 4(2) provides and plan to take the cash and invited Ricablanca to join the
punishes an impossible crimean act which, were it not

28
CRIMINAL LAW REVIEW DIGESTS
JUSTICE ROMEO CALLEJO NOTE: = Callejo Ponente

scheme. Ricablanca, reported the matter to the owner of is deemed complete from the moment the offender gains
Mega Foam, Joseph Dyhengco. possession of the thing, even if he has no opportunity to
Verification from company records showed that dispose of the same.
petitioner never remitted the subject check to Mega Foam. There can be no question that as of the time that
However, Baby Aquino had already paid Mega Foam in petitioner took possession of the check meant for Mega
cash as replacement for the dishonored check. Foam, she had performed all the acts to consummate the
Dyhengco filed a Complaint with the NBI and crime of theft, had it not been impossible of
worked out an entrapment operation with its agents. With accomplishment in this case. Since the crime of theft is not
the help of Ricablanca, petitioner and Valencia were a continuing offense, petitioner's act of receiving the cash
arrested upon receiving the marked money. The NBI filed replacement should not be considered as a continuation of
a criminal case for qualified theft against the two. The RTC the theft. At most, the fact that petitioner was caught
found the accused guilty of qualified theft. The CA receiving the marked money was merely corroborating
modified the judgment by reducing the sentence of evidence to strengthen proof of her intent to gain.
Valencia and Capitle but Jacontos sentence remained. Petitioner Gemma T. Jacinto was found guilty of
impossible crime.
Issue: WON a worthless check can be the object of theft -
NO!
PEOPLE VS RAFAEL BALMORES Y CAYA
Ratio: Under Article 308, in relation to Article 310 of RPC,
the personal property subject of the theft must have some Facts: Balmores is being prosecuted for the crime of
value, as the intention of the accused is to gain from the estafa through falsification of a security. It is alleged that
thing stolen. This is further bolstered by Article 309, where Balmores tore off the bottom (cross wise) of a genuine 1/8
the law provides that the penalty to be imposed on the unit Phil Charity Sweepstakes Ticket. This way, the real
accused is dependent on the value of the thing stolen. number on such ticket was removed and that by
The requisites of an impossible crime are: (1) that substituting and using an ink Balmores allegedly wrote
the act performed would be an offense against persons or 07400 instead which is actually the winning number. (the
property; (2) that the act was done with evil intent; and (3) removal of the bottom portion and writing in ink the
that its accomplishment was inherently impossible, or the number was pleaded guilty to by Balmores)
means employed was either inadequate or ineffectual. Balmores presented the ticket as genuine to the
To be impossible under this clause, the act PCSO so he could claim the money. However, he was not
intended by the offender must be by its nature one able to perform all the acts of execution which would
impossible of accomplishment. There must be either (1) produce the crime of estafa through falsification of a
legal impossibility, or (2) physical impossibility of security because Bayani Miler, the employee to whom the
accomplishing the intended act in order to qualify the act ticket was presented, immediately discovered the
as an impossible crime. Legal impossibility occurs where falsification and caused Balmores apprehension. (in short,
the intended acts, even if completed, would not amount to no exchange of money and ticket took place)
a crime. Factual impossibility occurs when extraneous
circumstances unknown to the actor or beyond his control Issue: (real issue is actually whether there was estafa)
prevent the consummation of the intended crime.
In this case, petitioner performed all the acts to As to the impossible crime part: since the falsification of
consummate the crime of qualified theft, which is a crime the ticket was so obvious, is the consummation of the
against property. Were it not for the fact that the check crime actually impossible?
bounced, she would have received the face value thereof,
which was not rightfully hers. Therefore, it was only due to Digesters explanation: the ticket was obviously falsified
the extraneous circumstance of the check being unfunded, (as in very patent on its face na spurious siya) hence, it is
a fact unknown to petitioner at the time, that prevented the argued that impossible naman maconsummate yung crime
crime from being produced. The thing unlawfully taken by of estafa since nobody will give the money in exchange for
petitioner turned out to be absolutely worthless, because the ticket (kasi nga obvious na fake --- as what happened
the check was eventually dishonored, and Mega Foam in the case).
had received the cash to replace the value of said
dishonored check. So question is, do we have an impossible crime here?

Side issue: When is the crime of theft produced? Theft Held: NO!
is produced when there is deprivation of personal property IMPOSSIBLE CRIME PART: the recklessness
due to its taking by one with intent to gain. Unlawful taking and clumsiness of Balmores in falsifying the ticket did not

29
CRIMINAL LAW REVIEW DIGESTS
JUSTICE ROMEO CALLEJO NOTE: = Callejo Ponente

make the crime impossible under Art.4 Par.2. Examples of Lizada, on 4 different occasions (August First Case,
impossible crimes are as follows: trying to kill another by November 5 Second Case, October 22 Third
putting an arsenic substance in the latters soup but it Case, and September 15 Fourth Case of 1998),
turns out the substance was just common salt; or when with lewd designs, did then and there willfully,
one tries to murder a corpse. In this case, even if the ticket unlawfully and feloniously had carnal knowledge with
was patently falsified on its face, it cannot be said that it the victim against her will and consent.
was impossible to consummate the crime of estafa thru
falsification. EVIDENCE OF THE PROSECUTION:
OTHER MATTERS: It is argued that for the June o Rose Orillosa had 3 children, Analia, Jepsy, and
29, 1947 draw (which is the date when the winning Rossel. Orillosa after being separated to her
number of 07400 was drawn) there couldve been no husband, met Lizada and lived together as
genuine 1/8 unit PCSO ticket because the PCSO issued husband and wife.
only units for each ticket. Also, it was not shown that the o Sometime in 1996, Analia was in her room when
number removed from the ticket is not the same as that Lizada entered, laid on top of her, removed her
written in ink. T-shirt and underwear. Lizada then inserted his
The court will not take judicial notice of the fact finger in her vagina. He removed his finger and
that only units of tickets has been issued because it is inserted his penis in her vagina. Momentarily, she
not of common knowledge. Besides, if it were true that no felt a sticky substance coming out from his penis.
1/8 units were issued, then it only supports the She also felt pain in her sex organ. Satiated,
prosecutions theory that balmores ticket was spurious. accused-appellant dismounted but threatened to
Note that in this case, Balmores pleaded guilty to kill her if she divulged to anyone what he did to
removing the true and unidentified number of the ticket her. Accused-appellant then returned to his room.
and substituting in ink a new figure. There wouldve been The incident lasted less than one hour. Petrified
no need for such removal and substitution if the original by the threats on her life, Analia kept to herself
number was the same as that written in ink. what happened to her.
o Sometime in August 1997, accused-appellant
(note: in this case, SC said Balmores is merely guilty of an entered again the room of Analia, placed himself
attempt to commit estafa. But since what is involved here on top of her and held her legs and arms. He
is falsification of a government obligation --- sweepstake then inserted his finger into her sex organ
ticket --- he still has to suffer the fully brunt of the penalty ("fininger niya ako"). Satiated, accused-appellant
of the law). left the room. During the period from 1996 to
1998, accused-appellant sexually abused private
PARAS DISSENT: There was an impossible crime. The complainant two times a week.
falsification was inherently inadequate and is certainly to o On November 5, 1998, Analia was in the sala of
be detected. In short, Balmores could not have succeeded their house studying her assignments. Lizada
in chasing the ticket. In fact, the matter of falsification was was also in the sala. Rossel tended the video
immediately detected by Miler. shop while his mother was away. Analia went into
Also the fact that only 1/4units of the ticket was her room and lay down in bed. She did not lock
actually issued couldve proved in court (thus taken judicial the door of the room because her brother might
notice of) if only Balmores had counsel to assist him. enter any time. She wanted to sleep but found it
(Balmores is actually an illiterate and during the entire difficult to do so. Lizada went to his room next to
proceeding from the lower court, he waived his right to be the room of Analia. He, however, entered the
assisted by counsel) room of Analia. He was wearing a pair of short
pants and was naked from waist up. Analia did
not mind Lizada entering her room because she
People vs. Lizada knew that her brother, Rossel was around.
However, Lizada sat on the side of her bed,
*This is an automatic review of the decision of RTC finding placed himself on top of her, held her hands and
accused-appellant Freddie Lizada guilty beyond legs and fondled her breasts. She struggled to
reasonable doubt of 4 counts of qualified rape and meting extricate herself. Lizada removed her panty and
on him the death penalty for each count. touched her sex organ. Lizada inserted his finger
into her vagina, extricated it and then inserted his
FACTS: penis into her vagina. Lizada ejaculated. Analia
THE CHARGES: Lizada was charged with 4 counts of felt pain in her sex organ. Momentarily, Rossel
qualified rape under 4 separate Information. That passed by the room of Analia after drinking water

30
CRIMINAL LAW REVIEW DIGESTS
JUSTICE ROMEO CALLEJO NOTE: = Callejo Ponente

from the refrigerator, and peeped through the 2. W/N Lizada is guilty of consummated acts of
door. He saw Lizada on top of Analia. Lizada saw lasciviousness defined in Article 336 of the
Rossel and dismounted. Lizada berated Rossel Revised Penal Code or attempted rape under
and ordered him to go to his room and sleep. Article 335 of the said Code, as amended in
Rossel did. Lizada then left the room. Analia relation to the last paragraph of Article 6 of
likewise left the room, went out of the house and the Revised Penal Code. (I think sir will focus
stayed outside for one hour. Rose arrived home on this issue)
at 6:00 p.m. However, Analia did not divulge to 3. W/N Lizada is guilty beyond reasonable doubt
her mother what Lizada had just done to her. and also W/N Lizada is guilty of qualified rape.
o On November 9, 1998, Rose left the house.
Lizada was in the sala of the house watching HELD:
television. Analia tended the video shop. 1. NO.
However, Lizada told Analia to go to the sala. 2. Attempted Rape.
She refused, as nobody would tend the video 3. YES. NO, 2 counts of simple rape.
shop. This infuriated Lizada who threatened to
slap and kick her. Analia ignored the invectives Summary:
and threats of Lizada and stayed in the video First Criminal Case: guilty beyond reasonable doubt of
shop. When Rose returned, a heated argument simple rape.
ensued between Lizada and Analia. Rose sided Second Criminal Case: guilty beyond reasonable doubt of
with her paramour and hit Analia. This prompted attempted rape.
Analia to shout. "Ayoko na, ayoko na." Shortly Third and Fourth Criminal Case: guilty beyond reasonable
thereafter, Rose and Analia left the house on doubt of 2 counts of simple rape.
board the motorcycle driven by her mother to
retrieve some tapes which had not yet been FIRST CRIMINAL CASE: SIMPLE RAPE
returned. When Rose inquired from her daughter
what she meant by her statement, "ayoko na, Lizada avers that the Information for this Case is defective
ayoko na," she told her mother that accused- because the date of the offense "on or about August 1998"
appellant had been touching the sensitive parts alleged therein is too indefinite, in violation of Rule 110,
of her body and that he had been on top of her. Section 11 of the Revised Rules on Criminal Procedure
Rose was shocked and incensed. which reads:
o Analia and her mother went to the Police Station "Sec. 11. Date of commission of the offense. It
where Analia gave her Affidavit-Complaint. Analia is not necessary to state in the complaint or information
was also examined by the medico-legal officer. the precise date the offense was committed except when it
The following is the conclusion made by the is a material ingredient of the offense. The offense may be
officer: 1). No evident sign of extragenital alleged to have been committed on a date as near as
physical injuries noted on the body of the subject possible to the actual date of its commission.
at the time of examination. 2). Hymen, intact and Lizada further asserts that the prosecution failed
its orifice small (1.5 cms. in diameter) as to to proved that he raped Analia in August 1988. The OSG
preclude complete penetration by an average- argued that that the date "on or about August 1998" is
sized adult Filipino male organ in full erection sufficiently definite. After all, the date of the commission of
without producing any genital injury. the crime of rape is not an essential element of the crime.
Failure to specify the exact dates or time when the rapes
DEFENSES AND EVIDENCE OF THE ACCUSED: occurred does not ipso facto make the information
Lizada denied any allegations and claimed that Rose defective on its face. The reason is obvious. The precise
actually coached her children to testify against him date or time when the victim was raped is not an element
because Rose wanted to manage their business and of the offense. The gravamen of the crime is the fact of
take control of all the properties they acquired during carnal knowledge under any of the circumstances
their coverture. Also, Rose was so exasperated enumerated under Article 335 of the Revised Penal Code.
because he had no job. As long as it is alleged that the offense was committed at
any time as near to the actual date when the offense was
ISSUE: committed an information is sufficient.
1. First Criminal Case: W/N the information was
defective because the date of the offense on or SECOND CRIMINAL CASE: ATTEMPTED RAPE
about August 1988 is too indefinite.

31
CRIMINAL LAW REVIEW DIGESTS
JUSTICE ROMEO CALLEJO NOTE: = Callejo Ponente

Lizada avers that he is not liable for rape. His 4. The non-performance of all acts of execution was due
contention is correct. The collective testimony of Analia to cause or accident other than his spontaneous
and her younger brother Rossel was that on November 5, desistance."
1998, Lizada who was wearing a pair of short pants but The first requisite of an attempted felony consists
naked from waist up, went on top of her, held her hands, of two elements, namely:
removed her panty, mashed her breasts and touched her "(1) That there be external acts;
sex organ. However, Lizada saw Rossel peeping through (2) Such external acts have direct connection with the
the door and dismounted. He later left the room of Analia. crime intended to be committed."
In light of the evidence of the prosecution, there was no
introduction of the penis of Lizada into the aperture or Acts constitutive of an attempt to commit a felony
within the pudendum of the vagina of private complainant. should be distinguished from preparatory acts which
Hence, Lizada is not criminally liable for consummated consist of devising means or measures necessary for
rape. accomplishment of a desired object or end. One
In light of the evidence on record, we believe that perpetrating preparatory acts is not guilty of an attempt to
Lizada is guilty of attempted rape and not of acts of commit a felony. However, if the preparatory acts
lasciviousness. constitute a consummated felony under the law, the
Article 336 of the Revised Penal Code reads: malefactor is guilty of such consummated offense. It must
"Art. 336. Acts of Lasciviousness. Any person who shall be borne in mind, however, that the spontaneous
commit any act of lasciviousness upon other persons of desistance of a malefactor exempts him from criminal
either sex, under any of the circumstances mentioned in liability for the intended crime but it does not exempt him
the preceding article, shall be punished by prision from the crime committed by him before his desistance.
correccional." In light of the facts established by the
For an accused to be convicted of acts of prosecution, we believe that Lizada intended to have
lasciviousness, the prosecution is burdened to prove the carnal knowledge of private complainant. The overt acts of
confluence of the following essential elements: Lizada proven by the prosecution were not mere
preparatory acts. By the series of his overt acts, Lizada
"1. That the offender commits any act of lasciviousness or had commenced the execution of rape which, if not for his
lewdness. spontaneous desistance, will ripen into the crime of rape.
2. That it is done under any of the following Although Lizada desisted from performing all the acts of
circumstances: execution, his desistance was not spontaneous as he was
a. By using force or intimidation; or impelled to do so only because of the sudden and
b. When the offended party is deprived of reason unexpected arrival of Rossel. Hence, Lizada is guilty only
or otherwise unconscious; or of attempted rape.
c. When the offended party is under 12 years of
age." THIRD AND FOURTH CRIMINAL CASES: 2 counts of
"Lewd" is defined as obscene, lustful, indecent, SIMPLE RAPE
lecherous. It signifies that form of immorality which has
relation to moral impurity; or that which is carried on a Lizada avers that the prosecution failed to prove
wanton manner. his guilt beyond reasonable doubt. The physical evidence
The last paragraph of Article 6 of the Revised belies Analias claim of having been deflowered by Lizada
Penal Code reads: on four different occasions. The contention of Lizada,
"There is an attempt when the offender however, does not persuade the Court. The fact that
commences the commission of a felony directly by overt Analia remained a virgin from 1996 up to 1998 does not
acts, and does not perform all the acts of execution which preclude her having been repeatedly sexually abused by
should produce the felony by reason of some cause or Lizada. Analia being of tender age, it is possible that the
accident other than his own spontaneous desistance." penetration of the male organ went only as deep as her
The essential elements of an attempted felony labia. Whether or not the hymen of private complainant
are as follows: was still intact has no substantial bearing on Lizada's
"1. The offender commences the commission of the felony commission of the crime. Even, the slightest penetration of
directly by overt acts; the labia by the male organ or the mere entry of the penis
2. He does not perform all the acts of execution which into the aperture constitutes consummated rape. It is
should produce the felony; sufficient that there be entrance of the male organ within
3. The offender's act be not stopped by his own the labia of the pudendum.
spontaneous desistance; SC agree with Lizada, however, that he is guilty
only of 2 counts of simple rape, instead of qualified rape.

32
CRIMINAL LAW REVIEW DIGESTS
JUSTICE ROMEO CALLEJO NOTE: = Callejo Ponente

The evidence on record shows that Lizada is the common- prove that said beginning of execution, if carried to its
law husband of Rose, the mother of private complainant, complete termination following its natural course,
Analia. As of October 1998, Analia was still 13 years old, without being frustrated by external obstacles nor by
and under Article 335 as amended by Republic Act 7659, the voluntary desistance of the perpetrator, will
the minority of the private complainant, concurring with the logically and necessarily ripen into a concrete offense.
fact that accused-appellant is the common-law husband of - Thus, in case of robbery, in order that the simple act
the victim's mother, is a special qualifying circumstance of entering by means of force or violence another
warranting the imposition of the death penalty. However, person's dwelling may be considered an attempt to
said circumstance was not alleged in the Informations as commit this offense, it must be shown that the
required by Section 8, Rule 110 of the Revised Rules on offender clearly intended to take possession, for the
Criminal Procedure which was given retroactive effect by purpose of gain, of some personal property belonging
this Court because it is favorable to the accused. Hence, to another. In the instant case, there is nothing in the
even if the prosecution proved the special qualifying record from which such purpose of the accused may
circumstance of minority of private complainant and reasonably be inferred.
relationship, the Lizada being the common-law husband of - in offenses not consummated, as the material
her mother, Lizada is guilty only of simple rape. damage is wanting, the nature of the action intended
(accion fin) cannot exactly be ascertained, but the
same must be inferred from the nature of the acts
PEOPLE VS LAMAHANG executed (accion medio).
- Acts susceptible of double interpretation, that is, in
Lamahang was caught by a policeman the act of making favor as well as against the culprit, and which show
an opening with an iron bar on the wall of a store of cheap an innocent as well as a punishable act, must not and
goods. The accused had only succeeded in breaking one cannot furnish grounds by themselves for attempted
board and in unfastening another from the wall, when the nor frustrated crimes.
policeman showed up, who instantly arrested him and
placed him under custody. The lower court found him
guilty of attempted robbery. PEOPLE V. EDUARDO SAMPIOR

Issue: Is he guilty of attempted robbery? No such thing as frustrated rape. Merest touch of the male
organ upon the labia of the pudendum, no matter how
Held: NO. He is guilty of attempted trespass to dwelling slight, rape is consummated.
- The attempt to commit an offense which the Penal
Code punishes is that which has a logical relation to a Evelyn Sampior (18) is the eldest of Eduardos 9 children.
particular, concrete offense; that, which is the She was left in their house with only 2 little kid sisters and
beginning of the execution of the offense by overt acts a baby brother, while Eduardo and his other sons went out
of the perpetrator, leading directly to its realization to harvest palay. Evelyns mom was also out then, selling
and consummation. The attempt to commit an fruits in a trade center.
indeterminate offense, inasmuch as its nature in At 10am that day, Eduardo returned home, told
relation to its objective is ambiguous, is not a juridical the 2 little girls to go out and play. The only ones left in the
fact from the standpoint of the Penal Code. house therefore, are Evelyn, her dad Eduardo and the
- There is no doubt that in the case at bar it was the infant baby who was then sleeping. Eduardo suddenly
intention of the accused to enter Tan Yu's store by pulled Evelyn towards him and began to take off her shirt
means of violence, passing through the opening and underwear. Evelyn resisted, but Eduardo persisted.
which he had started to make on the wall, in order to He was able to force her to lie down on the floor, then he
commit an offense which, due to the timely arrival of removed his clothes and mounted her. He held his penis
the police, did not develop beyond the first steps of its and inserted it into Evelyns vagina. Afterwards, he
execution. ordered her to get dressed and to tell no one about it,
- But it is not sufficient, for the purpose of imposing otherwise, he would kill the entire family. Then he left the
penal sanction, that an act objectively performed house.
constitute a mere beginning of execution; it is At 3pm, Eduardo returned home smelling of
necessary to establish its unavoidable connection, liquor. He sexually abused Evelyn again.
like the logical and natural relation of the cause and Days later, Evelyn told her mom about the
its effect, with the deed which, upon its incident and reported the matter to the police. 2 separate
consummation, will develop into one of the offenses complaints for rape were filed against Eduardo (one for
defined and punished by the Code; it is necessary to

33
CRIMINAL LAW REVIEW DIGESTS
JUSTICE ROMEO CALLEJO NOTE: = Callejo Ponente

the 10am session, the other for 3pm). Trial court found room kneeling before Crysthel whose pajamas and panty
Eduardo guilty. were already removed, while his short pants were down to
In this appeal, Eduardo does not seek an his knees. According to Corazon, Campuhan was forcing
acquittal, but only a reduction of the penalty of reclusion his penis into Crysthels vagina.
perpetua. He argues that no rape was consummated. Corazon called for help and Campuhan was
Evelyn was examined in a general hospital by Dr. Toledo subdued. The barangay officials were called. Physical
who found that her hymen was still intact. He also alleges examination of the victim yielded negative results. No
that when Evelyn was on the witness stand, she testified evident sign of extra-genital physical injury was noted by
that there was no full penile penetration. He wants to be the medico-legal officer on Crysthels body as her hymen
declared guilty only of frustrated rape. was intact.
Campuhan said he was innocent but the trial
ISSUE: w/n Eduardo Sampior is guilty of consummated court found him guilty of statutory rape and sentenced him
rape YES! to the extreme penalty of death.

RULING: Eduardos claim that according to Evelyns Campuhans defense was that it was almost
testimony there was no full penile penetration is inconceivable that Corazon could give such a vivid
contradicted by the records. On the witness stand, Evelyn description of the alleged sexual contact when from where
categorically and convincingly testified that there was she stood she could not have possibly seen the alleged
complete phallic penetration. A candid narration by a rape touching of the sexual organs of Campuhan on Crysthel.
victim deserves credence particularly where no ill motive is He asserts that the absence of any external signs of
attributed to her that would make her falsely testify against physical injuries or of penetration of Crysthels private
the accused. No woman in her right mind would admit to parts more than bolsters his innocence.
having been raped and subject herself and her family to
the shame concomitant with a rape prosecution unless the Issue: There is no doubt that Campuhan is guilty of rape.
charges are true. A daughter would not accuse her father However, is it attempted rape or consummated rape?
of incestuous rape unless it were true.
On the matter of the intact hymen, a.k.a the Held: It was attempted rape.
VIRGO INTACTA THEORY: a broken hymen or laceration What consummates rape anyway? Statutory rape
of any part of the female genitalia is not a prerequisite for is consummated by carnal knowledge of a woman below
rape conviction. Medical examination of the victim is only 12 years. Jurisprudence has held that in concluding that
corroborative evidence and is not required. carnal knowledge took place, full penetration of the vaginal
Eduardo points to the 1927 case of People v. orifice is not an essential ingredient, nor is the rupture of
Erinia where the court found the accused guilty only of the hymen necessary; the mere touching of the external
frustrated rape because there was no conclusive evidence genitalia by the penis capable of consummating the sexual
of penile penetration. This ruling, however, has been act is sufficient to constitute carnal knowledge. But the act
overturned. The crime of frustrated rape is non-existent of touching should be understood here as inherently part
in our criminal law. The merest touch of the male of the entry of the penis into the labias of the female organ
organ upon the labia of the pudendum, no matter how and not mere touching alone of the mons pubis or the
slight, consummates the rape. pudendum.
Accused guilty of 2 counts of rape. Reclusion It was held that when an accused failed to
perpetua plus 50k civil indemnity, 50k moral damages, achieve an erection, had a limp or flaccid penis, or an
exemplary damages for 25k all for both counts of rape. oversized penis which could not fit into the victim's vagina,
nonetheless rape was consummated if the accused
repeatedly tried, but in vain, to insert his penis into the
PEOPLE V. CAMPUHAN victims vagina and reaches the labia of her pudendum or
that the penis of the accused touched the middle part of
Facts: Ma. Corazon P. Pamintuan, mother of four (4)-year her vagina.
old Crysthel, went down from the second floor of their Thus, touching when applied to rape cases does
house to prepare Milo for her children. At the ground floor not simply mean mere epidermal contact, stroking or
she met Primo Campuhan, a helper of her brother, who grazing of organs, a slight brush or a scrape of the penis
was then busy filling small plastic bags with water to be on the external layer of the victims vagina, or the mons
frozen into ice in the freezer located at the second floor. pubis. There must be sufficient and convincing proof that
As she was busy preparing the drinks, she heard one of the penis indeed touched the labias or slid into the female
her daughters cry, "Ayoko, ayoko!" prompting her to rush organ, and not merely stroked the external surface
upstairs. There, she saw Campuhan inside her childrens thereof, for an accused to be convicted of consummated

34
CRIMINAL LAW REVIEW DIGESTS
JUSTICE ROMEO CALLEJO NOTE: = Callejo Ponente

rape because the labias, which are required to be slightest penetration of the female organ, i.e., touching of
"touched" by the penis, are by their natural situs or either labia of the pudendum by the penis, there can be no
location beneath the mons pubis or the vaginal surface, consummated rape; at most, it can only be attempted
and to touch them with the penis is to attain some degree rape, if not acts of lasciviousness.
of penetration beneath the surface. Thus, the conclusion
that touching the labia majora or the labia minora of the
pudendum constitutes consummated rape. ARISTOTEL VALENZUELA vs. People and CA June 21,
In this case, rape was not consummated because 2007
it merely constitutes a "shelling of the castle of orgasmic
potency," or as earlier stated, a "strafing of the citadel of Valenzuela and Calderon were charged in an
passion." There was no bombardment of the drawbridge" information with the crime of theft. The two were sighted
according to Justice Bellosillo. Why? The prosecution outside the Super Sale Club (a supermarket within SM
utterly failed to discharge its onus of proving that North EDSA) by security guard Lago. Valenzuela, wearing
Campuhans penis was able to penetrate Crysthels a Receiving Dispatching Unit ID, was seen hauling a push
vagina however slight. Furthermore Corazons testimony cart with cases of Tide (twice) and unloading these cases
could not prove that she saw inter-genital contact. in an open parking space, where Calderon was waiting.
Campuhans kneeling position rendered an unbridled Valenzuela then called a cab and the two loaded the
observation impossible. Not even a vantage point from the cartoons of Tide and boarded the vehicle. Lago tried to
side of the accused and the victim would have provided stop them by asking for a receipt, but the two reacted by
Corazon an unobstructed view of Primos penis fleeing on foot, prompting Lago to fire a warning shot.
supposedly reaching Crysthels external genitalia. Valenzuela and Calderon were apprehended and 4 cases
Lastly, Crysthel said that Campuhans penis did of Tide Ultramatic, 1 case of Ultra 25 grams and 3 cases
not penetrate her organ. Under Art. 6 of detergent were recovered.
(attempted/frustrated), in relation to Art. 335 (rape), of the Valenzuela and Calderon pleaded not guilty on
RPC, rape is attempted when the offender commences arraignment and claimed to be innocent bystanders.
the commission of rape directly by overt acts, and does According to Calderon, he went to the said supermarket
not perform all the acts of execution which should produce with his neighbor Rosulada to withdraw from his ATM. Due
the crime of rape by reason of some cause or accident to the long queue, they decided to buy snacks and went
other than his own spontaneous desistance. All the outside after hearing a gunshot. Suddenly, they were
elements of attempted rape - and only of attempted rape - grabbed by a security guard. As for Valenzuela, he and his
are present in the instant case, hence, the accused should cousin Gregorio were walking in the parking lot to ride a
be punished only for it. tricycle when they saw Lago fire a shot. People started
running and he was apprehended by Lago. During
SCIENCE BONUS (Justice Callejo says we should Valenzuelas cross-examination, he admitted that he had
know this): been employed as a bundler of GMS Marketing,
The pudendum or vulva is the collective term for assigned at the supermarket though not at SM.
the female genital organs that are visible in the perineal RTC: Valenzuela and Calderon guilty of consummated
area, e.g., mons pubis, labia majora, labia minora, the theft. The RTC found credible the testimonies of the
hymen, the clitoris, the vaginal orifice, etc. prosecution witnesses and established the convictions on
The mons pubis is the rounded eminence that the positive identification of the accused as perpetrators of
becomes hairy after puberty, and is instantly visible within the crime.
the surface. Only Valenzuela filed a brief with CA, causing the
The next layer is the labia majora or the outer lips dismissal of Calderons appeal. Valenzuela argued in CA
of the female organ composed of the outer convex surface that he should only be convicted of frustrated theft since at
and the inner surface. The skin of the outer convex the time he was apprehended, he was never placed in a
surface is covered with hair follicles and is pigmented, position to freely dispose of the articles stolen.
while the inner surface is a thin skin which does not have CA: affirmed RTC (consummated theft). Valenzuela filed
any hair but has many sebaceous glands. Directly beneath Petition for Review.
the labia majora is the labia minora.
As stated above, the labia majora must be ISSUE: WON the theft should be deemed as
entered for rape to be consummated, and not merely for consummated or merely frustrated?
the penis to stroke the surface of the female organ. Thus,
a grazing of the surface of the female organ or touching RULING: Consummated. Theft is already produced
the mons pubis of the pudendum is not sufficient to upon the taking of personal property of another
constitute consummated rape. Absent any showing of the

35
CRIMINAL LAW REVIEW DIGESTS
JUSTICE ROMEO CALLEJO NOTE: = Callejo Ponente

without the latters consent. There is no frustrated consider: that the felony is not produced, and that such
theft. failure is due to causes independent of the will of the
Each felony under the Revised Penal Code has a perpetrator. The second factor ultimately depends on the
subjective phase, or that portion of the acts constituting evidence at hand in each particular case. The first,
the crime included between the act which begins the however, relies primarily on a doctrinal definition attaching
commission of the crime and the last act performed by the to the individual felonies in the Revised Penal Code as to
offender which, with prior acts, should result in the when a particular felony is not produced, despite the
consummated crime. After that point has been breached, commission of all the acts of execution.
the subjective phase ends and the objective phase begins. So, in order to ascertain whether the theft is
It has been held that if the offender never passes the consummated or frustrated, it is necessary to inquire as
subjective phase of the offense, the crime is merely to how exactly is the felony of theft produced.
attempted. On the other hand, the subjective phase is Parsing through the statutory definition of theft under
completely passed in case of frustrated crimes, for in such Article 308, there is one apparent answer provided in the
instances, subjectively the crime is complete. language of the law that theft is already produced
Truly, an easy distinction lies between upon the tak[ing of] personal property of another
consummated and frustrated felonies on one hand, and without the latters consent.
attempted felonies on the other. So long as the offender The ability of the offender to freely dispose of
fails to complete all the acts of execution despite the property stolen is not a constitutive element of the
commencing the commission of a felony, the crime is crime of theft. It finds no support or extension in Article
undoubtedly in the attempted stage. Whether a crime is 308, whether as a descriptive or operative element of theft
frustrated or consummated necessitates an initial or as the mens rea or actus reus of the felony. To restate
concession that all of the acts of execution have been what this Court has repeatedly held: the elements of the
performed by the offender. The critical distinction crime of theft as provided for in Article 308 of the Revised
instead is whether the felony itself was actually produced Penal Code are: (1) that there be taking of personal
by the acts of execution. The determination of whether the property; (2) that said property belongs to another; (3) that
felony was produced after all the acts of execution had the taking be done with intent to gain; (4) that the taking
been performed hinges on the particular statutory be done without the consent of the owner; and (5) that the
definition of the felony. It is the statutory definition that taking be accomplished without the use of violence
generally furnishes the elements of each crime under the against or intimidation of persons or force upon things.
Revised Penal Code, while the elements in turn unravel For the purpose of ascertaining whether theft is
the particular requisite acts of execution and susceptible of commission in the frustrated stage, the
accompanying criminal intent. question is again, when is the crime of theft produced?
there is only one operative act of execution by There would be all but certain unanimity in the
the actor involved in theft the taking of personal position that theft is produced when there is
property of another. It is also clear from the provision deprivation of personal property due to its taking by
that in order that such taking may be qualified as theft, one with intent to gain. Viewed from that perspective, it
there must further be present the descriptive is immaterial to the product of the felony that the offender,
circumstances that the taking was with intent to gain; once having committed all the acts of execution for theft, is
without force upon things or violence against or able or unable to freely dispose of the property stolen
intimidation of persons; and it was without the since the deprivation from the owner alone has already
consent of the owner of the property. ensued from such acts of execution.
As applied to the present case, the moment Valenzuela We are satisfied beyond reasonable doubt that
obtained physical possession of the cases of detergent the taking by the petitioner was completed in this case.
and loaded them in the pushcart, such seizure motivated With intent to gain, he acquired physical possession of the
by intent to gain, completed without need to inflict violence stolen cases of detergent for a considerable period of time
or intimidation against persons nor force upon things, and that he was able to drop these off at a spot in the parking
accomplished without the consent of the SM Super Sales lot, and long enough to load these onto a taxicab.
Club, Valenzuela forfeited the extenuating benefit a Indeed, we have, after all, held that unlawful
conviction for only attempted theft would have afforded taking, or apoderamiento, is deemed complete from the
him. moment the offender gains possession of the thing, even if
the theft would have been frustrated only, once he has no opportunity to dispose of the same.
the acts committed by petitioner, if ordinarily sufficient to Insofar as we consider the present question,
produce theft as a consequence, do not produce [such unlawful taking is most material in this respect.
theft] by reason of causes independent of the will of the Unlawful taking, which is the deprivation of ones
perpetrator. There are clearly two determinative factors to personal property, is the element which produces the

36
CRIMINAL LAW REVIEW DIGESTS
JUSTICE ROMEO CALLEJO NOTE: = Callejo Ponente

felony in its consummated stage. At the same time, The authorities continued to appeal to the
without unlawful taking as an act of execution, the offense accused to surrender peacefully but they refused. Finally,
could only be attempted theft, if at all. the police and military authorities decided to launch an
With these considerations, we can only conclude offensive and assault the place. This resulted in injuries to
that under Article 308 of the Revised Penal Code, theft the girls, Mimie and Mary Choco as well as to 2 of the
cannot have a frustrated stage. Theft can only be accused Ronaldo and Reynaldo Canasares.
attempted or consummated. The presumed inability of the
offenders to freely dispose of the stolen property does not ISSUE:
negate the fact that the owners have already been 1) WON the crime committed was merely attempted
deprived of their right to possession upon the completion 2) WON the mitigating circumstance of voluntary
of the taking. There is no language in Article 308 that surrender should be appreciated
expressly or impliedly allows that the free disposition of
the items stolen is in any way determinative of whether HELD/ RATIO:
the crime of theft has been produced. 1) No!
The defense contends that the crime of
robbery/theft has three consecutive stages: 1) the giving
PEOPLE V. SALVILLA 2) the taking and 3) the carrying away or asportation. And
without asportation the crime committed is only attempted.
FACTS: 4 were charged with the crime of robbery with There is no question that in robbery, it is required
serious physical injuries and serious illegal detention but that there be a taking of personal property belonging to
only Salvilla appealed. another. This is known as the element of asportation the
A robbery was staged by the 4 accused at the essence of which is the taking of a thing out of the
New Iloilo Lumber Yard. They were armed with possession of the owner without his privity and consent
homemade guns and hand grenade. They entered the and without the animus revertendi. In fact, if there is no
establishment and told Rodita (employee) that it was a actual taking, there can be no robbery. Unlawful taking of
hold-up. Salvilla pointed his gun at Severino Choco personal property of another is an essential part of the
(owner), Mary and Minnie (2 daughters with one minor) crime of robbery.
and told Severino that all they needed was money. Salvillo insists that while the "giving" has been
Severino told Mary to get a paper bag wherein he placed proven, the "taking" has not. And this is because neither
P20,000.00 cash (P5,000.00, according to the defense) he nor his three co-accused touched the P5,000.00 given
and handed it to Salvilla. Thereafter, Severino pleaded by Severino nor the latter's wallet or watch during the
with the 4 accused to leave the premises as they already entire incident; proof of which is that none of those items
had the money but they paid no heed. Instead, one were recovered from their persons.
accused took the wallet and wristwatch of Severino after However, Rodita testified that Severino put
which the latter, his two daughters, and Rodita, were P20,000.00 inside a paper bag and subsequently handed
herded to the office and kept there as hostages. it to Salvillo. Also, the other accused took the wallet and
Thereafter, Salvilla told Severino to produce wristwatch of Severino. In respect of the P50,000.00 from
P100,000.00 so he and the other hostages could be the Mayor Rodita declared that the Mayor handed the
released. Severino answered that he could not do so amount to her and that she thereafter gave the amount to
because it was a Saturday and the banks were closed. one of the holduppers. The "taking" was, therefore,
In the meantime, police and military authorities sufficiently proved.
had surrounded the premises of the lumber yard. They It is no defense either that Salvillo and his co-
negotiated with the accused using a loud speaker and accused had no opportunity to dispose of the personalities
appealed to them to surrender with the assurance that no taken. That fact does not affect the nature of the crime,
harm would befall them as he would accompany them From the moment the offender gained possession of the
personally to the police station. The accused refused to thing, even if the culprit had no opportunity to dispose of
surrender or to release the hostages. the same, the unlawful taking is complete.
The OIC Mayor arrived and joined the It has been held that the crime is consummated
negotiations. Salvilla demanded P100,000.00, a coaster, when the robber acquires possession of the property,
and some raincoats. She offered them P50,000.00 even if for a short time, and it is not necessary that the
instead, explaining the difficulty of raising more as it was a property be taken into the hands of the robber, or that he
Saturday. The accused agreed to receive the same and to should have actually carried the property away, out of the
release Rodita. The P50,000 was given and Rodita physical presence of the lawful possessor, or that he
released. should have made his escape with it.

37
CRIMINAL LAW REVIEW DIGESTS
JUSTICE ROMEO CALLEJO NOTE: = Callejo Ponente

Therefore, the conviction for consummated HELD/RATIO: An essential element of murder and
robbery is affirmed. homicide, whether in their consummated, frustrated or
attempted stage, is intent of the offenders to kill the
2) No! victim immediately before or simultaneously with the
To be mitigating, a surrender must have the infliction of injuries. Intent to kill is a specific intent
following requisites: (a) that the offender had not been which the prosecution must prove by direct or
actually arrested; (b) that the offender surrendered himself circumstantial evidence, while general criminal intent is
to a person in authority or to his agent; and (c) that the presumed from the commission of a felony by dolo.
surrender was voluntary. evidence to prove intent to kill in crimes against persons
In this case, the "surrender" by the accused may consist, inter alia, in the means used by the
hardly meets these requirements. They were asked to malefactors, the nature, location and number of wounds
surrender by the police and military authorities but they sustained by the victim, the conduct of the malefactors
refused until only much later when they could no longer do before, at the time, or immediately after the killing of the
otherwise by force of circumstances when they knew they victim, the circumstances under which the crime was
were completely surrounded and there was no chance of committed and the motives of the accused. If the victim
escape. The surrender of the accused was held not to be dies as a result of a deliberate act of the malefactors,
mitigating as when he gave up only after he was intent to kill is presumed.
surrounded by the constabulary and police forces. Their In the present case, the prosecution mustered the
surrender was not spontaneous as it was motivated more requisite quantum of evidence to prove the intent of
by an intent to insure their safety. And while it is claimed petitioners to kill Ruben. Esmeraldo and Ismael
that they intended to surrender, the fact is that they did not pummeled the victim with fist blows. Even as Ruben fell to
despite several opportunities to do so. There is no the ground, unable to defend himself against the sudden
voluntary surrender to speak of. and sustained assault of petitioners, Edgardo hit him three
times with a hollow block. Edgardo tried to hit Ruben on
the head, missed, but still managed to hit the victim only in
RIVERA vs. PEOPLE OF THE PHILIPPINES the parietal area, resulting in a lacerated wound and
cerebral contusions. In addition, even if Edgardo did not
FACTS: While Ruben went to a store to buy food, hit the victim squarely on the head, petitioners are still
Edgardo Rivera, one of the accused, mocked the former criminally liable for attempted murder.
for being jobless. A heated exchange of words between Article 6 of the RPC provides for the essential
the two soon followed. The following day, Ruben, together elements of an attempted felony, namely:
with his 3-yr old daughter, went to the store, again, to buy 1. The offender commences the commission of the
food. Then, accused Esmeraldo, Ismael and Edgardo felony directly by overt acts;
Rivera emerged from their house and ganged up on 2. He does not perform all the acts of execution
Ruben. They mauled Ruben with fist blows and he fell to which should produce the felony;
the ground. While in that helpless position, Edgardo hit 1. The offenders act be not stopped by his own
Ruben three times with a hollow block on the parietal area, spontaneous desistance;
while the other two continued mauling him. Although 2. The non-performance of all acts of execution was
feeling dizzy, Ruben managed to stand up. Ismael threw a due to cause or accident other than his
stone at him, hitting him at the back. When the policemen spontaneous desistance
arrived, the accused fled to their house. The Rivera The first element of an attempted felony consists of
brothers were then convicted by the trial court of frustrated two elements:
murder. On appeal, the CA modified the RTC decision and 1. Presence of external acts; and
convicted the accused of attempted murder. 2. Such external acts have direct connection with
The accused insist that the prosecution failed to the crime intended to be committed.
prove that they had the intention to kill Ruben when they In the case at bar, petitioners, who acted in concert,
mauled him with a hollow block. Furthermore, even if they commenced the felony of murder by mauling the victim
had intent to kill Ruben, the prosecution failed to prove and hitting him three times with a hollow block; they
treachery, thus, they should be guilty only of attempted narrowly missed hitting the middle portion of his head. If
homicide. Edgardo had done so, Ruben would surely have died.

ISSUE: W/N the accused should be found guilty only


of attempted homicide instead of attempted murder. LEONIDAS EPIFANIO Y LAZARO VS PEOPLE
NO.

38
CRIMINAL LAW REVIEW DIGESTS
JUSTICE ROMEO CALLEJO NOTE: = Callejo Ponente

Facts: Crisaldo and his cousin, Allan were walking to their all the acts to complete the commission of the crime) but
respective homes. Since the pavement going to Crisaldos nevertheless, the crime is not consummated because of
house was narrow, Allan walked ahead of Crisaldo. the intervention of causes independent of the will of the
Suddenly, Crisaldo felt the stab of a bladed offender.
weapon on his back. His attacker was their uncle, Lazaro In homicide cases, all the acts of execution would
(also known as uncle Iyo). Lazaro stabbed Crisaldo again have been performed if the wound inflicted is mortal and
but only hit the latters left arm. When Allan heard could cause the victims death barring medical
Crisaldos cry of pain, he rushed to help and upon seeing intervention. (if no intent to kill: consummated physical
their uncle, asked him why he did such a thing. Lazaro ran injuries --- if with intent to kill: homicide)
away. Intent to kill may be proved by: 1) motive; 2)
Crisaldo was first brought to Allans house where nature or number of weapons used; 3) nature and number
the wound was wrapped in a blanket. Then he was of wounds inflicted; 4) manner by which the crime was
transferred to the Penaplata hospital where first aid committed and 5) words uttered by the offender at the time
treatment was administered. Thereafter, he was the injuries were inflicted.
transferred to the Davao Medical Center where he stayed In this case, intent to kill is very evident because
for 3weeks to recuperate. of the manner of execution and of the number of wounds
Lazaro was then charged with Frustrated Murder. that was inflicted. However, Lazaro failed to perform all the
He pleaded not guilty to the charge and instead set up an acts of execution because Allan came and he was forced
alibi (he was sleeping at home when the incident took to run away. Lazaro did not voluntarily desist from
place and that when he found out what happened he stabbing Crisaldo, he had to stop because Allan
rushed to help). recognized him. Hence, the subjective phase has not been
RTC: Guilty completed.
CA: Affirmed Moreover, no evidence was presented to prove
that Crisaldo wouldve died from the wounds were it not for
Issue: Whether or not the crime of frustrated murder was the timely medical attendance. Without such proof, the
proved beyond reasonable doubt? character of the wound is doubtful.

Held: NO!
Lazaro does not question the conviction but JINGGOY ESTRADA V. SANDIGANBAYAN (Feb 26,
instead seeks that his offense be lowered to attempted 2002)
murder. It is alleged that there is no evidence to prove that
Crisaldos injuries were life-threatening that it wouldve Facts: As an offshoot of the impeachment proceedings
caused his death were it not for the timely medical against Joseph Estrada, five criminal complaints against
intervention. He noted that the physicians findings said the former President and members of his family, his
the wounds would heal within 15-30days barring associates, friends and conspirators were filed with the
complications. There was no notation that the injury was Ombudsman. One of the charges was for plunder and
life threatening. among the respondents was petitioner Jinggoy Estrada,
It must be stressed that it is not the gravity of the then mayor of San Juan, Metro Manila.
wounds alone which determines whether a crime is Estrada filed several motions (motion to quash
attempted or frustrated. The question is whether the and suspend, very urgent omnibus motion) which were all
assailant has passed the subjective phase of the denied.
commission of the offense. Estrada claims that respondent Sandiganbayan
In the leading case of US vs Eduave, an acted without or in excess of jurisdiction or with grave
attempted crime is when the offender is thwarted by a abuse of discretion amounting to lack of jurisdiction in:
foreign force such that he is unable to perform all the acts 1) not declaring that R.A. No. 7080 as applied to him was
which should produce the crime as a consequence. The in denial of his right to the equal protection of the laws;
subjective phase in the commission of a crime is that 2) not holding that the Plunder Law does not provide
portion of the acts from the time the commission of the complete and sufficient standards;
crime is instituted up until the time the last act is 3) sustaining the charge against petitioner for alleged
performed. After that time, the phase is objective. offenses, and with alleged conspirators, with which and
Note that in attempted crimes, the offender never with whom he is not even remotely connected - contrary to
passes the subjective phase. He is unable to performs all the dictum that criminal liability is personal, not vicarious -
the acts of execution which would produce the crime. On results in the denial of substantive due process;
the other hand, a crime is frustrated when the offender has
passed the subjective phase (meaning, he has performed ISSUE/RATIO:

39
CRIMINAL LAW REVIEW DIGESTS
JUSTICE ROMEO CALLEJO NOTE: = Callejo Ponente

1) WON the Plunder law as applied to Estrada was in enable the latter to amass, accumulate or acquire ill-gotten
denial of his right to equal protection-- NO wealth in the aggregate amount of P4,097,804,173.17. As
the Amended Information is worded, however, it is not
A careful examination of the Amended certain whether the accused conspired with each
Information will show that it is divided into three (3) parts: other to enable the former President to amass the subject
(1) the first paragraph charges Erap with the crime of ill-gotten wealth.
plunder together with petitioner Jinggoy, Atong Ang, and In light of this lack of clarity, petitioner cannot be
others; (2) the second paragraph spells out in general penalized for the conspiracy entered into by the other
terms how the accused conspired in committing the crime accused with the former President as related in the
of plunder; and (3) the following four sub-paragraphs second paragraph of the Amended Information but only for
describe in detail the predicate acts constitutive of the the predicate acts he allegedly committed as related in
crime of plunder pursuant to items (1) to (6) of R.A. No. sub-paragraph (a) of the Amended Information which were
7080, and state the names of the accused who committed allegedly done in conspiracy with the former President
each act. whose design was to amass ill-gotten wealth amounting to
Contrary to petitioners posture, the allegation is more than P4 billion.
that he received or collected money from illegal gambling There is no denying the fact that the plunder of
on several instances. The phrase on several instances an entire nation resulting in material damage to the
means the petitioner committed the predicate act in series. national economy is made up of a complex and manifold
It matters little that sub-paragraph (a) did not utilize the network of crimes. In the crime of plunder, therefore,
exact words combination or series as they appear in different parties may be united by a common purpose. In
R.A. No. 7080. These two terms are to be taken in their the case at bar, the different accused and their different
popular, not technical, meaning, the word series is criminal acts have a commonality to help the former
synonymous with the clause on several instances. President amass, accumulate or acquire ill-gotten wealth.
Sub-paragraphs (a) to (d) in the Amended Information
2) WON plunder law provide sufficient and complete alleged the different participation of each accused in the
standards to guide the courts in dealing with accused conspiracy. The gravamen of the conspiracy charge,
alleged to have contributed to the offense?YES therefore, is not that each accused agreed to receive
protection money from illegal gambling, that each
Section 2. Any public officer who, by himself or in misappropriated a portion of the tobacco excise tax, that
connivance with the members of his family, relatives by each accused ordered the GSIS and SSS to purchase
affinity or consanguinity, business associates, shares of Belle Corporation and receive commissions from
subordinates or other persons, amasses, accumulates or such sale, nor that each unjustly enriched himself from
acquires ill-gotten wealth through a combination or series commissions, gifts and kickbacks; rather, it is that each
of overt or criminal acts as described in Section 1(d) of them, by their individual acts, agreed to participate,
hereof in the aggregate amount or total value of at least directly or indirectly, in the amassing, accumulation
Fifty million pesos (P50,000,000.00) shall be guilty of the and acquisition of ill-gotten wealth of and/or for
crime of plunder and shall be punished by reclusion former President Estrada.
perpetua to death. Any person who participated with the In the American jurisdiction, the presence of
said public officer in the commission of an offense several accused in multiple conspiracies commonly
contributing to the crime of plunder shall likewise be involves two structures: (1) the so-called wheel or circle
punished for such offense. In the imposition of penalties, conspiracy, in which there is a single person or group (the
the degree of participation and the attendance of hub) dealing individually with two or more other persons or
mitigating and extenuating circumstances, as provided by groups (the spokes); and (2) the chain conspiracy, in
the Revised Penal Code, shall be considered by the court. which there is successive communication and cooperation
in much the same way as with legitimate business
3) WON in sustaining the charge against petitioner for operations between manufacturer and wholesaler, then
alleged offenses, and with alleged conspirators, with which wholesaler and retailer, and then retailer and consumer.
and with whom he is not even remotely connected - From a reading of the Amended Information, the case at
contrary to the dictum that criminal liability is personal, not bar appears similar to a wheel conspiracy. The hub is
vicarious - resulted in the denial of substantive due former President Estrada while the spokes are all the
process; --NO accused, and the rim that encloses the spokes is the
common goal in the overall conspiracy, i.e., the amassing,
The allegations in the Amended Information, it is accumulation and acquisition of ill-gotten wealth.
clear that all the accused named thru their individual
acts, conspired with former President Estrada to

40
CRIMINAL LAW REVIEW DIGESTS
JUSTICE ROMEO CALLEJO NOTE: = Callejo Ponente

4) WON the allegation of conspiracy in the Amended Vitug, Dissenting:


Information is too general NO Allegation of conspiracy is not enough. It is neither right
nor just, to cast criminal liability on one for the acts or
The requirements on sufficiency of allegations deeds of plunder that may have been committed by
are different when conspiracy is not charged as a crime in another or others over which he has not consented or
itself but only as the mode of committing the crime as in acceded to, participated in, or even in fact been aware of.
the case at bar. There is less necessity of reciting its Such vicarious criminal liability is never to be taken lightly
particularities in the Information because conspiracy is not but must always be made explicit not merely at the trial but
the gravamen of the offense charged. The conspiracy is likewise, and no less important, in the complaint or
significant only because it changes the criminal liability of information itself in order to meet the fundamental right of
all the accused in the conspiracy and makes them an accused to be fully informed of the charge against him.
answerable as co-principals regardless of the degree of
their participation in the crime.[49 The liability of the Kapunan, and Buena, J, Dissenting:
conspirators is collective and each participant will be The requirement for complete allegations on the
equally responsible for the acts of others,[50 for the act of particulars of the indictment is based on the right of the
one is the act of all accused to be fully informed of the nature of the charge
In the case at bar, the second paragraph of the against him, so that he may adequately prepare for this
Amended Information alleged in general terms how the defense pursuant to the due process clause of the
accused committed the crime of plunder. It used the words Constitution. The fact, however, is that it is the
in connivance/conspiracy with his co-accused. These prosecution which determines the charges to be filed and
words are sufficient to allege the conspiracy of the how the legal and factual elements in the case shall be
accused with the former President in committing the crime utilized as components of the information. It is not for the
of plunder. accused, usually a layman, to speculate upon the
purposes and strategy of the prosecution and be
5. WON bail should be granted? Hearings must be thereafter prejudiced through erroneous guesswork.
conducted first to determine if the evidence of petitioners Thus, since the People dictate what he should be charged
guilt is strong as to warrant the granting of bail to with, fairness demands that he should not be convicted of
petitioner. a crime with which he is not charged or which is not
necessarily included therein.
During the pendency of the case, Jinggoy prayed
that he be allowed to post bail due to his serious medical Ynares-Santiago, Dissenting:
condition which is life-threatening to him if he goes back to There is no showing in the records that Mayor Estrada
his place of detention. Basing its finding on the earlier consciously adopted a common plan or joined in
testimony of Dr. Anastacio, the Sandiganbayan found that concerted action with President Estrada and Governor
petitioner failed to submit sufficient evidence to convince Singson to commit any two of the four criminal acts in the
the court that the medical condition of the accused amended information or conspired to commit more than
requires that he be confined at home and for that purpose once the receipt and transmission of jueteng money. If the
that he be allowed to post bail. petitioner was aware that the money entrusted to him for
The crime of plunder is punished by R.A. No. delivery came from illegal gambling, it is established that
7080 with the penalty of reclusion perpetua to death. mere knowledge, acquiescence, or agreement to
Under our Rules, offenses punishable by death, reclusion cooperate (in the transmission of jueteng funds in this
perpetua or life imprisonment are non-bailable when the case) is not enough to constitute one as a conspirator of
evidence of guilt is strong, to wit: the crime (in this case, plunder) with a view to a
The constitutional mandate makes the grant or furtherance of the common design and purpose.
denial of bail in capital offenses hinge on the issue of Petitioner states that he is linked to only P2,000,000.00 of
whether or not the evidence of guilt of the accused is jueteng money but the Ombudsman seeks to hold him
strong. This requires that the trial court conduct bail responsible with his father for the aggregate amount of
hearings wherein both the prosecution and the defense P4,097,804,173.17 of ill-gotten wealth. Petitioners
are afforded sufficient opportunity to present their criminal act is alleged to be contributing to the crime of
respective evidence. The burden of proof lies with the plunder. This construction of the law by the Prosecution
prosecution to show strong evidence of guilt.[60]This is dangerous if not ominous.
Court is not in a position to grant bail to the petitioner as
the matter requires evidentiary hearing that should be Sandoval-Gutierrez, Dissenting:
conducted by the Sandiganbayan. My theory is that there are four separate conspiracies with
no overall goal or common purpose to commit the crime of

41
CRIMINAL LAW REVIEW DIGESTS
JUSTICE ROMEO CALLEJO NOTE: = Callejo Ponente

plunder. For one, there is no allegation in the Amended called upon to help him execute the affidavit. He gave his
Information that petitioner agreed with the former confession thereafter with assistance of Atty. Falgui. He
President and the other accused to acquire and amass ill- admitted that he, together with the others, including
gotten wealth by misappropriating the tobacco excise tax Ferdinand (security guard) planned the kidnapping.
allocated for Ilocos Sur; by ordering the GSIS and SSS to Thereafter, the men that Pagalasan mentioned in
purchase shares of Belle Corporation and receive his confession were arrested. The men holding
commission from such sale; and enriching himself from Christopher learned about this and stated that the people
commissions, gifts and kickbacks named and arrested were innocent and werent involved.
They also asked for a 3 million ransom. The following
morning however, the policemen were able to rescue
PEOPLE V. PAGALASAN Christopher without paying the ransom money.
Pagalasan was thereafter charged with
Facts: Crime: Two counts of Kidnapping for Ransom (of kidnapping for ransom. However, after a few days, he
George Lim and son Christopher Lim-10yo) substituted his counsel Atty. Falgui with Atty. Fontanilla.
Spouses George and Desiree Lim had 3 children, Pagalasan retracted his extrajudicial confession, saying
one of whom is Christopher. They resided in Villa that he was tortured and held at gun point when he
Consuelo Subdivision in General Santos City. They hired executed it and that he wasnt assisted by counsel of his
a security guard, Ferdinand Cortez. choice.
On Sept. 4, 1994, around 11 pm, 4 armed men As a defense, Pagalan says that at the time of
barged into their house through the kitchen door. The the incident, he was riding a tricycle which stopped near
intruders all wore bonnets and when they entered through the Lim house. He saw the masked men who saw him as
the kitchen door, they dragged security guard Ferdinand well. He alleges that the men poked their guns at him and
with them with his hands tied. The men barged into the forced him to participate in the kidnapping and he was
bedroom of the spouses Lim. They demanded that the designated as the driver. He further claims that he was
spouses cooperate with their demands if they didnt want tortured to force him to write the confession and that he
to get hurt. They ransacked the house, got cash and other was mauled.
valuables. Thereafter, the men gave wife Desiree a The cases filed against Pagalasan are illegal
handwritten note. The note threatened the spouses not to possession of fire arms and kidnapping for ransom AND
get the military involved and that they will be getting in serious illegal detention. He was acquitted for possession
touch with the couple soon. and was convicted for kidnapping.
The men demanded that George give them the
key to the car. He complied. The men dragged George Issue: W/N Pagalasan should be convicted of kidnapping
and one of his children, Christopher, to the car. The men with ransom.
drove along the national highway and blindfolded George
and Christopher. After some time driving, the car stopped Held: Pagalasan is convicted of only kidnapping, not
at Sitio Tupi and 2 of the men alighted bringing with them kidnapping with ransom, the latter being punishable with
Christopher. Then the driver again started the car to death.
transport George to Maasim. In this case, the evidence on record inscrutably
Meanwhile, the police were informed of the shows that the appellant and his three cohorts were armed
kidnapping and policemen were dispatched for with handguns; two of them had hand grenades, and all of
investigation and set up a checkpoint. The masked driver them had masks over their faces. They gained entry into
with George halted when he saw the checkpoint, switched the Lim residence after overpowering the security guard
off the headlights and took off his mask. The driver turned Ferdinand and the housemaid Julita, and tying their hands
out to be the appellant in this case, Michael Pagalasan. behind their backs. One of the masked men remained in
The policemen approached the car, indentified themselves the sala, while the three others barged into the bedroom of
and asked for the passengers names. Although George George and Desiree, and kidnapped George and his ten-
gave a false name, the policemen saw his hands were year-old son Christopher. The appellant and his cohorts
shaking and they opened the door of the car. They forced father and son to board Georges car. The
arrested Pagalasan and inspected the vehicle, finding a appellant drove the car, dropped off Christopher and his
handgun and a grenade. cohorts at Sitio Tupi, and drove on with George in the car
At the police station, Pagalasan was placed towards the direction of Maasim.
under custodial investigation. It is said that the police The collective, concerted and synchronized acts of
inquired if Pagalasan wanted to execute an affidavit and if the appellant and his cohorts before, during and after the
he knew a lawyer. Pagalasan said he wanted to and that kidnapping constitute indubitable proof that the appellant
he didnt knew any lawyer. Thereafter, Atty. Falgui was and his three companions conspired with each other to

42
CRIMINAL LAW REVIEW DIGESTS
JUSTICE ROMEO CALLEJO NOTE: = Callejo Ponente

attain a common objective: to kidnap George and the same occasion and from the same situs. As a
Christopher and detain them illegally. The appellant was a matter of procedural law, the appellant may be
principal by direct participation in the kidnapping of the two convicted of slight illegal detention under the
victims. Information for kidnapping for ransom as the former is
HOWEVER, the prosecution failed to prove that he necessarily included in the latter crime.
had knowledge of and concurred with the said demand for The SC held that Pagalasan is guilty of slight illegal
ransom so he cannot be convicted of the graver crime of detention. Article 268 of the Revised Penal Code which
kidnapping for ransom, only kidnapping. reads:
To warrant the imposition of the death penalty for the Art. 268. Slight illegal detention. The penalty
crime of kidnapping and serious illegal detention for of reclusion temporal shall be imposed upon any private
ransom, the prosecution must prove beyond reasonable individual who shall commit the crimes described in the
doubt the following: (a) intent on the part of the accused to next preceding article without the attendance of any of the
deprive the victim of his liberty; (b) actual deprivation of circumstances enumerated therein.
the victim of his liberty; (c) motive of the accused, which is The same penalty shall be incurred by anyone
extortion of ransom from the victim or any other who shall furnish the place for the perpetration of the
person. The qualifying circumstance which must be crime.
alleged in the Information and proved by the prosecution If the offender shall voluntarily release the person
as the crime itself by words and overt acts of the accused so kidnapped or detained within three days from the
before, during and after the kidnapping and detention of commencement of the detentio, without having attained
the victim. Neither actual demand for nor actual payment the purpose intended, and before the institution of criminal
of ransom is necessary for the crime to be committed. proceedings against him, the penalty shall be prision
In this case, the prosecution was able to prove mayor in its minimum and medium periods and a fine not
beyond reasonable doubt that the appellant conspired exceeding seven hundred pesos. (As amended by
with three others to kidnap the victims. However, it Republic Act No. 18).
failed to prove that they intended to extort ransom While the epigraph or title of the article mentions only
from the victims themselves or from some other slight illegal detention, kidnapping committed in
person, with a view to obtaining the latters connection with the lower offense of slight illegal detention
release. The kidnapping by itself does not give rise to the is also covered by the article.
presumption that the appellant and his co-conspirators In this case, the appellant is a private
purpose is to extort ransom from the victims or any other individual. George had been kidnapped and detained
person. illegally by the appellant and his cohorts, but only for less
The only evidence adduced by the prosecution to than a day. George regained his freedom after the
prove the element of extorting ransom are the three appellant had been arrested at the intersection of the
handwritten letters. There was no demand for ransom in national highway and Espina Road. There is no evidence
exchange for George and Christophers liberty. While that the appellant and his cohorts kidnapped George for
there is a demand for ransom of P3,000,000 in the purpose of extorting ransom for his release. There is
the second letter, and a demand for the release of Ronie likewise no evidence that they inflicted any serious
Puntuan within three days in the third letter, the said physical injuries on George, or simulated public authority,
demands are in consideration of Christophers release or threatened to kill him. Furthermore, there is no evidence
from custody, and not that of George. that the appellant and his cohorts intended to detain the
There is no evidence that the signatory and sender victim for more than three days.
of the second letter is a co-conspirator of the appellant, Although the appellant and his co-conspirators
the latter is not bound by the said letter, conformably to kidnapped George and Christopher on the same occasion
Section 28, Rule 130 of the Revised Rules of Evidence. and from the same situs, the appellant is guilty of two
separate crimes: kidnapping under Article 267 of the
Issue: W/N Pagalasan should be convicted of slight illegal Revised Penal Code, and slight illegal detention under
detention under Article 268 of the Revised Penal Code, for Article 268 of the Revised Penal Code. The appellant
kidnapping George. and his co-conspirators were animated by two sets of
separate criminal intents and criminal resolutions in
The prosecution may have failed to prove that the kidnapping and illegally detaining the two
appellant and his co-conspirators intended to extort victims. The criminal intent in kidnapping Christopher
ransom for Georges release; however, as a matter of was separate from and independent of the criminal
substantive law, the appellant may be held guilty of intent and resolution in kidnapping and detaining
two separate crimes, although he and his co- George for less than three days. In the mind and
conspirators kidnapped George and Christopher on conscience of the appellant, he had committed two

43
CRIMINAL LAW REVIEW DIGESTS
JUSTICE ROMEO CALLEJO NOTE: = Callejo Ponente

separate felonies; hence, should be meted two The affirmative defense of self-defense may be
separate penalties for the said crimes: one for complete or incomplete. It is complete when all the three
kidnapping under Article 267 of the Revised Penal essential requisites are present; it is incomplete if only
Code and another for slight illegal detention under unlawful aggression on the part of the victim and any of
Article 268 of the same code. The felony of slight illegal the two essential requisites were present. Unlawful
detention is necessarily included in the crime of aggression on the part of the victim is a condition sine qua
kidnapping for ransom; thus, the appellant may be non to self-defense, complete or incomplete.
convicted of the former crime under an Information for The right of self-defense proceeds from necessity
kidnapping for ransom. and limited by it. The right begins where necessity does,
There is conspiracy when two or more persons and ends where it ends. There is however, a perceptible
agree to commit a felony and decide to commit difference between necessity and self-defense. Self-
it. Conspiracy as a mode of incurring criminal liability must defense excuses the repulse of a wrong; necessity
be proven separately from and with the same quantum of justifies the invasion of a right. Hence, it is essential to
proof as the crime itself. Conspiracy need not be proven self-defense that it should be a defense against a present
by direct evidence. To hold an accused guilty as a co- unlawful attack.
principal by reason of conspiracy, he must be shown to Self-defense is an act to save life; hence, it is
have performed an overt act in pursuance or furtherance right and not a crime. x x x It is a settled rule that to
of the complicity. There must be intentional participation in constitute aggression, the person attacked must be
the transaction with a view to the furtherance of the confronted by a real threat on his life and limb; and the
common design and purpose. peril sought to be avoided in imminent and actual, not
merely imaginary. But what then is the standard? We rule
that the test should be: does the person invoking the
SENOJA V. PEOPLE defense believe, in due exercise of his reason, his life or
limb is in danger? Hence, when an inceptual/unlawful
Facts: Exequiel Senoja, Fidel Senoja (they were brothers), aggression ceases to exist, the one making a defense has
Jose Calica and Miguel Lumasac were drinking gin in the no right to kill or injure the former aggressor. After the
hut of Crisanto Reguyal. Leon Lumasac suddenly barged danger has passed, one is not justified in following up his
in, holding a bolo and was looking for his brother Miguel adversary to take his life.
whom he suspected of drying up the ricefield he was IN THIS CASE, there were two events
plowing. However, when Senoja (Exequiel) approached concerned: 1) The arrival of Leon who was armed with a
Leon, the latter tried to hack him so he embraced Leon bolo and 2) When Leon demanded for his bolo because
and Jose took Leons bolo. After the confrontation, Leon he wanted to go home already after the commotion inside
wanted to get his bolo back because he wanted to go the house, and then eventually left with a threat. Quoting
home. After getting it back, Leon walked out of the place the appellate court, the SC said that the victim had already
followed by Senoja. Suddenly, Senoja stabbled Leon at left the hut. At that point in time, the victim was simply
the back. When Leon turned around, Senoja continued walking toward his home; he had stopped being an
stabbing him until he fell to the ground. Then petitioner ran aggressor. It was Senoja who wanted a confrontation this
towards the barangay road and threw away the knife he time. It was Senoja who was now the unlawful aggressor
used to stab Leon. in this second phase of their confrontation.
Petitioner admitted killing the victim but invoked
the affirmative defense of self-defense. His version said
that after the commotion inside the house, Leon left but UNITED STATES V. DOMEN
with a threat that something will happen to Senoja. Senoja
followed Leon as the latter was making his way home. Facts: Domen and the deceased Victoriano Gadlit
When Leon realized that Senoja was following him, Leon quarrelled about a carabao of the defendant Domen which
walked back towards him and suddenly hacked Senoja at Gadlit said had gotten into his corn patch. The deceased
the left side of his head and right thigh. Unable to evade attacked the defendant and struck him with a piece of
the treacherous attack by Leon, Senoja drew his colonial wood called Japanese, about a vara in length and about
knife and stabbed Leon in self-defense, inflicting upon him the size of ones wrist. The deceased struck at the
multiple wounds which caused his death. accused four or five times and that the accused did not
retreat but struck back wounding the deceased at the
Issue: W/N Senoja merely acted in self-defense forearm.

Held: No, Senoja is guilty of HOMICIDE. Issue: W/N there was reasonable necessity for the means
employed by the defendant to repel the attack?

44
CRIMINAL LAW REVIEW DIGESTS
JUSTICE ROMEO CALLEJO NOTE: = Callejo Ponente

Issue: W/N Beatriz has a legitimate claim for self-


Held: Yes, defendant must be ACQUITTED. defense?
Quoting US v. Molina, the Court said: 1) During
an unlawful attack by another and while a struggle is going Held: None!
on and the danger to his person or to his life continues, There was no unlawful aggression on the part of
the party assaulted has a right to repel the danger by Marciano. Hence, there is no reason to consider the other
wounding his adversary, and if, necessary, to disable him; elements of self-defense lack of sufficient provocation
2) the fact that a person when assaulted does not flee and reasonable necessity of the means employed
from his assailant is not sufficient reason for declining in a because these elements presuppose the existence of
proper case to uphold the rational necessity of the means unlawful aggression.
employed in repelling the illegal attack. Beatriz alleges that before she stabbed
The retreat to the wall doctrine says that it is the Marciano, he pushed her head on account of which she
duty of a person assailed to retreat as far as he can before felt dizzy and hit her leg against something. The court did
he is justified in meeting force with force. This principle not believe this argument on the ground that it was not
has now given way in the United States to the stand supported by evidence. Moreover, even if this was to be
ground when in the right rule. A true man, who is without believed, the court said that a slight push of the head with
fault, is not obliged to fly from an assailant, who, by the hand does not constitute the unlawful aggression
violence or surprise, maliciously seeks to take his life or do contemplated by the law. Unlawful aggression as an
him enormous bodily harm. element of self-defense is not necessarily implied in any
IN THIS CASE, the accused did not provoke the act of aggression against a particular person, when the
assault. The accused was where he had the right to be. author of the same does not persist in his purpose or
The law did not require him to retreat when his assailant when he desists therefrom to the extent that the person
was rapidly advancing upon him in a threatening manner attacked is no longer in peril.
with a deadly weapon. The accused was entitled to do The court then gave certain doctrines which
whatever he had reasonable grounds to believe at the illustrated certain acts which do not constitute unlawful
time was necessary to save his life or to protect himself aggression, such as: hard blow on the head without
from great bodily harm. The element of practicability made specifying whether he used his hand or any instrument,
it impossible for him to determine during the heat of a this being the only act preceding the stabbing of the victim;
sudden attack whether he would increase or diminish the holding the accused by the necktie and giving him a blow
risk to which exposed by standing his ground or stepping on the neck with the back of the hand without injuring him;
aside. His resistance was not disproportionate to the a shove or an attempt to strike with a bench or chair, all of
assault. The wound was inflicted, not on what is usually a which took place in a bar. (NB: Names of these cases
vital part of the body but on the arm as one would naturally were not stated, court only cited Gazette dates)
strike to defend himself. Court considered the following mitigating
circumstances: (1) obfuscation because she was
abandoned by Marciano with whom she had been living
PEOPLE VS. YUMAN with for years; and (2) lack of instruction she was an
illiterate.
Facts: Marciano Martin and accused Beatriz Yuman
without being married, lived as husband and wife for about
3 or 4 years until Marciano decided to leave their common PEOPLE VS. DE LA CRUZ
dwelling. Beatriz went to look for Marciano at the cockpit
of Mandaluyong. From there, they rode a vehicle wherein Facts: The deceased Leoncio Naos, together with the
they Marciano intimated to Beatriz his determination to spouses Cabasan, lived in a house owned by de la Cruz.
end their relations. After Marciano rudely shunned away De la Cruz demanded that the spouses vacate the house.
Beatrizs suggestion that they go home together, Beatriz Naos then intervened in behalf of the spouses and told
pulled out a penknife and stabbed Marciano on the right de la Cruz to permit them to stay. De la Cruz disliked this
lumbar region which damaged his kidney. Thereafter, intervention. The following day, de la Cruz, carrying his
Marciano ran away but Beatriz, with penknife in hand, loaded rifle and accompanied by 2 men, returned to the
pursued him. Beatriz only stopped when Marciano came place of the spouses. At that time, Naos had with him a
across a traffic policeman, Eduardo Dizon. Beatriz was kris and a small bolo, as was his practice in going to the
then arrested and eventually charged with homicide field, for the purpose of defense against the Moros and the
because Marciano died the day after. Beatriz claims self- animals. Both the kris and the bolo were sheathed. When
defense. de la Cruz saw Naos approaching him he fired causing 9
wounds in the left leg and one in the right. TC found de la

45
CRIMINAL LAW REVIEW DIGESTS
JUSTICE ROMEO CALLEJO NOTE: = Callejo Ponente

Cruz guilty of homicide. He argues that he was only acting Cajurao admitted the stabbing, but claimed he
in self-defense. was defending himself. He said that at around 9 PM,
Betita shouted at him, Putang ina ka, ari pa na, nakit-an
Issue: W/N De la Cruz has a legitimate claim for self- na ta! Betita also accused him of being a braggart and a
defense? liar. At about 10:00 p.m., Cajurao went out of the gym and
seated himself on a concrete bench nearby, beside the
Held: None! There was no unlawful aggression! trunk of a mango tree. Betita followed and shouted at him
According to the prosecution witnesses, Naos was about saying, When you are in a group you are a braggart.
15 yards away from de la Cruz when he fired his rifle. This Now, we are here outside. The Cajurao replied, saying,
fact was also supported by the experiments made by the Boy, what is this? Betita retorted, You came here just to
Constabulary which showed that the distance of the look for trouble! The appellant stood up and was about to
wounds in the leg of Naos could only have been made if leave, but Betita slapped him on the face. Betita then fled
the latter was about 15 yards away. This means that to the stall of Pordios and took hold of the makeshift lamp.
Naos, who was 15 yards away and had both his bolo and As he was about to throw the lighted lamp at the appellant
kris sheathed at the time, did not pose any real and who was about four meters away, the latter walked slowly
imminent danger to the life and limb of de la Cruz. to Betita and asked, Why did you slap me, Boy? The
The fact that de la Cruz had gone to the house appellant pushed Betitas hand aside, the hand that held
with his rifle loaded in advance, and the fact that he fired the lamp, and pulled out a knife from his waist. The
at the deceased without any prior provocation which could appellant then stabbed Betita on his right nipple. He threw
properly be considered as such, are indicative not only of his knife in a grassy area and fled from the scene.
de la Cruzs intention to defend himself in case of Cajurao claims he stabbed Betita because the
aggression, but also to provoke and commit the same. latter took hold of the Tanduay lamp on the stall of
Furthermore, de la Cruz, apart from having a Pordios and was about to throw it at him. Cajuraos
loaded rifle in hand, was also accompanied by 2 men. witness testified that Betita was about to throw the lamp at
Naos wouldve been dumb had he attacked de la Cruz. Cajurao. This impelled the appellant to rush to where
The point of the matter is that there was no real or at least Betita was. Before the lamp could be thrown at him, he
imminent aggression on the part of Naos. stabbed Betita. According to Cajurao, the victims act of
Court considered 2 mitigating circumstances: (1) slapping him and attempting to throw the lighted lamp at
lack of intention to cause so grave a wrong; and (2) him constituted unlawful aggression on the part of the
voluntary surrender. latter. Thus, there was no provocation on his part; the
means he used to repel the unlawful aggression of Betita
was reasonable.
People v Cajurao The trial court did not believe Cajuraos claim of
Memory aid: NIPPLE STAB self-defense.

Cajurao was charged with murder (qualified by treachery) Issue: Was this valid self-defense? NO.
for stabbing Santiago Betita.
On November 29, 1993, the residents of Held: Second (credible witnesses for prosecution), third
Poblacion, Surallah, South Cotabato were in a festive (prosecution witnesses immediately reported crime a
mood. There was carnival in the municipal plaza. There day after, while defense witnesses did not), and fifth
was also a disco in the town gym. Pacita Pordios put up a reason (defense witness found not credible) are more
stall outside the gym. She used a makeshift lamp (a bottle evidentiary issues, than crim issues. Read original if you
of Tanduay with kerosene) to light her stall. want to know in detail.
Cajurao and his friend Danosos tried to enter the First. Like alibi, self-defense is a weak defense
gym, but as they didnt have any tickets, they were refused because it is easy to fabricate. When the accused
entry. interposes self-defense, he thereby admits having killed
At around 10:30 PM, the victim Betita went to the victim. The burden of proof is shifted on him to prove
Pordios stall and got her lamp. Pordios got angry with with clear and convincing evidence the confluence of the
Betita, but the latter just ignored Pordios. Suddenly, essential requisites of a complete self-defense, namely:
someone threw a stone, prompting people to scamper (a) unlawful aggression on the part of the victim; (b)
away. And then, out of nowhere, Cajurao sped towards reasonable necessity of the means employed to prevent or
Betita and stabbed him on the right nipple. Cajurao ran repel it; and (c) lack of sufficient provocation on the part of
away but was soon caught by some volunteers, led by the person defending himself. The accused must rely on
Domingo Tecson. the strength of his own evidence and not on the weakness
of the evidence of the prosecution; because even if the

46
CRIMINAL LAW REVIEW DIGESTS
JUSTICE ROMEO CALLEJO NOTE: = Callejo Ponente

prosecutions evidence is weak, the same can no longer and right hand arched backward, ready to throw the bottle
be disbelieved. The appellant failed to discharge his at the appellant. This was not the case.
burden. Assuming that Betita did slap the appellant on the
Fourth. The flight of the appellant, his throwing face, the appellants evidence shows, however, that Betita
away the knife used to stab the victim, his failure to report anticipated that the appellant would retaliate and forthwith
the stabbing and to surrender himself to the police ran away to the stall of Pacita and took hold of the knife.
authorities and to thereafter claim that he killed Betita in From that moment, the inceptive unlawful aggression on
self-defense, all these belie his claim that he killed the the part of Betita had ceased to exist; there was no longer
victim in self-defense. a need for the appellant to still pursue the victim and kill
Sixth. There can be no self-defense, complete or him. In fine, when the appellant stabbed the victim, he did
incomplete, unless there is clear and convincing proof of so to retaliate.
unlawful aggression on the part of the victim. The Court found that there was no treachery either.
unlawful aggression, a constitutive element of self- So he was liable for homicide.
defense, must be real or at least imminent and not merely
imaginary. A belief that a person is about to be attacked is
not sufficient. Even an intimidating or threatening attitude PEOPLE V CATBAGAN
is by no means enough. Unlawful aggression (Pretty long case because it involved 3 victims -
presupposes an actual or imminent danger on the life or Mickey).
limb of a person. Mere shouting, an intimidating or Memory aid: 3 victims.
threatening attitude of the victim does not constitute
unlawful aggression. Unlawful aggression refers to an Catbagan was charged of homicide, murder and frustrated
attack that has actually broken out or materialized or at the murder. He claims self-defense and lawful performance of
very least is clearly imminent; it cannot consist in oral duty.
threats or merely a threatening stance or posture.he A birthday party was being held for Danilo
settled rule in jurisprudence is that when unlawful Lapidante. A guest of his, Air Force and PSG man, Sgt
aggression ceases, the defender no longer has the right to Suico was really excited and started firing shots into the
kill or even wound the former aggressor. Retaliation is not air with his armalite rifle. Policeman Catbagan heard the
a justifying circumstance.Upon the cessation of the shots, and since the election ban was at full effect that
unlawful aggression and the danger or risk to life and limb, time, he went to the house of Lapidante to investigate. No
the necessity for the person invoking self-defense to one confessed to the shooting so Catbagan went home,
attack his adversary ceases. If he persists in attacking his embarrassed.
adversary, he can no longer invoke the justifying Coincidentally, before Catbagan got to
circumstance of self-defense. Self-defense does not justify Lapidantes house, Sgt. Suico and his friend Lacaden,
the unnecessary killing of an aggressor who is retreating went back home to exchange the armalite with a pistol.
from the fray. After a while, Catbagan, along with his friend
In this case, Pordios (the stall owner) testified Fababier, returned to the party to investigate again. Suico
that the appellant stabbed Betita even as the latter moved told him that the shots were just part of the celebration.
over to the next stall, still holding the lamp with the lighted Suddenly, a piece of stone hurled from the
wick which he took from her stall to defend himself from direction of the celebrants house landed on a tree and
the appellant. Betita had anticipated that the appellant thence to the body of Catbagan. Irritated and reacting
would assault him. Betitas fears proved to be well- thereto, Catbagan directed Fababier to look for the one
founded, as the appellant rushed to where he was and who threw the stone.
stabbed him on the right nipple. Pordios did not testify The prosecution claimed that at that moment,
that before the stabbing, Betita was about to throw the Sgt. Suico got out of the pedestrian steel gate and
bottle at the appellant. extended his hand towards Catbagan in the street as he
On cross-examination by defense counsel, introduced himself as being a PSG. Completely ignoring
Pordios testified that before the appellant stabbed Betita, the gesture of the latter, Catbagan drew out his .9mm
the latter was merely holding the bottle in his right hand, automatic pistol and with both hands holding the gun, fired
on the level of the right shoulder, with his elbow by the successively at Suico, who when hit stretched out his
side of the body. hand, shouting Huwag (Dont) Pare. Despite this
In fine, Betita was in a defensive position when Catbagan fired more shots at the victim who fell on the
he was stabbed. If, as claimed by the appellant, Betita pavement, bloodied and dying from mortal wounds.
was about to throw the bottle at him, surely Betitas right As the shots were fired, Jun Lacaden who was
hand would have been raised above his head, his body taking a nap on the front seat of the owner-type jeep
parked on the other side of the street was abruptly

47
CRIMINAL LAW REVIEW DIGESTS
JUSTICE ROMEO CALLEJO NOTE: = Callejo Ponente

awakened. Not fully aware of what happened, he lawful exercise.


disembarked therefrom without knowing what to do. These requisites are absent in this case. Appellant
Unexpectedly, two shots were also fired at him by was not performing his duties at the time of the shooting,
Catbagan. One bullet found its mark in the body of Jun because the men he shot had not been indiscriminately
Lacaden who then fell down. firing guns in his presence, as he alleges. The trial court
Almost simultaneously, Catbagan directed his said that that Catbagan had no personal knowledge that it
attention to Lapidante who was then inside their was Suico who had been firing the Armalite.
compound in the vicinity of their steel main gate. Upon the At most, appellant was in the house of the
prompting of his wife Rosita for him to run and evade the Lapidantes to determine who had fired the gunshots that
assailant, the celebrant turned towards the main door of were heard by the neighborhood. But the fatal injuries that
their house. But before he could reach the safety of their he inflicted on the victims were not a necessary
abode, two rapid shots were aimed by Catbagan at him, consequence of the performance of his duty as a police
one of which hit him in the upper part of his body. officer.
As a consequence of the injuries they sustained, His presence at the scene of the incident should be
Sgt. Suico died on the spot; Lapidante later died in the distinguished from his act of shooting them. His presence
hospital in Lagro, Quezon City; whereas Jun Lacaden had was justified, his act of shooting was not. He was duty-
to be treated and confined at the East Avenue Medical bound to find out who had fired the gun that day and to
Center, Quezon City. maintain peace and order in the neighborhood. But his act
Appellant argues that he was justified in shooting of shooting of the victims cannot be justified.
the victims, as he was merely defending himself and There is an important distinction between the
fulfilling his sworn duties. He claims that upon reaching the present case and People v. Cabrera. In the latter, the
house of Lapidante, a rock was thrown at him. He also disturbance had been created by the victim in the
claimed that two people rushed out to him - Lacaden who presence of the accused, who therefore had the duty to
had an ice pick and Suico who aimed a gun at him. immediately intervene and subdue the former, who was
Threatened of his safety, he drew his own gun while causing danger. In the present case, appellant had no
stepping backward and fired at the aggressors. After, he personal knowledge of who had fired the gunshots. Thus,
saw Lapidante rushing back to his house, shouting akin his duty at the time was simply to determine who was the
na yung mahaba! He fired a warning shot, uttering: subject of the complaints of the residents of the village. It
Tumigil ka, huwag kang kikilos. Lapidante, however, did was never shown, though, that the shooting was in
not heed Catbagans warning and continued rushing furtherance of or was a necessary consequence of his
towards his house, as if to get something. Fearing that performance of such duty.
Lapidante might be able to get hold of the long gun, To be sure, the right to kill an offender is not
Catbagan fired a shot at him once. absolute, and may be used only as a last resort, and
under circumstances indicating that the offender cannot
Issue 1: Will the defense of fulfilling his lawful duties lie? otherwise be taken without bloodshed. The law does not
No. clothe police officers with authority to arbitrarily judge the
necessity to kill. It must be stressed that the judgment and
Appellant invokes his lawful performance of duty as discretion of police officers in the performance of their
one such circumstance, arguing that his presence at the duties must be exercised neither capriciously nor
scene of the incident were all in consonance with the oppressively, but within reasonable limits. In the absence
legitimate performance of a sworn duty. Citing these of a clear and legal provision to the contrary, they must act
specific facts (complaints of neighbors, indiscriminate in conformity with the dictates of a sound discretion, and
shots, the gun ban), he argues that he was justified in within the spirit and purpose of the law.
shooting the victims. In effect, his contention is that he
was justified in maintaining public order, as well as in Issue 2: Is the defense of self-defense valid?
protecting and securing life and property because he was
a policeman. We should look at the circumstances of the shooting in the
Although he is correct in arguing that he had the case of each victim.
legal obligation to maintain peace and order, he was not As to Suico, no valid self-defense because the
justified in shooting the victims. Two requisites must means employed were not reasonable, but he is granted a
concur before this defense of lawful performance of duty mitigating circumstance because of lack of sufficient
can prosper: 1) the accused must have acted in the provocation and the presence of unlawful aggression.
performance of a duty or in the lawful exercise of a right or Unlawful aggression is an actual physical assault, or at
office; and 2) the injury caused or the offense committed least a threat to inflict real imminent injury, upon a person.
should have been the necessary consequence of such In case of threat, it must be offensive and strong,

48
CRIMINAL LAW REVIEW DIGESTS
JUSTICE ROMEO CALLEJO NOTE: = Callejo Ponente

positively showing the wrongful intent to cause injury -- as of aggression or by some external acts showing the
in this case. Thus, Suicos act of aiming a cocked gun at commencement of actual and material unlawful
appellant is sufficient unlawful aggression. aggression.
The means employed by the person invoking Catbagan also argued that Lapidante rushed
self-defense is reasonable if equivalent to the means of towards his house to take a more advantageous position.
attack used by the original aggressor. Whether or not the Court ruled against him. Referred to here is the rule that if
means of self-defense is reasonable depends upon the it is clear that the purpose of the aggressor in retreating --
nature or quality of the weapon, the physical condition, the or, as in this case, Lapidantes rushing towards his house -
character, the size and other circumstances of the - is to take a more advantageous position to ensure the
aggressor; as well as those of the person who invokes success of the attack already begun, the unlawful
self-defense; and also the place and the occasion of the aggression is considered still continuing; and the one
assault. resorting to self-defense has a right to pursue and disable
The nature and the number of gunshot wounds -- the former.
debilitating, fatal and multiple -- inflicted by appellant on Obviously, this rule does not apply to Lapidante,
the deceased shows that the means employed by the because 1) there was no clear purpose in his act of
former was not reasonable and commensurate to the retreating to take a more advantageous position; and 2)
unlawful aggression of the latter. The unreasonableness since he never attacked appellant in the first place, the
becomes even more apparent from the fact, duly admitted former could not have begun any unlawful aggression and,
by appellant himself, that Suico had obviously been hence, would not have had any reason to take a more
inebriated at the time of the aggression. It would have advantageous position. How could there have been a
thus been easier for the former to have subdued the victim continuation of something that had never been started? If
without resorting to excessive means. any aggression was begun in this case, it was by Suico,
Finally, as to the element of lack of sufficient not by Lapidante.
provocation on the part of the person resorting to self- As to Lacaden, Catbagan claims Lacaden rushed
defense, appellant has sufficiently established that he towards him with an ice pick. But the evidence does not
went to the house of the Lapidantes to find out who had support his cliam.
fired the gunshots earlier that day. There was therefore Moreover, the evidence showed that Lacaden
absolutely no provocation from him, either by unjust was shot in the BACK. The wound in the back of the victim
conduct or by incitement, that would justify Suicos acts of clearly shows that he was shot while his back was turned
cocking and aiming a gun at him. to appellant. Hence, there was no unlawful aggression on
Not having proven all the elements of self- the part of the former.
defense, appellant cannot use it to justify sufficiently his
fatal shooting of Suico. Having proven a majority of the SC: homicide for Suico (no treachery, plus mitigating
elements, however, the former may still be credited with a circumstance of incomplete self-defense and voluntary
mitigating circumstance in accordance with Article 13 of surrender) and Lapidante (no treachery, mitigating
the RPC. circumstance of voluntary surrender), less serious physical
As to Lapidante, he allegedly rushed towards his injuries for Lacaden (no treachery, intent to kill, mitigating
house to get hold of the mahaba, so appellant had no circumstance of voluntary surrender ).
other recourse but to shoot him. The purpose of the victim
in rushing towards his house was supposedly to recover
the advantage he had previously enjoyed. Hence, it is
argued that unlawful aggression was present.
No unlawful aggression. Unlawful aggression presupposes
an actual, sudden and unexpected attack or imminent
danger thereof. Such aggression refers to an attack that PEOPLE V. DECENA
has actually broken out or materialized or is at the very
least clearly imminent; it cannot consist merely of any oral FACTS:
threat or intimidating stance or posture. Here, the Complainant Renelyn UDE (assisted by her
perceived danger was more in the mind of appellant than mother, Erlinda AGUIRRE) filed a complaint for
in reality. His act of running towards his house can hardly RAPE against Edwin DECENA before the RTC of
be characterized as unlawful aggression. It could not Aklan. The information stated:
have imperiled Catbagans life. Court ruled that a threat That on or about the 9th day of March,
even if made with a weapon or the belief that a person [is] 1995, in the afternoon, in Barangay
about to be attacked, is not sufficient, but that it is Dumga, Municipality of Makato,
necessary that the intent be ostensibly revealed by an act Province of Aklan, Republic of the

49
CRIMINAL LAW REVIEW DIGESTS
JUSTICE ROMEO CALLEJO NOTE: = Callejo Ponente

Philippines, and within the jurisdiction of would not allow UDE to go to another
this Honorable Court, the above-named man because UDE is for him.
accused, with lewd design and with o UDE collaborated AGUIRREs
intimidation, did then and there wilfully, declarations on material points
unlawfully and feloniously have sexual regarding the incident of Mar. 9, 1995.
intercourse with the offended party, UDE told the Court in the afternoon of
RENELYN UDE, a woman, against her the incident she was alone in their
will and without her consent thereby house; her mother had gone some
inflicting upon the latter, physical place; DECENA took the top part of her
injuries, to wit: clothing and then totally undressed her;
xxx DECENA tied her hands with a towel;
CONTRARY TO LAW." holding his scythe on her neck,
DECENA pleaded not guilty and testified in his DECENA succeeded in having sexual
defense that AGUIRRE had a motive against him intercourse w/ her.
[AGUIRRE wanted to reconcile with her legal o According to UDE, DECENA has been
husband] and claimed that he was working in the using her for around 6 months and for
house of Perseverancia TUBAO on the date and more than 40 times.
time in question. TUBAO testified to corroborate o In the afternoon of Mar 12, 1995,
the defense of alibi. DECENA wanted to have sexual
Prosecution presented UDE and Dr. Emma intercourse w/ UDE but she ran away
CORTES, who rendered the medico legal report and hid under the Dumga Bridge until
on the examination conducted on UDE. her mother found her. UDE was crying
The evidence for the prosecution was because her vagina was swelling and
summarized by the trial court as follows: UDE did not want to go back to their
o UDE is the 12-year old daughter of house. They went to the Brgy. Captain
AGUIRRE by her legal spouse, Ramon of Dumga to report the incident. The
Ude, from whom she is separated in Brgy. Captain accompanied them to the
fact. DECENA is AGUIRREs common- Makato Police Station. At about 5PM,
law spouse, and has been living with DECENA was arrested by the
them since UDE was only in grade 3. policemen of their town, and UDE (w/
o On Mar. 9, 1995, AGUIRRE was asked her mother AGUIRRE) was able to
by DECENA to borrow the fishing net return home.
(hudhud) of her uncle whose house On Mar. 12, 1995 at 5:45 PM, UDE was
was around 1 km from their house. examined by Dr. CORTES. Her internal
DECENA accompanied her but left examinations as stated in the medico-legal report
ahead. were as follows: admits 1 finger with ease; old
o AGUIRRE was not able to borrow the laceration 3:00 oclock position (Labia Minora); no
fishing net so she returned home signs of external physical injuries In her
immediately testimony, she said that DECENAs hymen was
o She found a naked DECENA on top of quite thick and the old laceration was deep it
her equally naked daughter UDE on the was not a superficial [laceration]. Old laceration
stairs of her small nipa house. DECENA was not connected w/ rape because a new
was having sexual intercourse with laceration in the hymen would have healed in 2 to
UDE. On UDEs neck was a scythe held 3 days. Old laceration was likely inflicted more
by DECENA; UDEs hands were bound than 2 weeks before examination. Vaginal smear
by a towel. revealed no semen.
o After watching for 5 seconds, AGUIRRE Trial court ruled in favor of UDE claiming that her
left and went to the back of her house testimony was credible [gave straightforward,
for air. She was so angry she boloed a consistent, and intelligent answers even if she
banana plant. She did not use the bolo was traumatized and accused exercised some
on DECENA because she was afraid kind of moral ascendancy on her]. The defense of
DECENA might use the scythe he was alibi was rejected by the trial court because the
holding on UDEs neck. house of TUBAO was only 150 meters from the
o AGUIRRE confronted DECENA why he house of AGUIRRE, where the rape took place.
did that to UDE. DECENA said that he

50
CRIMINAL LAW REVIEW DIGESTS
JUSTICE ROMEO CALLEJO NOTE: = Callejo Ponente

Trial court convicted DECENA for the crime of Damages: 50K (civil indemnity) + 50K (moral
Rape as defined and penalized under Art. 335 of damages) 50K (exemplary damages is deleted
the RPC, as amended by (Sec 11) of the Death absent aggravating circumstance)
Penalty Law, aggravated by the fact that said
accused is the common-law spouse of the
mother of UDE, the victim. DECENA was PEOPLE V. ELYBOY SO
sentenced to suffer DEATH. And ordered to
indemnify UDE w/ 50K for actual damages and FACTS:
50K for exemplary damages.
On June 10, 1991, Elyboy SO was charged with
The case is brought before the Supreme Court
murder before the RTC-Manila for the death of
on automatic review.
Mario TUQUERO. The information reads:
That on or about June 3, 1991, in the
ISSUE: w/n trial court erred in imposing the death penalty
City of Manila, Philippines, the said
considering that the supposed qualifying circumstances
accused did then and there willfully,
were not alleged in the information? YES. He should be
unlawfully and feloniously, with intent to
convicted only of simple rape. Penalty of death reduced to
kill and with treachery and evident
reclusion perpetua.
premeditation, attack, assault and use
DECENA claims that the qualifying
personal violence upon the person of
circumstances that a girl should be eighteen
one Mario Tuquero y Alas by then and
years of age and that the offender is "the
there stabbing him several times with a
common-law spouse of the parent of the victim"
fan knife on different parts of his body,
were not alleged in the information. In his reply
thereby inflicting upon said Mario
brief, DECENA reiterates his plea for reduction of
Tuquero Y Alas mortal wounds which
the penalty for the reason that the information
were the direct and immediate cause of
charges only simple rape.
his death thereafter.
This Court has ruled that the circumstances Contrary to law.
under the amendatory provisions of Section 11 of
Republic Act 7659, the attendance of any of SO pleaded not guilty.
which mandates the single indivisible penalty of The prosecution established the following facts:
death, are in the nature of qualifying o On June 2, 1991, at around 9PM, SO
circumstances which cannot be proved as such met his lady friend, Teresita DOMINGO,
unless alleged with particularity in the information in a jeep in Quiapo bound for Pasig.
unlike ordinary aggravating circumstances which Since SOs house is walking distance to
affect only the period of the penalty and which DOMINGOs house, DOMINGO
may be proven even if not alleged in the requested SO to bring her home.
information. o While walking on their way to
It would be a denial of the right of the accused to DOMINGOs house, they passed the
be informed of the charge against him and house of SOs first cousins (Estbean,
consequently, a denial of due process, if he is Edgar, and Emy). SO saw his cousin
charged with simple rape and will be convicted of Edgar with Ronnie Tan and 3 others and
its qualified form punishable by death although noticed that a drinking spree was taking
the attendant circumstance qualifying the offense place.
and resulting in capital punishment was not o Upon seeing SO, Edgar greeted him by
alleged in the indictment under which he was saying that Bingbong Crisologo is
arraigned. coming and then invited SO to drink and
Procedurally, then, while the minority of UDE and requested that SO introduce DOMINGO.
her relationship to the DECENA were established SO answered that he cannot introduce
during the trial, DECENA can only be convicted DOMINGO because she is his.
of simple rape because he cannot be punished o SO proceeded to bring DOMINGO
for a graver offense than that with which he was directly to her house. After bringing
charged. Accordingly, the imposable penalty DOMINGO home, SO passed by his
is reclusion perpetua. cousins house to honor their invitation.
o After an exchange of pleasantries,
Edgar offered SO a bottle of beer. SO
declined because it was already passed

51
CRIMINAL LAW REVIEW DIGESTS
JUSTICE ROMEO CALLEJO NOTE: = Callejo Ponente

10PM and he was on his way home. o SO stayed in the alley for 30 minutes
Edgar convinced SO to drink a little and until the policemen arrived. SO
stay awhile so that SO could also meet surrendered.
Edgars future brother-in-law TUQUERO o As a result of the stabbing incident,
who was arriving with Emy. SO decided TUQUERO suffered 18 stab wounds on
to stay. different parts of his body, with at least 4
o Soon after, Emy and TUQUERO fatal wounds causing his death.
arrived. TUQUERO was a manager of a SO claimed self-defense alleging that it was his
restaurant in Paris and arrived in the cousins who started the fight.
Philippines on March 7, 1991. Emy, a
registered nurse, met TUQUERO
The RTC convicted SO for MURDER qualified by
treachery and sentenced him to reclusion
sometime in March 1991 and they
perpetua.
started living in as husband and wife at
her parents house. TUQUERO, SO appeals.
unknown to Emy, was legally married to
a certain Evelyn Tuquero. ISSUE/S: w/n RTC erred in disregarding his claim of self-
o The group, consisting of Esteben, defense? NO.
Edgar, SO, Ronnie, TUQUERO, and w/n RTC erred in finding that there was
Emy resumed their drinking spree. treachery? NO.
o After the group consumed 4 cases of w/n RTC erred in disregarding the exempting
beer and before 3AM, Emy went inside circumstance of insanity? NO
the house to sleep. While Emy was
sleeping, she was awakened by noise Self Defense:
coming from the group outside. It turned SO maintains that he stabbed the victim in
out SO had a misunderstanding and legitimate self-defense and invokes in his favor
altercation w/ somebody and he was the constitutional presumption of innocence
shouting loudly. claiming that, despite his plea of self-defense, the
o After pacifying the protagonists, prosecution retains the burden of proving his guilt
TUQUERO advised SO to go home beyond reasonable doubt. This argument
because SOs voice was disturbing the deserves no credit in light of the established and
neighbors. SO ran towards home. time-honored rule that when self-defense is
o At around 4:00AM of June 3, invoked, the burden of evidence shifts to the
TUQUERO and Emy decided to leave appellant to show that the killing was justified and
for Fairview, Quezon City to get the that he incurred no criminal liability therefor. He
papers of a vehicle owned by must rely on the strength of his own evidence
TUQUERO which is being held by the and not on the weakness of the prosecution's
Bureau of Customs. evidence, for, even if the latter were weak, it
o While TUQUERO and Emy were waiting could not be disbelieved after his open admission
for a taxi, SO suddenly appeared from of responsibility for the killing.
behind, and stabbed TUQUERO at the
back several times with an 11-in fan
He must prove the essential requisites of self-
defense, to wit: (a) unlawful aggression on the
knife with a white handle. Emy shouted
part of the victim, (b) reasonable necessity of the
for help.
means employed to repel the aggression, and (c)
o When TUQUERO was about to run, he
lack of sufficient provocation on the part of the
slid and fell to the ground lying on his
accused.
back. SO took advantage of the
situation and repeatedly stabbed The initial and crucial point of inquiry is
TUQUERO on the front part of his body. whether there was unlawful aggression on the
Emy pleaded to SO to stop stabbing part of the victim for absent this essential
TUQUERO but SO ignored her and element, no claim of self-defense can be
continued stabbing TUQUERO. successfully interposed. If there is no unlawful
o SO fled from the crime scene and ran to aggression, there is nothing to prevent or to repel
a dark alley. Emy brought TUQUERO at and the second requisite of self-defense would
the UERMM Hospital. have no basis.

52
CRIMINAL LAW REVIEW DIGESTS
JUSTICE ROMEO CALLEJO NOTE: = Callejo Ponente

SOs claim that TUQUERO attacked him with a The testimony of Dr. Omer Galvez, Chief of the
knife fails to convince us. The record reveals Child & Adolescent Service of the National
glaring and serious inconsistencies in SOs Center For Mental Health (NCMH) and attending
testimony that makes it totally unworthy of physician of SO when he was confined at the
credence. SO testified that he was able to wrest National Center for Mental Health from June 8,
the knife from TUQUERO because the latter's 1985 to December 2, 1985, only established the
thrust was slow. However, this contradicts his previous confinement of appellant at the NCMH
statement during the same cross-examination, and that appellant showed signs of psychosis or
that the incident happened so fast and that insanity at the time. The rest of his testimony
TUQUEROs attack was sudden. consisted merely of assumptions, possibilities,
Even if we allow SOs contention that TUQUERO and generalities.
was the initial unlawful aggressor, we still cannot A perusal of SOs testimony further negates his
sustain his plea of self-defense. After SO plea of insanity. SOs recall of the events that
successfully wrested the knife from TUQUERO, transpired before, during and after the stabbing
the unlawful aggression had ceased. After the incident, as well as the nature and contents of his
unlawful aggression has ceased, the one making testimony, does not betray an aberrant mind. His
the defense has no more right to kill or even memory conveniently blanks out only as to the
wound the former aggressor. number of wounds he inflicted on the victim. This,
Appellant's claim of self-defense is, likewise, appellant attributes to insanity but we are far from
contradicted and negated by the physical convinced. A man may act crazy but it does not
evidence on record. The victim sustained necessarily and conclusively prove that he is
eighteen (18) stab wounds on different parts of legally so.
his body. Of the eighteen (18), four (4) were fatal The presence of his reasoning faculties, which
stab wounds. The presence of a large number of enabled him to exercise sound judgment and
wounds on the part of the victim, their nature and satisfactorily articulate the aforesaid matters,
location disprove self-defense and instead sufficiently discounts any intimation of insanity of
indicate a determined effort to kill the victim. appellant when he committed the dastardly
felonies. (citing People v. Aquino)
Treachery: In the present case, the defense has failed to
Considering the number and nature of the wounds inflicted adduce sufficient evidence to overthrow the
by appellant on the victim, the testimony of the presumption of sanity. The State, thus, continues
prosecution witness Emy So that appellant unexpectedly its guard against sane murderers who seek to
and suddenly attacked the victim from behind, and the fact escape punishment through a general plea of
that appellant suffered not a single injury, we agree with insanity.
the trial court that the killing was attended by treachery.
This clearly illustrates that appellant, in the commission of
Appeal DISMISSED.
the crime, employed means, methods and form in its
Side Issue: Credibility of Witness [Emy So]:
execution which tended directly, and especially to ensure
We give no credence to appellant's argument. Long
its execution without risk to himself arising from the
settled in criminal jurisprudence is the rule that when the
defense which the victim might make.
issue is one of credibility of witnesses, appellate courts will
generally not disturb the findings of the trial court,
Insanity:
considering that the latter is in a better position to decide
The law presumes every man to be sane. A the question, having heard the witnesses themselves and
person accused of a crime who pleads the observed their deportment and manner of testifying during
exempting circumstance of insanity has the the trial, unless it has plainly overlooked certain facts of
burden of proving it. substance and value that, if considered, might affect the
In order that insanity may be taken as an result of the case.
exempting circumstance, there must be complete In the instant case, although Emy So readily
depreciation of intelligence in the commission of admitted that her relationship with appellant was not
the act or that the accused acted without the close, she explained that it was SO who had ill-feelings
least discernment. Mere abnormality of his against her family and bore a grudge. The defense has not
mental faculties does not exclude imputability. shown such degree of partiality on the part of prosecution
witness Emy So as would cast doubt on her credibility and

53
CRIMINAL LAW REVIEW DIGESTS
JUSTICE ROMEO CALLEJO NOTE: = Callejo Ponente

impeach her testimony, especially when said testimony is overlooks the circumstance that as found by the Court of
"not inherently improbable in itself". Appeals, the challenge was to go out, i.e., to fight outside
The fact alone that the victim was Emy So's the building, it not being logical that the fight should be
live-in partner does not impair her testimony. The Court held inside the office building in the plain view of
has time and again ruled that mere relationship of the subordinate employees. Even applying the rules in
witness to the victim does not automatically impair his duelling cases, it is manifest that an aggression ahead of
credibility and render the testimony less worthy of faith the stipulated time and place for the encounter would
and credit. be unlawful; to hold otherwise would be to sanction
unexpected assaults contrary to all sense of loyalty and
fair play. In the present case, assuming that De la
JUSTO v. COURT OF APPEALS Cuesta accepted the challenge of the accused, the
facts clearly indicate that he was merely on his way
FACTS: Appellant Justo was found guilty of the crime of out to fight the accused when the latter violently lay
assault upon a person in authority. Offended party hands upon him. The acceptance of the challenge did
Nemesio de la Cuesta is a duly appointed district not place on the offended party the burden of
supervisor of the Bureau of Public Schools. De la Cuesta preparing to meet an assault at any time even before
was leaving the office in order to take his meal when he reaching the appointed place for the agreed
saw appellant Justo conversing with Severino Caridad, encounter, and any such aggression was patently
academic supervisor. Appellant Justo requested De la illegal.
Cuesta to go with him (Justo) and Caridad to the office of
the latter. In Caridads office, appellant Justo asked about
the possibility of accommodating Miss Racela as a teacher PEOPLE v MARIVIC GENOSA
in the district of De la Cuesta. Caridad said that there was (January 15, 2004)
no vacancy, except that of the position of shop teacher.
Upon hearing Caridads answer, Appellant Justo sharply FACTS: Accused Marivic Genosa is charged with
addressed De la Cuesta:y Shet, you are a double parricide. She raises the claim of Battered Woman
crosser. One who cannot keep his promise. Syndrome (BWS) which allegedly constitutes self-
Appellant Justo then grabbed a lead paper weight from defense. She suffered battery from her deceased husband
the table of Caridad and challenged the offended party Ben Genosa whenever the latter is drunk. She testified
De la Cuesta to go out. Hence, upon Justos that one year after their marriage, her husband would slap
suggestion, De la Cuesta followed appellant Justo as her, pin her down the bed and sometimes beat her. The
they went out of Caridads office. Before they could go neighbors of the couple have witnessed their frequent
outside and when they were in front of the table of one quarrels. Accused Marivic has also visited doctors and
Carlos Bueno, a clerk in the division office, De la Cuesta psychiatrists during her marriage with deceased every
asked Appellant Justo to put down the paper weight but time she would be beaten by her husband.
instead Appellant Justo grabbed the neck and collar of On the day of the crime, accused was 8-months
the polo shirt of de la Cuesta which was torn. This caused pregnant and was looking for her husband as she was
De la Cuesta to box appellant Justo several times. De la afraid that he was gambling and might het drunk again.
Cuesta raised the claim of self-defense which was upheld Eventually that night, deceased came home from
by the lower courts. Hence, Appellant Justo now claims gambling and was drunk.
that the claim of self-defense by De la Cuesta should not Deceased Ben purportedly nagged accused
have been appreciated by the lower courts because of the Marivic for following him, even challenging her to a fight.
lack of unlawful aggression on his (Justos) part. She allegedly ignored him and instead attended to their
children who were doing their homework. Apparently
ISSUE: Whether or not there was unlawful aggression on disappointed with her reaction, Ben switched off the light
Appellant Justos part despite the alleged fact that there and, with the use of a chopping knife, cut the television
was a mutual agreement to fight. antenna or wire to keep her from watching television.
According to Marivic, Ben was about to attack her so she
HELD: Yes, there was unlawful aggression. Therefore the ran to the bedroom, but he got hold of her hands and
lower courts did not err in upholding the self-defense claim whirled her around. She fell on the side of the bed and
of De la Cuesta. (Appellant Justo loses) screamed for help. Ben left. At this point, Marivic
packed Bens clothes because she wanted him to leave.
RATIO: The argument that the offended party, De la Seeing his packed clothes upon his return home, Ben
Cuesta, cannot claim to have been unlawfully attacked allegedly flew into a rage, dragged Marivic outside of the
because he had accepted the accuseds challenge to fight, bedroom towards a drawer holding her by the neck, and

54
CRIMINAL LAW REVIEW DIGESTS
JUSTICE ROMEO CALLEJO NOTE: = Callejo Ponente

told her You might as well be killed so nobody would nag However, the techniques adopted by the woman in
me. Marivic testified that she was aware that there was a her effort to placate him are not usually successful, and
gun inside the drawer but since Ben did not have the key the verbal and/or physical abuse worsens. Each partner
to it, Ben got a three-inch long blade cutter from his senses the imminent loss of control and the growing
wallet. She however, smashed the arm of Ben with a tension and despair. Exhausted from the persistent
pipe, causing him to drop the blade and his wallet. Marivic stress, the battered woman soon withdraws emotionally.
then smashed Ben at his nape with the pipe as he was But the more she becomes emotionally unavailable, the
about to pick up the blade and his wallet. She thereafter more the batterer becomes angry, oppressive and
ran inside the (childrens) bedroom. abusive. Often, at some unpredictable point, the violence
According to Marivic, she thereafter ended the spirals out of control and leads to an acute battering
life of her husband by shooting him. She supposedly incident.
distorted the drawer where the gun was and shot Ben. The acute battering incident is said to be
After the incident, Marivic and the children locked the characterized by brutality, destructiveness and,
house and left. The lifeless body of Ben was discovered sometimes, death. The battered woman deems this
by the neighbors due to the awful smell which emanated incident as unpredictable, yet also inevitable. During this
from the couples house. phase, she has no control; only the batterer may put an
Accused Marivic raised the claim of self-defense end to the violence. Its nature can be as unpredictable as
for her life and defense of her unborn child. the time of its explosion, and so are his reasons for ending
it. The battered woman usually realizes that she cannot
ISSUES: reason with him, and that resistance would only
1. Whether or not there was a valid self-defense. exacerbate her condition.
NO At this stage, she has a sense of detachment from
2. Whether or not Marivic is entitled to any the attack and the terrible pain, although she may later
mitigating circumstance. YES (psychological clearly remember every detail. Her apparent passivity in
paralysis and passion and obfuscation) the face of acute violence may be rationalized thus: the
batterer is almost always much stronger physically, and
HELD: A battered woman has been defined as a woman she knows from her past painful experience that it is futile
who is repeatedly subjected to any forceful physical or to fight back. Acute battering incidents are often very
psychological behavior by a man in order to coerce her to savage and out of control.
do something he wants her to do without concern for her The final phase of the cycle of violence begins when
rights. Battered women include wives or women in any the acute battering incident ends. During this tranquil
form of intimate relationship with men. Furthermore, in period, the couple experience profound relief. On the one
order to be classified as a battered woman, the couple hand, the batterer may show a tender and nurturing
must go through the battering cycle at least twice. behavior towards his partner. He knows that he has been
Any woman may find herself in an abusive relationship viciously cruel and tries to make up for it, begging for her
with a man once. If it occurs a second time, and she forgiveness and promising never to beat her again. On
remains in the situation, she is defined as a battered the other hand, the battered woman also tries to convince
woman. herself that the battery will never happen again; that her
More graphically, the battered woman syndrome is partner will change for the better; and that this good,
characterized by the so-called cycle of violence, which gentle and caring man is the real person whom she loves.
has three phases: (1) the tension-building phase; (2) the In the instant case, we meticulously scoured the
acute battering incident; and (3) the tranquil, loving (or, at records for specific evidence establishing that appellant,
least, nonviolent) phase. due to the repeated abuse she had suffered from her
During the tension-building phase, minor battering spouse over a long period of time, became afflicted with
occurs -- it could be verbal or slight physical abuse or the battered woman syndrome. We, however, failed to
another form of hostile behavior. The woman usually tries find sufficient evidence that would support such a
to pacify the batterer through a show of kind, nurturing conclusion. More specifically, we failed to find ample
behavior; or by simply staying out of his way. What evidence that would confirm the presence of the essential
actually happens is that she allows herself to be abused in characteristics of BWS.
ways that, to her, are comparatively minor. All she wants The defense fell short of proving all three phases
is to prevent the escalation of the violence exhibited by the of the cycle of violence supposedly characterizing
batterer. This wish, however, proves to be double-edged, the relationship of Ben and Marivic Genosa. No doubt
because her placatory and passive behavior legitimizes there were acute battering incidents. In relating to the
his belief that he has the right to abuse her in the first court a quo how the fatal incident that led to the death of
place. Ben started, Marivic perfectly described the tension-

55
CRIMINAL LAW REVIEW DIGESTS
JUSTICE ROMEO CALLEJO NOTE: = Callejo Ponente

building phase of the cycle. She was able to explain in there was a sufficient time interval between the unlawful
adequate detail the typical characteristics of this stage. aggression of Ben and her fatal attack upon him. She had
However, that single incident does not prove the existence already been able to withdraw from his violent
of the syndrome. In other words, she failed to prove that behavior and escape to their childrens bedroom.
in at least another battering episode in the past, she During that time, he apparently ceased his attack and
had gone through a similar pattern. went to bed. The reality or even the imminence of the
How did the tension between the partners usually danger he posed had ended altogether. He was no longer
arise or build up prior to acute battering? How did Marivic in a position that presented an actual threat on her life or
normally respond to Bens relatively minor abuses? What safety.
means did she employ to try to prevent the situation from Had Ben still been awaiting Marivic when she came
developing into the next (more violent) stage? out of their childrens bedroom -- and based on past
Neither did appellant proffer sufficient evidence in violent incidents, there was a great probability that he
regard to the third phase of the cycle. She simply would still have pursued her and inflicted graver harm --
mentioned that she would usually run away to her then, the imminence of the real threat upon her life would
mothers or fathers house; that Ben would seek her out, not have ceased yet. Impending danger (based on the
ask for her forgiveness and promise to change; and that conduct of the victim in previous battering episodes) prior
believing his words, she would return to their common to the defendants use of deadly force must be shown.
abode. Threatening behavior or communication can satisfy the
In sum, the defense failed to elicit from appellant required imminence of danger. Considering such
herself her factual experiences and thoughts that would circumstances and the existence of BWS, self-defense
clearly and fully demonstrate the essential characteristics may be appreciated.
of the syndrome. We reiterate the principle that aggression, if not
The Court appreciates the ratiocinations given by the continuous, does not warrant self-defense. In the
expert witnesses for the defense. Indeed, they were able absence of such aggression, there can be no self-
to explain fully, albeit merely theoretically and defense -- complete or incomplete -- on the part of the
scientifically, how the personality of the battered woman victim. Thus, Marivics killing of Ben was not completely
usually evolved or deteriorated as a result of repeated and justified under the circumstances.
severe beatings inflicted upon her by her partner or
spouse. They corroborated each others testimonies, Mitigating Circumstances
which were culled from their numerous studies of It should be clarified that these two circumstances --
hundreds of actual cases. However, they failed to present psychological paralysis as well as passion and obfuscation
in court the factual experiences and thoughts that -- did not arise from the same set of facts.
appellant had related to them -- if at all -- based on which On the one hand, the first circumstance arose from
they concluded that she had BWS. the cyclical nature and the severity of the battery inflicted
by the batterer-spouse upon appellant. That is, the
BWS as Self-Defense repeated beatings over a period of time resulted in her
In any event, the existence of the syndrome in a psychological paralysis, which was analogous to an illness
relationship does not in itself establish the legal right of the diminishing the exercise of her will power without depriving
woman to kill her abusive partner. Evidence must still be her of consciousness of her acts.
considered in the context of self-defense. The second circumstance, on the other hand,
From the expert opinions discussed earlier, the Court resulted from the violent aggression he had inflicted on her
reckons further that crucial to the BWS defense is the prior to the killing. That the incident occurred when she
state of mind of the battered woman at the time of the was eight months pregnant with their child was deemed by
offense -- she must have actually feared imminent harm her as an attempt not only on her life, but likewise on that
from her batterer and honestly believed in the need to kill of their unborn child. Such perception naturally produced
him in order to save her life. passion and obfuscation on her part.
Settled in our jurisprudence, however, is the rule that
the one who resorts to self-defense must face a real Epilogue
threat on ones life; and the peril sought to be avoided We now sum up our main points. First, each of the
must beimminent and actual, not merely imaginary. phases of the cycle of violence must be proven to have
Unlawful aggression is the most essential element of characterized at least two battering episodes between the
self-defense. It presupposes actual, sudden and appellant and her intimate partner. Second, the final
unexpected attack -- or an imminent danger thereof -- on acute battering episode preceding the killing of the
the life or safety of a person. In the present case, batterer must have produced in the battered persons mind
however, according to the testimony of Marivic herself, an actual fear of an imminent harm from her batterer and

56
CRIMINAL LAW REVIEW DIGESTS
JUSTICE ROMEO CALLEJO NOTE: = Callejo Ponente

an honest belief that she needed to use force in order to beforehand to deliberate on her acts and to choose a less
save her life. Third, at the time of the killing, the batterer fatal means of eliminating her sufferings.
must have posed probable -- not necessarily immediate Appellant further alleges that the syndrome is
and actual -- grave harm to the accused, based on the already a recognized form of self-defense in the United
history of violence perpetrated by the former against the States and in Europe. In the US particularly, it is classified
latter. Taken altogether, these circumstances could as a post-traumatic stress disorder, rather than a form of
satisfy the requisites of self-defense. Under the existing mental illness. It has been held admissible in order to
facts of the present case, however, not all of these assess a defendant's perception of the danger posed by
elements were duly established. the abuser.
In view of the foregoing, Appellant Genosa pleads
PEOPLE v MARIVIC GENOSA that she be allowed to present evidence to prove that her
( GR No. 135981 September 29, 2000) relationship with her spouse-victim had afflicted her with
the syndrome. Allegedly, an expert can explain how her
FACTS: This case involved the "battered woman experiences as a battered woman had affected her
syndrome," which is alleged to be equivalent to self- perception of danger and her honest belief in its
defense. RTC found appellant guilty of parricide imminence, and why she had resorted to force against her
aggravated by treachery. Appellant has admitted the fact batterer.
of killing her husband and the acts of hitting his nape with Moreover, proof of insanity could have exempted
a metal pipe and of shooting him at the back of his head, appellant from criminal liability. If the accused had not
which of said acts actually caused the victim's death. performed the act voluntarily, then he could not have been
criminally liable. In the instant case, it is equally important
ISSUE: Whether the appellant should be examined by to determine whether Appellant Genosa had acted freely,
qualified psychologists or psychiatrists in order to intelligently and voluntarily when she killed her
determine her state of mind at the time of the killing. spouse. The Court, however, cannot properly evaluate her
battered-woman-syndrome defense, absent expert
HELD: Yes. The case is hereby REMANDED to the trial testimony on her mental and emotional state at the time of
court for the reception of expert psychological and/or the killing and the possible psychological cause and effect
psychiatric opinion on the "battered woman syndrome" of her fatal act.
plea.

RATIO: In seeking to be "examined and evaluated by PEOPLE V. FLORES Y PARAS, G.R. No. 177355, 2010
psychologists and psychiatrists to bring into evidence the
abuse inflicted upon her; [and] to determine whether such Facts: Flores was charged with qualified rape. AAA was
abuse will support the 'battered woman syndrome'," she the name of the 13 year-old victim (under RA 9262 VAW-
asks the Court to "re-evaluate the traditional elements" C, the names of women and child victims are withheld.)
used in determining self-defense and to consider the That the crime of rape was committed with the qualifying
"battered woman syndrome" as a viable plea within the circumstances of victim being under 18 years of age, the
concept of self-defense. accused is her stepfather, being the common-law spouse
Allegedly, there are four characteristics of the of her mother (BBB in this case), and that the rape was
syndrome: (1) the woman believes that the violence was committed in full view of the victims mother.
her fault; (2) she has an inability to place the responsibility Rape details: fateful evening of July 18, 2001, at
for the violence elsewhere; (3) she fears for her life and/or around eight oclock, Flores ordered her to ask her
her children's lives; and (4) she has an irrational belief that daughter AAA to sleep with them. Both AAA and BBB
the abuser is omnipresent and omniscient. Living in obeyed Flores for fear of his wrath. At around ten oclock
constant danger of harm or death, she knows that future in the evening, BBB was awakened by the pinch of her
beatings are almost certain to occur and will escalate over daughter, BBB was then shocked to see that Flores was
time. Her intimate knowledge of the violent nature of her already on top of her daughter, who was shouting Aray,
batterer makes her alert to when a particular attack is Aray, Nanay, Aray. She felt angry but could not do
forthcoming, and when it will seriously threaten her anything because Flores not only had a bladed weapon
survival. Trapped in a cycle of violence and constant fear, poked at her neck, but he also threatened to kill her if she
it is not unlikely that she would succumb to her shouted. BBB endured this horrifying episode for the next
helplessness and fail to perceive possible solutions to the thirty minutes. Sweetheart defense of Flores was stricken
problem other than to injure or kill her batterer. She is down by court.
seized by fear of an existing or impending lethal
aggression and thus would have no opportunity

57
CRIMINAL LAW REVIEW DIGESTS
JUSTICE ROMEO CALLEJO NOTE: = Callejo Ponente

Issue: Death penalty properly meted? Considering the age According to Bautista, the defendant waited on
of the victim at time of rape incident was not sufficiently the right side of the path near some guava trees and
established? stabbed Francisco Rivera with a knife in her right hand
when he arrived in front of her; that the injured man cried
Held: Yes. But Age not properly proven. In the case at "Aruy, Dios mio", while the defendant turned around and
bar, not only did the prosecution fail to present AAAs birth returned to the house of Maria Inguit, saying "Icao ay
certificate, but BBB, the victims mother herself, gave malaon na" (hacia tiempo ya)
contradictory statements on the true age of her daughter.
At one time she said that AAA was 13 years old, and yet Defendant-appelants version:
when asked about the year of AAAs birthday, she After they reached a narrow part a man suddenly threw his
declared that it was 1982. STILL, Flores cannot escape arms around her from behind, caught hold of her breasts
the penalty of death. Flores forgot the important fact that and kissed her, and seized her in her private parts; that
aside from AAAs minority, the qualifying circumstance she tried to free herself, but he held her and tried to throw
that the rape was committed in full view of AAAs mother her down; that when she felt weak and could do nothing
was also alleged in the Information. more against the strength of the man, she got a knife from
BUT (again!) despite this, the 2006 law her pocket, opened it, and stabbed him in defense of her
abolishing the death penalty in effect reduced his honor. She further testified that the man who attacked her
sentence to reclusion perpetua. did not say anything; that she asked him who he was but
he did not answer; that when she was assaulted she cried
for help, saying "Madre mia; Dios mio"; that when she was
PP VS GATUA seized, she was about two brazas behind her nearest
companion; that when she was face to face with her
assailant during the struggle she could scarcely recognize
his face in the darkness and could not be sure that it was
PEOPLE V DELA Cruz (1935 case)
Francisco Rivera.
She further testified that she was engaged in
1 braza is a measure of six feet
selling fruit, and that the fanknife in question was in a
Francisco Ramos, his wife along with Ramos and Santoyo
pocket of the overcoat she was wearing that day; that she
(4 persons) went to the house of Remedios Dela Cruz
went off with her friends without having an opportunity of
(defendant) and asked her to join the wake of Sion. The
changing her clothes.
defendant and her friends started to walk on their way to
their respective houses at around 9 P.M. They were later
Issue: Whether appellant is guilty of homicide.
on followed, 5 minutes later, by Francisco
Rivera(deceased-victim) and Bautista. Rivera and Bautista
Held: No. This case is similar to the case of Ah chong. The
overtook defendants party. When they reached a narrow
court held that a person is not criminally responsible when,
part of the path, Rivera went ahead of Bautista. At that
by reason of a mistake of facts, he does an act for which
time the members of the defendant's party were walking in
he would be exempt if the facts were as he supposed
single file and defendant was the hindmost. She was
them to be, but would constitute murder if he had known
about two brazas from the person immediately ahead of
the true state of facts at the time, provided that the
her. Francisco Ramos heard someone cry out "Aruy, Dios
ignorance or mistake of fact was not due to negligence or
mio". He went back and found that Francisco Rivera had
bad faith
been stabbed under the right breast. The wounded man
We cannot believe the testimony of Enrique
was taken to the hospital, where he died the next
Bautista, because Francisco Ramos, one of the witnesses
afternoon.
for the prosecution, testified that it was a dark night, and
Francisco Ramos testified that it took him about
Bautista himself said that he could scarcely see anyone in
two minutes to go back to the place where Rivera was. He
the darkness
found and that Bautista was with the wounded man, and
The appellant is an illiterate barrio girl, unable to
the defendant had started back towards the house of
write her name, and scarcely eighteen years old. We do
mourning. He overtook her. She had a knife in her hand.
not believe her story is a fabrication. In this connection it is
When they reached the house where the wake was being
to be noted that almost immediately after the incident in
held, Remedios de la Cruz stuck the knife into a table and
question took place, the appellant said she stabbed
said that she stabbed Francisco Rivera because he
Francisco Rivera because he embraced her. It is not
embraced her.
improbable that she was reluctant to relate in the presence
of all the people in the house of Maria Inguit (where the
Prosecutions version:
wake was held) the details of what had occurred.

58
CRIMINAL LAW REVIEW DIGESTS
JUSTICE ROMEO CALLEJO NOTE: = Callejo Ponente

The appellant stabbed the deceased only once, On the side:


although she retained possession of the knife, and The fact that she voluntarily surrendered to the lieutenant
undoubtedly could have inflicted other wounds on him if in the chapel and admitted to stabbing the deceased , and
she had desired. In other words she desisted as soon as the fact that she had acted in the immediate vindication of
he released her a grave offense committed against her a few moments
Appelant acquitted. before, and upon such provocation as to produce passion
or temporary loss of reason, should be considered as
mitigating circumstances in her favor.
PEOPLE VS JAURIGUE The aggravating circumstance that the killing was
done in a place dedicated to religious worship cannot be
Facts: Defendant Avelina Jarigue(girl) and appellant legally sustained as there is no evidence to show that
Amado Capino lived in the same barrio. Prior to the defendant had murder in her heart when she entered the
incident at hand, Capino had been courting Jarigue to no chapel. She should therefore be charged with homicide
avail. A month prior to the incident, Capino stole a hanky without aggravating circumstances and with mitigating
belonging to Jarigue bearing her nickname aveling while circumstances.
I was being washed. On another night, Jarigue was
feeding a dog under her house, when Capino approached
her and spoke to her of his love, which she flatly refused, PEOPLE VS. NARVAEZ
and he thereupon suddenly embraced and kissed her and
touched her boobs. She thereafter kept a long fan knife to FACTS: Mamerto Narvaez has been convicted of murder
protect herself. A few days later, Capino climbed up the (qualified by treachery) of David Fleischer and Flaviano
house of Jarigue and entered the room where she was Rubia. Narvaez shot Fleischer and Rubia when the two
sleeping. He felt her forehead with the intention of raping were constructing a fence (that would
her. She immediately screamed for help, which awakened prevent Narvaez from getting into his house and rice mill).
her parents and brought them to her side. Capino then Narvaez, who was taking a nap when he heard sounds of
came out from where he was hiding, under the bed, and construction, awoke and asked them to stop destroying his
kissed the hand of Jarigues father to beg for forgiveness. house and asking if they could talk things over. Fleischer
Several days later on the fateful night, her family went to responded with "No, gadamit, proceed, go ahead."
the local church where it was quite bright. When Jarigue Narvaez lost his "equilibrium," and shot Fleisher first, then
was left alone in the bench while her father tended to Rubia, who was running towards the jeep to get his gun.
some business, Capino sat beside Jarigue and placed his Both died.
hand on top of her thigh. On observing this highly Narvaez voluntarily surrendered and claimed he
improper conduct, Jaurigue stabbed Capino in the neck, killed in defense of his person and of his rights. The CFI
fatally causing a single wound from which he died. convicted him of murder qualified by treachery with the
Jaurigue surrendered without question. aggravating circumstance of evident premediation and the
mitigating circumstance of voluntary surrender.
Issue: WON defendant acted in the legitimate defense of Prior to the shooting, Fleischer and Co. (the
her honor and should be completely absolved from all company of Fleischer's family) was involved in a legal
criminal liability. battle with the Narvaez over certain pieces of property. At
the time of the shooting, the civil case was still pending for
Side issue: WON there were mitigating and aggravating annulment (settlers wanted granting of property to Fleisher
circumstances. and Co. to be annulled). Narvaez had leased his property
from Fleisher (though case pending and ownership
Held: She is not absolved from criminal liability. If the uncertain) to avoid trouble. On June 25, Narvaez received
defendant had killed Capino when he climbed up her a letter terminating the contract because he allegedly
house to rape her, she could have been perfectly justified didn't pay rent. He was given 6 months to remove his
in killing him. However, when the deceased sat beside house from the land. Shooting was barely 2 months after
defendant on the same bench in a well lit chapel with letter.
several people inside, including her own father and the
barrio lieutenant where there is no possibility of being ISSUE: W/N CFI erred in convicting Narvaez despite the
raped. She cannot be legally declared completely exempt fact that he was acting in defense of his person and of his
from criminal liability for fatally wounding the deceased rights.
since the means employer by her in the defense of her
honor was evidently excessive. HELD/RATIO: YES (with respect to rights) Narvaez
ordered released immediately coz his penalty was

59
CRIMINAL LAW REVIEW DIGESTS
JUSTICE ROMEO CALLEJO NOTE: = Callejo Ponente

reduced and he had already served it as he was position to defend himself. Voluntary surrender is not
imprisoned for 14 years. appreciated even if the accused submits himself to the
members of the barangay tanod who, by their presence in
Defense of his person - NO his house, precluded his escape)
The courts said that although the fencing of Narvaez
house was indeed a form of aggression against him, this Facts: The victim, Jessie Lacson and Edwin Velasco were
aggression was not done against his person but rather on gathering shells by the seashore. Because they were
his rights to property. However, in consideration of the thirsty, they went to the fishpond to get young coconuts.
violation of property rights, the courts referred to Art. 30 of The caretaker of said fishpond was Ignacio, who
the civil code, which recognizes the right of owners to sometimes stayed in the house located therein.
close and fence their land. But the Narvaez cant Jessie got a coconut. Ignacio shouted at him to
subscribe to the article because his ownership of the land put it down, which he did. Edwin, whom Ignacio did not
being awarded by the government was still pending, see, saw Ignacio fire his homemade gun at Jessie who
therefore putting ownership into question. It is accepted was hit on the left breast (yes, he died). At that time,
that Narvaez Iganacio was 40 meters away from Jessie while Edwin
was 6 meters away. Then Ignacio cranked his gun and
Defense of his rights YES (although incomplete) aimed at Edwin but did not fire. Edwin reported the
The argument of the justifying circumstance of self- shooting to the Barangay Tanod, who went to Ignacios
defense is applicable only if the 3 requirements are house (since he wasnt there, they waited for him to
fulfilled, according to Art. 11(1) RPC: arrive). Upon arrival, the latter was asked to and did
1. Unlawful aggression. surrender. When asked why he fired his gun at Jessie, he
2. Reasonable necessity of means employed to replied that Jessie stole some young coconut.
prevent or repel attack. Ignacios version was that he saw the two boys
3. Lack of sufficient provocation on part of coming out of his house with a basket. It so happened
person defending himself. that there were 28 pieces of crabs stocked in there. Since
Unlawful aggression due to the utterance of the 2 did not stop upon his request, he fired his gun at
Fleischer and the invasion of Narvaezs property was them (50 meters away) without intention to kill. He left and
clear. The pending case regarding ownership was decided informed the Kagawad about the incident and reported it
only over a year after the incident, and even then, to the Barangay Tanod.
Fleischer had given Narvaez until the end of the year to The RTC held that Ignacio failed to prove that he
leave the land. Lack of sufficient provocation was clear acted in lawful defense of the landowners property. There
because Narvaez was asleep in his house, then asked was no legal reason for him to shoot the victim, and
Fleischer to stop so they could talk. Firing a shotgun from unarmed minor at that time. The court qualified the killing
a window, however, was a disproportionate means of to murder because of the presence of treachery.
resistance. Ignacio appealed, alleging that the RTC erred in
Since not all requisites present, defendant is finding the qualifying circumstance of treachery attendant
credited w/ the special mitigating circumstance of in the case and in not appreciating the mitigating
incomplete defense, pursuant to A13(6) RPC. These circumstance of voluntary surrender. He said he merely
mitigating circumstances are: voluntary surrender & acted on impulse to stop them from fleeing and did not
passion & obfuscation. intend to kill anyone.

DISSENTS: Issue: W/N he is guilty of murder YES


Abad Santos: Self-defense in the penal code refers to
unlawful aggression on persons and not property. Ratio: The testimony of Edwin disproves his claim. They
Gutierrez, Jr.: Appellant defended from an attack on his did stop after Ignacio shouted at them. In fact, they were
property that was not coupled with an attack on his already facing him when he fired the shot. This was
person. There should be no special mitigating affirmed by the forensic guy who testified that the entry
circumstance of incomplete defense. The sentence should point of the bullet was at the chest and not at the back.
have been modified to prision mayor and the defendant This clearly shows that Ignacio killed the victim and did so
should have been ordered released immediately. without risk to himself. A killing is qualified by treachery
when the accused employs means, methods or forms of
execution therefore without risk to himself arising from the
PEOPLE v. IGNACIO defense which the offended party make. In this case,
(There is treachery when the accused unexpectedly and there was no risk to Ignacio because Jessie was only 14
deliberately shoots an unarmed minor who is not in a years old and unarmed.

60
CRIMINAL LAW REVIEW DIGESTS
JUSTICE ROMEO CALLEJO NOTE: = Callejo Ponente

With respect to voluntary surrender, the SC did intention as well as in the act of the person making the
not appreciate this. For a mitigating circumstance to be defense.
appreciated, 3 requisites must be satisfied: 1) the offender
has not actually been arrested, 2) the offender surrenders
himself to a person in authority or the latters agent, and 3) PEOPLE V. CHUA HIONG
the surrender is voluntary. The defense must show an
intent to surrender unconditionally to the authorities, Doctrine: Self-defense is also available in libel cases
because of an acknowledgment of guilt or because of a
wish to spare them the trouble and the expense Facts: Federico Chua Hiong is the uncle of Cesareo
concomitant to the search and capture of the accused. Gacheco. Gacheco and his family were defeated in a civil
Ignacions surrender was not voluntary; rather, he was case in the CFI of Manila, which, if not overturned by the
forced to give himself up because the members of the SC, would lead to Gacheco and co. losing 2/3s of the
barnagay tanod were already inside his house, thereby inheritance left by a Paulino Gacheco.
precluding his escape. Hiong sided with the party that defeated
Gacheco. This created tension and Gacheco wrote the
Chief Finance Agent of the Department of Finance
US V. AH CHONG charging Hiong with tax evasion and using a fake
citizenship. He then wrote a letter to Vice-President
Facts: Ah Chong was a cook in Ft. McKinley. He was Fernando Lopez accusing Hiong of illegal transactions
afraid of bad elements. One evening, before going to bed, with the government.
he locked himself in his room by placing a chair against A letter was written by a certain Benito Solipco to
the door. After having gone to bed, he was awakened by Hiong. (The SC says Solipco was undoubtedly if not
someone trying to open the door. He called out twice, Gacheco himself, acting under Gachecos inducement.) It
Who is there, but received no answer. Fearing that the said that the members of the Go Family Association, of
intruder was a robber, he leaped from his bed & called out which Gocheco belonged, told Solipco that they will make
again, If you enter the room I will kill you. But at that every vengeance against Hiong, such as paying some
precise moment, he was struck by the chair that had been persons to kill him, or reporting him to every Philippine
placed against the door, & believing that he was being Government Authority that he is a communist and other
attacked he seized a kitchen knife & struck & fatally kinds of vengeance. The letter warned Hiong to be careful
wounded the intruder who turned out to be his roommate. as the Go Family wee all his enemies now and that they
Trial Court convicted him of homicide. will make every vengeance against him at all cost. The
letter was contained in an envelope along with a rope
Issue: W/n Ah Chong was liable for the death of his which contained a note saying this serves for your
roommate. personal use. Hiong received threats on the phone and
was denounced as a communist through anonymous
Held: NO. Ah Chong must be acquitted because of letters.
mistake of fact. Gocheco then caused to be published articles
entitled Doubtful Citizenship in the Feb 11, 1952 issue of
Ratio: Had the facts been as Ah Chong believed them to the Manila Chronicle. It said that while the Commissioner
be, he would have been justified in killing the intruder of Immigration had certain evidences supporting the
under Article 11, par. 1, of the RPC, which requires, to Filipino citizenship of Hiong, the Commisisoners decision
justify the act, that there be: was based on questionable proofs. It then proceeded to
1. Unlawful aggression on the part of the person killed, enumerate the evidences such as:
2. Reasonable necessity of the means employed to
prevent or repel it, and 1. Mr. Frederico M. Chua Hiong and his amily, as
3. Lack of sufficient provocation on the part of the person shown, by the Master List of alien registered in
defending himself. 1941 with the Bureau of Immigration, were
If the intruder was really a robber, forcing his way registered under reg no.s. 199-461 to 199466.
into the room of Ah Chong, there would have been 3. The proceedings of the Board of Special Inquiry
unlawful aggression on the part of the intruder. There at the Port of Manila, under Chinese Board
would have been a necessity on the part of Ah Chong to Report No. 1451, show that Mr. Chua Hiong was
defend himself and/or his home. The knife would have admitted into the country as legitimate minor son
been a reasonable means to prevent or repel such of Chua Pe on September 23, 1913.
aggression. And Ah Chong gave no provocation at all. 4. A certified Chinese Marriage Certificate secured
Under A11 of the RPC, there is nothing unlawful in the from the local Civil Registrar shows tha his

61
CRIMINAL LAW REVIEW DIGESTS
JUSTICE ROMEO CALLEJO NOTE: = Callejo Ponente

marriage was performed by the Chinese Consul Issue: Whether or not Hiongs libelous publication was a
at the Chinese YMCA in 1926. proper act of self-defense in relation to Gochecos earlier
5. Affidavits sworn to by residents of Aparri, Doubtful Citizenship article.
Cagayan, the place where the allaged mother (of
Hiong) lives, and submitted by the Chief of Police Held: Yes. Self-defense applies to the crime of libel.
at the instance of the investigator in this case,
show that the alleged mother has never left Ratio: Self-defense is a mans inborn right. In a physical
Aparri, much less the Philippines, and therefore assault, retaliation becomes unlawful after the attack has
could not give birth to Hiong who was born in ceased, because there would be no further harm to repel.
China. But that is not the case when it is aimed at a persons
In response, Hiong caused Seriously Speaking to good name. Once the aspersion is cast its sting clings and
be published in the Manila Chronicle. It said: the one thus defamed may avail himself of all necessary
This investigation was only one of a series of means to shake it off. He may hit back with another libel
other investigations conducted by different agencies of our which, if adequate, will be justified.
government at the instigation of Mr. Gocheco, who Granting that the Seriously Speaking column of
appears to be obsessed with a persecution mania in order the Manila Chronicle caused by Hiong was libelous, is it
to besmirch my name and reputation and harass me and unnecessarily libelous? It was intended to counteract the
my family. impression left in the mind of the public by the article
To my eternal shame and misfortune, Mr. Doubtful Citizenship which Gocheco caused to be
Cesario T. Gocheco is my nephew. As such, he is published in the Manila Chronicle on Feb. 11, 1952.
cognizant of all of the facts of my life for he has known me Hiong was living as a Filipino, his livelihood
for the past 25 years.Why then this sudden concern depended mainly upon enterprises only Filipinos can
over my citizenship? Why this mad desire to bring harm to engage in. It is perfectly conceivable that any attempt to
me and my family? The reason is not hard to find assail his Filipino citizenship should meet the keenest
personal revenge is the moving passion in this drama of defense from him.
intrigues and persecution to which I and my family have To flout in public the genuineness of ones
been subjected. citizenship is slanderous, nobody would dare deny, the
more so Hiongs case for obvious reasons. The Doubtful
.It is easy to imagine the gloom, despondency and Citizenship column makes it appear that his citizenship
despair, that must have seized the Gocheco family when was acquired through questionable means and that an
the above decision was handed down as that would divest investigation is currently being conducted with respect to
them of everything that they now have and thus face stark the legality of his citizenship. Gochecos purpose was to
poverty. malign Hiong. Because he lost in the civil case, Gocheco
decided to air his grievances through the press.
..It is obvious that the name Benito Sulipco is fictitious, Hiongs Seriously Speaking Column is not
as it is the most natural thing that my enemies should necessarily libelous because Hiong is entitled to show
cowardly hide behind the cloak of anonymity, but, one Gochecos motive behind Doubtful Citizenship and to
need not stretch the imagination too far to be able to dispel the bad impression about him of those who had
guess the mastermind behind these threats. read it.

..For what could be better or more convenient to my


enemies than my untimely death, or for that matter, my PEOPLE VS. BATES
deportation from this country had they been able to prove [penned by J. Austria-Martinez; Justice Callejo has a
their charges filed with the different government agencies. concurring opinion in this case]
What better or more convenient weapon can my enemies
avail of then a this systematic and malicious persecution in FACTS:
order to coerce or cajole me into submitting to their Version of the Prosecution
demands that I should desist from proceeding with the civil Edgar Fuentes, Simon Fuentes and Jose Boholst left
case I have instituted against the Gocheco family which Barangay Esperanza, Ormoc City to deliver copra to a
shall ultimately reduce them to the poverty of the certain Fely Rodado at Barangay Green Valley, Ormoc
proverbial church-mouse? City. After delivering the copra, the three went headed
Because of the article above, Hiong was found back to Barangay Esperanza. As they were heading back,
guilty of libel by the RTC. He now appeals. Carlito Bates suddenly emerged from the thick banana
plantation, aiming his firegun against Boholst. The latter
grabbed Carlitos right hand and elbow and tried to wrest

62
CRIMINAL LAW REVIEW DIGESTS
JUSTICE ROMEO CALLEJO NOTE: = Callejo Ponente

possession of the firearm. While the two were grappling was already lying helpless on the ground. At that moment,
for possession, the gun fired, hitting Carlito who unlawful aggression on the part of Jose had ceased. It is
immediately fell to the ground. At that instant, Marcelo a settled rule that when unlawful aggression ceases, the
Bates (brother) and his son, Marcelo Bates, Jr. (nephew) defender has no longer any right to kill or wound the
emerged from the banana plantation, each brandishing a former aggressor, otherwise, retaliation and not self-
bolo. They immediately attacked Jose, hacking him defense is committed. Hence, the fact that unlawful
several times. Jose fell to the ground but the two kept on aggression on the part of Jose already ceased when
hacking him. Marcelo repeatedly hacked him rules out the possibility of
self-defense, whether complete or incomplete.
Version of the Defense
Ponciano Sano went to the house of Marcelo Bates to get Other Issues:
a chicken. While they were trying to catch a chicken, they The circumstance that an attack was sudden and
noticed Jose, Edgar, and Simon approach the house of unexpected to the person assaulted did not constitute the
Carlito. Thereafter, they saw Jose drag Carlito out of the element of alevosia necessary to raise homicide to
house while both were arguing and grappling. When murder, where it did not appear that the aggressor
Marcelo was about to approach them, Jose shot Carlito consciously adopted such mode of attack to facilitate the
with a gun. Marcelo attacked Jose but the latter also fired perpetration of the killing without risk to himself. Treachery
a shot at him. However, Marcelo was able to duck and cannot be appreciated if the accused did not make any
avoid being shot. Jose was about to shoot Marcelo for the preparation to kill the deceased in such manner as to
second time but the latter retaliated by hacking Jose with a insure the commission of the killing or to make it
bolo hitting him on the neck. impossible or difficult for the person attacked to retaliate or
The RTC of Ormoc City held Marcelo guilty of the defend himself.
crime of murder. There is nothing to indicate from the testimony of
Edgar that Marcelo and his son employed means and
ISSUE: Whether or not Marcelo acted in self-defense??? methods to insure that they will be able to attack Jose
without risk to themselves arising from any defense that
RULING: NO. Jose might make. There is no evidence to show that they
Under Article 11 of the Revised Penal Code, anyone who purposely remained hidden in the thick banana plantation
acts in defense of his person or rights do not incur any awaiting for the opportune time to attack Jose with
criminal liability provided that the following circumstances impunity.
concur: First, unlawful aggression on the part of the Hence, for failure of the prosecution to prove
victim; second, reasonable necessity of the means treachery or any other circumstance which would qualify
employed to prevent or repel it; and third, lack of sufficient the killing of Jose to murder, appellant should only be held
provocation on the part of the person defending himself. It liable for the crime of homicide punishable under Article
is a settled rule that when an accused admits killing the 249 of the Revised Penal Code.
victim but invokes self-defense, it is incumbent upon him Appellant was able to prove the mitigating
to prove by clear and convincing evidence that he acted in circumstance of voluntary surrender, as shown by the
self-defense; and as the burden of the evidence is thus testimony of Barangay Captain Feliseo Sano.
shifted to him, he must rely on the strength of his own Passion and obfuscation may not be properly
evidence and not on the weakness of the prosecution. appreciated in favor of Marcelo. To be considered as a
After scrutiny of the evidence presented, we mitigating circumstance, passion or obfuscation must arise
agree with the trial court that self-defense was not from lawful sentiments and not from a spirit of lawlessness
22
established by Marcelo. He testified that he initially or revenge or from anger and resentment. In the present
inflicted only a single hack wound on the neck of Jose case, clearly, Marcelo was infuriated upon seeing his
causing the latter to fall to the ground. He then went to the brother, Carlito, shot by Jose. However, a distinction must
aid of his brother Carlito but upon finding that he was be made between the first time that Marcelo hacked Jose
already dead, he went back to where Jose fell. Marcelo and the second time that the former hacked the latter.
admitted that at that time, Jose was in a lying position still When Marcelo hacked Jose right after seeing the latter
alive but hardly moving. Under such a situation, Jose shoot at Carlito, and if Marcelo refrained from doing
could have hardly put up any defense, much less, make anything else after that, he could have validly invoked the
an aggressive move against appellant. Despite Joses mitigating circumstance of passion and obfuscation. But
condition, Marcelo repeatedly hacked Jose. Granting that when, upon seeing his brother Carlito dead, Marcelo went
Jose was the one who first committed unlawful back to Jose, who by then was already prostrate on the
aggression, appellant was no longer justified in further ground and hardly moving, hacking Jose again was a
inflicting wounds upon Jose because at that time, the latter

63
CRIMINAL LAW REVIEW DIGESTS
JUSTICE ROMEO CALLEJO NOTE: = Callejo Ponente

clear case of someone acting out of anger in the spirit of unjust judgment.
revenge.
HELD: Under the pari delicto doctrine, where the parties
to a controversy are equally culpable or guilty, they shall
UBARRA V MAPALAD have no action against each other, and it shall leave the
parties where it finds them. This doctrine finds expression
FACTS: In a sworn letter-complaint dated 21 November in the maxims ex dolo malo non oritur actio' and 'in pari
1991 and addressed to then Court Administrator, Josue N. delicto potior est conditio defendentis.The Court found the
Bellosillo (Justice), complainant Atty. Manuel T. Ubarra, on application of the pari delicto theory in a criminal case to
behalf of his client Juanito A. Calderon, charges Judge be strange, to say the least. In the first place, the rule on
Luzviminda Mapalad (Judge), the Presiding Judge of the pari delicto is a rule in civil law. It is principally governed
MTC of Pulilan, Bulacan, with grave misconduct, for by Articles 1411 and 1412 of the Civil Code under the
knowingly rendering an unjust judgment, violation of the Chapter on Void or Inexistent Contracts, and presupposes
Canons of Judicial Ethics and the failure to decide within a situation where the parties are in culpability similarly
the mandated ninety-day period Criminal Case situated. That this rule can by no means apply in a
entitled Peo v.Cruda which involves the charge of Grave criminal ease is evidenced by the aforesaid Article 1411
Threats. There is also another action for Grave Threats which provides in part that "When the nullity proceeds
likewise entitled Peo v Cruda. T from the illegality of the cause or object of the contract,
Calderon is the offended party in the both and the act constitutes a criminal offense, both parties
criminal cases. He alleges in his affidavit that in the course being in pari delicto, they shall have no action against
of the trial of the first criminal case, he noted that accused each other, and both shall be prosecuted." Secondly, in
Roberto Crude worked as a houseboy of the Judge. By view of the broader grounds of public policy, the rule may
that time, he had already observed the Judge's partiality in not be invoked against the State. Thirdly, in the
favor of the said accused. prosecution of public crimes, the complainant is the State
The case was submitted for decision on 27 while the private offended party is but a complaining
March 1990. On 9 August 1991, Crude married witness. Any criminal act perpetrated by the latter on the
respondent's youngest sister. It was the respondent occasion of the commission of the crime, or which may
herself who solemnized that marriage at her office, as have given rise to the criminal act imputed to the accused
evidenced by the marriage contract. Despite such is not the act or conduct of the State and can by no means
marriage, respondent did not inhibit herself from hearing bind it under the doctrine of pari delicto. To rule otherwise
the case and instead proceeded to render and would be to establish a dangerous doctrine which would
promulgate, on 17 October 1991, a judgment acquitting irreparably weaken the very foundations of the criminal
Cruda, her brother-in-law. justice system and frustrate the administration of justice.
The answer to the letter-complaint, filed by the Whatever wrongful act may have been committed by the
respondent was devoted mostly to a narration of her offended party may only be invoked to justify the
sincere and honest efforts to reform and rehabilitate accused's own act or mitigate his liability.
Roberto Cruda. She denies having knowingly rendered an In spite of all this, however, the respondent may
unjust judgment in favor of her brother-in- law because not necessarily be liable for rendering an unjust judgment
she "was persuaded to dismiss the same not on account as there is no convincing evidence on record to show that
that the guilt of Cruda was not proven beyond reasonable she knew such judgment to be unjust and that she
doubt but by the very reason that both the private rendered the same with the conscious and deliberate
complainant and the accused therein were in pari delicto." intent to commit an injustice. She could only be, as she is
She admits, however, that she decided the case beyond hereby found, guilty of gross ignorance of the law.
ninety (90) days from the date it was submitted for The Court noted with grave concern the
decision, and pleads for this Court's understanding, respondent's revelation in her Answer that "she interceded
leniency and compassion considering that a MTC Judge is in the settlement of the cases pending against" Roberto
saddled not only with judicial functions, but quasi-judicial Cruda. By such admission, it is clear that the respondent
task as well which are enough to drain most of his/her acted as counsel for the accused. It is therefore evident
energy. that she is guilty of improper conduct, which could only
On 10 September 1992 the Court referred the serve to diminish public confidence in the integrity and
case to the Executive Judge of the RTC of Malolos, impartiality of the judiciary. Her behavior amounted to a
Bulacan who recommended that proper penalty be violation of Rule 2.01, Canon 2 of the Code of Judicial
imposed upon the respondent Judge. Conduct.
The Court decided to dismiss the Judge from the
ISSUE: WON the respondent have knowingly rendered an service with forfeiture of all benefits, except for the

64
CRIMINAL LAW REVIEW DIGESTS
JUSTICE ROMEO CALLEJO NOTE: = Callejo Ponente

monetary value of her accrued leaves, and with prejudice Trial court convicted Jesus of murder, and sentenced him
to re-employment in any branch or service of the to reclusion perpetua.
government, including government owned or controlled
corporations for grave misconduct, gross inefficiency and Issue: WON Jesus was merely performing a lawful act
neglect of duty, gross ignorance of the law and conduct with due care hence, cannot be held criminally liable for
prejudicial to the best interest of the service. the victims death - No!

WON Jesus is liable for murder - No! Homicide only


People of the Philippines vs. Jesus Retubado Ruling: The phrase state of necessity is of German origin.
Countries which have embraced the classical theory of
Facts: Someone played a joke on Edwin Retubado, the criminal law, like Italy, do not use the phrase. The
appellants younger brother who was mentally ill. justification refers to a situation of grave peril (un mal),
Someone inserted a lighted firecracker in a cigarette pack actual or imminent (actual o imminente). The word
and gave it to Edwin. While Edwin and his father were propiedad covers diverse juridical rights (bienes juridicos)
having dinner, it exploded. The suspect was their neighbor such as right to life, honor, the integrity of ones body, and
Emmanuel Caon, Jr. The matter was brought to the property (la vida, la integridad corporal, el pudor, el honor,
attention of the barangay captain who conducted an bienes patrimoniales) belonging to another. It is
investigation. It turned out that Emmanuel Caon, Jr. was indispensable that the state of necessity must not be
NOT the culprit. The appellant, however, was bent on brought about by the intentional provocation of the party
confronting Emmanuel Caon, Jr. Thereafter, the father of invoking the same.
Emmanuel Jr., 50 y.o. Emmanuel Caon, Sr., (pedicab The defense of a state of necessity is a justifying
driver) was confronted by Jesus when the former was on circumstance under Article 11, paragraph 4 of the RPC. It
his way home. Emmanuel Sr. ignored Jesus so the latter is an affirmative defense that must be proved by the
pushed the pedicab which nearly fell into a canal. Jesus accused with clear and convincing evidence. By admitting
followed Emmanuel Sr. to his house. His wife, Norberta causing the injuries and killing the victim, the accused
Caon was in the balcony of their house, above the porch must rely on the strength of his own evidence and not on
waiting for him to arrive. Emmanuel, Jr., meanwhile, was the weakness of the evidence of the prosecution. Whether
already asleep. Emmanuel Sr. demanded to know why he the accused acted under a state of necessity is a question
was being followed. Jesus told Emmanuel that he just of fact, which is addressed to the sound discretion of the
wanted to talk to Emmanuel Jr., but Emmanuel Sr. told the trial court.
appellant that his son was already asleep. Norberta went There is no basis to deviate from the findings of
down from the balcony and placed her hand on her the trial court that the appellant was the provocateur, the
husbands shoulder to pacify him. Jesus forthwith pulled unlawful aggressor and the author of a deliberate and
out a handgun from under his T-shirt and shot Emmanuel malicious act of shooting the victim at close range on the
on the forehead. The latter fell to the floor as the appellant forehead. The court came to this conclusion based on:
walked away from the scene. Emmanuel was brought to 1. Norberta Caons testimony.
the Tuburan District Hospital, but he died shortly 2. There is no evidence that the appellant informed the
thereafter. Jesus surrendered to the police but failed to police authorities that he killed the victim in a state of
surrender the firearm he used to kill the victim. necessity and that his brother, Edwin, threw the gun into
Jesus admitted shooting the victim but claimed the sea.
that he was merely performing a lawful act with due care 3. The appellant had the motive to shoot and kill the
hence, cannot be held criminally liable for the victims victim.
death. He testified that when he insisted that Emmanuel There is no treachery in the present case to
wake up his son, Emmanuel went to his room and qualify the crime to murder. To appreciate treachery, two
emerged therefrom holding a handgun. Jesus grabbed (2) conditions must be present, namely, (a) the
Emmanuels hand, they struggled for the gun but employment of the means of execution that give the
eventually, Emmanuel fell on his knees. Jesus pulled the person attacked no opportunity to defend himself or to
gun to the level of Emmanuels forehead, and the gun retaliate, and (b) the means of execution were deliberately
suddenly went off. Jesus then rushed to his house to or consciously adopted. The prosecution failed to adduce
change clothes. He placed the gun on the dining table. an iota of evidence to support the confluence of the
When he went back to the dining room his sister told him abovementioned conditions.
that their brother Edwin had taken the gun and thrown it The appellant is entitled to the mitigating
into the sea. circumstance of voluntary surrender.

65
CRIMINAL LAW REVIEW DIGESTS
JUSTICE ROMEO CALLEJO NOTE: = Callejo Ponente

VICKY TY VS PEOPLE that it reduced him to a mere instrument acting without


will.
Facts: What we have here is 7 cases of violations of In this case, Tys fear was not real and imminent.
BP22. Tys mother, Chua Lao So Un and her sister, Judy Her fear that her mothers health might deteriorate due to
Chua were confined at the Manila Doctors Hospital (note: the inhumane treatment of the hospital or that her mother
they were not confined at the same time. The mother was might commit suicide is speculative and not the
confined from 1990 until 1992, while the sister was uncontrollable fear contemplated by law. Moreover, Ty
confined from 1991-1992). Vicky Ty signed the had sufficient knowledge that the issuance of checks
Acknowledgment of Responsibility for Payment for the without funds may result in violation of BP22.
admission of the patients. The total hospital bills of the Ty also suggests that she may be exempted by
2patients amounted to P1,075,592.95. the justifying circumstance of state of necessity. For this,
Ty then executed a promissory note to assume the law prescribes the following requisites: 1) evil sought
payment of the obligation in instalments. Several to be avoided actually exists; 2) injury feared be greater
postdated checks against Metrobank and payable to the than that done to avoid it; and 3) there be no other
hospital were drawn. 7checks, each covering an amount practical and less harmful means of preventing it.
of P30K were all deposited on their due dates only to be In this case, the evil sought to be avoided is
dishonoured due to insufficiency of funds, with the merely expected or anticipated hence, this defense is not
account closed advice. Demand letters were sent but the applicable. Ty could have used other options to avoid
obligation remained unpaid. committing a crime such as giving jewelry or other forms
The defense of Ty was that she issued the of security. Moreover, for the state of necessity to be
checks because of an uncontrollable fear of greater availing, the greater injury feared should not have been
injury. She was allegedly forced to issue the checks brought about by negligence or imprudence, more so
because the hospital would not release her mother unless inaction of the actor. In this case, the issuance of the
the bills were paid. She alleges that because of the bounced checks was brought about by Tys failure to pay
unsettle bill, the hospital deprived her mother of room the hospital bills.
facilities (no aircon, refrigerator, tv, telephone line, late
delivery of food, refusal to change hospital gown and Others: 1) BP22 punishes the mere act of issuing a
bedsheets). Also, the hospital allegedly suspended the bounced check whether it was issued to pay an obligation
medical treatment of her mother. This debasing or to guarantee an obligation. The law itself creates a
treatment affected her mothers mental, psychological prima facie presumption of knowledge of insufficiency of
and physical health that her mother contemplated suicide. funds. Deceit is not an essential element of BP22. The
Thus, fearing this, she was compelled to issue the PN and gravamen of the offense is the issuance of a bad check.
the checks. 2) CA is correct in deleting the penalty of
TC: Guilty imprisonment. Ty was a first-time offender and she did not
CA: Guilty but deleted the penalty of act in bad faith. Administrative Circular 12-2000 merely
imprisonment. establishes a rule of preference in the application of the
penalty in BP22 but the discretion on what to impose rests
Issue: Whether Ty is justified because of the solely on the judge.
uncontrollable fear and state of necessity
circumstances?
CABANLIG VS SANDIGANBAYAN
Held: No! Ty is guilty!
The only question of law raised is whether the Prosections version:
defense of uncontrollable fear is tenable to warrant Tys A robbery occurred in Nueva Ecija but 4 days later, 3
exemption from criminal liability. For this exempting suspects were caught. All items were recovered except
circumstance to be invoked successfully the following for a vase and a small radio. Valino, one of those
must concur: 1) existence of an uncontrollable fear; 2) fear apprehended, knew where the location of the stolen items
must be real and imminent; and 3) the fear of an injury is were so 5 policemen decided to escort Valino to the place
greater than or at least equal to that committed. where the stolen items were hidden. They rode a jeep.
The threat that caused the uncontrollable fear While on their way, Valiano was able to grab one of the
must be of such gravity that an ordinary man would have polices M16 armalite. Cabanlig, who was behind Valino
succumbed to it. It should be based on a real, imminent or inside the jeep, saw what happened and decided to fire
reasonable fear for ones life or limb. The threat must not one shot at Valino, and after 3 seconds, fired another 4
be speculative, fanciful or remote. A person invoking this consecutive shots. Valino did not fire a shot. The next
circumstance must show that the compulsion was so great

66
CRIMINAL LAW REVIEW DIGESTS
JUSTICE ROMEO CALLEJO NOTE: = Callejo Ponente

day, somebody heard the police talking to a fellow policemen were still inside the jeep when Valino
policeman, saying that they salvaged Valino. suddenly grabbed the M16 Armalite.
Defenses version: - By suddenly grabbing the M16 Armalite from his
It was not a salvage. It was an act of self-defense and unsuspecting police guard, Valino certainly did not
performance of duty intend merely to escape and run away as far and fast
Sandiganbayan: Cabanlig liable for homicide as possible from the policemen. Valino did not have to
since he failed to show that the shooting was the grab the M16 Armalite if his sole intention was only to
necessary consequence of the due performance of duty flee from the policemen. If he had no intention to
(but the 4 others were acquitted since there was no engage the policemen in a firefight, Valino could
evidence of conspiracy) simply have jumped from the jeep without grabbing
the M16 Armalite.
Issue: w/n Cabanlig is liable for Valinos death - The Sandiganbayan had very good reasons in
steadfastly adhering to the policy that a law enforcer
SC: NO. Acquitted. must first issue a warning before he could use force
- Self-defense and fulfillment of duty operate on against an offender. However, the duty to issue a
different principles. Self-defense is based on the warning is not absolutely mandated at all times and at
principle of self-preservation from mortal harm, while all cost, to the detriment of the life of law enforcers.
fulfillment of duty is premised on the due performance The directive to issue a warning contemplates a
of duty. The difference between the two justifying situation where several options are still available to
circumstances is clear, as the requisites of self- the law enforcers. In exceptional circumstances such
defense and fulfillment of duty are different. as this case, where the threat to the life of a law
- While self-defense and performance of duty are two enforcer is already imminent, and there is no other
distinct justifying circumstances, self-defense or option but to use force to subdue the offender, the law
defense of a stranger may still be relevant even if the enforcer's failure to issue a warning is excusable.
proper justifying circumstance in a given case is
fulfillment of duty. For example, a policeman's use of
what appears to be excessive force could be justified PEOPLE VS ULEP
if there was imminent danger to the policeman's life or
to that of a stranger. If the policeman used force to Wapili, having a high fever and insensibly talking to
protect his life or that of a stranger, then the defense himself, was acting strangely in his home (nasisiraan na
of fulfillment of duty would be complete, the second ng ulo). His brother in law was trying to calm him down
requisite being present. but to know avail. Wapili locked himself in his room. Later
- Undoubtedly, the policemen were in the legitimate on, he went out naked and chased his brother in law
performance of their duty when Cabanlig shot Valino. (Leydan). Leydan and neighbours tried to tie him with
Thus, fulfillment of duty is the justifying circumstance rope but to no avail so he got loose in the village. Leydan
that is applicable to this case. To determine if this went to a policewoman to report the incident and while this
defense is complete, we have to examine if Cabanlig was happening, Wapili turned up in front of the
used necessary force to prevent Valino from escaping policewomans house to bang her vehicle so she called for
and in protecting himself and his co-accused assistance. Later on, SPO1 Ulep and 2 other police
policemen from imminent danger. officers went to the scene where they saw Wapili armed
- In this case, Valino was committing an offense in the with a bolo and a rattan stool (sabi naman ng relatives ni
presence of the policemen when Valino grabbed the Wapili wala siyang dalang bolo). Ulep fired a warning shot
M16 Armalite from Mercado and jumped from the jeep but Wapili charged towards them so Ulep shot him. Wapili
to escape. The policemen would have been justified fell to the ground. Ulep came closer then pumped another
in shooting Valino if the use of force was absolutely bullet to his head, literally blowing his brains out. Ulep:
necessary to prevent his escape. But Valino was not self-defense and fulfilment of a duty.
only an escaping detainee. Valino had also stolen the
M16 Armalite of a policeman. The policemen had the Issue: w/n Ulep is liable for the death of Wapili
duty not only to recapture Valino but also to recover
the loose firearm. By grabbing Mercado's M16 SC: YES. Liable for homicide
Armalite, which is a formidable firearm, Valino had - Before the justifying circumstance of fulfillment of
placed the lives of the policemen in grave danger. a duty under Art. 11, par. 5, of RPC may be
- Had Cabanlig failed to shoot Valino immediately, the successfully invoked, the accused must prove the
policemen would have been sitting ducks. All of the presence of two (2) requisites, namely, that he acted
in the performance of a duty or in the lawful exercise

67
CRIMINAL LAW REVIEW DIGESTS
JUSTICE ROMEO CALLEJO NOTE: = Callejo Ponente

of a right or an office, and that the injury caused or the in the several cases mentioned in Arts. 11 and 12,
offense committed be the necessary consequence of provided that the majority of such conditions be
the due performance of duty or the lawful exercise of present.
such right or office. The second requisite is lacking in - The Court likewise credited Ulep with the mitigating
the instant case. circumstance of voluntary surrender. The police
- During the first stage, the victim threatened the safety blotter of Kidapawan Municipal Police Station shows
of the police officers by menacingly advancing that immediately after killing Wapili, accused-
towards them, notwithstanding accused-appellant's appellant reported to the police headquarters and
previous warning shot and verbal admonition to the voluntarily surrendered himself
victim to lay down his weapon or he would be shot. As
a police officer, it is to be expected that accused-
appellant would stand his ground. Up to that point, his ORTEGA V. PEOPLE
decision to respond with a barrage of gunfire to halt
the victim's further advance was justified under the Facts: Joemar Ortega raped AAA (name withheld), the
circumstances. After all, a police officer is not required daughter of FFF who was a close friend of Joemars mom.
to afford the victim the opportunity to fight back. He was around 13 years old then and AAA was around 6
Neither is he expected - when hard pressed and in years old. During trial, the RTC and CA were not
the heat of such an encounter at close quarters - to impressed with the defense of denial and adjudged guilty
pause for a long moment and reflect coolly at his peril, of rape.
or to wait after each blow to determine the effects The important issue arose during the pendency
thereof. of the case in the SC. In 2006 (while the case was still
- However, Ulep cannot be exonerated from overdoing pending), RA 9344 (The Juvenile Justice and Welfare Act)
his duty during the second stage of the incident - was passed. One of its provisions found in sec. 64 of the
when he fatally shot the victim in the head, perhaps in act provides that ...cases of children fifteen (15) years old
his desire to take no chances, even after the latter and below at the time of the commission of the crime shall
slumped to the ground due to multiple gunshot immediately be dismissed and the child shall be referred
wounds sustained while charging at the police to the appropriate local social welfare and development
officers. Sound discretion and restraint dictated that officer. Thus, Ortega contends that he should not be
Ulep, a veteran policeman, should have ceased firing made to suffer the penalty of reclusion perpetua as he was
at the victim the moment he saw the latter fall to the 13 years old when the crime was committed.
ground. The victim at that point no longer posed a The OSG argued that Ortega is no longer
threat and was already incapable of mounting an covered by the provisions of Section 64 of RA 9344 since
aggression against the police officers. Shooting him in as early as 1999, Ortega was convicted by the RTC and
the head was obviously unnecessary. the conviction was affirmed by the CA in 2001. RA 9344
- Likewise, the evidence at hand does not favor his was passed into law in 2006, and with Ortega now
claim of self-defense. The presence of unlawful approximately 25 years old, he no longer qualifies as a
aggression is a condition sine qua non. There can be child as defined by RA 9344. Moreover, the OSG claimed
no self-defense, complete or incomplete, unless the that the retroactive effect of Section 64 of RA 9344 is
victim has committed an unlawful aggression against applicable only if the child-accused is still below 18 years
the person defending himself. In the present case, the old as explained under Sections 67 and 68 thereof.
records show that the victim was lying in a prone
position on the ground - bleeding from the bullet Issue: Should RA 9344 apply retroactively to Ortegas
wounds he sustained, and possibly unconscious - case?
when accused-appellant shot him in the head. The
aggression that was initially begun by the victim Held: Yes.
already ceased when accused-appellant attacked A retroactive application of RA 9344 should be
him. From that moment, there was no longer any given to Ortega pursuant to the well-entrenched principle
danger to his life. in criminal law - favorabilia sunt amplianda adiosa
- The Court appreciated the incomplete justifying restrigenda. Penal laws which are favorable to the
circumstance of fulfillment of a duty or lawful exercise accused are given retroactive effect.
of a right. Under Art. 69 of RPC, "a penalty lower by Furthermore, the deliberations of the Senate with
one or two degrees than that prescribed by law shall regard to RA 9344 show an intent for it to apply
be imposed if the deed is not wholly excusable by retroactively. As stated by Senator Santiago even after
reason of the lack of some of the conditions required final conviction if, in fact, the offender is able to prove that
to justify the same or to exempt from criminal liability at the time of the commission of the offense he is a minor

68
CRIMINAL LAW REVIEW DIGESTS
JUSTICE ROMEO CALLEJO NOTE: = Callejo Ponente

under this law, he should be given the benefit of the law. Discernment is the metal capacity to understand the
Senator Pimentel concurred with this statement. difference of right and wrong.
Ortega was only 13 years old at the time of the In this case, presence of discernment was
commission of the alleged rape. This was duly proven by deduced from the calculated acts of petitioner i.e. dragging
the certificate of live birth, by his testimony, and by the the victim in the vacant house so as not to be discovered
testimony of his mother. Furthermore, his age was never and quickly taking off when Teofisto discovered the crime.
assailed in any of the proceedings before the RTC and the In fact, upon prodding of petitioners dad, he hid at his
CA. As such, Ortega, at the time of the commission of the grandmothers house to evade arrest.
crime, was below 15 years of age. Under R.A. No. 9344,
he is exempted from criminal liability. However, this does MINOR ISSUE:
not mean that he is exempted from civil liability. Whether or not there petitioner, as he contends,
was deprived of preliminary investigation
No. No need for preliminary investigation.
Llave vs People According to the rules on criminal procedure, when there
is a valid warrantless arrest, preliminary investigation may
FACTS: Neil Llave, a 12 year old boy, was charged with not be conducted provided there was an inquest. In this
raping Debbielyn, a 7 year old girl, in Pasay City. After case there was a valid warrantless arrest and inquest
school, Debbielyn proceeded home, changed her clothes
and went to her mothers store to bring home unsold quail
eggs. On her way, she passed by a vacant house adjacent PEOPLE OF THE PHILIPPINES vs. ENRICO A.
to Teofistos house (their neighbor). She was suddenly VALLEDOR
pulled by petitioner to a pile of hollow blocks; forced her to
lay down on the cement. Petitioner removed his and the 3 informations were filed against Valledor: 1. murder (with
victims clothes. He lay on top of the victim, started kissing treachery and evident premeditation) for stabbing Elsa
her and inserted his penis inside the victims vagina. The Rodriguez on the chest; 2. Attempted murder (with
victim resisted to no avail. treachery and evident premeditation) for stabbing Ricardo
Thereafter Teofisto heard the victims cries and Maglalang but due to timely medical assistance, he was
went out to see what has happened. At that point, Llave able to survive; 3. Frustrated murder (with treachery and
took off. evident premeditation) for stabbing Roger Cabiguen on his
The parents of the victim, upon knowing the right forearm but due also to timely medical assistance, he
incident, found petitioner in the latters grandparents was able to survive.
house. Llave was arrested by the tanod. Victim Roger Cabiguen was inside his bedroom
The victim was brought to the Child Protection working on a lettering job together with his first cousin,
Unit of the PGH where Dr. Castillo found that no injury victim Elsa Rodriguez, and his friends, Simplicio Yayen
was found on the hymen and perineum, there was a and Antonio Magbanua. All of a sudden, Valledor entered
scanty yellowish discharge between the labia minora. the room; uttered Rogers nickname (Jer) and immediately
There was also a fresh abrasion of the perineal skin at 1 attacked him with a knife, but Roger was able to evade the
oclock position near the anus which can only be caused thrust and was stabbed instead on the right forearm.
by a blunt object such as erect penis or finger. The Valledor then stabbed Elsa Rodriguez on the chest and
findings, according to the Dr., were consistent with the said, I had my revenge, Elsa. Valledor then left.
claim that victim was sexually abused. Roger and Elsa were immediately brought to the
The RTC and the CA both convicted petitioner of hospital. On their way out, Antonio noticed a commotion
the crime of rape. Both courts found that petitioner, by his and saw victim Ricardo Maglalang, a neighbor, wounded.
conducts during the incident, acted with discernment. Antonio learned from the by-standers that Ricardo was
likewise stabbed by Valledor. Upon reaching the hospital,
ISSUE: Whether or not petitioner acted with discernment Elsa was declared dead on arrival. Roger on the other
hand was treated for the 5-centimeter wound sustained by
HELD: Yes he did! Article 12, par3 of the RPC exempts him on his right forearm. Victim Roger Cabiguen testified
from criminal liability persons who are over 9 years and that Valledor suspected him of killing his dog. Valledor
under 15 UNLESS he acted with discernment. The ratio also courted Elsa but she rejected him. Elsa even spat on
for the exemption is the absence of intelligence which is and slapped Valledor.
an essential part of a felony whether by dolo or culpa.
Intelligence is the power to determine the morality of Valledors defense of insanity:
human acts and to distinguish licit from illicit acts. Valledor was employed as provincial jail guard. His mother
Pacita noticed that Valledor was behaving abnormally. For

69
CRIMINAL LAW REVIEW DIGESTS
JUSTICE ROMEO CALLEJO NOTE: = Callejo Ponente

days he was restless and unable to sleep. He likewise positioned nearest to Valledor but the latter chose to stab
complained that their neighbors were spreading rumors Roger and Elsa; 2) Valledor called out the nickname of
that he was a rapist and a thief, prompting Pacita to bring Roger before stabbing him; 3) Simplicio Yayen and
his son to Dr. de Guzman. Pacita disclosed to Dr. de Antonio Magbanua who were likewise inside the room
Guzman that insanity runs in their family. After were left unharmed; 4) Valledor, a spurned suitor of Elsa,
examination, Dr. de Guzman diagnosed him as suffering uttered the words, I had my revenge, Elsa, after stabbing
from psychosis with schizophrenia. He prescribed a her; and 5) Valledor hurriedly left the room after stabbing
depressant (Thoracin), which kept Valledor sane for a the victims.
period two months. Evidently, the foregoing acts could hardly be said
Pacita noticed that Valledor was again acting strangely. to be performed by one who was in a state of a complete
She left to buy Thoracin but when she returned he was absence of the power to discern. Valledor was clearly
nowhere to be found. He was seen swimming across the aware and in control of what he was doing as he in fact
river and uttering that his family will be killed. The brgy. purposely chose to stab only the two victims. His obvious
Officials went to get him and on their way to the city, motive of revenge against the victims was accentuated by
Valledor jumped off the jeep. That afternoon, it was calling out their names and uttering the words, I had my
learned that Valledor killed and wounded his neighbors. revenge after stabbing them. Finally, his act of
Defense offered the findings of: Dr. Melendres- immediately fleeing from the scene after the incident
Valledor suffering from Psychosis or Insanity classified indicates that he was aware of the wrong he has done and
under Schizophrenia (deterioration from previous level of the consequence thereof.
functioning, auditory hallucination, ideas of reference, Valledor acts prior to the stabbing incident to wit:
delusion of control, suspiciousness, poor judgment and crying; swimming in the river with his clothes on; and
absence of insight), Psychoactive Substance Use jumping off the jeepney; were not sufficient to prove that
Disorder. he was indeed insane at the time of the commission of the
TC: Valledor guilty of Murder, Frustrated Murder crime.
and Attempted Murder. But since he was confined at the As consistently held by this Court, A man may act
Mental Hospital, the service of his sentence was crazy but it does not necessarily and conclusively prove
suspended. Valledor appealed. that he is legally so. Then, too, the medical findings
showing that Valledor was suffering from a mental
ISSUE: WON Valledor sufficiently invoked the defense of disorder after the commission of the crime, has no bearing
insanity to exempt him from liability? No. WON Valledor on his liability. What is decisive is his mental condition at
should be held liable of frustrated murder for stabbing the time of the perpetration of the offense. Failing to
Roger? No. only Attempted murder because the wound is discharge the burden of proving that he was legally insane
not fatal. when he stabbed the victims, he should be held liable for
his felonious acts.
RULING: In the eyes of the law, insanity exists when For stabbing Roger Cabiguen, Valledor should
there is a complete deprivation of intelligence in only be liable of attempted murder (not frustrated) for the
committing the act. Mere abnormality of the mental wound sustained was not fatal. The settled rule is that
faculties will not exclude imputability. The accused must where the wound inflicted on the victim is not sufficient to
be "so insane as to be incapable of entertaining a criminal cause his death, the crime is only attempted murder, since
intent." He must be deprived of reason and act without the the accused did not perform all the acts of execution that
least discernment because there is a complete absence of would have brought about death.
the power to discern or a total deprivation of freedom of
the will.
Since the presumption is always in favor of PEOPLE vs. JACINTO
sanity, he who invokes insanity as an exempting
circumstance must prove it by clear and positive evidence. FACTS: Herein accused Hermie Jacinto was charged with
And the evidence on this point must refer to the time the crime of raping a 5-year old child back in 2003. It must
preceding the act under prosecution or to the very moment be noted that at the time of the alleged commission of the
of its execution. crime, Jacinto was still a minor, being only 17 years of
In the case at bar, Valledor failed to discharge age. From the evidence of the prosecution, it was alleged
the burden of overcoming the presumption of sanity at the that the victim, AAA, together with her sister, CCC, were
time of the commission of the crime. The following sent by their father to buy cigarettes from the store. CCC
circumstances clearly and unmistakably show that came back to the house without AAA in tow, but the father
Valledor was not legally insane when he perpetrated the was not alarmed thinking that she was watching TV at the
acts for which he was charged: 1) Simplicio Yayen was house of her Aunt Rita. At the same time, witness Julito

70
CRIMINAL LAW REVIEW DIGESTS
JUSTICE ROMEO CALLEJO NOTE: = Callejo Ponente

testified that he saw Jacinto at the store as he placed AAA of the crime and the minors cunning and shrewdness.
on his lap. The three of them, Julito, Jacinto and AAA, left In the present case, we agree with the Court of
the store at the same time. Julito went to the house of Appeals that: (1) choosing an isolated and dark place to
Aunt Rita, while Jacinto, who held AAAs hand, went perpetrate the crime, to prevent detection[;] and (2) boxing
towards the direction of the lower area or place. They the victim xxx, to weaken her defense are indicative of
walked towards the rice fields near the house of the then 17 year-old Jacintos mental capacity to fully
Perochos. There he made her lie down on harrowed understand the consequences of his unlawful action.
ground, removed her panty and boxed her on the chest. Nonetheless, the corresponding imposable
Already half-naked from waist down, he mounted her, and, penalty should be modified considering that the victim is a
while her legs were pushed apart, pushed his penis into minor. Since the victim was only 5 years old when
her vagina and made a push and pull movement. She felt appellant defiled her in 2003, the law prescribing the death
pain and cried. Afterwards, appellant left and proceeded to penalty when statutory rape is committed applies.
the Perochos. She, in turn, went straight home crying. Her The following, however, calls for the reduction of
father heard her crying and calling out his name. She was the penalty: (1) the prohibition against the imposition of the
without slippers. He found her face greasy. There was penalty of death in accordance with Republic Act No.
mud on her head and blood was oozing from the back of 9346; and (2) the privileged mitigating circumstance of
her head. He checked for any injury and found on her minority of the appellant, which has the effect of reducing
neck a contusion that was already turning black. She had the penalty one degree lower than that prescribed by law,
no underwear on and he saw white substance and mud on pursuant to Article 68 of the Revised Penal Code.
her vagina. AAA told her father what Jacinto did to her. Under Article 68 of the Revised Penal Code,
Enraged, her father confronted Jacinto. when the offender is a minor under 18 years, the penalty
In 2007, the RTC rendered its decision finding next lower than that prescribed by law shall be imposed,
Jacinto guilty of the crime charged, sentencing him with but always in the proper period. However, for purposes
the penalty of reclusion perpetua. The CA affirmed the of determining the proper penalty because of the
decision, but sentenced him with an indeterminate penalty privileged mitigating circumstance of minority, the
of from six (6) years and one (1) day to twelve (12) years penalty of death is still the penalty to be reckoned
of prision mayor, as minimum, to seventeen (17) and four with. Thus, the proper imposable penalty for the accused-
(4) months of reclusion temporal, as maximum in appellant is reclusion perpetua.
accordance with RA 9344 or the Juvenile Justice Law,
which was enacted on April 28, 2006. (2) NO. Nevertheless, a CICL, whose judgment of
conviction has become final and executory only after
ISSUE(s): his disqualification from availing of the benefits of
(1) W/N the CA was correct in applying RA 9344 in suspended sentence on the ground that he/she has
computing the sentence of the accused. exceeded the age limit of twenty-one (21) years, shall
(2) W/N Jacinto is entitled to the Automatic still be entitled to the right to restoration,
Suspension of Sentence granted by RA 9344 to rehabilitation, and reintegration in accordance with
children in conflict with the law (CICL). RA 9344.

HELD/RATIO: RA 9344 warrants the suspension of sentence of a child in


(1) YES. He should have been sentenced with conflict with the law notwithstanding that he/she has
reculusion perpetua because the crime involved is reached the age of majority at the time the judgment of
statutory rape which is punishable by death penalty. conviction is pronounced. Thus:
SEC. 38. Automatic Suspension of Sentence. -
Sec. 6 of RA 9344 exempts a child above 15 years but Once the child who is under eighteen (18) years
below 18 years of age from criminal liability, unless the of age at the time of the commission of the
child is found to have acted with discernment, in which offense is found guilty of the offense charged, the
case, the appropriate proceedings in accordance with court shall determine and ascertain any civil
the Act shall be observed. Discernment is that mental liability which may have resulted from the offense
capacity of a minor to fully appreciate the consequences committed. However, instead of pronouncing the
of his unlawful act. Such capacity may be known and judgment of conviction, the court shall place the
should be determined by taking into consideration all the child in conflict with the law under suspended
facts and circumstances afforded by the records in each sentence, without need of application: Provided,
case. The surrounding circumstances must demonstrate however, That suspension of sentence shall
that the minor knew what he was doing and that it was still be applied even if the juvenile is already
wrong. Such circumstance includes the gruesome nature eighteen (18) years of age or more at the time

71
CRIMINAL LAW REVIEW DIGESTS
JUSTICE ROMEO CALLEJO NOTE: = Callejo Ponente

of the pronouncement of his/her guilt. one (21) years. Section 40 of the law and Section 48 of
(Emphasis supplied.) the Rule are clear on the matter. Unfortunately, appellant
On 10 September 2009, this Court is now twenty-five (25) years old.
promulgated the decision in Sarcia, saying: Be that as it may, to give meaning to the
The xxx provision makes no distinction as to the legislative intent of the Act, the promotion of the welfare of
nature of the offense committed by the child in a child in conflict with the law should extend even to one
conflict with the law, unlike P.D. No. 603 and who has exceeded the age limit of twenty-one (21) years,
A.M. No. 02-1-18-SC. The said P.D. and so long as he/she committed the crime when he/she was
Supreme Court (SC) Rule provide that the benefit still a child. The offender shall be entitled to the right to
of suspended sentence would not apply to a child restoration, rehabilitation and reintegration in accordance
in conflict with the law if, among others, he/she with the Act in order that he/she is given the chance to live
has been convicted of an offense punishable by a normal life and become a productive member of the
death, reclusion perpetua or life imprisonment. In community. The age of the child in conflict with the law at
construing Sec. 38 of R.A. No. 9344, the Court is the time of the promulgation of the judgment of conviction
guided by the basic principle of statutory is not material. What matters is that the offender
construction that when the law does not committed the offense when he/she was still of tender
distinguish, we should not distinguish. Since age.
R.A. No. 9344 does not distinguish between a Thus, appellant may be confined in an
minor who has been convicted of a capital agricultural camp or any other training facility in
offense and another who has been convicted of a accordance with Sec. 51 of Republic Act No. 9344.
lesser offense, the Court should also not Sec. 51. Confinement of Convicted Children in
distinguish and should apply the automatic Agricultural Camps and Other Training Facilities.
suspension of sentence to a child in conflict with A child in conflict with the law may, after
the law who has been found guilty of a heinous conviction and upon order of the court, be made
crime. to serve his/her sentence, in lieu of confinement
The legislative intent reflected in the Senate in a regular penal institution, in an agricultural
deliberations on Senate Bill No. 1402 (Juvenile Justice camp and other training facilities that may be
and Delinquency Prevention Act of 2005) further established, maintained, supervised and
strengthened the position of this Court to cover heinous controlled by the BUCOR, in coordination with
crimes in the application of the provision on the automatic the DSWD.
suspension of sentence of a child in conflict with the law. Following the pronouncement in Sarcia, the case
The pertinent portion of the deliberation reads: shall be remanded to the court of origin to effect
If a mature minor, maybe 16 years old to below appellants confinement in an agricultrual camp or other
18 years old is charged, accused with, or may training facility.
have committed a serious offense, and may have
acted with discernment, then the child could be
recommended by the Department of Social PEOPLE V. VALENTIN DOQUENA
Welfare and Development (DSWD), by the Local
Council for the Protection of Children (LCPC), or Nature: Appeal from an order of the Court of First
by [Senator Miriam Defensor-Santiagos] Instance of Pangasinan (convicting Valentin Doquena, 13
proposed Office of Juvenile Welfare and yrs, 9 months, and 5 days old, of homicide; having acted
Restoration to go through a judicial proceeding; with discernment in committing the said act.)
but the welfare, best interests, and restoration of
the child should still be a primordial or primary Facts: Between 1-2 pm of Nov. 19, 1938, Juan Ragojos
consideration. Even in heinous crimes, the and Epifanio Rarang were playing volleyball in the yard of
intention should still be the childs restoration, their school in Sual, Pangasinan. Valentin Doquena, the
rehabilitation and reintegration. xxx (Italics accused, intercepted the ball, and threw it a Ragojos, who
supplied in Sarcia.) was hit in the stomach. Miffed, Ragojos chased Doquena,
On 24 November 2009, the Court En Banc and upon catching him, slapped Doquena on the nape,
promulgated the Revised Rule on Children in Conflict with and punched him in the face. After doing this, Ragojos
the Law, which reflected the same position. These went back to Rarang to resume playing volleyball.
developments notwithstanding, we find that the benefits of Insulted, Doquena looked for something to throw at
a suspended sentence can no longer apply to appellant. Ragojos, finding none, he got his cousins (Romualdo
The suspension of sentence lasts only until the child in Cocal) knife, and confronted Ragojos. Ragojos denied
conflict with the law reaches the maximum age of twenty- Doquenas request for a fight and resumed playing.

72
CRIMINAL LAW REVIEW DIGESTS
JUSTICE ROMEO CALLEJO NOTE: = Callejo Ponente

Doquena stabbed the unaware Ragojos in the chest, The Aguilars demanded upon Jarco Marketing
thereby killing the latter. The court held that in committing the reimbursement of the hospitalization, medical bills and
the act, the accused acted with discernment and was wake and funeral expenses which they had incurred but
conscious of the nature and consequences of his acts, Jarco refused to pay. Jarcos argument was that it
therefore his defense that he was a minor was untenable observed the diligence of a good father of a family in the
(given that the Doquena was a 7th grade pupil, one of the selection, supervision and control of its employees. Also,
brightest in his class, and was an officer in the CAT that the mom was negligent in exercising care and
program), and thus convicted him of the crime of diligence over her daughter by allowing her to freely roam
homicide. The court ordered him to be sent to the Training around in a store. ZHIENETH too, was guilty of
School for Boys until he reaches the age of majority. Thus, contributory negligence since she climbed the counter,
the appeal by the accused, stating that to determine triggering its eventual collapse on her. They also
whether or not there was discernment on the part of the emphasized that the counter was made of sturdy wood
minor, the following must be taken into consideration: with a strong support.
a) The facts and circumstances which gave rise to TC: Not liable. The proximate cause of the
the act committed. accident was was Zhieneths act of clinging to it
b) The state of mind at the time the crime was CA: Liable. Jarco was negligent in maintaining a
committed structurally dangerous counter (defective, unstable); the
c) The time he had at his disposal child was absolutely incapable of negligence or tort.
d) The degree of reasoning of the minor
Issues:
Issue: WON the accused acted with discernment (1) Whether the death of ZHIENETH was accidental or
attributable to negligence -- NEGLIGENCE
Held: Decision affirmed. Yes, the accused acted with (2) In case of a finding of negligence, whether the same
discernment. Accused mistakes the discernment for was attributable to private respondents for maintaining a
premeditation, or at least for lack of intention, as a defective counter or to the mother and ZHIENETH for
mitigating circumstance. However, the DISCERNMENT failing to exercise due and reasonable care while inside
that constitutes an exception to the exemption from the store premises NEGLIGENCE OF JARCO
criminal liability of a minor under 15 years but over nine, MARKETING
who commits an act prohibited by law, is his MENTAL
CAPACITY to understand the difference between right and Ratio:
wrong, and such capacity may be known and should be (1) An accident pertains to an unforeseen event in which
determined by taking into consideration all the facts and no fault or negligence attaches to the defendant. It is "a
circumstances afforded by the records in each case, the fortuitous circumstance, event or happening; an event
very appearance, the very attitude, the very comportment happening without any human agency, or if happening
and behavior of said minor, not only before and during the wholly or partly through human agency, an event which
commission of the act, but also after and even during the under the circumstances is unusual or unexpected by the
trial. person to whom it happens."
On the other hand, negligence is the omission to
do something which a reasonable man, guided by those
JARCO MARKETING, LEONARDO KONG, JOSE TIOPE considerations which ordinarily regulate the conduct of
and ELISA PANELO, V. CA CONRADO C. AGUILAR human affairs, would do, or the doing of something which
and CRISELDA R. AGUILAR a prudent and reasonable man would not do. I is "the
failure to observe, for the protection of the interest of
Facts: Jarco Marketing Corporation is the owner of another person, that degree of care, precaution and
Syvel's Department Store. Petitioners Leonardo Kong, vigilance which the circumstances justly demand, whereby
Jose Tiope and Elisa Panelo are the store's managers. such other person suffers injury." Test: Did the defendant
Private respondents are parents of Zhieneth Aguilar in doing the alleged negligent act use that reasonable care
(ZHIENETH). and caution which an ordinarily prudent person would
Criselda and Zhieneth (6 years old) were at the have used in the same situation? If not, then he is guilty of
department store. Criselda was signing her credit card slip negligence.
when she heard a loud thud. She looked behind her and The tragedy was no accident and ZHIENETH's
beheld her daughter pinned beneath the gift-wrapping death could only be attributed to negligence. Petitioner
counter structure. She was crying and shouting for help. Panelo and another store supervisor were then already
She was brought to Makati Medical Center, where she personally informed of the danger posed by the unstable
died after 14 days. counter. Yet, neither initiated any concrete action to

73
CRIMINAL LAW REVIEW DIGESTS
JUSTICE ROMEO CALLEJO NOTE: = Callejo Ponente

remedy the situation. They have miserably failed to significance that person ascribes to the said act. Hence a
discharge the due diligence required of a good father of a person may not intend to shoot another but may be aware
family. of the consequences of his negligent act which may cause
injury to the same person in negligently handling an air
(2) Conclusive presumption that favors children below 9 rifle. It is not connect, therefore, to argue, as petitioner
years old in that they are incapable of contributory does, that since a minor above nine years of age but
negligence. Even if we attribute contributory negligence to below fifteen acted with discernment, then he intended
ZHIENETH and assume that she climbed over the such act to be done. He may negligently shoot his friend,
counter, no injury should have occurred if we accept thus did not intend to shoot him.
petitioners' theory that the counter was stable and sturdy. The basic reason behind the enactment of the
If that was the truth, a frail six-year old could not have exempting circumstances embodied in Article 12 of the
caused the counter to collapse. RPC: the complete absence of intelligence, freedom of
The mother was also not negligent. She just action, or intent, or on the absence of negligence on the
momentarily released the child's hand from her clutch part of the accused. The element of intelligence is
when she signed her credit card slip. At this precise necessary to determine the morality of human acts to
moment, it was reasonable and usual for her to let go of distinguish a licit from an illicit act. If absent, this would
her child. Further, at the time ZHIENETH was pinned mean that the accused-child would have no intelligence in
down by the counter, she was just a foot away from her doing the crime and so the law exempts him from criminal
mother; and the gift-wrapping counter was just four meters liability.
away from her mother and did not loiter as petitioners lt is for this reason, therefore, why minors nine
would want to impress upon us. years of age and below are not capable of performing a
criminal act. On the other hand, minors above nine years
of appeal but below fifteen are not absolutely exempt.
G.R. No. 75256 January 26, 1989 However, they are presumed to be without criminal
JOHN PHILIP GUEVARRA, Petitioner, vs. HONORABLE capacity, but which presumption may be rebutted if it could
IGNACIO ALMODOVAR, Respondent. be proven that they were "capable of appreciating the
FACTS: nature and criminality of the act, that is, that (they) acted
with discernment.
Petitioner Guevarra was 11 years old when the incident Intelligence" as an element of dolo actually
happened. He was playing with 4 other children, including embraces the concept of discernment as used in Article 12
his best friend Almine, in their backyard one morning. of the RPC and as defined in the aforecited case of
They were taget shooting tansan(s) with an air rifle People vs. Doquena, supra. It could not therefore be
borrowed from a neighbor, around 15-20 meters away. In argued that discernment is equivalent or connotes 'intent'
the course of the game, Almine was hit by the pellet on his for they refer to two different concepts. Intelligence, which
left collar bone, causing his death. includes discernment, is a distinct element of dolo as a
The Fiscal exculpated Guevarra taking into means of committing an offense.
consideration his age and the fact that the occurrence In evaluating felonies committed by means of
appeared to be an accident. As a result, the victims culpa, three (3) elements are indispensable, namely,
parents appealed to the Ministry of Justice, and the latter intelligence, freedom of action, and negligence. Obviously,
ordered the Fiscal to file a case for Homicide Through intent is wanting in such felonies. However, intelligence
Reckless Imprudence. It was stated in the information that remains as an essential element, hence, it is necessary
Guevarra was over 9 years but below 15 years of age that a minor above nine but below fifteen years of age be
and acting with discernment fired the air rifle and due to possessed with intelligence in committing a negligent act
his carelessness and negligence, caused the death of which results in a quasi-offense. For him to be criminally
Almine. Guevarras parents filed a motion to quash, liable, he must discern the rightness or wrongness of the
contending that the Information contains averments which effects of his negligent act. Indeed, a minor over nine
if true would constitute a legal excuse or justification. years of age but below fifteen may be held liable for a
quasi-offense under Article 365 of the RPC.
ISSUE: W/N Guevarra is guilty of the crime charged Case remanded to trial court for trial.
despite his age (11) and lack of discernment.

HELD/RATIO: We learned earlier that intent and PEOPLE V. SARCIA (GR. No. 169641, 2009)
discernment are different the former refers to the desired
of one's act while the latter relates to the moral Facts: Sarcia was charged with rape. AAA was the five
year-old victim. Some rape details: In 1996, appellant

74
CRIMINAL LAW REVIEW DIGESTS
JUSTICE ROMEO CALLEJO NOTE: = Callejo Ponente

removed AAAs shorts and underwear. He also removed ROWENO POMOY v. PEOPLE
his trousers and brief. Thereafter, he ordered [AAA] to lie
down on her back. Then, he lay on top of her and inserted The Case:
his penis into [AAAs] private organ. Appellant made an up- A petition for Review by Appellant Pomoy, against the
and-down movement("Nagdapadapa tabi"), AAA felt ruling of RTC Iloilo and the CA, that found him guilty
severe pain and exclaimed Aray. Saricas conviction was of homicide
then affirmed upon appeal, crediting AAAs testimony and The TC and CA found that the death of Tomas
her cousins as well, despite certain inconsistencies. Balboa, who was shot with a .45 service pistol, with
Relevant Fact: Meanwhile, when accused- deliberate intent and decided purpose to kill, and
appellant was detained at the New Bilibid Prison pending without any justifiable cause or motive, did then and
the outcome of his appeal before this Court, (R.A.) No. there willfully, unlawfully and feloniously assault the
9344, the Juvenile Justice and Welfare Act of 2006 took deceased Balboa. Hence this petition.
effect on May 20, 2006. The RTC decision and CA
decision were promulgated on January17, 2003 and July The Facts according to the Prosecution:
14, 2005, respectively. The promulgation of the sentence Balboa was a teacher in Concepcion College of
of conviction of accused handed down by the RTC was Science and Fisheries in Concepcion, Iloilo.
not suspended as he was about 25 years of age at that About 7:30 am of 4 January 1990, some policemen
time, in accordance with Article 192 of (P.D.) No. 603, The arrived to arrest Balboa, allegedly due to a robbery
Child and YouthWelfare Code, etc. He is now 31 years of back in December 1989. Balboa complied and was
age. Thus, the retroactivity of RA 9344 is at issue which detained in Camp Jalandoni, Iloilo with Edgar
affords the accused , so long as he was under 18 at the Samudio, another suspect of the robbery case.
time of the criminal incident. Automatic suspension of About 2 pm, petitioner Pomoy, the police sergeant,
sentence is also available even if the child reached 18 came for Balboa for tactical interrogation at the
at the time of the promulgation of judgment(Sec. 38). investigation room. At that time, Pomoy had a service
gun (a .45 caliber pistol)
Relevant Issue: WON RA 9344 still applies in favor of When Pomoy and Balboa were in the building near
accused? the investigation room, and two (2) gunshots were
heard. When they came to the source of the gunshot
Held: NO. But death penalty reduced to reclusion sounds, Pomoy was seen still holding the .45 caliber
perpetua. Case remanded to court a quo for appropriate pistol, facing Balboa, who was lying in a pool of blood,
disposition under Sec.51. about two (2) feet away.
Firstly, Section 38 does not distinguish WON When the Commanding Officer arrived, he disarmed
child is guilty of capital offense or a lesser one, and so Pomoy, and had Balboa brought to the hospital but
automatic suspension of sentence can be afforded even in was stopped by a doctor in the premises, saying it
a heinous crime. Nonetheless, while Sec. 38 of R.A. No. was unnecessary as Balboa is already dead.
9344 provides that suspension of sentence can still be Upon the request of Mrs. Balboa, an autopsy was
applied even if the child in conflict with the law is already conducted in NBI Iloilo with findings by the medico-
(18) years of age or more at the time of the legal officer, as follows: Cause of Death: Hemorrhage,
pronouncement of his/her guilt, Sec. 40 of the same law massive secondary gunshot wounds on chest and
limits the said suspension of sentence until the said child abdomen.
reaches the maximum age of 21. Since he is now already
31 years old, the question on the suspension of sentence The Facts according to the Defense:
is now moot and academic. Petitioner Pomoy generally adopts the narration of
However, he is still entitled to the disposition facts in the trail court and CA decisions,
measure in Section 51 which reads:. Confinement of Defense had the following witnesses:
Convicted Children in Agricultural Camps and Other Erna Basa (the lone eyewitness to the incident)
Training Facilities.A child in conflict with the law may, who says, while she was working about 2pm,
after conviction and upon order of the court, be made to heard some noise and exchange of words that
serve his/her sentence, in lieu of confinement in a regular were unclear but seemed like trouble. She opens
penal institution, in an agricultural camp and other training the door, and seeing one meter away, finds
facilities that may be established, maintained, supervised Pomoy and Balboa grappling for possession of
and controlled by theBUCOR, in coordination with the the gun from Pomoys holster. It all happened so
DSWD. fast that the gun was pulled out of the holster, a
shot was fired, but she wasnt sure who pulled
the trigger.

75
CRIMINAL LAW REVIEW DIGESTS
JUSTICE ROMEO CALLEJO NOTE: = Callejo Ponente

Eden Legaspi, she also hears the commotion neither aggravating nor mitigating circumstances
from the outside, but remained seated where she proven.
was. She witnesses Erna Basa go and open the
door. Eden Legaspi only stood up after shots Issues:
were fired and hears on of the two men fall down. 3. W/N the prosecution failed to overturn the defense?
Dr. Salvador Mallo Jr., the medico-legal officer YES, the prosecution failed.
who conducted the autopsy. He finds two (2) a) Did Pomoy had full control of the gun, as the
entrance wounds on Balboa, ones trajectory CA ruled? NO.
going upward, and the other downward. b) Did the safety lock feature, requirement of
Pomoy the petitioner, he notes that once he pressure and two gunshots necessarily conclude
opened the door to meet Balboa, the latter a determined effort to kill instead of an accident,
suddenly approached him to take hold of the gun as the CA held? NO.
in the holster. Pomoy also notes he loaded and c) Did the number and location of the gunshot
cocked his gun before going to Camp Jalandoni wounds necessarily conclude deliberate intent?
that day. Though Balboa is shorter, Pomoy notes NO.
Balboa was bigger in build. Pomoy however, b) W/N there was a exempting circumstance of accident,
prevented Balboa from taking his gun. After a few as in Art 12, par. 4.. YES.
seconds of grappling, the gun was forced out of c) W/N there was Self Defense. NO.
the holster, it fired to the right side of the victim.
Held:
Trial Court and CA Decisions: 1. The CA failed to see the prosecutions failure to
Pomoy was held guilty based on the following: 1) overturn the allegations of the accident, as an exempting
Petitioner had substantial control of the gun, 2) the circumstance in Article 12, which exculpates the actor
gun was locked prior to the grabbing incident, hence when the harm was done without his fault or negligence
unlocked by the petitioner, 3) location of the wounds but rather on circumstances unforeseen or out of his
do not support the assertion of the grabbing of the control. Thus, in determining whether an accident
gun, 4) as the OSG said, an accident was unlikely attended the incident, courts must take into account the
since there were two gunshot wounds, on two dual standards of (1) lack of intent to kill and (2)
different angles and distant parts of the body, instead absence of fault or negligence. (See below)
of merely one. The OSG said that it is an oft-repeated
principle that the location, number and gravity of the a. POMOY HAD NO FULL CONTROL. According to
wounds inflicted on the victim have a more revealing the facts, the Petitioner was NOT in control of the
tale of what actually happened during the incident. gun when it fired, mainly through the testimonial
The appellate court cited People v. Reyes saying that evidence of Erna Basa. According to the cross,
revolvers are not prone to accidental firing since it when she began to see the incident, the gun was
need to be cocked and pressure is needed to be still in the holster, at the side of the petitioner. She
exerted on the trigger. also mentioned both the petitioner and the deceased
Furthermore, the CA debunked the alternative plea of had their hands on the gun, while it was INSIDE the
self-defense. It held that petitioner had miserably holster, at that point they were both already
failed to prove the attendance of unlawful aggression, grappling for possession. She mentions that both
an indispensable element of this justifying gunshots happened during the grappling, but
circumstance. because of the wrestling of the two, she could not
Also, the CA altered the trial courts ruling in see where the gun was pointed towards. This was
appreciating the aggravating circumstance of abuse because as Pomoys right hand and Balboas left
of public position. The CA said that for the hand were scuffling for possession, Pomoy
aggravating circumstance to apply, he must use his continued to use his left hand to subdue Balboa.
influence, prestige and ascendancy which his office
gives him in realizing his purpose. If the accused The foregoing account clearly demonstrates that the
could have perpetrated the crime without occupying petitioner did NOT have control of the gun to
his position, then there is no abuse of public position. consider any willful intent to kill the deceased.
(People vs. Joyno, 304 SCRA 655, 670). The CA According to the witness, the deceased persistently
ruled that in this case there was no showing of a tried to wrest the weapon from the petitioner, while
premeditated plan, nor did the petitioner take he resolutely tried to thwart those attempts. The CA
advantage of his public position. Hence there were therefore, had no firm basis to conclude that Pomoy
had full possession of the gun.

76
CRIMINAL LAW REVIEW DIGESTS
JUSTICE ROMEO CALLEJO NOTE: = Callejo Ponente

The elements of accident are as follows: 1) the accused


b. NO CLEAR SHOWING OF DETERMINED was at the time performing a lawful act with due care; 2)
EFFORT. Since it is now undisputed that both the resulting injury was caused by mere accident; and 3)
petitioner and victim struggled aggressively for on the part of the accused, there was no fault or no intent
possession of the gun, the eyewitness account of to cause the injury.
Basa clearly illustrated the fact that in the fierce and From the facts, it is clear that all these elements
vicious frenzied grappling, it supports the were present. At the time of the incident, petitioner was
conclusion that the safety lock was accidentally an investigator for the PNP. Thus, he was in the lawful
released, and the force of either man was strong performance of his duties that, under the instructions of his
enough to fire the gun, putting the necessary superior, he fetched the victim from the latters cell for a
pressure. Not to mention that it was admitted that routine interrogation. Also, it was in the lawful performance
Pomoy cocked the gun earlier that day. of his duty as a law enforcer that petitioner tried to defend
Also, the fact that two gunshots were fired were his possession of the weapon when the victim suddenly
attributed to the nature of the gun and not a tried to remove it from his holster. As an enforcer of the
conclusion of deliberate intent. As the petitioner law, petitioner was duty-bound to prevent the snatching of
himself testified, he said that a caliber .45 semi- his service weapon by anyone, especially by a detained
automatic pistol, when fired, immediately slides person in his custody. Such weapon was likely to be used
backward throwing away the empty shell and returns to facilitate escape and to kill or maim persons in the
immediately carrying again a live bullet in its vicinity, including petitioner himself.
chamber. Thus, the gun can, as it did, fire in Petitioner cannot be faulted for negligence. He
succession. Verily, the location of, and distance exercised all the necessary precautions to prevent his
between the wounds and the trajectories of the service weapon from causing accidental harm to others.
bullets jibe perfectly with the claim of the petitioner: As he so assiduously maintained, he had kept his service
the trajectory of the first shot going downward from gun locked when he left his house; he kept it inside its
left to right thus pushing Balboas upper body, tilting holster at all times, especially within the premises of his
it to the left while Balboa was still clutching working area. At no instance during his testimony did the
petitioners hand over the gun; the second shot accused admit to any intent to cause injury to the
hitting him in the stomach with the bullet going deceased, much less kill him.
upward of Balboas body as he was falling down and The participation of petitioner, if any, in the
releasing his hold on petitioners hand. victims death was limited only to acts committed in the
Thus the reliance of the CA in People v. Reyes course of the lawful performance of his duties as an
was misplaced. This case involves a semi-automatic enforcer of the law. The removal of the gun from its
pistol, the mechanism of which is very different from holster, the release of the safety lock, and the firing of the
that of a revolver, the gun used in Reyes. Unlike a two successive shots -- all of which led to the death of the
revolver, a semi-automatic pistol, as sufficiently victim -- were sufficiently demonstrated to have been
described by petitioner, is prone to accidental firing consequences of circumstances beyond the control of
when possession thereof becomes the object of a petitioner. At the very least, these factual circumstances
struggle. create serious doubt on the latters culpability.

c. THE LOCATION OF THE WOUNDA ARE 3. There was NO SELF DEFENSE.


IRRELEVANT, though ordinarily it would be. In this
case though, they are inconsequential where both Pomoy put forth self defense as an alternative defense,
the victim and the accused were grappling for that granting arguendo that he intentionally shot Balboa,
possession of a gun, the direction of its nozzle may he claims he did so to protect his life and limb from real
continuously change in the process, such that the and immediate danger.
trajectory of the bullet when the weapon fires
becomes unpredictable and erratic. In this case, the The SC said that Self-defense is inconsistent with the
eyewitness account of that aspect of the tragic exempting circumstance of accident, in which there is no
scuffle shows that the parties positions were intent to kill. On the other hand, self-defense necessarily
unsteady, and that the nozzle of the gun was neither contemplates a premeditated intent to kill in order to
definitely aimed nor pointed at any particular target. defend oneself from imminent danger. Apparently, the
fatal shots in the instant case did not occur out of any
1. The ELEMENTS of ACCIDENT WERE ALL PRESENT conscious or premeditated effort to overpower, maim or kill
in this case. the victim for the purpose of self-defense against any
aggression; rather, they appeared to be the spontaneous

77
CRIMINAL LAW REVIEW DIGESTS
JUSTICE ROMEO CALLEJO NOTE: = Callejo Ponente

and accidental result of both parties attempts to possess stand up and brought him to the main road. Lani asked
the firearm. Ricky who stabbed him and Ricky replied that it was
Since the death of the victim was the result of an appellant who stabbed him. Then Docloy Cortez arrived at
accidental firing of the service gun of petitioner -- an the scene on board his tricycle. Accordingly, Ricky was put
exempting circumstance as defined in Article 12 of the on the tricycle and taken to the Romblon Provincial
Revised Penal Code -- a further discussion of whether the Hospital. He was sent to the Romblon provincial hospital
assailed acts of the latter constituted lawful self-defense is but died while being operated on.
unnecessary.
Petitioners story:
RULING: Though timeless is the legal adage that facts Same story as the prosecution. . . . until. . .. he then went
found by the trial court and appellate court are conclusive, to his house, locked the door with a nail, and went to
the Supreme Court however may overturn the same when sleep. However, he was awakened at around 9:30 p.m. by
certain crucial facts or details are overlooked and when loud noises coming from Ricky and his three companions.
upon a petition, a reexamination is imperative. Due to the He peeped through the window grills of his house and
appreciation of facts of the accident, credibility of the admonished them not to make any loud noises. Ricky,
witnesses creating a reasonable doubt, and upholding the who was then already inebriated, was incensed; he pulled
presumption of innocence, the appellant was therefore out a balisong, pushed the door, and threatened to stab
ACQUITTED. the petitioner. The petitioner pushed their sala set against
the door to block the entry of Ricky, but the latter
continued to push the door open with his hands and body.
Toledo v People (by Justice Callejo) The petitioner ran to the upper portion of their house and
got his bolo. He returned to the door and pushed it with all
Prosecutions version: his might using his left hand. He then pointed his bolo,
On September 16, 1995, appellant (Noe Toledo y which was in his right hand, towards Ricky. The bolo
Tamboong) went to a black-smith who made the design of accidentally hit Ricky on the stomach, and the latter lost
his bolo. When he went home to Tuburan, Odiongan, his balance and fell to the floor. The petitioner, thereafter,
Romblon late in the afternoon, appellant saw the group of surrendered to the barangay captain at 11:00 a.m.
Lani Famero, Michael Fosana, Rex Cortez and Ricky He claimed accidental death (Article 12 par 4) in the trial
Guarte (deceased) drinking gin at the house of the court and CA then changed his theory to Self-defense
Spouses Manuel and Eliza Guarte, Rickys parents. (Article 11 par 1)
Appellants house is about five (5) meters away from the
house of Spouses Guarte. Appellant requested the group Issue: Whether or not accidental death (article 12) or even
of Ricky to refrain from making any noise. Thereupon, Self-defense (article 11) applies.
appellant proceeded inside his house and went to sleep.
Around 9:00 p.m., Gerardo Faminia, Eliza Guartes brother Held: No Accidental Death nor Self Defense. He is still
arrived at the Guarte house and asked for any left-over liable for the crime of homicide.
food. Eliza prepared dinner for him and after Gerardo Rule on change of theory: It is a matter of law that when a
finished eating, he went home accompanied by Ricky. party adopts a particular theory and the case is tried and
Gerardos home is about twelve (12) meters away from the decided upon that theory in the court below, he will not be
Guarte home. Minutes later, Ricky came back and permitted to change his theory on appeal. The case will be
together with Lani, Rex and Michael, went to sleep at the reviewed and decided on that theory and not approached
Guarte house. They had not laid down for long when they and resolved from a different point of view. To permit a
heard stones being hurled at the roof of the house. The party to change his theory on appeal will be unfair to the
stoning was made three (3) times. Ricky rose from bed adverse party.
and peeped through a window. He saw appellant stoning It is an aberration for the petitioner to invoke the
their house. Ricky went out of the house and proceeded to two defenses at the same time because the said defenses
appellants house. Ricky asked appellant, his uncle, why are intrinsically antithetical.(quoting People v Javier 377
he was stoning their house. Appellant did not answer but SCRA 300 (2002). There is no such defense as accidental
met Ricky at the doorstep of his (appellants) house and, self-defense in the realm of criminal law.
without any warning, stabbed Ricky on the abdomen with Self-defense under Article 11, paragraph 1 of the
a bolo. Eliza had followed his son Ricky and upon seeing Revised Penal Code necessarily implies a deliberate and
that Ricky was stabbed, shouted for help. Lani heard positive overt act of the accused to prevent or repel an
Elizas cry for help and immediately rushed outside the unlawful aggression of another with the use of reasonable
house. Lani saw Ricky leaning on the ground and means. The accused has freedom of action. He is aware
supporting his body with his hands. Lani helped Ricky of the consequences of his deliberate acts. The defense is

78
CRIMINAL LAW REVIEW DIGESTS
JUSTICE ROMEO CALLEJO NOTE: = Callejo Ponente

based on necessity which is the supreme and irresistible C. To prove self-defense, the petitioner was
master of men of all human affairs, and of the law. From burdened to prove the essential elements thereof, namely:
necessity, and limited by it, proceeds the right of self- (1) unlawful aggression on the part of the victim; (2) lack of
defense. The right begins when necessity does, and ends sufficient provocation on the part of the petitioner; (3)
where it ends. Although the accused, in fact, injures or kills employment by him of reasonable means to prevent or
the victim, however, his act is in accordance with law so repel the aggression. Unlawful aggression is a
much so that the accused is deemed not to have condition sine qua non for the justifying circumstances of
transgressed the law and is free from both criminal and self-defense, whether complete or incomplete. Unlawful
civil liabilities. aggression presupposes an actual, sudden, and
On the other hand, the basis of exempting unexpected attack, or imminent danger thereof, and not
circumstances under Article 12 of the Revised Penal Code merely a threatening or intimidating attitude. We agree
is the complete absence of intelligence, freedom of action, with the ruling of the CA that the petitioner failed to prove
or intent, or the absence of negligence on the part of the self-defense, whether complete or incomplete:
accused. The basis of the exemption in Article 12, The evidence on record revealed that there is no
paragraph 4 of the Revised Penal Code is lack of unlawful aggression on the part of Ricky. While it was
negligence and intent. The accused does not commit established that Ricky was stabbed at the doorstep of
either an intentional or culpable felony. The accused appellants house which would give a semblance of verity
commits a crime but there is no criminal liability because to appellants version of the incident, such view, however,
of the complete absence of any of the conditions which is belied by the fact that Ricky arrived at appellants house
constitute free will or voluntariness of the act. An accident unarmed and had only one purpose in mind, that is, to ask
is a fortuitous circumstance, event or happening; an event appellant why he threw stones at his (Rickys) house. With
happening wholly or partly through human agency, an no weapon to attack appellant, or defend himself, no sign
event which under the circumstances is unusual or of hostility may be deduced from Rickys arrival at
unexpected by the person to whom it happens. appellants doorstep.Ricky was not threatening to attack
Self-defense, under Article 11, paragraph 1, and nor in any manner did he manifest any aggressive act that
accident, under Article 12, paragraph 4 of the Revised may have imperiled appellants well-being. Rickys want of
Penal Code, are affirmative defenses which the accused is any weapon when he arrived at appellants doorstep is
burdened to prove, with clear and convincing evidence. supported by the fact that only one weapon was presented
Such affirmative defenses involve questions of facts in court, and that weapon was the bolo belonging to
adduced to the trial and appellate courts for resolution. By appellant which he used in stabbing Ricky.Thus,
admitting killing the victim in self-defense or by accident appellants version of the events does not support a finding
without fault or without intention of causing it, the burden is of unlawful aggression
shifted to the accused to prove such affirmative defenses. Appellant was not justified in stabbing
He should rely on the strength of his own evidence and Ricky.There was no imminent threat to appellants life
not on the weakness of that of the prosecution. If the necessitating his assault on Ricky.Unlawful aggression is
accused fails to prove his affirmative defense, he can no a condition sine qua non for the justifying circumstance of
longer be acquitted. self-defense.For unlawful aggression to be appreciated,
there must be an actual, sudden, unexpected attack or
imminent danger thereof, not merely a threatening or
Evidence of petitioner incredible and of barren weight. intimidating attitude.In the absence of such element,
Reasons: appellants claim of self-defense must fail.
A. If the testimony of the petitioner is to be D. With the failure of the petitioner to prove self-
believed, the force of the struggle between him and the defense, the inescapable conclusion is that he is guilty of
victim would have caused the door to fall on the petitioner. homicide as found by the trial court and the CA.He cannot
However, the petitioner failed to adduce real evidence that even invoke Article 12, paragraph 4 of the Revised Penal
the door of his house was destroyed and that he sustained Code
any physical injuries, considering that he was only five
inches away from the door.
B. If the door fell to the sala of the house of the TAMBOONG V. PEOPLE
petitioner, the victim must have fallen on top of the door. It
is incredible that the bolo of the petitioner could have hit Facts: After picking up his bolo from the blacksmith,
the stomach of the victim. The claim of the petitioner that appellant went home late in the afternoon where he saw a
he managed to step aside and avoid being crushed by the group of people including deceased drinking gin near his
door belies his claim that the bolo accidentally hit the house. He then requested the group to refrain from
victim on the stomach. making any noise, he then proceeded to his house and

79
CRIMINAL LAW REVIEW DIGESTS
JUSTICE ROMEO CALLEJO NOTE: = Callejo Ponente

slept. At around 9pm he was awakened by loud noises attempt to prove his defense is based solely on his
coming from the drinking group, he peeped through the testimony, but the courts did not think much of it. The
window grills of his house and admonished them not to evidence on record reveal that there is no unlawful
make any load noises. The deceased, already drunk, was aggression on the part of the victim, while it was
incensed, he pulled out his balisong and pushed the door, established that the deceased was stabbed at the
and threatened to stab the petitioner. Petitioner then doorstop of the accused. The presence of the balisong on
pushed the sala set against the door to block the entry of the person of the deceased was also not fully established,
the deceased, but the latter continued to push the door hence the deceased was approaching without any weapon
open. It was then that petitioner ran up his house and got and no clear sign of hostility. For aggression to be
his bolo. He returned to the door and pushied with all his appreciated there must be an actual , sudden, unexpected
might , he then pointed his bolo towards the deceased. attack or imminent danger thereof. Not merely a
The bolo accidentally hit the deceased on the stomach threatening or intimidating attitude and the accused must
and died. Petitioner thereafter surrendered to the tanod at present proof of positively strong act of real aggression.
around 11a.m. Appelant was not justified in stabbing the victim.
The trial court found him guilty of homicide with There was no imminent threat to appelants life
mitigating circumstance of voluntary surrender. The TC did necessitating his assault on the victim. Unlawful
not give credence to the testimony of petitioner that his aggression is a condition sine qua non for the justifying
bolo accidentally hit the victim on the stomach. CA circumstance of self-defense.
affirmed the judgment. Petitioner contends that he acted in
complete self-defense when the victim was hit accidentally
by his bolo. PEOPLE VS. MORALES

Issue: WON petitioner is guilty beyond reasonable doubt Note: En Banc case (Callejo was part of this). Automatic
of homicide. review coz they were charged with death penalty
kidnapping with ransom
Held: He is Guilty of homicide. Petitioner claims Art 12 par
4 any person who, while performing a lawful act with due FACTS: On Nov 9, 1994 MORALES and MALIT et al.
care, causes an injury by mere accident without fault or (Morales and Malit lang ang na-charge coz the others
intention of causing it is exempted from criminal liability. were at large), abducted 5 people while they were on
However petitioner also claims Art 11 par 1 anyone who board a L-300 van at San Vicente, Bacolor, Pampanga, for
acts in defense of his person or rights against unlawful the purpose of extorting ransom money from the parents
aggression cannot be criminally liable (justified). The of the said victims with threat to kill the said victims if their
courts did not look kindly on this, it is an aberration for the parents failed to deliver the ransom money. The victims
petitioner to invoke the two defenses at the same time were brought and detained in Bataan until the father of
because the said defenses are intrinsically antithetical. victims, Feliciano Tan, paid and delivered to Morales et. al
There is no such defense as accidental self-defense in the amount of P92,000.00. Morales was caught and
criminal law. pleaded not guilty upon arraignment. After a series of
Self-D necessarily implies a deliberate and testimonies from the victims, the police and the prosecutor
positive overt act of the accused to prevent an unlawful (recounting the kidnapping), MORALES and MALIT
aggression of another with the use of reasonable means. testified for their own defense. They denied under oath
The accused has freedo of action. He is aware of the that they willingly participated in the kidnapping,
consequences of his deliberate acts. Although the interposing the defense of having acted under the impulse
accused in fact kills the victim, his act is in accordance of an uncontrollable fear of an equal or greater injury.
with law so much so that the accused is deemed not to They stated that they met the other co-accused because
have transgressed the law and is free from both criminal they were supposedly invited for a construction job. On
and civil liabilities. On the other hand, basis of exempting their way to the construction site (via commute), one of
circumstances is the complete absence of intelligence, them flagged down an L300 (the one the victims were
freedom of action or intent, or the absence of negligence riding) and poked a gun at the driver. Morales and Malit,
on the part of the accused. The basis therefore is lack of who got scared, started walking away but they were forced
negligence and intent. An accident is a fortuitous into the van at gunpoint. They pleaded to be released
circumstance, an event happening partly through human because they did not want any involvement with the crime
agency, an event which under the circumstances is but, the other co-accused responded with more threats,
unusual or unexpected by the person to whom it happens. including threats to their families. Morales and Malit did a
The petitioner however failed to prove that the victim was number of acts (was involved in the initial abduction,
killed by accident without fault or intention on his part. His feeding/guarding the children while they are w/ them,

80
CRIMINAL LAW REVIEW DIGESTS
JUSTICE ROMEO CALLEJO NOTE: = Callejo Ponente

instructing the Feliciano to go to Gumi for the ransom) Facts: Paderna, his girlfriend Amelita, and other teenagers
before they were released. Notwithstanding their went to downtown General Santos City to witness the
defenses, both of them were charged by the RTC and commemoration of the landing of General Paulino Santos
sentenced to death. and the first batch of National Land Settlement
Administration settlers in Dadiangas before WWII. They
ISSUES: stayed until 1am and boarded a tricycle on the way home,
1. W/N Morales can be granted the exempting which dropped them off in the Silway Bridge because the
circumstances of irresistible force and/or driver refused to go further out of fear of hold-uppers.
uncontrollable fear of an equal or greater injury. NO When they reached the middle, another tricycle
arrived. On board were Tami and Bagatao, who ordered
Under Article 12 of the Revised Penal Code, a person is them to stop. Bagatao was armed with a gun. Some of
exempt from criminal liability if he acts under the the teenagers fled while four remained. Tami ordered
compulsion of an irresistible force, or under the impulse of them to leave one girl, Amelita behind while the rest were
an uncontrollable fear of equal or greater injury, because ordered to leave.
such person does not act with freedom. In the case of Tami and Bagatao took Amelita to the compound
People vs. Del Rosario, however, we held that for such of the beachhouse of the Shellane company where
defense to prosper the duress, force, fear intimidation Bagatao raped her. She was brought to the Veres Ice
must be present, imminent and impending, and of such Plant where he raped her again. She was raped a third
nature as to induce a well-grounded apprehension of time when they took her to Bagataos uncles house. All
death or serious bodily harm if the act be done. A threat of the while, Tami served as a look out and assisted Bagatao
future injury is not enough. By not availing of the chance to by removing Amelitas pants, underwear, and by
escape (since the other accused were waiting for them at spreading her legs when she refused to do so, all on
the distance of 1 kilometer) their allegation of fear or Bagataos orders.
duress becomes untenable. For it to apply: it is necessary After, Amelitas hands were tied up while
that the compulsion be of such a character as to leave no Bagatao slept. Tami told Amelita he would release her out
opportunity to escape or self-defense in equal combat. of pity and brought her to the Saavedra Saway Elementary
The fear (threats against family members), were not real School. However, Tami raped Amelita twice, in two
as they were not supported by evidence. And even if they separate rooms. After, she was brought to Silway Bridge
were real, they were not of imminence as to prevent any and was told to go home with the threat that if she told the
chance of escape, and that this fear they allegedly police, she and her family would be killed.
suffered does not suffice to grant them the exempting Naturally, Amelita told her family and her
circumstance. boyfriend and they went to the police. The RTC found
Bagatao and Tami guilty for the crime of abduction with
2. W/N conspiracy was proven beyond reasonable rape on 3 counts committed with the use of a deadly
doubt YES weapon and were sentenced to 3 reclusion pereptuas
each. Tami was additionally found guilty of rape on 2
The acts done by the appellants (was involved in the initial counts and was sentenced an additional 2 reclusion
abduction, feeding/guarding the children while they are w/ perpetuas (total of 5).
them, instructing the father to go to Gumi for the ransom) Tami appealed and denied participation in the
clearly shows that there was close coordination, indicating crimes charged. He maintained that he did not conspire
a common purpose or design. Conspiracy exists when two with Bagatao to abduct Amelita and that it was not proven
or more persons come to an agreement concerning the that he agreed to commit the crime. He claimed that he
commission of a felony and decide to commit it. Where all was just following the orders of his cousin when he acted
the accused acted in concert at the time of the as a lookout.
commission of the offense, and it is shown by such acts
that they had the same purpose or common design and Issue: W/N he is guilty of conspiring with Bagatao YES
were united in its execution, conspiracy is sufficiently
established. It must be shown that all participants Ratio: It is a well-settled rule that for collective
performed specific acts with such closeness and responsibility to be established, it is not necessary that
coordination as to indicate a common purpose or design to conspiracy be proved by direct evidence of a prior
commit the felony. agreement to commit the crime. It is sufficient that at the
time of the commission, all the accused acted in concert
showing that they had the same purpose or common
PEOPLE v. TAMI design and were united in its execution. In this case, the
simultaneous and participatory acts of Tami and Bagatao

81
CRIMINAL LAW REVIEW DIGESTS
JUSTICE ROMEO CALLEJO NOTE: = Callejo Ponente

reveal a community of design. To hold an accused liable - Night: Theres a party at the house of Ruizs niece.
as co-principal by reason of conspiracy, he must be shown The policemen were invited. They parked in front of
to have performed an overt act pursuant to it. This may the VISLU office. Ruiz arrived then started shouting
consist of active participation in the actual commission of WRONG PARKING! WRONG PARKING! He sent a
the crime, or may consist of moral assistance to his co- companion to call the policemen. The policemen
conspirators by being present at the time of commission or obliged to talk to Ruiz. Pero nagkainitan na naman.
by exerting moral ascendancy over the other conspirators Ruiz left.
by moving them to execute or implement the conspiracy. - Later that night: Riding a pickup, Ruiz and his
Tami was not only present; he actually took part in the companions went back to the VISLU office and
sexual assault. started firing at the policemen who were about to
To be free of culpability, a person invoking leave. One died. Others were injured.
irresistible force or uncontrollable fear must show that the - Ayun, they got convicted of murder and 2 counts of
force exerted was such that it reduced him to a mere frustrated homicide. Hence, this appeal.
instrument acting not only without his will but against his - The defense claims that the lower court failed to
will as well. Compulsion must be of such character as to appreciate in favor of Ruiz the mitigating
leave him no opportunity for self-defense in equal combat circumstances of (1) voluntary surrender, (2)
or for escape. Tami failed to proved this, not taking drunkenness which is not habitual and (3) having
advantage of the numerous opportunities he had to leave acted in vindication of a grave offense.
the scene.
ISSUE: W/N the erred in not appreciating the mitigating
Other stuff: circumstances.

1. Tami said it was impossible for both of them to have HELD/RATIO: NO to 1 and 2. YES to 3.
had multiple sexual intercourse with Amelita in such short This Court finds that the first two alleged mitigating
a time (around 2 hours) Court said there is nothing circumstances cannot be appreciated in favor of said
incredible about this, given their physical built and age accused. There is nothing in the affidavit and testimony of
Jesus G. Ruiz that he intended to surrender when he went
2. Tami claims the court erred in imposing the penalty of 5 to the PC headquarters. The Idea of surrender must have
reclusion pereptuas, at most it should only have been 2 been far from his mind because according to him he just
counts of forcible abduction with rape. The SC said that reported the shooting incident to prevent further
he was guilty of 1 forcible abduction with rape and 4 bloodshed.
counts of rape pursuant to People v. Jose: Where the four The defense claims that the accused Jesus G.
accused forcibly abducted Maggie de la Riva and each of Ruiz was intoxicated at the time of the shooting incident
them raped her, the SC held that even while the first act of because he was allegedly drinking Tanduay liquor. He
rape was being performed, the crime of forcible abduction even offered a prosecution witness to join him. This
had already been consummated, so that each of the three conclusion is without any basis. Not all persons who drink
succeeding crimes of the same nature cannot legally be Tanduay liquor get drunk. In fact, the prosecution witness
considered as still connected with the abduction. In other did not testify that Jesus G. Ruiz was drunk or intoxicated.
words, they should be detached and considered The record has no evidence that shows that the liquor
independently of the of forcible abduction and therefore, taken by Jesus G. Ruiz was of such quantity as to have
the former can no longer be complexed with the latter. blurred his reason and deprived him of self control. Said
circumstance must first be established before
VINDICATION drunkenness may be considered as a mitigating
circumstance. Although the last paragraph of Art. 15,
1. THE PEOPLE OF THE PHILIPPINES, plaintiff- Revised Penal Code fails to provide for the degree of
appellee, vs. JESUS G. RUIZ and ALFREDO GUNO, intoxication needed to mitigate the penalty for an offense,
accused appellants. it should be such an intoxication that would diminish the
agent's capacity to know the injustice of his acts, and his
FACTS: will to act accordingly.
- Morning: Accused Ruiz [VISLU (a labor union) However, this Court finds that the mitigating
president] fought with policemen because of the latter circumstance of having acted in vindication of a grave
allowed civilians to use the Honda the former offense should be appreciated in favor of the accused
donated. He called the policemen stupid so medyo Ruiz in so far as the killing of Sgt. Bito is concerned.
nagkainitan. Ruiz left. The evidence shows beyond moral certainty that
accused Jesus G. Ruiz deeply offended as he was then

82
CRIMINAL LAW REVIEW DIGESTS
JUSTICE ROMEO CALLEJO NOTE: = Callejo Ponente

called (abusador) and challenged by the policeman who Melchor escaped. Quidato, Eduardo, and Jonathan were
died to a draw. brought to the hospital. Quidato was pronounced dead on
The question of whether or not a certain personal arrival. Eduardo died two hours later.
offense is grave is dependent upon such factors as the Two informations were filed. One for the murder
social standing of the person, the place, and the time of Quidato and the other for Eduardos.
when the insult was made. In the case at bar, the accused The RTC found them (Bacabac, Jose, Jesus,
Ruiz was the President of VISLU. Considering that he was Edzel and Jonathan) all guilty. They all appealed but only
called an (abusador) and challenged to a draw by the Bacabac filed a brief and only his was given due course.
deceased in the presence of other policemen and right in
front of his own office building where his laborers were Issue: Whether or not Bacabac should be credited with the
then supposed to be reporting preparatory to a loading mitigating circumstance of immediate vindication of a
job, the act of the accused Ruiz in subsequently killing the grave offense. NO.
policeman is attended by the mitigating circumstance of
having acted in vindication of a grave offense. Ratio: Bacabac is not entitled to the mitigating
circumstance of immediate vindication of a grave offense.
For such mitigating circumstance to be credited, the act
BACABAC V. PEOPLE should be, following Article 13, paragraph 5 of the Revised
Penal Code, "committed in the immediate vindication of a
Facts: Hernani Quidato (the victim) was at a dance with grave offense to the one committing the felony
Eduardo Selibio (Eduardo) and Melchor Selibio (Melchor). (delito), his spouse, ascendants, descendants,
And so were Jonathan Bacabac (Jonathan) and Edzel legitimate, natural or adopted brothers or sisters, or
Talanquines (Edzel). relatives by affinity within the same degree." The
Jonathan and Edzel left the dance hall. Quidatos offense committed on Edzel was "hitting" his ear with a
group also left. They encountered Jonathan and Edzel. stick (according to Jesus), a bamboo pole (according to
The two groups had a misunderstanding. Edzel). By Edzel's own clarification, "[he] was hit at [his]
On his way home, Jesus Delfin Rosadio (Jesus), ear, not on [his] head." That act would certainly not be
noticed a commotion. He soon saw that Melchor was classified as "grave offense." And Edzel is Bacabacs
"hugging" Edzel, and later "tying" Jonathan "with his nephew, hence, not a relative by affinity "within the same
hands." He then saw the victim hit Edzel with a "stick." He degree" contemplated in Article 13, paragraph 5 of the
thus told the victim and his companions that Edzel is the Revised Penal Code.
son of Councilor Jose Talanquines, Jr. (Jose), whereupon
Eduardo told Jesus to go away for they might shoot him. Note: There was a long discussion about conspiracy and
Jesus left and went to Edzel's house to report to his father the SC said there was conspiracy. There was also
what he had witnessed. Meanwhile, Edzel and Jonathan treachery. Bacabac is guilty of murder.
escaped.
The victim and his companions then headed for
home. They met Pat. Ricardo Bacabac (Bacabac), with PEOPLE VS. IGNAS
Edzel and Jonathan who are Bacabacs nephews; Also
there were Edzel's father, Jose, Edzels mother, and his June Ignas Y Sanggino and Wilma Grace Ignas are
two sisters. Bacabac and Jose were carrying M-16 husband and wife. However, Wilma was having an affair
armalites, while Jonathan and Edzel had a piece of wood with Nemesio Lopate.
and a revolver, respectively. Later on, Wilma left for Taiwan. She sent 4
Jesus then pointed to the victim and his letters, 2 of which are meant for Romenda Fogayao and
companions as the ones who had manhandled Jonathan the other 2 for Nemesio. In her letter for Romenda, Wilma
and Edzel. The victim apologized, explaining that he and instructed the latter to reveal to June her affair with
his companions mistook Jonathan and Edzel for other Nemesio.
persons. Jesus blurted out, however, "You are just Romenda informed June that Wilma was having
bragging that you are brave. You are only bullying small an affair with Nemesio. She added that the two had spent
children. Bacabac, at that instant, fired his armalite into a day and a night together in a room at Dangwa Inn in
the air, while Jose fired his armalite ("as if spraying his rifle Manila. June got furious. He uttered There will be a day
from right to left") at the victim and Eduardo, even hitting for that Nemesio. I will kill that Nemesio.
Jonathan in the thigh as he (Jonathan) "was on the move Two gunshots were heard by the witnesses in the
to strike Quidato with a piece of wood." Eduardo fell. And evening at the Trading Post (some kind of a vegetable
so did Quidato who was in a kneeling position, and as he market) in La Trinidad, Benguet. The fallen victim was
was raising his hands in surrender, Jose shot him again.

83
CRIMINAL LAW REVIEW DIGESTS
JUSTICE ROMEO CALLEJO NOTE: = Callejo Ponente

Nemesio. He was brought to the hospital but then he died


upon arrival. November 12, 1934
The RTC found June guilty of murder aggravated FACTS: Between 11 and 12 o'clock on the night of May
by treachery, nighttime, and specially aggravated by the 13, 1934, Marciano Retubado, the deceased, and Vicente
use of an unlicensed firearm, with no mitigating Matbagon, the defendant, had a fight at the cockpit in
circumstance and sentenced to the penalty of death by Cebu Province. The fight resulted from a remark made by
lethal injection. the Matbagon respecting the tuba sold by the niece of
Retubado. Shortly after they bit each other, Retubado
ISSUE: called his son and they started home. When they came
Whether or not the RTC erred when it ruled opposite a colo tree, about fifty meters from the cockpit,
that the killing of the deceased was attended Matbagon approached Retubado and stabbed him in the
by evident premeditation, treachery and breast. Emiliano Retubado cried for help. Rufino Surigao
nighttime??? -- YES was the first to arrive. Retubado struck the Matbagon on
Whether or not the RTC erred in the head with the bottle that he was carrying. The bottle
appreciating the alleged use of an was broken and the light went out. A struggle between the
unlicensed .38 caliber firearm as an accused and the deceased followed. Retubado received in
aggravating circumstance??? -- NO all four wounds. He died in a few minutes from the wounds
that he had received on the breast and on the left side of
RULING: The amended information does not definitely the chest. The accused then ran away.
and categorically state that the unlawful killing was Lower Court: Guilty of murder because the crime
attended by the aggravating or qualifying circumstances of was committed with treachery; aggravating circumstance
treachery, evident premeditation, and nocturnity. of nocturnity was offset by the mitigating circumstance of
The 2000 Revised Rules of Criminal Procedure passion and obfuscation, crime committed a few minutes
requires that the qualifying and aggravating circumstances after he was bitten by the deceased.
must be specifically alleged in the information. Although
the Revised Rules of Criminal Procedure took effect only Issue: Whether the court erred in the appreciation of the
on December 1, 2000 or long after the fatal shooting of aggravating and mitigating circumstances. YES
Nemesio, as a procedural rule favorable to the accused, it
should be given retrospective application. Hence, absent SC: Evidence does not justify the finding of the trial judge
specific allegations of the attendant circumstances of as to the classification of the crime or the appreciation of
treachery, evident premeditation, and nocturnity in the the modifying circumstances. There could not be, under
amended information, it was error for the trial court to the circumstances of this case, both treachery and the
consider the same in adjudging appellant guilty of murder. aggravating circumstance of nocturnity, because the
As worded, we find that the amended information under nocturnity would be included in the treachery as an
which June was charged and arraigned, at best indicts him inseparable incident, and should not be considered
only for the crime of homicide. Any conviction should, separately (U. S. vs. Salgado).
thus, fall under the scope and coverage of Article 249 of With respect to the mitigating circumstance of
the Revised Penal Code. passion and obfuscation, it was improperly appreciated in
Under R.A. No. 8294, which took effect on this case. At least half an hour intervened between the
July 8, 1997, where murder or homicide is committed fight at the cockpit and the stabbing. The accused in
with the use of an unlicensed firearm, the separate waiting for the deceased near the colo tree and in
penalty for illegal possession of firearm shall no attacking him was actuated by a desire for revenge.
longer be imposed since it becomes merely a special The attack was not the result of a sudden impulse of
aggravating circumstance. This Court has held in a natural and uncontrollable fury (People vs. Hernandez).
number of cases that there can be no separate As to the question of whether or not the crime
conviction of the crime of illegal possession of firearm was committed with alevosia or treachery, if the accused
where another crime, as indicated by R.A. No. 8294, is had been hiding behind the tree and had stabbed the
committed. Although R.A. No. 8294 took effect over a deceased without warning, the crime would undoubtedly
year after the alleged offense was committed, it is have been committed with treachery, but that is not what
advantageous to June insofar as it spares him from a took place in this case. The accused was waiting near the
separate conviction for illegal possession of firearms and colo tree, and when a braza away he was seen by the son
thus should be given retroactive application. of the deceased, and presumably by the deceased. The
accused with a knife in his hand walked up to the
deceased and stabbed him in the breast. Under these
PEOPLE v VICENTE MATBAGON circumstances we think it is clear that there was no

84
CRIMINAL LAW REVIEW DIGESTS
JUSTICE ROMEO CALLEJO NOTE: = Callejo Ponente

treachery or alevosia as that term is used in the Revised deceased returned with a slingshot (Indian pana) which he
Penal Code. used, hitting accused-appellants father in the mouth.
As to whether or not nocturnity should be taken Palabrica reported the matter to the police. The next day,
into account as an aggravating circumstance in this case, he was told by his sister that the deceased and some
tt was said in the case of People vs. Trumata and companions were looking for him. He, therefore, looked for
Baligasa, that nocturnity should not be estimated as an the deceased and found him on Ylagan Extension Street,
aggravating circumstance, since the time for the playing billiards with some companions. When the
commission of the crime was not deliberately chosen by deceased saw him, he said, so you are here, while
the accused; that if it appears from the record that the drawing the knife tucked in his waist. Accused-appellant
accused took advantage of the darkness for the more claimed that he then stabbed the deceased in the stomach
successful consummation of his plans, to prevent his with the knife he was carrying and ran away. (When asked
being recognized, and that the crime might be perpetrated why the deceased had two stab wounds, accused-
unmolested, the aggravating circumstance of nocturnity appellant said that after the deceased had been stabbed,
should be applied (U.S. vs. Billedo). In the present case he fell down near a pedicab and that he may have
none of the foregoing reasons exists for appreciating sustained injuries as a result).
nocturnity as an aggravating circumstance.
Prosecutions version: The prosecution presented an
Separate Opinions MALCOLM and eyewitness, Domingo Lombreno, Jr., the caretaker of the
GODDARD, JJ., dissenting: We are of the opinion that the billiard hall where the incident happened. He testified that
crime committed was murder qualified as such because of that night, Silvano played a billiard game with Andrew
the presence of treachery and that consequently the Limpio. While the game was in progress, Lombreno, Jr.
judgment should be affirmed. said he noticed accused-appellant shoving people out of
his way as he walked towards the deceased, who was
HULL, J., dissenting: The RPC provides that then waiting for his turn at the billiard table. He said that
nocturnity is an aggravating circumstance when the when accused-appellant arrived, he said to Silvano, So
circumstance "may facilitate the commission of the you are here! Then accused-appellant stabbed the
offense." This language is clear and contemplates that deceased and quickly made his exit. According to
each case will be considered on its merits. There is no Lombreno, Jr., the deceased was unarmed.
need of an elaborate argument to expound the meaning of Trial Court rendered a decision finding accused-
the section if the plain words therein used are given their appellant guilty of murder qualified by treachery with the
every-day current use. aggravating circumstance of evident premeditation
The test fixed by the statute is an objective one. If I (penalty: death). Accused-appellants claims that he acted
read the majority opinion right, before nocturnity can be in the immediate vindication of a grave offense committed
considered, it must meet the objective test fixed by the against his father (mitigating circumstance).
statute and a subjective test fixed by the majority opinion.
To hold that a sudden, murderous assault committed on a Issues:
dark night has no greater chance of success than if 1. WON the crime commited was murder qualified with
committed in broad day-light, does violence to the treachery with the aggravating circumstance of evident
experience of mankind. premeditation -YES!
2. WON court erred in not appreciating the mitigating
circumstance of immediate vindication of a grave offense
PEOPLE VS. ROBERTO PALABRICA committed against his father. -NO!

Facts: An information for murder was filed against Roberto There was treachery in the commission of the crime as
Palabrica alleging that he stabbed Vic Jun Silvano using a shown by the following: (1) the employment of means of
bladed weapon, with intent to kill and with treachery and execution which gave the deceased no opportunity to
evident premeditation, causing his death. Accused- defend himself or to retaliate and (2) the deliberate and
appellant Palabrica testified solely in his defense. conscious adoption by accused-appellant of the means of
Palabricas version: Palabricas family owned a store execution. The deceased Silvano was unsuspecting when
along the national highway leading to San Carlos City, attacked as he was waiting for his turn at the billiard table
Negros Occidental. One evening, the deceased, Silvano, when accused-appellant stabbed him. Moreover, the
was in his store for some beer. Silvano had a quarrel with weapon used and the nature of the injuries inflicted,
another customer which Palabrica tried to pacify. Silvano showing that accused-appellant aimed at a vital spot of the
resented this and pulled out a knife. Palabrica broke a deceaseds body, establish that accused-appellant
bottle and, upon seeing this, the deceased ran away. The

85
CRIMINAL LAW REVIEW DIGESTS
JUSTICE ROMEO CALLEJO NOTE: = Callejo Ponente

deliberately adopted the particular mode of attack to Caber admitted killing Ramirez but interposed
ensure the commission of the offense with impunity. that he did so in self defense. Caber declared that he was
The killing of Silvano was premeditated. Contrary 63yrs old, and that he worked at Caltex gasoline station.
to his claim, accused-appellant had a reason for attacking One morning, on his way to work as he was alighting from
Silvano (Silvano hit his dads mouth using an Indian pana). the pedicab, he heard someone shout Manong Caber,
Although he denied it, accused-appellant could not have someone is going to kill you! and turned to see Ramirez
helped harboring ill will towards the deceased. He even who was about to stab him. He was able to parry the blow
admitted that on he went around looking for the deceased. and turn the knife towards his attacker and in the process
He went to many places and did not stop until he found stabbing him. (back story to why Ramirez wanted to kill
him at the billiard hall. In addition, it is to be noted that Caber: Allegedly, Cabers wife filed a rape case against
accused-appellant armed himself throughout the search Ramirez as a result of which Ramirez was detained. So in
with a long knife. All these convince us that the killing of short, Caber is arguing that after being released, Ramirez
the deceased had been premeditated. wanted to kill him because of revenge. Also, Caber insists
The accused-appellant had practically an entire that he holds no grudge against Ramirez for what the
day to reflect on the consequences of his act. Thus, the latter did to his wife)
following elements of evident premeditation: (1) the time TC: Caber is guilty with mitigating circumstance
when the accused determined to commit the crime; (2) an of passion/obfuscation and qualifying circumstance of
act manifestly indicating that the accused had clung to his premeditation and treachery.
determination; and (3) sufficient lapse of time between
such determination and execution to allow him to reflect Issue: 1) Whether there was self defense? 2) Whether
upon the consequences of his act were established in this there was premeditation and treachery? 3) Whether there
case. was passion/obfuscation?43
Accused-appellant claims that the mitigating
circumstance of immediate vindication of a grave offense Held: 1) No self defense!
should have been appreciated in his favor. The The defense that Caber killed Ramirez in self
circumstance cannot be appreciated where, as here, defense has no merit. Upon invoking self defense, it was
the accused had sufficient time to recover equanimity. the burden of Caber to prove that: 1) the victim was guilty
For in this case, the incident at the store, from which of unlawful aggression; 2) there was reasonable necessity
reason accused-appellant claims he stabbed the for the means employed; and 3) that there was no
deceased, happened the night before. Thus, he had sufficient provocation on part of the person making
sufficient time to regain his composure. In fact, he defense.
sought the assistance of the police. This cannot be Proof of unlawful aggression by the victim is an
reconciled with his present claim that he acted in the indispensable element of self defense. But note that even
immediate vindication of a grave offense committed if there was unlawful aggression in the beginning, once it
against his father. has ceased, hostility on the part of the person making
The killing is murder, qualified by treachery. As defense should also cease.
the generic aggravating circumstance of evident The circumstance of running after Ramirez and
premeditation was alleged and proven and as there is no then stabbing him twice belies the argument of self
mitigating circumstance, the trial court correctly sentenced defense. Assuming that indeed, Ramirez initially attacked
accused-appellant to death Caber, the fact that the former ran away after doing so
means the unlawful aggression has ceased. Cabers act of
pursuing Ramire when there was no more threat towards
PEOPLE VS FRANCISCO CABER him belies self defense.

Facts: Julian Rama saw Francisco Caber chasing 2) No premeditation and treachery!
Teodolfo Ramirez with a bladed weapon which was locally For evident premeditation, prosecution failed to
known as pisao. Rama was able to identify them because prove: 1) the time when Caber determined that he will
Caber was his fellow tanod while Ramirez was his commit the crime; 2) an act manifestly indicating that he
kumpadre. had clung to his determination; and 3) sufficient lapse of
Ramirez ran towards Rama asking for help. time between the determination and execution to allow
Rama raised his hands telling Caber to stop. Although him the opportunity to reflect on his actions. Prosecution
Ramirez used Rama as shield Caber was still able to stab only presented the circumstance that Caber wanted
Ramirez on the chest, twice. Caber surrendered to Rama revenge because Ramirez raped his wife.
whereas Ramirez was pronounced dead on arrival at the Theres also no treachery. To prove treachery it
hospital. must be shown that: 1) at the time of the attack, the victim

86
CRIMINAL LAW REVIEW DIGESTS
JUSTICE ROMEO CALLEJO NOTE: = Callejo Ponente

was not in a position to defend himself; and 2) the neighbors mauled him. He ran home to get a knife tapos
accused consciously adopted the particular means, nagwala na siya, wanted to pick a fight to whoever he
methods or form of attack employed by him. In this case, sees. He claims that in the process, Alfredo tried to stab
Ramirez sought cover behind Rama hence, he knew the him then the latter ran away pero nadapa daw so
danger facing him and tried to escape it albeit nasaksak niya sarili niya
unsuccessfully. There can be no treachery under those LC, CA: guilty of homicide but with a mitigating
circumstances. circumstance: passion or obfuscation

3) No mitigating circumstance of passion/obfuscation! Issue: tama ba na may passion or obfuscation?


In order that passion/obfuscation might be
appreciated it must be shown that: 1) an act both unlawful SC: meron
and sufficient to produce such a condition of mind [of Passion and obfuscation exist when
passion/obfuscation]; and 2) said act which produced the (1) there is an act, both unlawful and sufficient to produce
obfuscation must not be so far removed from the time of such a condition of the mind, and
the commission of the crime. This is because after a (2) the said act which produced the obfuscation was not
considerable length of time, the perpetrator might have far removed from the commission of the crime by a
already calmed down. Moreover, the act must be shown to considerable length of time, during which the perpetrator
have arisen from lawful sentiments and not that of might recover his normal equanimity.
lawlessness revenge There is passion and obfuscation when the
In this case, Caber admitted himself that he crime was committed due to an uncontrollable burst
harboured no ill feelings and that he didnt even know of passion provoked by prior unjust or improper acts,
that Ramirez had already been released. This in itself or due to a legitimate stimulus so powerful as to
belies the claim that he acted out of overcome reason.
passion/obfuscation. Furthermore, the stabbing In this case it was established that petitioner
incident took place 3days after the rape took place. and his wife had a violent altercation and that
Thus, the act which supposedly caused the petitioner was mauled by his neighbors after he
passion/obfuscation was so far removed from the kicked some of them for laughing at him. These
stabbing. In United States v. Sarikala the Court ruled that events and circumstances prior to the killing of
the lapse of more than 24 hours, reckoned from the Alfredo Gonzales could have caused unusual
commission of the act which produced the passion or outbursts of passion and emotion on petitioners part.
obfuscation up to the time of the commission of the felony, These resulted in the tragic stabbing of the victim thus
constituted a considerable period of time after which such entitling petitioner to the mitigating circumstance
circumstance would no longer be deemed present. analogous to passion and obfuscation.
NOTE: mitigating circumstance of voluntary
surrender is appreciated. Caber has not yet been arrested PASSION AND OBFUSCATION
when voluntarily surrendered himself to a person in
authority. PEOPLE V. VENTURA

Spouses Jaime and Aileen Bocateja were sleeping in their


DANAFRATA VS PEOPLE room. At around 2am, Jaime was roused from his sleep by
accused Felix Ventura (armed with firearm) and Arante
Accused Danafrata and his wife were having an argument Flores (with a bladed weapon) who were able to stealthily
in the streets when the latter hit the former with a plastic enter the house by cutting a hole in the kitchen door.
chair while Danafrata hit her back (slugging match). Ventura pointed the gun at Jaimes face, announce a hold
Accused ran home then came back again, kicking the up and hit Jaime on the head. The 2 then struggled for the
neighbours he encountered. Because of his behaviour, he gun, and since Jaime was winning, Ventura called on
was mauled by 3 neighbors so he had no choice but to go Flores to stab Jaime. Flores did stab him 3 times. When
home again to get a knife. He went back to retaliate. He wife Aileen saw her husband in danger, she cried for help
saw the father of one of those who mauled him so and Flores stabbed her 4 times (she died eventually). The
accused challenged him to a fight. However, he spotted spouses niece who was sleeping upstairs, ran downstairs
one of the 3 and so he stabbed Alfredo in the chest. and recognized Flores as a former employee of the
spouses butcher shop. She called on their neighbors for
Accuseds version: inaway nga siya ng asawa niya so help. Ventura and Flores then fled.
napahiya siya sa mga tao sa paligid niya. He was so The police intercepted the accused and from
humiliated he kicked a table but because of this, 3 them recovered a .38 caliber revolver with bullets and a

87
CRIMINAL LAW REVIEW DIGESTS
JUSTICE ROMEO CALLEJO NOTE: = Callejo Ponente

blood-stained knife. Bombo radio covered their arrest and showed their careful and deliberate plan of
when asked why, Ventura answered that he suspects that carrying out a killing.
his wife was carrying on an affair with Jaime. It turned out - There was clear conspiracy between the
that Venturas wife was a maid for the Bocateja spouses. accused. Theyre both principals
Ventura saw his wife wearing a new ring, and the wife said - There was taking advantage of superior strength
it was from Jaime who was courting her. She was in killing Ailleen. To take advantage of superior
dismissed as a maid when Aileen found out their illicit strength means to purposely use excessive force
relations. After Ventura and his wife split, the former tried out of proportion to the means of defense
to confirm the truth from Flores who worked for Bocatejas available to the person attacked. It depends on
meat shop. He confirmed the affair and helped Ventura the age, size and strength of the parties, and is
with the crime. considered whenever there is a notorious
Trial court: guilty of attempted murder (Jaime), inequality of forces between the victim and the
with aggravating circumstance of evident premediation, aggressor, which is taken advantage of by him in
dwelling, nighttime and breaking of door to gain entrance. the commission of the crime. Unlike in treachery,
Reclusion temporal max. Also, guilty of murder (Aileen), where the victim is not given the opportunity to
aggravated by abuse of superior strength, dwelling, defend himself or repel the aggression, taking
nighttime, breaking of door. Death. advantage of superior strength does not mean
that the victim was completely defenseless.
Issue: w/n not guilty of murder (accused argue homicide Abuse of superiority is determined by the excess
lang) MURDER! of the aggressor's natural strength over that of
the victim, considering the momentary position of
Ruling: [for passion and obfuscation] Court ruled out the both and the employment of means weakening
mitigating circumstance of passion and obfuscation the defense, although not annulling it. Hence, the
as mitigating circumstance. While it is true that fact that Aileen attempted to fend off the attack
jealousy may give rise to P&O, it is necessary that the on her and her husband by throwing nearby
act which produces the obfuscation was NOT far objects, such as an electric cord, at appellant
removed from the commission of the crime by a Flores does not automatically negate the
considerable length of time, during which the possibility that the latter was able to take
perpetrator might recover his normal equanimity. This advantage of his superior strength.
is the same with immediate vindication of a grave
offense it cannot be considered where sufficient
time elapsed for the accused to regain his composure. PEOPLE V. SALAZAR
Here, Venturas suspicions were aroused a
week before the stabbing incident, when he first FACTS: The accused is a moro native of Zamboanga.
confronted his wife about the ring. Also, on the day One morning, he invited his common-law wife to go with
when they planned to commit the crime, 10 hours lapsed him to gather nipa for the repair of their house. Romana
from the time they left their home, armed with weapons then arrived and invited Maxima to accompany her to her
already, until they entered the Bocateja residence. Within house to get palay. Because of the invitation of Romana,
that time, Ventura even had time to change his clothes at Maxima refused to go with her husband, which aroused
a relatives house, accused even had dinner at the market. his anger. At that time, the accused already entertained
At the victims residence, they waited 3 hours (because the suspicion that his wife was having illicit relation with
they didnt want to be caught) before they actually carried Fortunato, the husband of Romana, to the extent that he
out their plan. Certainly, there was enough time that believed that the child his wife was bearing was the result
passed for Venturas emotions to cool. of such illicit relation. This incident started the accused on
a killing rampage leaving in its wake sixteen dead and
Other crim-related things: some wounded.
- There was evident premeditation. The execution
of the act was preceded by cool thought. It ISSUE: W/N the accused may avail of mitigating
requires: (1) the time when the accused circumstance of obfuscation arising from jealousy? NO.
determined to commit the crime; (2) an act
manifestly indicating that the accused clung to his HELD: Such cannot be invoked in favour of the
determination; and (3) sufficient lapse of time accused considering that his relationship with his
between such determination and execution to common-law wife was illegitimate. In addition, many
allow him to reflect upon the circumstances of his days had already passed from the discovery of the
act. Also, the fact that they were armed only alleged infidelity of his common-law wife before he

88
CRIMINAL LAW REVIEW DIGESTS
JUSTICE ROMEO CALLEJO NOTE: = Callejo Ponente

committed the crime allegedly in vindication of his careful study, there is no evidence that supports
honor. As a matter of fact he admitted having planned Rubens testimony.
his vengeance long before the opportune moment Though the exceptional circumstance provided for in
came to carry it out. Art 247 of the RPC couldve been applied, but the
accused failed to present any evidence in his favor.
The Court ruled that for this to apply, there must be at
PEOPLE V. RUBEN TAKBOBO least clear and convincing evidence, to apply such
exemption. The burden of evidence now, has been
Facts: shifted to accused, which he failed to give credence
Accused Ruben Takbobo, a middle-aged fisherman to. He needs strength of his own evidence and not the
from Cebu, is charged with killing his wife, Lucia, weakness of the prosecution
using a knife AND bolo, which instantaneously killed Note also that the Court finds no reason to doubt the
her. testimony of the child.
Though pleading guilty, the court still orders Also, the Court held that his statement that he killed
prosecution and defense to submit evidence as to his wife by accident AND that he was really aiming for
motive. Cadiz for sleeping with her is NEGATED by the
Lucia was killed on March 25, 1991, at night time. number of stab wounds certified in the medico-legal
This was all witnessed by their youngest daughter report, and affirmed by the testimony of the child. Also
Madilyn, who woke up in the middle of the night due the report shows that the deceased was wearing
to the noise of the quarreling. panties.
She testifies as a child witness that she saw her Police Inspector Singco, who took accuseds affidavit
father grab the bolo and hunting knife towards her and statement also testified that accused did not
mom, before hacking Lucia at the feet, then the neck, mention the act of infidelity at that time. The normal
then the hands, then the armpit, then on the breast. human reaction to such incident is to include such fact
Shortly after the incident, accused Ruben went to the in the first step of the investigation.
authorities voluntarily, explaining the story, without THEREFORE, there being no passion or obfuscation
any mention of his motive for killing her, nor the fact attending, the same cannot be appreciated. The dual
that he came home from fishing shortly before the requisites therefor: (1) there be an act both unlawful
killing. and sufficient to produce such condition of mind; and
However, it was mentioned that the accused has a (2) said act was not far removed from the
short temper, and on two separate occasions, wielded commission of the crime by a considerable length
the bolo to his other daughters, but only with minor of time, during which the perpetrator might recover his
cuts. moral equanimity.
Accused though, told the court that he killed Lucia HOWEVER, the mitigating circumstances of voluntary
because he caught her sleeping with another man. surrender and voluntary plea of guilty (which was
The testimony of the accused states that coming made prior to the prosecutions presentation of
home at 3:00 am from fishing, he finds his neighbor, evidence) must be appreciated.
Cadiz Catulong, sleeping with his wife and that his SIDE NOTE: Though there are two mitigating and not
wife isnt wearing underwear. Accused tried to kill aggravating circumstances, the penalty for parricide
Cadiz, but Lucia pushed him away, causing Cadiz to being reclusion perpetua, the same is indivisible.
escape through the window. Therefore, the penalty cannot be lowered to the next
In the process, he struck his wife, then ran outside to lower degree. This doctrine is applicable to indivisible
look for Cadiz but to no avail penalties, thus no matter how many mitigating
In this appeal, Accused Ruben is claiming for circumstances you have in an indivisible penalty like
mitigating circumstances of (a) passion and reclusion perpetua, you cannot lower it.
obfuscation, (b) voluntary surrender, and (c) voluntary
plea of guilty.
PEOPLE V. LOPEZ
Issue: Can those mitigating circumstances be claimed by
Ruben? Doctrine: The exercise of a lawful right cannot be the
proper source of obfuscation that may be considered a
Ruling: Not for Passion and Obfuscation, only for mitigating circumstance
Voluntary Surrender and Voluntary Plea of Guilty
The reason why passion and obfuscation would not Facts: The Lopez family (Placido, Luding, and their
apply is that, after the High Courts thorough and children) lived in a shanty on a patch of land owned by

89
CRIMINAL LAW REVIEW DIGESTS
JUSTICE ROMEO CALLEJO NOTE: = Callejo Ponente

one Perla Castro in Baguio City. Castro had been seeking until Sola decided to leave Hicks. Sola found another afro-
the ouster of the Lopezes from her land since 1993. In american lover in Wallace Current. When Hicks learned
fact, the Lopez spouses even signed an Acknowledgment about this he went to Currents house to confront the two.
Receipt that they received the amount of five thousand While conversing, Hicks said God damn, Ive made up my
pesos (P5,000.00) as an assistance from Perla Castro to mind as he was about to grab his revolver. Current got
transfer voluntarily their shanty which they have illegally hold of Hicks hand but the latter slapped it away. Current
constructed on Castros land. The Lopezes never left ran inside a room just as Hicks drew his revolver and shot
anyway. Sola, who was close by in the sala of the house, on the left
Castro sold her land to Liwayway Maramat. One side of the breast. Sola died. Hicks was charged and
day, Maramat and Castro decided to inspect the found guilty of murder, sentenced to death.
excavations done over the land. When they arrived there,
Perla immediately confronted the Lopezes about their Issue: W/N there is the mitigating circumstance of passion
digging and told them to stop. She told them to dig instead and obfuscation?
on the fifty-square meter lot supposedly waived in their
favor by a certain Josie Ramos. Held: None!
Perla then asked Maramat to see for herself the Generic aggravating circumstance of premeditation
new lot where the Lopezes were to transfer. As an SC held that the crime was attended with the aggravating
afterthought, Perla called on Joselito (the son of Lopez) to circumstance of premeditation because it found, according
show him the new site. While the 3 were talking, Joselito to one of the witnesses, that before the crime, the witness
suddenly grabbed the hair of Perla at the back and started and Hicks were drinking and the latter, while cleaning a
hacking her with a bolo. Perla died from the wounds. revolver said that Solas time had come. SC found that
Joselito was prosecuted for the crime of murder. Hicks deliberately and after due reflection had resolved to
Joselito contends that the trial court should have kill the woman who had left him for another man, and in
appreciated the mitigating circumstance of passion and order to accomplish his perverse intention with safety,
obfuscation. notwithstanding the fact that he was already provided with
a clean and well-prepared weapon and carried other
Issue: Was there a mitigating circumstance of passion or loaded cartridges besides those already in his revolver, he
obfuscation? NO. entered the house, greeting everyone courteously and
conversed with his victim, in what appeared to be a proper
Ruling: For passion and obfuscation to be properly manner, disguising his intention and claiming her by his
appreciated, it must arise from lawful sentiments. In this apparent repose and tranquility, doubtless in order to
case, the trial court was correct in not appreciating the successfully accomplish his criminal design, behaving
mitigating circumstance when it ruled that the act of Castro himself properly as he had planned to do beforehand.
in demanding that the Lopez family vacate her land and
transfer elsewhere and discontinue their excavation on the Absence of mitigating circumstance
land was not unlawful and unjust as she was exercising As against the two foregoing aggravating circumstances
her right to her land. no mitigating circumstances is present, not even that
The exercise of a lawful right cannot be the mentioned in paragraph 7 of article 9 of the Penal Code,
proper source of obfuscation that may be considered to wit loss of reason and self-control produced by
a mitigating circumstance. Since 1993, Castro had been jealousy as alleged by the defense, inasmuch as the
seeking the ouster of the Lopezes who were unjustly only causes which mitigate the criminal responsibility
occupying her land. This was further bolstered by a written for the loss of self-control are such as originate from
promise manifested by the Lopezes in their legitimate feelings, not those which arise from
Acknowledgment Receipt to vacate the subject land and vicious, unworthy, and immoral passions.
after receiving P5,000.00 supposedly for their new house.
Despite the fact that was an exchange of harsh words
between Perla Castro and Luding Lopez, this cannot PEOPLE VS GUILLERMO BELLO
overturn the fact that the Castro had long been unjustly KEYWORD: White Slave Trade
deprived of the possession of her own land.
FACTS: Bello is a 54 year old widower who was having a
common law relationship with Alicia Cervantes who was
UNITED STATES VS. HICKS (once you go black) then only 24 years old. Because of financial strain, Bello
induced Alicia to work as a public hostess in Marings Bar;
Facts: Agustina Sola was the mistress of Augustus Hicks, to which she agreed. Bello was very infatuated with Alicia
an afro-american. They lived together for about 5 years that he watched her dance all the time in the bar. One

90
CRIMINAL LAW REVIEW DIGESTS
JUSTICE ROMEO CALLEJO NOTE: = Callejo Ponente

night, Bello saw Alicia enter a movie house with another ABUSE OF CONFIDENCE AND OBVIOUS
guy. Bello took her out of the movie house and reminded UNGRATEFULNESS - There is nothing to show that the
her to be discreet about her personal conduct. assailant and his common-law wife reposed in one
One night, he approached Alicia while the latter another any special confidence that could be abused, or
was working in the bar to ask for some money. He was any gratitude owed by one to the other that ought to be
sent away by Maring, the bar owner, telling him to stop respected, and which would bear any relation, or
bothering Alicia as he was already an old man. On his way connection, with the crime committed. None is inferable
home, Bello encountered Justo and Luis Marasigan who from the fact that the accused was much older than his
said so this is the man whose wife is being used by victim, or that he was penniless while she was able to earn
Maring for white slave trade. Infuriated, Bello went to the a living and occasionally gave him money, since both lived
bar, got hold of Alicia from behind with his left hand and together as husband and wife. Neither is it shown that the
stabbed her several times with a balisong his right hand. accused took advantage of any such special confidence in
He surrendered immediately by going to the municipal order to carry out the crime.
building.
The trial court made a finding of treachery, PASSION AND OBFUSCATION the remarks of the
evident premeditation, and in cold blood and without Marasigan brothers hurt the feelings of Bello
provocation. In the dispositive portion however, the trial prompting him to indulge in heavy drinking and
court considered the aggravating circumstances of (1) thereafter, plead to Alicia to leave her work. This
nighttime (2) abuse of confidence and obvious constitutes passion and obfuscation.
ungratefulness and (3) superior strength offset only by the
mitigating circumstance of voluntary surrender. Bello was
convicted to die through electrocution. Pelonia v People

ISSUE: Pelonia was charged with murder.


Whether or not the aggravating circumstances of
treachery, evident premeditation, superior There was a fiesta. Deceased Ignacio Nacilla and his
strength, nighttime and abuse of confidence and friends went with his friends to the house of Pelonia for
obvious ungratefulness may be appreciated. dinner. After preparing dinner (with a bolo!), Pelonia told
NO the group into the dining room. However, Nacilla refused
Whether or not the mitigating circumstance of and said that he was not there to eat, but to kill.
passion and obfuscation may be appreciated - Apparently, Nacilla had a grudge against Pelonia, because
YES Pelonia ratted him out to the Marines for being abusive,
and the Marines manhandled him for it. The deceased
HELD: was a head taller than Pelonia.
TREACHERY there was no treachery. True, Alicia may His friends tried to calm Nacilla down, but he was
have been stabbed at the back yet this is but a really mad and told Pelonia that Pelonia was depending
continuation of earlier stabbing which Bello inflicted in the so much on his garrand rifle, his issued firearm as
breast, hypogastric region and left wrist. The back stab member of the CHDF.
was inflicted when Alicia was about to run. Incensed at the comment and the fact that it was
spoken in his own home and in front of his visitors, and the
EVIDENT PREMEDITATION there was no evident pre- fact that Nacilla was not even invited, Pelonia went
meditation. Bello carried the Balisong not because he upstairs to his room to get his rifle. He fired a warning
planned on killing Alicia but because he always brings it shot, but Nacilla did not budge. The accused wife even
for protection. The killing was a spur of the moment. pleaded with Nacilla to leave, but he refused. So, Pelonia
shot him.
SUPERIOR STRENGTH there was no superior strength. Pelonia claimed self-defense. He claimed that
Bello was an old man and invalid [baldado] while Alicia Nacilla got Pelonias cooking bolo and was about to thrust
was in the prime of her youth, and not infirm. The facts are it, so he shot him.
not sufficient to draw a comparison of their relative The CA gave Pelonia 3 mitigating circumstances:
strength. voluntary surrender, immediate vindication of a grave
offense, and sufficient provocation.
NIGHTIME although the killing was done at night, it was
not purposely sought or taken advantage of by Bello. In Was there self-defense?
fact, Marings bar was well lit. SC: No self-defense because no unlawful aggression.
Court did not believe that Nacilla was able to get the bolo

91
CRIMINAL LAW REVIEW DIGESTS
JUSTICE ROMEO CALLEJO NOTE: = Callejo Ponente

during the time that Pelonia went to his room to get the The circumstance of abuse of superior strength, qualifying
rifle. the crime of murder, which the trial court found to have
been proven, has not been established beyond a
Can immediate vindication and sufficient provocation reasonable doubt. In the case of United States vs. Devela,
be appreciated together in this case? this court said that "the mere fact that the number of the
SC: No. The mitigating circumstance of having acted in assailants is superior to that of those attacked by them is
the immediate vindication of a grave offense was properly not sufficient to constitute the aggravating circumstance of
appreciated. Pelonia was humiliated in front of his guests abuse of superiority." In this case we have the
and kin in his own house. It is settled, however, that the photographs of the body of the Yu Hiong showing that he
mitigating circumstance of sufficient provocation had a strong constitution: but there is no evidence of the
cannot be considered apart from the circumstance of physical constitution of the accused Epifanio Diokno and
vindication of a grave offense. These two Roman Diokno. Therefore, we cannot determine whether
circumstances arose from one and the same incident or not said accused were physically stronger than the
so that they should be considered as only one deceased and whether or not they abused such
mitigating circumstance. superiority.
Neither does this court find the existence of the
Guilty of homicide. other circumstance qualifying murder, that is, evident
premeditation, proven beyond a reasonable doubt
because, even assuming that both the accused went to
People vs. EPIFANIO DIOKNO and ROMAN DIOKNO San Pablo, Laguna, each carrying the knife used by him in
attacking Yu Hiong, it being customary for the people of
Yu Hiong was a vendor of sundry goods in Lucena. said province to carry it, it cannot be inferred with certainty
Salome Diokno, engaged to Yu Hiong, invited the latter to from the mere fact that they carried knives that their
go with her. Yu Hiong and Salome Diokno went to the intention in going to San Pablo was to look for the
house of Vicente, Salome's cousin. As they found nobody deceased in order to kill him. In order that premeditation
in the house, they went on their way up to San Pablo, may be considered either as an aggravating circumstance
Laguna. Roman Diokno telegraphed his father Epifanio or as a qualifying circumstance, it must be evident, that is,
Diokno, who was in Manila, informing him that Salome had the intention to kill must be manifest and it must have
eloped with the Chinese Yu Hiong. Epifanio and Roman been planned in the mind of the offender and carefully
went to San Pablo, Laguna. They saw Yu Hiong coming meditated. It is not enough that it arose at the moment of
down the stairs. When Yu Hiong saw them, he ran the aggression.\
upstairs and they pursued him. As the Chinese found the Therefore, there having been neither abuse of
door of the house locked, he shouted that it be opened for superior strength nor evident premeditation, the crime
him. At that moment, he was overtaken by the Epifanio committed by the accused is simple homicide.
who carried balisong. Yu Hiong fell on his knees and
implored pardon. In that situation Roman Diokno stabbed MITIGATING- *read Reyes book. As a general rule, you
him with the knife in the back and later in the left side. cannot have both vindication of a grave offense and
Epifanio Diokno also stabbed him once. Yu Hiong fell on passion or obfuscation in 1 case. This case is an
the landing of the stairs in the balcony, and there he was exception.
again stabbed repeatedly. Then Roman Diokno said: The presence of the immediate vindication of
"Enough, father." Yu Hiong lost consciousness. a grave offense to said accused, may be taken into
consideration in favor of the two accused, because
ISSUE: WON Ramon should be acquitted? No. WON although the elopement took place on January 4,
abuse of superior strength was present so as to qualify the 1935, and the aggression on the 7th of said month and
crime of murder? No. Guilty only of homicide. year, the offense did not cease while Salome's
whereabouts remained unknown and her marriage to
RULING:The testimony of the eyewitnesses leave no the deceased unlegalized. Therefore, there was no
room for doubt that Roman Diokno cooperated with his interruption from the time the offense was committed
father and stabbed the deceased Yu Hiong with a knife in to the vindication thereof. Our opinion on this point is
different parts of the body. Furthermore, the deceased based on the fact that the herein accused belong to a
stated in his ante mortem declaration that it was Roman family of old customs to whom the elopement of a
Diokno who inflicted the necessarily mortal wound in his daughter with a man constitutes a grave offense to
back, which caused his death. their honor and causes disturbance of the peace and
tranquility of the home and at the same time spreads
AGGRAVATING

92
CRIMINAL LAW REVIEW DIGESTS
JUSTICE ROMEO CALLEJO NOTE: = Callejo Ponente

uneasiness and anxiety in the minds of the members


thereof. THE PEOPLE OF THE PHILIPPINES vs. RUFELINO
The presence of the having acted upon an ZAPATA and FERNANDICO TUBADEZA
impulse so powerful as naturally to have produced
passion or ofuscation, may also be taken into FACTS:
consideration in favor of the accused. The fact that 1. Feb 15, 1951; nighttime (around 8pm): Fausta
the accused saw the deceased run upstairs when he Tubadeza (Fausta), a 60 year old woman, was
became aware of their presence, as if he refused to cutting firewood when she was approached by
deal with them after having gravely offended them, the two accused, Zapata and Tubadeza. Zapata
was certainly a stimulus strong enough to produce in confronted her, saying You are the woman who
their mind a fit of passion which blinded them and led bewitched my wife.Using a piece of wood, he
them to commit the crime with which they are then beat her while Tubadeza dragged her
charged. towards the house Councilor Simeon Tubadeza.
The fact of surrendering himself immediately to Her husband Mariano heard what was happening
the agents of persons in authority, should also be taken and rushed towards the commotion, but Zapata
into consideration in favor of the accused Epifanio Diokno. threatened him. Being old himself, he was
crime of homicide defined and punished in article 249 of helpless.
the Revised Penal Code. Three mitigating circumstances 2. The Councilor told the two accused to go back to
must be taken into consideration in favor of the accused Faustas house, and when they returned, they
Epifanio Diokno and two in favor of the accused Roman brought back a bottle of wine & a bottle of oil,
Diokno, with no aggravating circumstance, thus saying Here are the ingredients for witchcraft
authorizing the imposition of the penalty next lower to that which we took from her house. The councilor
prescribed by law (reclusion temporal in its full extent), or then wrote on a statement that Fausta practiced
prision mayor in its full extent, in the period that this court witchcraft on Zapata's wife and had the same
deems applicable, which is the medium period in this thumbmarked by Fausta. Her husband was also
case, in accordance with the provisions of article 64, rule forced to sign it. Fausta died that same evening
5, that is eight years and one day of prision mayor. due to her injuries.
3. Tubadezas defense: alibi; Zapatas defense:
LAUREL, J., Fausta admitted being a witch and it was her
Agree as to others, except: the mitigating circumstance of husband himself who kicked her to death. This
immediate vindication of a grave offense. It should be was held by the SC to be unworthy of belief.
observed that the proximate cause of the tragedy was the 4. Tubadeza was also held as co-principal: while
elopement of Salome, the daughter of Epifanio and the there existed no previous understanding between
sister of Roman. Salome and the deceased had been the tw, yet it may be implied from the acts of
engaged for about a year and the evidence shows that the Tubadeza (helping, dragging, accompanying),
elopement took place at the instance of Salome herself. that they had the same unity of purpose in the
Under existing legislation, a woman eighteen years of age execution of the act.
or over, can contract marriage without the consent of her
parents. If she leaves the parental home for this purpose, ISSUE: What are the aggravating and mitigating
neither she nor her lover commits any offense. The act of circumstances to be appreciated in this case?
the deceased in eloping with Salome, at the invitation of
the latter was not a "grave offense" which called for or HELD:
justified immediate vindication. Aggravating
Evident premeditation not applicable
DIAZ, J., Nocturnity not applicable; Evidence fails to show
Crime should be murder. lIt is so qualified by the proven that nighttime was purposely sought by
fact that abuse of superior strength. said court, resolving appellants to commit the crime
the question whether or not the circumstance of abuse of Abuse of superior strength APPLICABLE.
superior strength should be taken into consideration in a Evidence positively demonstrates that they
case where two persons attack another, there being no disregarded the age and sex of the deceased, it
disparity in physical strength between the attackers and appearing that she was a frail woman of 65,
the attacked, and the former committed the aggression weighing only around 100 pounds and only 4 feet
with arms, the latter having only a small rod to defend and 8 inches in height, while Zapata and
himself, sustained the affirmative. Tubadeza were 32 and 27 years of age, when
the crime was committed.

93
CRIMINAL LAW REVIEW DIGESTS
JUSTICE ROMEO CALLEJO NOTE: = Callejo Ponente

Mitigating: mitigating circumstance of passion and obfuscation, the


Lack of intent to commit so grave a wrong following elements should concur:
APPLICABLE. It was evident that they merely (1) There should be an act both unlawful
wanted to denounce her as a witch before and sufficient to produce such
councilor Tubadeza when she was beaten and condition of mind;
dragged to the councilor's house, but that she (2) The act which produced the obfuscation
received a beating more than she could take. was not far removed from the
Obfuscation: APPLICABLE. It clearly appears commission of the crime by a
that appellants committed the crime in the belief considerable length of time, during
that the deceased had cast a spell of witchcraft which the perpetrator might recover
upon the wife of Zapata which caused her his normal equanimity.
serious illness. Accused-claims that he saw the cheaters in
their underwear. Wife-Analie claims that they were
SENTENCE: 2 mitigating & 1 aggravating = penalty in its merely lying down beside eachother. Nonetheless, in
minimum period. either case it is easy to see how accused-Pansensoy
acted with obfuscation because of jealousy upon
discovering his legitimate wife in the company of
PEOPLE V. PANSENSOY another man and the brazen admission by this man
that he loved his wife. Moreover, this is aggravated by
Facts: Accused-Pansensoys legally-married wife-Analie the fact that the cheaters brought with them the child
had an affair with the victim-Reyes, a jeepney driver. The of Pansensoy.
victim-Reyes and wife-Analie were renting a house in Extreme emotional pain could result from
Rizal and this was the place where the accused- such a situation and produce such passion and
Pansensoy caught the cheaters and is also where victim- anguish in the mind of a betrayed husband as to
Reyes was shot in the head at close range by the deprive him of self-control. To be blinded by passion
Accused-Pansensoy. and obfuscation is to lose self-control. In this case,
Accused-Pansensoy learned of the house the there is a clear showing that there were causes
cheaters were hiding because of his friend Bisaya who naturally tending to produce such powerful passion
told him that he saw the cheaters together with accused- as to deprive the accused of reason and self-control.
pansensoys son board a jeepney on their way to the Further, the killing was not far removed from the
house. Bisaya accompanied accused to the house. act producing the passion. Only a few minutes have
Accused knocked on the door of the house but elapsed.
he was not able to enter because wife-analie prevented 2.A. Treachery cannot co-exist with passion
him. Instead he sat on a bench outside the house. and obfuscation. The reason for this is that in passion,
Victim-Reyes went out and confronted the the offender loses his control while in treachery the means
accused. Accused asked victim-reyes whether he loved employed are consciously adopted. One who loses
Analie. Reyes answered Yes. Accused asked victim- reason and self-control cannot deliberately employ a
reyes whether he was single. Reyes answered Yes. At this particular means, method or form of attack in the
point, although not stated in the case, Accused- execution of the crime.
Pansensoy must have pointed a .38 caliber to the head of 2.B. Similarly, the aggravating circumstance of
victim-reyes. [BTW, accused is a security guard] Accused- evident premeditation cannot co-exist with the
Pansensoy counted one to three. ONE TWO BANG! circumstance of passion and obfuscation. The essence of
Victim-Reyes sprawled on the ground and died. premeditation is that the execution of the criminal act must
be preceded by calm thought and reflection upon the
Issue: 1. Murder or Homicide? resolution to carry out the criminal intent during the space
2. Is the mitigating circumstance of passion and of time sufficient to arrive at a composed judgment.
obfuscation present?
Held:
CRIME - SC said HOMICIDE because there was PEOPLE vs. PAGAL (NO USEFUL FACTS)
no treachery or evident premeditation. Further, the crime
is mitigated by passion and obfuscation. FACTS: Accused-appellants Pagal and Torcellino were
1. Before discussing why there is no treachery or charged with the crime of robbery with homicide, with the
premeditation. The mitigating of passion and obfuscation generic aggravating circumstances of nightime purposely
must first be discussed. In order to be entitled to the sought to better accomplish their criminal design; evident
premeditation; in disregard of the respect due the offended

94
CRIMINAL LAW REVIEW DIGESTS
JUSTICE ROMEO CALLEJO NOTE: = Callejo Ponente

party; and with abuse of confidence, the accused being their way to the house of Didoy Elican. As they were
then employees of the offended party. When the case was walking along the road at they met petitioner who collared
called for arraignment, the accused entered a plea of the victim, saying, Get it if you will not get it tonight, I will
guilty but they were allowed afterwards to prove the kill you. Thereafter, petitioner immediately stabbed the
mitigating circumstances of sufficient provocation or threat victim on the chest with a Batangueo knife. The place was
on the part of the offended party immediately preceding illuminated by a street light 3 to 4 arms length away from
the act, and that of having acted upon an impulse so the petitioner, enabling Dante Reginio to easily recognize
powerful as to produce passion and obfuscation. The the latter who happened to be his barangay mate.
RTC, after considering the 4 aggravating circumstances Dante Reginio and Nelson Magbanua executed a
and mitigating circumstance of only plea of guilt, found sworn statement identifying the petitioner as the culprit.
them guilty of the crime charged, sentencing them with the On cross-examination, Dante Reginio was
penalty of death. The case was elevated to the SC by confronted with an affidavit of desistance allegedly
virtue of the mandatory review on account of the penalty of executed by him and Nelson Magbanua stating, among
death imposed on the accused. others, that they both realize that it might be another
person who stabbed Clemente Del Gracia since it was
ISSUE: W/N the RTC erred in not appreciating in favor of dark that night of the incident.
the accused the mitigating circumstances of (1) sufficient Dante Reginio, however, denied knowledge of
provocation and (2) passion or obfuscation. the aforequoted affidavit and claimed that his signature
appearing thereon was a forgery.
HELD/RATIO: NO, the RTC is correct. On the other hand, the defense evidence
As a rule, two or more mitigating consisted of denial and alibi. Petitioner declared that at
circumstances arising from the same act cannot be 6:00 pm. of the night of the incident, he was in San Jose,
considered as separate and distinct circumstances Antique, waiting for the arrival of his wife from Iloilo City.
but should be treated as one. Thus, in this case, the The following day, his friend told him that he was the
mitigating circumstance of sufficient provocation cannot be suspect in the killing of Clemente Dela Gracia. For fear
considered because the alleged provocation which caused that he might be incarcerated, he went into hiding, but his
the obfuscation arose from the same incident, which is the mother convinced him to surrender to the police station.
alleged maltreatment and/or ill-treatment caused by the Nelson Magbanua admitted that he signed an
victims towards the accused-appellants. affidavit of desistance. He stressed, however, that he
As to the circumstance of passion and knew it was the petitioner who stabbed the victim but he
obfuscation, it cannot be treated as mitigating if the yielded to the pleas of petitioners wife and signed the
crime involved was planned and calmly meditated affidavit because he pitied her as she was then pregnant.
before its execution, such as in this case of robbery The trial court found the petitioner guilty beyond
where the appellants are expected to have carefully reasonable doubt of the crime of homicide.
planned its execution. Thus, in People vs. Daos, a case On appeal, petitioners conviction for the crime of
of robbery with homicide, this Court rejected the claim of homicide was affirmed but the penalty was modified
the appellants therein that passion and obfuscation should appreciating the mitigating circumstance of VOLUNTARY
have been estimated in their favor, because the death of SURRENDER.
the victim therein took place on the occasion of a robbery,
which, before its execution, had been planned and calmly Issue:
meditated by the appellants. 1. On the veracity of the affidavit of desistance
Finally, the maltreatment that appellants claim allegedly executed by Dante Reginio and Nelson
the victim to have committed against them occurred much Magbanua shows that the prosecution failed to
earlier than the date of the commission of the crime. establish beyond reasonable doubt the identity of
Provocation in order to be a mitigating circumstance must the culprit.
be sufficient and immediately proceeding the act. We hold
that the trial court did not commit any error in not Held: The contention is without merit. Dante Reginio
appreciating the said mitigating circumstances in favor of declared that the signature appearing above his type-
the appellants. written name on the affidavit of desistance was not his,
while Nelson Magbanua stated that he merely signed the
affidavit out of pity for the petitioners wife.
LUCES V PEOPLE OF THE PHILIPINES
2. The mitigating circumstance of voluntary
Facts: At 6:30 in the evening Dante Reginio, Nelson surrender appreciated by the CA.
Magbanua, and the victim, Clemente Dela Gracia, were on

95
CRIMINAL LAW REVIEW DIGESTS
JUSTICE ROMEO CALLEJO NOTE: = Callejo Ponente

Held: The Court of Appeals erred in appreciating the same offered to accompany her home. On their way they met
in favor of the petitioner. To benefit an accused, the four men, one of whom turned out to be a brother of the
following requisites must be proven, namely: (1) the accused who identified Eddie Basite as the person
offender has not actually been arrested; (2) the offender described by Sonia. They asked her to look for him but
surrendered himself to a person in authority; and (3) the she refused. Sonia proceeded instead to Monsoyohoy to
surrender was voluntary. A surrender to be voluntary must wait for her uncle Nazario Habungan who, she learned
be spontaneous, showing the intent of the accused to earlier, was going home and would pass by Monsoyohoy.
submit himself unconditionally to the authorities, either When she was already with his uncle and on their
because he acknowledges his guilt, or he wishes to save way to the police station, they saw the accused. Her uncle
them the trouble and expense necessarily incurred in his asked him to go with them to the police station but Basite
search and capture. Voluntary surrender presupposes fled. They pursued him and eventually caught him. Went
repentance. to the police station, underwent medical examination then
In the case at bar, petitioner surrendered to filed an Information for Rape against the accused.
the authorities in order to disclaim responsibility for TC found him guilty of rape. Basite contends the
the killing of the victim. This hardly shows any TC should have considered the mitigating circumstance of
repentance or acknowledgment of the crime on the voluntary surrender. He explains that he voluntarily
part of the petitioner. Moreover, at the time petitioner surrendered to then Barangay Captain Gilbert Sacla, and
surrendered, there was already a pending warrant of willingly went with him and complainants relatives to the
arrest against him. His arrest by that time was police station.
imminent. Hence, he should not be credited with the
mitigating circumstance of voluntary surrender. ISSUE: WON voluntary surrender should be appreciated
NO

PEOPLE V. BASITE HELD: A surrender to be voluntary must be


Voluntary Surrender | Justice Bellosillo spontaneous, showing the intent of the accused to
submit himself unconditionally to the authorities,
FACTS: Sonia Pa-ay, (19 yo student of midwifery, polio either because he acknowledges his guilt, or he
victim) was in Natuel, Buguias, Benguet, on her way to her wishes to save them the trouble and expense
parents home in Tinoc, Ifugao, to get her allowance. As necessarily incurred in his search and capture. If none
she was walking, she met Eddie Basite who was headed of these two reasons impelled the accused to
towards the opposite direction. A few seconds later, Sonia surrender, because his surrender was
heard footsteps behind her. When she looked back she obviously motivated more by an intention to insure
saw Eddie Basite following her. He reached her, held her his safety, his arrest being inevitable, the surrender is
by both hands and told her to go down with him. Sonia not spontaneous.
resisted. But the accused Eddie Basite pulled out a knife The conduct of accused-appellant after the
from his waistband, thrust it at her neck and threatened to commission of the offense, of running away after having
stab her if she continued to resist. He ordered her to lie been stabbed by Sonia and of fleeing from her relatives
down on the ground and out of fear she obeyed. when they tried to bring him to the authorities, do not show
The accused committed the act. When he was voluntary surrender as contemplated under the law. It
through with the sexual assault, he warned her not to appears that basite willingly went to the police
relate the incident to anyone or else he would stab her. authorities only to escape the wrath of Sonias
Upon seeing that the accused had laid down his knife relatives who were pursuing him and who appeared to
beside her head while he was putting on his clothes, Sonia be thirsting for his blood.
grabbed the knife and stabbed him on the left shoulder.
Wounded, the accused ran away.
Sonia tried to put on her clothes, but losing her 21. PEOPLE OF THE PHILIPPINES, plaintiff-appellee,
balance she rolled down the cliff and lost consciousness. vs. JUANITO ABELLA, DIOSDADO GRANADA,
When she recovered, she felt pain all over her body and BENJAMIN DE GUZMAN, and EDGARDO
could not find her bearings in her weakened state. She fell VALENCIA, accused-appellants.
asleep and woke up at around midnight. She made her
way up the mountain by the light of the moon. She FACTS: It all started with an altercation during a
decided to continue on her way to her parents house in basketball game. 3 days later, the 5 victims bodies were
Tinoc, Ifugao. found in the Pasig River. Victims were Marlon and Joseph
Along the way she passed by a house where she Ronquillo, Erwin and Andres Lojero and Felix Tamayo.
was offered camote to eat. Some soldiers arrived and

96
CRIMINAL LAW REVIEW DIGESTS
JUSTICE ROMEO CALLEJO NOTE: = Callejo Ponente

Marlons hands were tied at the back with an crime. The SC is sufficiently satisfied that their guilt was
electric cord, he had wounds and died from a gunshot to proven beyond reasonable doubt.
the head. Andres hands were also bound at the back with The killing was characterized by treachery. Though
a rope, his genitals were cut off and had wounds in his treachery should normally attend at the inception of the
body as well, cause of death was asphyxia by aggression, the facts show that the victims were first
strangulation. Josephs hands were tied at the back with a seized and bound and then slain, hence treachery is
basketball t-shirt, had wounds and died of strangulation as present. In this case, it is enough to point out that the
well. Erwins and Felixs body had abrasions and burns, victims hands were tied at the back when their bodies
there were cord impressions on his wrists, a fracture in his were found floating in Pasig River. This fact clearly shows
skull and died by drowning. that the victims were rendered defenseless and helpless,
The accused are Abella, Granada, De Guzman, thereby allowing the appellants to commit the crime
Valencia (all surnames), for MURDER, qualified by without risk at all to their persons. The circumstance of
treachery and evident premeditation. There were other abuse of superior strength was absorbed in treachery.
accused but their names were dropped from the The appellants move to clear their names cannot
information later on. be accepted as voluntary surrender. For a surrender to be
March 1992, the victims Ronquillo brothers were voluntary, it must be spontaneous and should show the
played 3 rounds of basketball in Sta. Mesa Manila against intent of the accused to submit himself unconditionally to
the team of Joey de los Santos. The Ronquillos brothers the authorities, either because (1) he acknowledges his
rd
won the first 2 rounds but the 3 one ended in a brawl. guilt or (2) he wishes to save the government the trouble
Later that afternoon, Joey went back to the place carrying and expense necessarily included for his search and
2 pillboxes but were apprehended. So, Joey and his capture. When the accused goes to a police station
brother just threw stones at the Ronquillos house. The merely to clear his name and not to give himself up,
neighbors saw this and ran after them and mauled them. voluntary surrender may not be appreciated.
That night, the victims were in front of the
Ronquillos house. Suddenly, a white Ford Fiera without a
plate number stopped in front of the group. There were 10- People v. Diva
13 people on board, including Joey and his brother. The
passengers in the Fiera alighted, faces covered with FACTS:
handkerchiefs and they were armed. The victims tried to Maximo and Cesaria Diva (DIVA Spouses) were
run but a shot was fired and Felix (victim) was shot. They charged with the murder of Ananias Bano
were boxed, kicked and hit and dragged into the van. They (BANO) at the junction of the provincial road and
were brought to a basement in a compound where witness of a trail leading to the house of the Diva
Elena saw them being mauled, whipped with a gun, Spouses in Barrio Santiago, San Francisco,
beaten with steel tubes and lead pipes. They also had a Cebu, on the afternoon of June 3, 1961.
blowtorch and the victims hands were tied. The victims The information alleged that on June 3, 1962,
begged for mercy. Afterwards, they were herded back to DIVA Spouses w/ intent to kill, conspired and
the car, seeming almost dead. 2 days later, their bodies mutually helped each other w/ evident
were found in the Pasig river. premeditation and treachery, and taking
Accused defense was an alibi, that they are INC advantage of superior strength, attacked BANO
members and were attending a panata that night. w/ bolo weapons inflicting 8 wounds, 2 of which
According to them, when they read their names in the were fatal.
newspapers as the perpetrators of the crime, they Based on the evidence of the prosecution:
consulted the INC Central Office and were then o Prior to the incident, BANO was a
accompanied by a lawyer to go to the police station to resident of Barrio Himinsolan, 4 km.
clear their names, They were later on identified in a away from Barrio Santiago, both of the
police line-up. The RTC convicted all of them for municipality of San Francisco, Cebu.
MURDER, with a penalty of reclusion perpetua. BANO married Alejandra Diva Aclon
(daughter of Teodora Diva de Aclon and
ISSUE: W/N the accused are entitled to the mitigating
niece of Maximo Diva (ergo, Teodora is
circumstance of voluntary surrender.
the sister of Maximos father, Raymundo
Diva). BANO resided in Barrio
HELD/RATIO: No.
Himinsolan until Alejandra died in 1958.
First of all, the witnesses were able to fully establish o In 1961, BANO took Justa Senor as his
and prove and connect the appellants-accused to the common law wife. He used to visit the

97
CRIMINAL LAW REVIEW DIGESTS
JUSTICE ROMEO CALLEJO NOTE: = Callejo Ponente

land of his deceased wife w/c adjoins him. Already weak, BANO picked up a
the land of Raymundo Diva, about 150 piece of wood and with it, hit the bolo
meters to the house of the DIVA held by Cesaria causing her to drop the
Spouses. During this year, as adjoining bolo on the ground.
landowners, a boundary dispute arose o While picking the bolo, Maximo
between BANO and Maximo, who was delivered another blow on the right face
then the caretaker of his fathers land. of BANO. BANO stopped retreating and
The dispute was eventually brought to exchanged blow for blow w/ Maximo
court, and was still pending in the CFI- (who received wounds on the face,
Cebu at the time the incident in question hand, and arms). Meanwhile, Cesaria
happened. chickened out and ran away.
o In September 1961, after the boundary o Weak and bleeding, BANO was left on
dispute case was filed in court, BANO the roadside by Maximo. Two persons,
went to Barrio Santiago to attend the together with Justa, approached the
wedding of a relative. On his return (at scene of the fight. BANO turned over
the junction of the provincial road and the bolo to Justa w/ instructions to
the road leading to the house of the deliver it to the authorities. The other 2
BANO Spouses [junction]), BANO was persons helped BANO and led him
ambushed by Maximo Diva and his towards to the Barrio. BANO died at the
younger brother, who were both armed steps of the stairs of the house of
with bolos. Nothing serious happened Andres Icoy, a school teacher of
because by-standers intervened. Until Santiago.
his eventual death, no other unusual From the medical certificate issued by Dr. Olitres
event happened. Maximo didnt show who autopsied the cadaver, BANO suffered: (1)
any belligerence towards BANO wound at the right lower jaw; (2) wound near the
whenever they saw each other. side of the mouth; (3) wound about the right
o On March 1962, BANO got ill of El Tor clavicle; (4) wound at the right side of the chest;
and became well due to timely medical (5) wound at the side of the upper part of right
assistance. On April, Justa Senor fell forearm; (6) wound at the left arm; (7) wound at
seriously ill while giving birth but also the medial side of the left scapular region; (8)
got well due to proper medical wound on his thigh. (3) & (4) were fatal and
assistance at the Southern Islands BANO ultimately died of of hemorrhage and the
Hospital where she was confined for 16 destruction of internal organs like the lung and
days. big blood vessels.
o To offer thanksgiving to the Lord and the Based on the evidence, DIVA Spouses after the
patron saint of Barrio Santiago, BANO incident nor did they surrender to the barrio
and Justa made a pilgrimage on June 3, lieutenant Rosalio Diva, Maximos uncle, and a
1962 to the chapel of Santiago, passing resident of their immediate neighborhood. The
by a road which was 150 meters from following day, the chief of police of San Francisco
the house of the BANO Spouses, was informed that Maximo Diva had surrendered
without any untoward incident. On their to the police authorities of the next town of Poro.
return, BANO and Justa took the same Defense: Maximo Diva admits killing BANO but
route, but at the junction, BANO was claims he acted in self-defense. Cesaria Diva
suddenly rushed upon by the DIVA claims that she did not participate in the fight
Spouses. Maximo was armed with a between her husband and BANO. The theory of
bolo while Cesaria with a bolo and a the defense is: DIVA Spouses were working in
sangalab, a sort of scythe used for the coconut plantation of their father in the
cutting grass. afternoon of June 3, 1962, when BANO
o The deceased retreated to avoid the introduced himself surreptitiously in the coconut
hacking blows of Maximo, but while he plantation and attacked Maximo Diva from
was defending himself against Maximo, behind. So Maximo Diva had to defend himself.
Cesaria sneaked behind BANO and The fight started in the coconut plantation about
delivered a bolo-blow on his back. five meters to the provincial road and lasted for
BANO continued to retreat while about fifteen minutes. Maximo Diva received nine
Maximo continued to inflict blows on wounds in the different parts of his body,

98
CRIMINAL LAW REVIEW DIGESTS
JUSTICE ROMEO CALLEJO NOTE: = Callejo Ponente

although only six were listed by Dr. Olitres in his prosecution, in the struggle between two giants who
medical certificate. On the other hand, Cesaria fought for no less than fifteen minutes armed with mortal
Diva claims that while the fight between her weapons, without exposing herself to being hit by the
husband and the deceased was going on she blows of one of them, nay, of the deceased, had she
was all the time shouting for help but succor did approached them and mingled in the fight. We are
not come persuaded, that upon the evidence, the participation
TRIAL COURT found the SPOUSES guilty of of Cesaria Diva in the aggression of the victim is of
murder and sentenced them to reclusion doubtful veracity. It is more reasonable to believe her
perpetua. testimony that she has been crying and calling for help
The accused appealed from the decision. during all the time the struggle was going on but that no
help came. Cesaria Diva did not take part in the
ISSUE/S: commission of the crime, and, therefore, conspiracy did
(1) w/n DIVA Spouses are entitled to mitigating not exist.
circumstance of voluntary surrender? No.
(2) w/n there was conspiracy between DIVA Spouses? No. DISPOSITION: Decision modified - Cesaria Diva is
acquitted of the offense charged on reasonable doubt and
(1) MITIGATING CIRCUSTANCE OF VOLUNTARY Maximo Diva is found guilty of homicide, with the benefit of
SURRENDER the mitigating circumstance of voluntary surrender.
After the incident. Maximo Diva left the scene thereof,
went to the municipality of Poro, a neighboring town, Side Issues:
where Dr. Olitres lived to have his wounds treated by the 1. There is lack of premeditation on the part of the
said doctor, and after the treatment of his wounds, he accused. The evidence reveals that until the
surrendered to the chief of police of the said town. The incident occurred (June 3, 1962) nor did Maximo
trial court considered this act of Maximo Diva as flight, Diva show any belligerence towards BANO
and, therefore, indicative of guilt. whenever they saw each other in Himinsolan and
To be entitled to the mitigating circumstance of Santiago. To properly appreciate the
voluntary surrender, the law does not require that the circumstance of evident premeditation, it is
perpetrator must give himself up to the authorities in the necessary to establish with proof, as clear as the
municipality where the offense was committed. All that proof of the crime itself, that (1) the time when
the law requires is for the offender to surrender to the the offender determined to commit the crime; (2)
authorities to save the government the trouble and an act manifestly indicating that the culprit has
expense of looking for him in order to arrest him. clung to his determination; and (3) a sufficient
Appellant Maximo Diva surrendered to the authorities lapse of time between the determination and the
the day following the incident. He did not wait for the execution to allow him to reflect. None of the
authorities to arrest him. Thus, an accused who foregoing requisites are present in the case at
presented himself in the municipal building five days bar.
after the commission of the crime to post the bond for 2. There is no treachery. The allegation of treachery
his temporary liberty was credited with the mitigating is not conclusively proven by the prosecution.
circumstance of voluntary surrender. The fact that a Although the deceased was suddenly attacked,
warrant of arrest had already been issued is not a bar but the deceased was able to retreat to avoid
to the consideration of this mitigating circumstance being hit by the hacking blows. So that he was
because the law does not require that the surrender only hit when he was already in the act of
be prior to the order of arrest. defending himself against the attack of the
By parity of reasoning, therefore, appellant accused.
Maximo Diva's voluntary surrender to the chief of police of 3. Defense of self-defense is untenable. Evidence
the municipality of Poro should be considered to mitigate showed Maximo laid in waiting behind a clamp of
his criminal liability because the law does not require iring-iring shrurbs beside the provincial road
him to surrender to the authorities of the municipality armed w/ a bolo, w/o perhaps the knowledge of
of San Francisco where the offense was committed. his wife of his purpose and suddenly attacked
BANO.Having admitted the killing, it was
(2) PARTICIPATION OF CESARIA DIVA incumbent upon him to prove by positive
Cesaria Diva was on her six or seven months pregnancy evidence or with convincing credibility his claim of
at the time of the incident, and in her condition then self-defense. A primordial requisite for self-
obtaining, it is rather doubtful that she would take such defense is unlawful aggression. This appellant
active part, as narrated by the witnesses for the Maximo Diva failed to prove.

99
CRIMINAL LAW REVIEW DIGESTS
JUSTICE ROMEO CALLEJO NOTE: = Callejo Ponente

4. Prosecution is not guilty of willful suppression of pull out the knife from his waist and stab the victim on the
evidence. chest. The one-inch long wound in appellant's left hand
was too superficial to support his claim that it was inflicted
while he was parrying the thrust of the victim. The mere
PEOPLE v. QUIMPO fact that he was wounded does not prove indubitably his
claim that he acted in self-defense. Nor that the victim and
FACTS: Accused-appellant Jimmy Dela Cruz y Quimpo not he was the aggressor. Note that appellant did not
was charged with and found guilty of the crime of murder present a knife during the trial to bolster his case. The
and was sentenced to reclusion perpetua. The Information witnesses for the prosecution denied that the victim was
alleged that on or about the 1st day of September, 1998 in armed with a knife and, indeed, none was recovered from
the evening, in Barangay Tigayon, Municipality of Kalibo, the scene of the crime.
Province of Aklan, Republic of the Philippines, and within
the jurisdiction of this Honorable Court, the above-named * The issue on voluntary surrender was not fully discussed
accused, while armed with a knife, with treachery and with in the case. It was only mentioned at the end of the case,
intent to kill, did then and there willfully, unlawfully and as follows:
feloniously attack, assault and stab one Arnulfo The trial court was correct when it considered
Inocencio, inflicting upon the latter physical injuries. the mitigating circumstance of Quimpos voluntary
Two eyewitnesses, Jovelyn Felizario, cousin of surrender to the barangay captain. Appellant
Arnulfo, and Glen Cipriano testified that in the evening of spontaneously and unconditionally placed himself in
September 1, 1998, several visitors were in her house at the hands of the authorities, and saved them the time
Tigayon, Kalibo, Aklan since it was the birthday of her and effort attendant to a search. The testimony of
brother, Jonel. At around 11:00 that evening, Arnulfo barangay captain Isberto and the police officer on this
Inocencio, appellant Jimmy dela Cruz, and brothers point was not contradicted by the prosecution. Thus, we
Glenn, Gilbert and Greg Cipriano were having a drinking find that the trial court correctly imposed the minimum of
session. Arnulfo played a guitar while appellant sang the penalty prescribed by law for the crime of murder
along. Afterwards, appellant requested Arnulfo to give the which isreclusion perpetua.
guitar to Gilbert. Arnulfo obliged and rose to hand the
guitar to Gilbert. When Arnulfo returned to his seat, VOLUNTARY SURRENDER
appellant suddenly drew his knife from his waist and
stabbed Arnulfo. According to the witness, appellant then NOTE: Know crimpro provisions
pointed at Arnulfo and said, "There, he is already dead."
The witness added she was just two meters away from the PEOPLE VS CALPITO
victim and the appellant when the stabbing incident
happened. FACTS: Calpito was charged with Robbery with Homicide.
Appellant admits that he killed the victim, Arnulfo Initially, Calpito entered a plea of not guilty, but after
Inocencio. However, he avers he did it in self-defense. He reinvestigation and re-arraignment, changed his plea to
claims that it was Arnulfo who attacked him first and that guilty. Court then charged Calpito of Murder instead of
he had no recourse but to stab Arnulfo. robbery with Homicide due to prosecutions failure to
sufficiently prove robbery.
ISSUE: W/N Quimpo acted in self-defense NO. (Side facts for side issue) Calpito argued that
since he was a minor (16 years old) when he committed
HELD: By invoking self-defense, the burden is placed the crime, although his birth certificate could not be
upon appellant to prove clearly and convincingly the verified, he should be credited with mitigating
elements thereof: unlawful aggression on the part of the circumstance of minority. RTC didnt consider this and no
victim, reasonable necessity of the means employed to mitigating circumstance was applied.
prevent or repel the aggression, and lack of sufficient
provocation on his part. Although all the three elements ISSUE: (related to topic hinde expressly sinabi but I just
must concur, self-defense must rest firstly on proof of assumed given the topic) W/N Calpito should be credited
unlawful aggression on the part of the victim. If no unlawful with mitigating circumstance of voluntary plea of guilty
aggression has been proved, no self-defense may be even though he pleaded not guilty on the first arraignment.
successfully pleaded, whether complete or incomplete. In YES.
this case, appellant's testimony miserably failed to prove
the existence of unlawful aggression. He claims that it was RATIO:
the victim who, without provocation on his part, suddenly The requisites of this circumstance are:
attacked him. To defend himself, he was constrained to (1) that the offender spontaneously confessed his guilt;

100
CRIMINAL LAW REVIEW DIGESTS
JUSTICE ROMEO CALLEJO NOTE: = Callejo Ponente

(2) that the confession of guilt was made in open court, Ratio: Takbobo admitted his guilt in open court prior
that is, before the competent court that is to try the case; to the presentation of evidence by the prosecution,
and which is a requisite for this mitigating circumstance.
(3) that the confession of guilt was made prior to the But despite the presence of two mitigating circumstances
presentation of evidence for the prosecution. (other is voluntary surrender) without any aggravating
In this case, upon re-arraignment, appellant, in the circumstance, the court did not agree with the
presence of his counsel, and in open court, voluntarily recommendation of the SG to reduce the penalty to
pleaded guilty to the crime charged before the prosecution reclusion temporal. This would patently run counter to the
presented its evidence. This mitigating circumstance rules for the application of indivisible penalties under Art.
should therefore be considered in computing the proper 63.
penalty. Art. 246 defines the crime of parricide and
imposes the penalty of reclusion perpetua to death.
SIDE ISSUE: W/N minority can be credited to Calpito Applying Art. 63, when the penalty is composed of two
notwithstanding unverified birth certificate. YES. indivisible penalties, the penalty cannot be lowered by one
degree, no matter how many mitigating circumstances are
SIDE RATIO: It is established that during the crime, present. Par. 5 of Art 64 (Rules for application of penalties
Calpito is indeed below eighteen. Though his real age with three periods) applies only to divisible penalties.
cannot be ascertained and that the birth certificate that he
presented is not satisfactory, throughout the trial, however,
he has consistently stated his age as being below 18 at PEOPLE OF THE PHILIPPINES vs. GREGORIO
the time of the commission of the crime. His conviction is MAGALLANES
affirmed but his penalty is reduced. [Doctrine: plea of Guilt]

FACTS: Accused-appellant Magallanes was a "mananari"


PEOPLE v. TAKBOBO or gaffer of fighting cocks. He and his friends were walking
towards the cockpit. Along their way, they met deceased
Facts: Ruben Takbobo, a middle aged fisherman, was Virgilio Tapales who was drinking in a store. Tapales
charged with killing his wife, Lucia, by hacking and called one of Magallanes friends (Cempron) who were
stabbing the latter with a knife and bolo. Takbobo went to walking with him. For some unknown reason, Tapales
the authorities and told them what happened. then directed his attention to the appellant who was
Their daughter testified in court that Takbobo had walking a few steps behind Cempron. Tapales held the
the propensity for inexplicable resort to violence against appellant by his shirt slapped him and strangled his neck.
members of his family (one of her fingers was cut and the But seeing a knife tucked in Tapales' waist, the appellant
third finger of her older sister was split by a bolo wielded pulled out the knife and slashed at Tapales to loosen his
by their father). grip. The appellant succeeded in wounding the face and
Takbobo said that the reason he killed his wife neck of Tapales who let go of the appellant and fled for his
was because he caught her sleeping with another man. life. Insatiated, the appellant pursued Tapales and when
He arrived home at 3am from his fishing activity when he the latter fell, the appellant stabbed him several more
his wife sleeping with their neighbor. He tried to kill him by times. Later, the appellant surrendered to the police
stabbing him but his wife pushed the man who authorities. Accused-appellant Magallanes was charged
immediately jumped out the window. As a result, his wife by the prosecution for Murder. During arraignment,
was hit by his thrust. He then found out that his wife had accused-appellant Magallanes expressed his
no panty. He tried searching but failed to find the other willingness to enter a plea of guilty to the lesser
man. He immediately reported the incident to the police offense of homicide with the mitigating circumstances
though he was not able to execute his affidavit as he was of plea of guilty and voluntary surrender. The
very confused. prosecution refused to lower the charge from murder to
Takbobo entered a plea of guilty. The RTC homicide, hence, trial ensued after which, a decision was
found him guilty of parricide but did not appreciate the rendered finding the appellant guilty of the crime of
mitigating circumstance of passion and obfuscation, murder.
voluntary surrender and voluntary plea of guilty. Thus, this
appeal. ISSUE: Whether or not accused Magallanes should be
convicted of Murder or homicide.
Issue: Did the court err in not considering the mitigating
circumstance of voluntary plea of guilty - YES HELD: Homicide.

101
CRIMINAL LAW REVIEW DIGESTS
JUSTICE ROMEO CALLEJO NOTE: = Callejo Ponente

The appellant asseverates that the killing of Magallanes] guilt. Hence, even though it was a
Tapales was not attended by treachery which would qualified confession, he still admitted his guilt
qualify it to murder, hence, he should have been convicted anyway. Otherwise, if we do not interpret the law this way,
of the crime of homicide only. On the other hand, the then the effect would be that the prosecution would simply
prosecution insists that the killing was treacherous counteract this plea of guilt with unfounded allegations of
because it was perpetrated while the defenseless Tapales aggravating circumstances.]
was running away from the appellant, thereby giving the
latter opportunity to stab Tapales at the back without
warning. On this issue we find for the accused-appellant. ZENON R. PEREZ, petitioner, vs. PEOPLE OF THE
Absent the qualifying circumstance of treachery, PHILIPPINES and SANDIGANBAYAN, respondents.
we therefore find the appellant guilty only of the crime of
homicide. Moreover, a careful scrutiny of the records of FACTS:
this case reveals that the trial court had erroneously failed - Petitioner was the town treasurer and used public
to appreciate in mitigation of the appellant's penalty the funds of a town in Bohol admitting that part of the
circumstances of voluntary surrender and plea of guilty. money was used to pay for the loan of his late
On record is the appellant's willingness to brother, another portion was spent for the food of his
enter a plea of guilty but to the lesser crime of family, and the rest for his medicine.
homicide. It only remains to consider briefly whether - Filing of the appropriate criminal case against
the appellant's plea of guilty in the form it was entered petitioner was recommended by the Auditor.
constitutes a voluntary confession of guilt before the - An administrative case was filed against petitioner to
court as defined in paragraph 7 of Article 13 of the which he filed an answer reiterating his earlier verbal
Revised Penal Code. In People v. Yturriaga where the admission before the audit team.
accused who was charged with murder entered a qualified - He eventually was able to remit amounts equivalent to
plea of guilty by claiming that the alleged qualifying that which he used.
circumstance of evident premeditation did not exist, we - Later, petitioner was charged and conviction before
said that: the Sandiganbayan with malversation of public funds.
Although the confession was qualified and
introduction of evidence became necessary, the ISSUE: W/N the penalty may be mitigated.
qualification did not deny the defendant's guilt
and, what is more, was subsequently fully HELD/RATIO: YES.
justified. It was not the defendant's fault that In all cases, persons guilty of malversation shall also
aggravating circumstances were erroneously alleged suffer the penalty of perpetual special disqualification and
in the information and mitigating circumstances a fine equal to the amount of the funds malversed or equal
omitted therefrom. If such qualification could deprive to the total value of the property embezzled.
the accused of the benefit of plea of guilty, then the The failure of a public officer to have duly
prosecution could nullify this mitigating circumstance forthcoming any public funds or property with which he is
by counteracting it with unfounded allegations of chargeable upon demand by any duly authorized officer,
aggravating circumstances. shall be prima facie evidence that he has put such missing
WHEREFORE, the judgment appealed from is funds or property to personal uses. (Underscoring
hereby MODIFIED by convicting the appellant Gregorio supplied)
Magallanes of the crime of homicide only with the The amount malversed totalled P72,784.57. The
mitigating circumstances of voluntary surrender and plea prescribed penalty is reclusion temporal in its maximum
of guilty in his favor. period to reclusion perpetua, which has a range of
seventeen (17) years, four (4) months and one (1) day to
[Note: In simple terms, a plea of guilt is a mitigating forty (40) years.
circumstance. But the question is, how about if the However, the commission of the crime was
accused pleaded guilty of a lesser crime than that charged attended by the mitigating circumstance akin to
against him? Will it be considered a mitigating voluntary surrender. As correctly observed by the
circumstance of plea of guilt? Yes! For example, in this Sandiganbayan, petitioner restituted the full amount even
case, the Court is saying that accused-appellants before the prosecution could present its evidence. That is
confession is qualified. It is qualified because although he borne by the records.
admitted the commission of a crime (homicide only), he It bears stressing that the full restitution of the
did not admit to the crime that was charged against him amount malversed will not in any way exonerate an
(murder). However, this qualified confession, according to accused, as payment is not one of the elements of
the court, did not deny the defendants [accused extinction of criminal liability. Under the law, the refund of

102
CRIMINAL LAW REVIEW DIGESTS
JUSTICE ROMEO CALLEJO NOTE: = Callejo Ponente

the sum misappropriated, even before the commencement store approximately two arms length from him. Then he
of the criminal prosecution, does not exempt the guilty saw Rodrigo handing a bolo to his brother Rodolfo and an
person from liability for the crime. At most, then, payment ice-pick one foot long to their cumpadre, saying at the
of the amount malversed will only serve as a mitigating same time, The one in white shirt. In a swift, sudden
circumstance akin to voluntary surrender, as provided for motion, the cumpadre bluntly stabbed Carlos Reyes on
in paragraph 7 of Article 1387 in relation to paragraph the chest, asking his companions, Ito ba? By this time,
1088 of the same Article of the Revised Penal Code. Rodrigo was six arms length away watching the whole
But the Court also holds that aside from incident. Meanwhile, Rodolfo, still holding the bolo, served
voluntary surrender, petitioner is entitled to the as a back-up, standing near his cumpadre. He also
mitigating circumstance of no intention to commit so shouted Walang makiki-alam! Carlos, wounded and
grave a wrong, again in relation to paragraph 10 of Article bleeding, fell on his back. After which, the three ran away
13. in the same direction. Danilo was able to clearly see what
The records bear out that petitioner transpired because the place was well-lighted by electric
misappropriated the missing funds under his custody and lights emanating from the store and the lamp post.
control because he was impelled by the genuine love for Danilo helped the parents of Carlos Reyes
his brother and his family. Per his admission, petitioner institute the case. Rodrigo Hilario said the event never
used part of the funds to pay off a debt owed by his took place and that Danilo implicated him because Danilo
brother. Another portion of the misappropriated funds went had a grudge against him. He said he was roving with his
to his medications for his debilitating diabetes. fellow barangay tanods that night.
Further, as shown earlier, petitioner restituted all The RTC found Rodrigo Hilario guilty of murder,
but Eight Thousand Pesos (P8,000.00) of the funds in less qualified by evident premeditation, and with the generic
than one month and a half and said small balance in three aggravating circumstance of treachery without any
(3) months from receipt of demand of COA on January 5, mitigating circumstance, and sentenced him to reclusion
1999. Evidently, there was no intention to commit so grave perpetua.
a wrong.
Issue: Whether or not the accused is liable despite the
error in personae.

Held: Yes! Sc affirms the conviction.


Ratio: The fact that the accused killed a person other than
PP v. HILARIO their intended victim is of no moment. According to Art. 4
of the Revised Penal Code, criminal liability is incurred by
Facts: One afternoon, accused Rodrigo Hilario, together any person committing a felony although the wrongful act
with his brother Rodolfo, and someone who appears to be done be different from that which is intended. One who
their cumpadre (whose name is uknown) went to the commits an intentional felony is responsible for all the
house of Danilo Manzanares. Manzanares was a watch consequences which may naturally or logically result
repairman and the Hilarios, (Manzanares uncles because therefrom, whether foreseen or intended or not. The
they were the siblings of Danilos mom) visited him to have rationale of the rule is found in the doctrine, el que es
the bracelet of Rodolfos watch restored. While Danilo was causa de la causa es causa del mal causado, or he who is
busy fixing the bracelet, the three were conversing nearby. the cause of the cause is the cause of the evil caused. [10
He inadvertently heard Rodrigo saying, Pare, nandyan na The accused performed voluntary acts. Their purpose was
ang taong titirahin natin, si Berong. In response, Rodolfo to kill. Hence, notwithstanding the mistake in the identity of
remarked, Padilim tayo. After 30 minutes, the three left the victim, the accused are still criminally liable.
and proceeded to the Barangay Hall which is only two It is to be noted that the lower court, in finding the
houses away. appellant guilty of murder, qualified the killing by evident
premeditation. Evident premeditation, however, may not
At about 8:30 in the evening, Danilo went to
properly be taken into account when the person whom the
Mang Jacks store. There, he saw Berong and the victim
defendant proposed to kill was different from the one who
Carlos Reyes in front of the store squatting and talking to
became his victim. When the person decided to kill a
each other. Both were wearing white shirts. A little later,
different person and premeditated on the killing of the
Berong removed his white shirt. Fate must be smiling on
latter, but when he carried out his plan he actually killed
him that night because uncannily, this innocent act would
another person, it cannot properly be said that he
later save his skin at the expense of Carlos.
premeditated on the killing of the actual victim. Thus
At this juncture, Danilo saw Rodrigo, Rodolfo and
premeditation was not aggravating in the case of People
their cumpadre approaching from the other side of the
vs. Guillen, where the accused had deliberately intended

103
CRIMINAL LAW REVIEW DIGESTS
JUSTICE ROMEO CALLEJO NOTE: = Callejo Ponente

to assassinate former President Manuel Roxas but he The song evokes the bitterest passions. This is not the
killed instead Simeon Varela and wounded others. This first time the song "My Way" has triggered violent behavior
doctrinal rule applies here. resulting in people coming to blows. In the case at bar, the
few lines of the song depicted what came to pass when
the victims and the aggressors tried to outdo each other in
PEOPLE VS. GEMOYA their rendition of the song.

The neighborhood of Barrio Malagamot, Panacan, Davao FACTS: Brothers Servillano, Melton and Michael Ferrer
City were awakened by a commotion. Armando Gemoya were having their drinking spree at their house but later
and Candelario Aliazar, together with their relatives, decided to proceed to Tidbits Videoke Bar to continue their
Ronilo and Rolly Tionko, went towards the house of Irene drinking spree and to sing. Thereafter, Jaime Palaganas
Lantapon. They were armed with pipe, wood, and an arrived together with Ferdinand Palaganas (nephew) and
improvised bow and arrow locally called Indian Pana. Virgilio Bautista. When Jaime Palaganas was singing,
Addressing a group of people who were huddled together, Melton Ferrer sang with him as the latter was familiar with
Ronilo stopped and demanded an explanation for what the song (My Way). Jaime Palaganas got irritated and
happened to his brother-in-law. They replied that nothing insulted. He felt that he was being mocked by Melton
happened to him and advised them to go home. Ronilo Ferrer, that caused him to went to the Ferrers table and
ignored them and the four went to the house of the uttered statements which began the fight.
Alferezes. They saw Wilfredo Alferez standing by the road Ferdinand sought help to Rujjeric Palaganas.
waiting for a taxi. The four rushed at him. Ronilo beat him They went to the Bar and upon seeing the Ferrers outside,
with a cylindrical wood, Rolly with a pipe, while Candelario Ferdinand pointing at the Ferrers instructed Rujjeric to
held his arms behind him. Armando aimed his Indian shoot them. Rujjeric Palaganas shot Servillano, Melton
pana at Wilfredo and the latter was hit on his left chest. and Michael with the use of unlicensed firearm. As a
Edgardo and his daughter, Rosalie Jimenez rushed to his result, Melton was killed, Servillano was fatally wounded
aid. But Rosalie was hit on the left ear by Armando. Then and Michael was shot in his right shoulder.
the four ran away. Wilfredo was brought to the hospital but
he died upon arrival. Rosalie, on the other hand, was ISSUE: Whether or not the use of unlicensed firearm is a
declared out of danger. special aggravating circumstance which should be
The RTC found Armando and Ronilo guilty of appreciated by the court at the case at bar? YES.
murder and frustrated homicide.
HELD: Both TC and the appellate court were correct that
ISSUE: Whether or not the RTC erred in convicting the aggravating circumstance of use of unlicensed must
Armando and Ronilo of the crime of frustrated homicide for be applied against petitioner since the same was alleged
the wounding of Jimenez??? -- YES in the informations filed against him before the RTC and
proven during the trial. However, such must be
RULING: The hitting of Rosalie was accidental as the considered as a special aggravating circumstance,
second Indian pana was meant for Wilfredo. The intent and not a generic aggravating circumstance.
to kill Rosalie is absent. However, they are still liable for Generic aggravating circumstances are those
the consequences of their felonious act. Mistake in the that generally apply to all crimes such as those mentioned
identity of the victim, which may either be error in in Article 14, paragraphs No. 1, 2, 3, 4, 5, 6, 9, 10, 14, 18,
personae (mistake of the person), or aberratio ictus 19 and 20, of the RPC. It has the effect of increasing the
(mistake in the blow), is neither exempting nor penalty for the crime to its maximum period, but it cannot
mitigating. They cannot therefore escape the criminal increase the same to the next higher degree. It must
liability resulting from the injury suffered by Rosalie. always be alleged and charged in the information, and
NOTE: Gemoya is entitled to the mitigating must be proven during the trial in order to be appreciated.
circumstance of voluntary surrender. Moreover, it can be offset by an ordinary mitigating
circumstance. On the other hand, special aggravating
circumstances are those which arise under special
conditions to increase the penalty for the offense to its
PALAGANAS VS. PEOPLE (My Way)
maximum period, but the same cannot increase the
CHICO-NAZARIO, J. (Panganiban, C.J., Chairperson,
penalty to the next higher degree. Examples are quasi-
Ynares-Santiago, Austria-Martinez, Callejo, Sr.,
recidivism under Article 160 and complex crimes under
J.J., concur.):
Article 48 of the Revised Penal Code. It does not change
the character of the offense charged. It must always be
alleged and charged in the information, and must be

104
CRIMINAL LAW REVIEW DIGESTS
JUSTICE ROMEO CALLEJO NOTE: = Callejo Ponente

proven during the trial in order to be appreciated. proceeded to the market place, which was just about 5
Moreover, it cannot be offset by an ordinary mitigating meters away. Thereat, he saw at a butchers shop a knife
circumstance. which he took and he right away returned to the barangay
It is clear from the foregoing that the meaning hall. When appellant returned, Segundina was sitting on a
and effect of generic and special aggravating low rattan stool. In front of her were Nancy and Julie. They
circumstances are exactly the same except that in case of did not notice appellants return, especially Segundina who
generic aggravating, the same CAN be offset by an had her back to appellant. Appellant suddenly stabbed
ordinary mitigating circumstance whereas in the case of Segundina on the left portion of her back. He then ran
special aggravating circumstance, it CANNOT be offset by away leaving the knife at the victims back with the jacket
an ordinary mitigating circumstance. he had covered it with, hanging by the knifes handle.
Aside from the aggravating circumstances Appellant surrendered right away to the police. Segundina
abovementioned, there is also an aggravating died in the morning of the following day. The appellant
circumstance provided for under Presidential Decree No. does not deny stabbing Segundina Cay-no. However, he
1866, as amended by Republic Act No. 8294, which is a maintains that neither treachery nor evident premeditation
special law. Its pertinent provision states: If homicide attended the commission of the crime. Appellant testified
or murder is committed with the use of an unlicensed that he was teasing Segundina that he be her baggage
firearm, such use of an unlicensed firearm shall be boy of the clothes that she was vending. Segundina got
considered as an aggravating circumstance. mad and humiliated him in front of many people. Trial
We already held in several cases that with the court found the appellant guilty of the crime of murder.
passage of RA8294 on 6 June 1997, the use of an
unlicensed firearm in murder or homicide is now Issues:
considered as a SPECIAL aggravating circumstance 1. WON the information for murder was sufficient -YES!
and not a generic aggravating circumstance. RA 8294 2. WON there was treachery -YES!
applies to the instant case since it took effect before the 3. WON there is a mitigating circumstance of passion and
commission of the crimes in 21 April 1998. Therefore, the obfuscation and sufficient provocation -NO!
use of an unlicensed firearm by the petitioner in the instant
case should be designated and appreciated as a 1. The Information, as written, consists of two paragraphs.
SPECIAL aggravating circumstance and not merely a The first paragraph contains the allegations of the date,
generic aggravating circumstance. time, place, the acts constituting the offense, and the
As was previously established, a special name of the victim. Written in a separate paragraph are
aggravating circumstance cannot be offset by an ordinary the aggravating circumstances of evident premeditation,
mitigating circumstance. Voluntary surrender of petitioner treachery, abuse of superior strength and craft, alleged as
in this case is merely an ordinary mitigating circumstance. attending the commission of the crime.The fact that the
Thus, it cannot offset the special aggravating qualifying circumstances were recited in the second
circumstance of use of unlicensed firearm. In accordance paragraph and not in the first paragraph of the Information,
with Article 64, paragraph 3 of the Revised Penal Code, as commonly done, is a matter of form or style for which
the penalty imposable on petitioner should be in its the prosecution should not be faulted.
maximum period. The test of sufficiency of Information is whether it
enables a person of common understanding to know the
charge against him, and the court to render judgment
PEOPLE VS. WILSON LAB-EO properly. Significantly, the appellant never claimed that he
was deprived of his right to be fully apprised of the nature
Facts: Segundina Cayno was engaged in the business of of the charges against him because of the style or form
selling rummage goods. One day, she displayed the adopted in the Information.
goods in front of the public market. Nancy Gaoan and The New Rules on Criminal Procedure now
Julie Dangla went to see Segundina to be massaged by require that both the qualifying and aggravating
the latter. Before noontime, while Nancy and Julie were circumstances must be specifically alleged in the
plucking the white hair strands of Segundina, appellant Information to be appreciated as such. Under the old
Wilson Lab-eo arrived and approached his aunt, Rules, only the qualifying circumstances were required to
Segundina. Appellant sat down in front of his aunt and be alleged in the Information, and aggravating
uttered something to her in a very soft voice. Nancy and circumstances, even if not alleged, could still be
Julie did not hear what he said because of her distance appreciated, except in cases where an aggravating
from them. What they only heard was Segundinas answer circumstance would result in the imposition of the death
which was uttered in a loud angry voice saying you get penalty.
out because I might suffer high blood. Appellant then

105
CRIMINAL LAW REVIEW DIGESTS
JUSTICE ROMEO CALLEJO NOTE: = Callejo Ponente

2. Treachery attended the stabbing of the victim. As a rule, assigned to him. He was also tasked to make almost daily
a sudden attack by the assailant, whether frontally or from deposits to the collections of the City Treasurer to the
behind, is treachery if he deliberately adopted such mode PNB.
of attack with the purpose of depriving the victim of a Whenever Hipol was absent, it was Lerma
chance to either fight or retreat. To constitute treachery, Roque who was ordered to deposit money. Thus, one day,
two conditions must concur: (1) the employment of means Roque was instructed to gather all deposit slips covering
of execution which tend directly and specially to insure the all deposits of funds of the City Treasurers Office with
accomplishment of the crime without risk to the assailant PNB. Roque then opened the unlocked desk drawer of
arising from the defense the victim might make; and (2) a Hipol as was her practice. She inadvertently stumbled
deliberate or conscious adoption of the means of upon 3PNB deposit slips inside Hipols drawer which did
execution. When a victim is unexpectedly attacked from not appear to have been actually deposited. It was later on
behind, depriving him of any opportunity to defend himself, verified that indeed they were not verified.
undeniably there is alevosia. Craft was likwise absorbed in Upon further inspection, more slips that were not
treachery as shown by the fact that the appellant hid the deposited were found. The COA conducted an audit it was
knife under his jacket to prevent the victim from seeing it found that around P2M++ collections were made but not
and from being alerted of an impending assault. deposited. Hipol denies the accusation of malversation of
public funds.
3. In order to be entitled to the mitigating circumstance of TC: Guilty. Reclusion perpetua as penalty among
passion and obfuscation, the following elements should others.
concur: (1) there should be an act both unlawful and
sufficient to produce such condition of mind; (2) the act Issue: Whether Hipol is guilty? And whether aggravating
which produced the obfuscation was not far removed from circumstance of taking advantage of public office should
the commission of the crime by a considerable length of be appreciated?
time, during which the perpetrator might recover his
normal equanimity. In asking the appellant to leave, the Held: Hipol is guilty! But no aggravating circumstance.
victim did not do anything unlawful. There is an absolute (Constitutional and criminal procedure arguments
lack of proof that the appellant was utterly humiliated by were made --- I wont discuss this anymore). But the gist
the victims utterance. Nor was it shown that the victim is: Constitutional argument --- Hipol argues that there was
made that remark in an insulting and repugnant manner. unreasonable search and seizure. But note that this
Neither was the mitigating circumstance of sufficient Constitutional proscription does not concern itself with the
provocation by the victim proven. From the testimonies of relation between private individuals. Criminal Procedure ---
witnesses, it was shown that it was in fact the appellant amendment of the information. But the Court said the
who provoked the victim. Moreover, this Court has held amendment only referred to the amount involved and not
that the provocation sufficient to mitigate an offense must to the crime charged.
be proportionate to the gravity of the retaliatory act. Conviction for malversation of public funds or
Treachery attended the stabbing of Segundina property require proof that: 1) the offender is a public
Cay-no, thereby qualifying the killing to murder which is officer; 2) he has the custody or control of funds or
punishable by reclusion perpetua to death. However, with property by reason of the duties of his office; 3) the funds
the mitigating circumstance of voluntary surrender, the or property involved are public funds or property for which
appellant shall suffer the penalty of reclusion perpetua he is accountable; and 3) he has appropriated, taken or
instead of death. misappropriated, or has consented, or through
abandonment or negligence permitted, the taking by
RA 8294 or RA 8353 another person of such funds or property.
In this case, the facts are clear that Hipol is a
PEOPLE V JOHN PETER HIPOL public officer and that he is tasked to regularly handle
Dunno why this is under firearms. Should be under public funds. Even if the duty of depositing them is not his
aggravating circumstance of taking advantage of public official task, the fact remains that he had custody of the
office. money.
Hipol points the blame to the missing money to
Facts: Hipol was employed as Cash Clerk II at the City the City Treasurer and the Cashier against whom the COA
Treasurers office in Baguio. His duty included assisting also filed charges against for the shortage of city funds.
the cashier in the preparation of payments of vouchers, Nevertheless, under Article 217, the failure of the public
correspondences, daily cash reports...etc., and other officer to present such public funds or property upon
communications and documents necessary in connection demand by a duly authorized officer is prima facie
with the handling of cash and other duties which may be evidence that he has put such missing funds or property to

106
CRIMINAL LAW REVIEW DIGESTS
JUSTICE ROMEO CALLEJO NOTE: = Callejo Ponente

personal use. Being an accountable officer, Hipol may be on an unsuspecting victim, depriving the latter of any real
convicted of malversation even in the absence of direct chance to defend himself, thereby ensuring its commission
proof of misappropriation as long as there is evidence of without risk to the aggressor, and without the slightest
shortage in his accounts which he cannot explain. provocation on the part of the victim. The treacherous
Nevertheless, the aggravating circumstance manner in which the crime was committed is shown by the
of taking advantage of public office cannot be sudden and unexpected attack upon the unsuspecting and
appreciated in this case. The element of taking apparently unarmed victims and also by the deliberate
advantage of public office is inherent in the crime of manner in which the assault was perpetrated. In this case,
malversation of public funds or property. Said crime a totally unsuspecting Jelord Velez held onto his brother
cannot be committed without the abuse of public Jerry on board their motorcycle on their way home
office. blissfully unaware of the onrushing peril behind them. The
Also, Sol Gen said that the crime was already attendance of treachery qualifies the killing to Murder.
economic sabotage. SC: No such thing as economic The trial court improperly applied the aggravating
sabotage as aggravating. (note: Mickey, from Glenn circumstance of taking advantage of public position. To
notes) appreciate this aggravating circumstance, the public
officer must use the influence, prestige or ascendancy
which his office gives him as a means by which he
PEOPLE V. VILLAMOR realizes his purpose. Test: Did the accused abuse his
office to commit the crime? In this case, there was no
Around dusk, brothers Jerry and Jelord Velez were on showing that Villamor took advantage of his being a
their way home on board a motorcycle after having dinner policeman to shoot Jelord or that he used his
at a friends house. From behind them appeared a influence, prestige or ascendancy in killing the victim.
speeding motorcycle, which they ignored. Suddenly, Villamor could have shot Jelord even without being a
gunshots rang out from behind them and they abruptly policeman. In other words, if the accused could have
turned towards the direction of the gunfire. The lights of perpetrated the crime even without occupying his
their motorbike fell on the attackers, and they clearly position, there is no abuse of public position. The
identified the latter to be PO3 Renato Villamor and Brgy. mere fact that accused-appellant is a policeman and used
Capt. Jessie Maghilom (both accused). The assailants his gun to kill is not sufficient to establish that he misused
fired at them a second time and then fled. Jerry the driver his public position in the commission of the crime.
sustained wounds in the abdomen and elbow, Jelord died
from the first gunshot (didnt say which part).
Villamor and Maghilom were indicted for murder PEOPLE V. TABION
(Jelord) with treachery and frustrated murder (Jerry) but
Maghilom remained at large. Villamor posted an alibi (he Where the information charging accused with rape failed
was then acting as a security escort for Mayor Yap). Not to allege minority and relationship (of victim to accused),
so important: the Velezes and the Yaps are political rivals. he cannot be convicted of qualified rape. Simple rape only,
Trial court: guilty of murder with aggravating circ as proved in trial.
of taking advantage of his public position. Death. For
frustrated murder also with the aggravating circ of taking Regin Tabion (16) was at home weaving baskets when
advantage of public position as a policeman, guilty. her dad, Dominador Tabion called her to wash plates. The
Reclusion temp, max. latter was her only companion. Tabion then told her to go
into their room, lie on the bed and remove her panty.
Ruling: SC agrees with trial court ruling that there was Tabion had with him a large hunting knife which he used
treachery BUT NO ABUSE OF PUBLIC AUTHORITY. to threaten his daughter. He successfully raped his
There is treachery when the offender commits daughter and warned the latter to not tell anyone about the
any of the crimes against persons, employing means, incident, otherwise, hed kill her and the whole family. This
methods or forms in the execution thereof which tend went on 10 times, until she couldnt bear the pain
directly and specially to insure its execution, without risk to anymore. She confessed to her mom and a case was
himself arising from the defense which the offended party filed. Tabion denied having raped his daughter and said
might make. The two conditions for the same are present that his wife merely had a grudge on him because he was
(1) that at the time of the attack, the victim was not in a an NPA.
position to defend himself, and (2) that the offender Trial court ruling: guilty of qualified rape. Penalty
consciously adopted the particular means, method or form of death.
of attack employed by him. The essence of treachery is
the swift, sudden and unexpected attack by the aggressor

107
CRIMINAL LAW REVIEW DIGESTS
JUSTICE ROMEO CALLEJO NOTE: = Callejo Ponente

Ruling: Tabion may only be convicted of simple, not policeman. Diosdada instinctively followed suit and sat
qualified rape. beside Mario.
The court is convinced that Tabion raped his own They cruised towards Roxas Boulevard. The
daughter Regin. The testimony of the victim is replete with driver then asked Mario why he was carrying a "deadly
details; she was categorical, straightforward, unshaken weapon," to which Mario answered, "for self-defense since
and unwavering even during the grueling cross- he was a polio victim." The driver and another policeman
examination. Victims testimony was also fully who were both seated in front grilled Mario. They
corroborated with the testimony of the examining frightened him by telling him that for carrying a deadly
physician. Accuseds defense of alibi cannot prevail over weapon outside his residence he would be brought to the
victims testimony. Indeed, no young girl of decent repute Bicutan police station where he would be interrogated by
would allow an examination of her private parts or subject the police, mauled by other prisoners and heckled by the
herself to the shame, embarrassment and humiliation of a press. As they approached Ospital ng Maynila, the mobile
public trial, if she has not in fact been raped. car pulled over and the 2 policemen in front told the
However, it must be noted that the constitution Montecillos that the bailbond for carrying a "deadly
grants the accused the inviolable right to be informed of weapon" was P12,000.00. At this point, the driver asked
the nature and cause of the accusation against him. This how much money they had. Without answering, Mario
means that every element of the offense must be alleged gave his P1,000.00 to Diosdada who placed the money
in the complaint or information. The accused "is presumed inside her wallet.
to have no independent knowledge of the facts that Diosdada was then made to alight from the car.
constitute the offense" charged. She was followed by the driver and was told to go behind
RA 7659, which took effect on December 31, the vehicle. There, the driver forced her to take out her
1993, imposes the death penalty in the event rape is wallet and rummaged through its contents. He counted her
attended by any one of the seven new special money. She had P5,000.00 in her wallet. The driver
circumstances enumerated in the said statute. took P1,500.00 and left her P3,500.00. He instructed her
Pursuant to the above-mentioned constitutional to tell his companions that all she had wasP3,500.00.
right of the accused, the death penalty may be imposed While going back to the car the driver demanded from her
only if the information has alleged and the evidence has any piece of jewelry that could be pawned. Ruefully, she
proven both the age of the victim and her relationship to removed her wristwatch and offered it to him. The driver
the offender. In the instant case, the age of the victim declined saying, "Never mind," and proceeded to board
was not alleged in the Information filed against the car. Diosdada, still fearing for the safety of her brother,
appellant. Because not all the elements of qualified followed and sat beside him in the car.
rape were alleged in the Information, the death Once in the car, Diosdada was directed by the
sentence cannot be meted out to him. policeman at the front passenger seat to place all her
In the crime of rape, the relationship between money on the console box near the gearshift. The car then
the offender and the victim is aggravating. proceeded to Harrison Plaza where the Montecillos were
Accused guilty of simple rape only, the told to disembark. From there, their dreadful experience
punishment was lowered (as to period of imprisonment) over, they went home to Imus, Cavite.
but moral and exemplary damages were awarded. The three policemen were charged and convicted
with robbery. Court of Appeals affirmed.

FORTUNA V. PEOPLE ISSUE: W/N the aggravating circumstance of abuse of


GR 135784, December 15, 2000 public position should be appreciated against the
(Was given the wrong citation, so I just googled for policemen? YES.
this case instead.)
HELD: To our mind, the success of the accused in taking
FACTS: While Diosdada Montecillo and her brother Mario their victims' money was premised on threats of
were standing at the corner of Mabini and Harrison Streets prosecution and arrest. This intense infusion of fear was
waiting for a ride home, a mobile patrol car of the Western intimidation, plain and simple. As a police officer, it is
Police District with 3 policemen on board stopped in front his primary duty to avert by all means the commission
of them. The policeman seated on the right at the front of an offense. As such, he should not have kept his
seat alighted and without a word frisked Mario. He took silence but, instead, should have protected the
Marios belt, pointed to a supposedly blunt object in its Montecillos from his mulcting colleagues. This
buckle and uttered the word "evidence." Then he motioned accused-appellant failed to do. His silence then could
to Mario to board the car. The terrified Mario obeyed and only be viewed as a form of moral support which he
seated himself at the back together with another zealously lent to his co-conspirators.

108
CRIMINAL LAW REVIEW DIGESTS
JUSTICE ROMEO CALLEJO NOTE: = Callejo Ponente

We however observe that the courts below failed aggravating circumstances of treachery, nighttime, and
to appreciate the aggravating circumstance of "abuse of contempt of or with assault to public authorities.
public position." The mere fact that the 3 accused were De Mesa questions the inclusion of aggravating
all police officers at the time of the robbery placed circumstances of treachery, nighttime, and assault to
them in a position to perpetrate the offense. If they public authorities.
were not police officers they could not have terrified
the Montecillos into boarding the mobile patrol car Issue: Was there an aggravating circumstance of
and forced them to hand over their money. Precisely it contempt of or with assault to public authorities?
was on account of their authority that the Montecillos
believed that Mario had in fact committed a crime and Ruling: No.
would be brought to the police station for The requisites of such circumstance are: (1) the
investigation unless they gave them what they public authority is engaged in the discharge of his duties
demanded. and (2) he is not the person against whom the crime is
Accordingly, the penalty imposed should be committed.
modified. Under Art. 294, par. (5), of The Revised Penal In this case, the aggravating circumstance
Code, the penalty for simple robbery is prision does not exist as the crime was committed against the
correccional in its maximum period to prision mayor in its barangay chairman himself and at the time that he
medium period. In view of the aggravating circumstance of was killed, he was not engaged in the discharge of his
abuse of public position, the penalty should be imposed in duties as he was in fact playing a card game (tong its)
its maximum period while the minimum shall be taken from with his neighbors.
the penalty next lower in degree, which is arresto The case also mentioned that the other
mayor maximum to prision correccional medium in any of aggravating circumstances of nighttime and treachery
its periods the range of which is four (4) months and one were not present. Since the aggravating circumstances of
(1) day to four (4) years and two (2) months. treachery, nighttime, and contempt of or with assault to
public authorities were not proved, De Mesa should be
@ PP v. PEDRO held guilty of homicide and not of murder.

PEOPLE V. DE MESA PEOPLE VS. TAC-AN (the BRONX Gang)

Doctrine: The requisites of aggravating circumstance of Facts: In two criminal cases, the trial court found Renato
contempt of or with assault to public authorities are: (1) Tac-An guilty of qualified illegal possession of firearm and
the public authority is engaged in the discharge of his murder, imposing upon him the penalty of death in both
duties and (2) he is not the person against whom the cases. Tac-An was a good friend and fellow Bronx Gang
crime is committed. member of Francis Escano III. They were both attending
rd
3 year high school in Divine World College in Tagbilaran
Facts: Patricio Motas was a Barangay Chairman of City. Tac-An was 18 years old while Escano was 15.
Barangay Sta. Cruz Putol, San Pablo City. One night, was Escano left the gang after his mom told him to stop
shot dead while playing tong-its with some townmates at a hanging out with Tac-An upon learning that the latter had
neighborhood store. Hernando De Mesa was accused of been carrying a gun around with him. From that point on,
committing the crime. It was said that De Mesa harboured their relationship soured. They got into a fist fight and
ill feelings towards Motas. In one instance, he threatened derogatory graffitis against the Bronx gang and Tac-An
Motas saying "May araw ka rin Chairman. Papatayin kita." were written on the walls of the school saying. Tac-An
After the crime was committed, one of the witnesses blamed Escano for this.
overheard De Mesa conversing with other men after the During an English class Tac-An got up from his
crime saying "Sigurado akong patay iyong putang inang si chair and approached the teacher to ask a question,
Chairman." leaving on his seat his scrapbook. When he returned to his
De Mesa put up the defense of alibi saying that chair he found Escano sitting on his scrapbook. They got
he was not at the scene of the crime but was instead into a fist fight. After they were seaparated, Tac-An
watching tv at home. This was corroborated by his wife. sneaked out of the school, went home and got his gun. He
The trial court charged him guilty of murder returned 15mins later during math class, under Mr.
because of the circumstantial evidence of testimony of the Pasilbas. Upon entering the room he fired his gun and
witnesses, flight of the accused, and motive to kill, demanded for Escano. The students ran towards the
presented by the prosecution. It also appreciated the teacher for protection. As Escano was running towards the
door, Tac-An shot him on the head. The trial court found

109
CRIMINAL LAW REVIEW DIGESTS
JUSTICE ROMEO CALLEJO NOTE: = Callejo Ponente

that the crime was aggravated with the circumstance of statute, to the disadvantage of an accused, we do not
contempt or insult to public authority. believe that a teacher or professor of a public or
recognized private school may be regarded as a "public
Issue: W/N the crime was committed in contempt or with authority" within the meaning of paragraph 2 of Article 14
insult to public authority? of the Revised Penal Code, the provision the trial court
applied in the case at bar.
Held: Nope! In addition, the SC also found no aggravating
A teacher or professor is not a public authority circumstance of evidence premeditation for the simple
Article 152 of the Revised Penal Code, as amended by reason that no sufficient evidence was presented to prove
Republic Act No. 1978 and Presidential Decree No. 299, that Tac-An had formed the intention and determination to
provides as follows: take Escanos life.
Similarly, the special aggravating circumstance of
Art. 152. Persons in authority and agents of acting while under the influence of Dangerous Drugs was
persons in authority. Who shall be deemed as also deleted because there was no medical proof
such. In applying the provisions of the presented to show that Tac-An was high when he
preceding and other articles of this Code, any committed the crime.
person directly vested with jurisdiction, whether So in the end, SC found no aggravating as well
as an individual or as a member of some court or as mitigating circumstances.
government corporation, board, or
commission, shall be deemed a person in
authority. A barrio captain and a barangay PEOPLE VS SAMUDIO
chairman shall also be deemed a person in
authority. FACTS: Herein accused appellant Antonio Samudio was
A person who by direct provision of law with three friends having a drinking spree in his place
or by election or by appointment by competent when they decided to transfer to Ely Samudios house.
authority, is charged with the maintenance of While they were drinking there, the victim, Baldomero San
public order and the protection and security of life Juan, Barangay Captain, passed by and was offered some
and property, such as a barrio councilman, barrio drinks by Antonios group. Thereafter, Benjamin Samudio,
policeman and barangay leader and any person uncle of Antonio, whose house was located approximately
who comes to the aid of persons in authority, 20 meters from Elys house, heard a commotion. When he
shall be deemed an agent of a person in went there, Benjamin saw Antonio stab Baldomero twice
authority. with a knife locally known as palas. While Antonio was
In applying the provisions of Articles 148 stabbing Baldomero, two of his companions held the
and 151 of this Code, teachers, professors and victim by his shoulder. Benjamin tried to intervene but to
persons charged with the supervision of public no avail.
or duly recognized private schools, colleges and Antonios group went out of the house and when
universities, and lawyers in the actual he passed by the house of a barangay councilman,
performance of their professional duties or on the Antonio told the latter that he had killed Baldomero.
occasion of such performance,shall be deemed Meanwhile, Ruben San Juan, the victims son,
persons in authority. rushed to Elys house upon knowing the incident.
Careful reading of the last paragraph of Article Suddenly, Antonio came and threatened Ruben. Antonio
152 will show that while a teacher or professor of a public left Elys house again and proceeded to the house of
or recognized private school is deemed to be a "person in another barriomate where he asked that the members of
authority," such teacher or professor is so deemed the CAFGU be called so he can surrender.
only for purposes of application of Articles 148 (direct In his defense, Antonio admitted sole
assault upon a person in authority), and 151 responsibility but interposed self-defense. According to
(resistance and disobedience to a person in authority him, Baldomero confronted him about a work which he
or the agents of such person) of the Revised Penal (Antonio) was contracted for by Baldomero; that Antonio
Code. In marked contrast, the first paragraph of Article claimed he was not paid for such work and that Baldomero
152 does not identify specific articles of the Revised Penal suddenly hit him prompting him to get hold of the knife and
Code for the application of which any person "directly stab Baldomero.
vested with jurisdiction, etc." is deemed "a person in Trial Court convicted Samudio of the crime of
authority." Because a penal statute is not to be given a murder and sentenced to reclusion perpatua.
longer reach and broader scope than is called for by the It is alleged in the Information that the killing was
ordinary meaning of the ordinary words used by such qualified by treachery, evident premeditation, abuse of

110
CRIMINAL LAW REVIEW DIGESTS
JUSTICE ROMEO CALLEJO NOTE: = Callejo Ponente

superior strength and disregard of respect due to the Memory aid: Nawasi, pumatay!
offended party on account of his rank.
Mangsant was charged with murder (qualified by
ISSUE: Whether the aggravating and mitigating treachery) for killing a 14-year old girl by stabbing her in
circumstances may be appreciated the back multiple times. Charged with aggravating: evident
premeditation, disregard of sex and taking advantage of
HELD: superior strength.
TREACHERY there was no treachery. When treachery Upon arraignment he pleaded "not guilty" but
is alleged, the manner of the attack must be proven (to during the trial and before the presentation of the evidence
show that the manner of attack was done to ensure the for the prosecution, said plea was changed to that of
victims defenselessness). In this case, the only "guilty". He testified that the 14-year old girl and he, were
eyewitness to the stabbing, Benjamin, did not see the lovers, having agreed to marry in the following May; that
initial stage and particulars of the attack on the afternoon of April 7, 1937, the date alleged in the
information, he visited his fiancee, and as in the course of
EVIDENT PRE-MEDITATION the court simply said that the conversation, she revealed that she loved another
the 3 requisites of evident premeditation was not present man, he became so obfuscated that he wounded her with
(bahala na kayo sa 3 elements nay un, hahaba lang a knife until she was lifeless. Nasawi, pumatay!
digest)
SC:
SUPERIOR STRENGTH although the accused- As to aggravating:
appellants were many, number alone does not determine No premeditation according to the description or
superior strength especially when the aggressors took no account of the crime given in the information.
advantage of their combined strength such as in this case. Disregard of sex [or age] cannot be
considered because it has never been proved nor
DISREGARD OF THE OFFENDED PARTY DUE TO HIS admitted by the defendant that in committing the
RANK this cannot be appreciated. Although crime he had intended to offend or insult the sex [or
Baldomero was a barangay captain, there is no age] of the victim.
showing that Antonio deliberately intended to Neither may the aggravating circumstance of
disregard or insult the respect due to Baldomero. It is abuse of superior strength be taken into account just
essential that the deliberate intent to offend or insult because of the fact that the defendant is a man and the
the rank of the victim must be shown. The aggravating deceased a woman, inasmuch as this circumstance is
circumstance of with insult or in disregard due to rank inherent in the crime committed and is moreover absorbed
is appreciated against an accused only when there is by the treachery which, in this case, qualifies the crime as
proof of fact of disregard and deliberate intent to murder.
insult the rank of the victim.
As to mitigating circumstances:
VOLUNTARY SURRENDER all the requisites were Not proper to consider lack of instruction,
satisfied. Antonio was not yet arrested; he surrendered inasmuch as he admitted that he had studied in the first
before persons in authority (CAFGU) and his surrender grade in a public elementary school. Lack of instruction
was voluntary [alternative circumstance] cannot apply to one who has
studied in the first grade in a public school, but only to him
OTHER ISSUES: who really has not received any instruction.
Acted upon an impulse so powerful as naturally
SELF DEFENSE there was no indication that there was to have produced obfuscation NO. It cant be considered
an unlawful aggression on the part of Baldomero. in his favor because the revelation by the deceased that
she loved another man, under the circumstances in which
CONSPIRACY conspiracy was not proved. Benjamin, it was made, was not sufficient to produce that mental
the only witness of the prosecution, did not witness the blindness which the RPC recognizes as mitigating. (tama
initial stage of the killing from which community of design pa ba to? mickey)
among the accused can be deduced. At most, the friends
of Antonio can only be convicted as accomplices.
PEOPLE VS. ANTONIO REYES
AGE
Dr. Aurora Lagrada, a spinster of about70 years old, lived
PEOPLE V. MANGSANT alone in her 2-storey house. Reyes house was about 4-5

111
CRIMINAL LAW REVIEW DIGESTS
JUSTICE ROMEO CALLEJO NOTE: = Callejo Ponente

meters away from the doctor's house. Reyes was able to FACTS:
gain entry into the house of Lagrada without the latter 1. Complainant, Vilma M. Concel (Vilma) is a 70
knowing. Armed with a bolo, Reyes stole one Rolex y.o. retired schoolteacher. She had 11 children
wristwatch, 1 gold bracelet, 1 gold ring with birthstone of by her late husband, and was the recipient of an
Jade, 1 pass book from Lagrada (amounting to P80,000). award as one of the outstanding mothers of the
On the occasion of the said robbery, Reyes stabbed province of Negros Occidental. Accused Nerio is
Lagrada several times in the different parts of her body a 28 y.o. field coordinator for ABS-CBN Radio.
directly causing her death. The trial court convicted Reyes Vilma, the victim, was his teacher in Grade 1.
of robbery with homicide and sentencing him to suffer the
2. Vilma was lying asleep in her bedroom insider
penalty of death.
her sari-sari store when she was awakened by
someone groping her breasts. The man
ISSUE: WON the trial court is correct in convicting Reyes?
undressed her, lay on top of her, took off his
YES.
clothes, while poking a knife at her. She tried to
grapple for possession of the knife, suffering cuts
HELD: To sustain a conviction of the accused for robbery
on her palm, but Nerio succeeded in ravishing
with homicide, the prosecution is burdened to prove the
her. Before he left, he told her he would be back
essential elements of the crime. The accused must be
the next day.
shown to have the principal purpose of committing
robbery, the homicide being committed either by reason of 3. Together with her daughter, Vilma went to the
or on occasion of the robbery. The homicide may precede police station a total of 3 times in connection with
robbery or may occur thereafter. What is essential is that the incident. At first she only complained of
there is a nexus, an intrinsic connection between the Trespass to Dwelling, Physical Injuries and
robbery and the killing. The latter may be done prior to or Grave Threats. On the third time, she finally told
subsequent to the former. However, the intent to commit them about the rape and she was examined.
robbery must precede the taking of the victim's life. Upon filing of the complaint, Nerio fled to Capiz.
Furthermore, the constituted crimes of robbery and He stayed there until he was arrested by the
homicide must be consummated. A homicide is Presidential Anti-Organized Crime Commission.
considered as having been committed on the occasion or 4. DEFENSE: Sweetheart theory and the sex was
by reason of the robbery when the motive of the offender consensual. Nerio told the Court that they were
in killing the victim is to deprive the latter of his property, to lovers and had three trysts prior to the incident.
eliminate an obstacle to the crime, to protect his On the first, Vilma asked him to help her transfer
possession of the loot, to eliminate witnesses, to prevent a potted plant. While he was washing her hands,
his being apprehended or to insure his escape from the she groped his groin and pulled him into the
scene of the crime. Appellant stated that he barged into bedroom. She performed oral sex on him and he
the house of the victim to rob her, and that he stabbed the complained about the pain. She removed her
victim when she was about to shout and because he was false teeth (!!!) and they proceeded to have
drunk. The appellant then took the victim's money and intercourse. For the second incident, Vilma
personal belongings and fled from the scene of the crime. allegedly gave him P300 after they had sex. The
The trial court correctly convicted the appellant of robbery third time he had trouble getting an erection so
with homicide. she performed oral sex on him again, she
ART. 294. Robbery with violence against or mounted him, they had sex, and she handed him
intimidation of persons Penalties. Any person guilty of P100 this time.
robbery with the use of violence against or any person 5. Trial Court found him guilty, hence this appeal.
shall suffer:
(a)the taking of personal property with the use of violence ISSUES
or intimidation against a person; (b) the property thus a) Whether or not guilt has been proven beyond
taken belongs to another; (c) the taking is characterized by reasonable doubt; YES
intent to gain or animus lucrandi and (d) on the occasion b) Whether the court erred in appreciating the
of the robbery or by reason thereof, the crime of homicide, aggravating circumstance of insult or in disregard
which is therein used in a generic sense, was committed. of the respect due the offended party on account
of her rank and age YES

PEOPLE OF THE PHILIPPINES vs. HILGEM NERIO Y HELD:


GIGANTO a) Nerio: unbelievable that Vilma can identify him due to
her faulty eyesight and that even if he used to be her

112
CRIMINAL LAW REVIEW DIGESTS
JUSTICE ROMEO CALLEJO NOTE: = Callejo Ponente

pupil, his physical appearance has changed PEOPLE V. JOSEPH MARQUITA AND ALEJANDRO
considerably since then. Plus there was delay in MARQUITA
reporting the incident.
Facts: Joseph, Alejandro and their friend the victim-
SC: Complainant recognized accused-appellant not deceased Sergio Pampilo were drinking in the house of
because he was once her former pupil but because victim-Pampilo. A small altercation ensued among them
she had seen his face on the date in question. This is because Pampilo didnt want Joseph and Alejandro to
the reason she was able to identify accused-appellant pass through his dike [whatever this was, the case did not
when his photograph was shown to her. As for the explain]. But as the altercation was growing, victim-
delay, complainant, an old lady, was ashamed to tell Pampilo grabbed a bottle of Tanduay Kulafu and struck
the police that she had been raped. The defense has Joseph Marquita in the face. Joseph felt the blood on his
utterly failed to show why complainant, a face and went on a rampage and stabbed the victim-
septuagenarian in the twilight of her life, a widow, and Pampilo in the stomach with a bolo. Alejandro tried to
a mother of eleven children, who testified not knowing prevent what was happening but he couldnt so he just ran
accused-appellant except when she was his Grade 1 away.
teacher, would file such a malicious charge against After stabbing Pampilo, Joseph went on a
him. rampage and also stabbed the sleeping family of Pampilo,
Vilma was, and is, respected not only in the his wife and 3 daughters. Two other children of Pampilo
community, having once been a candidate of their survived and were able to escape.
barangay, but in the entire province of Negros
Occidental, being one of the recipients of an awards Issue: Is the aggravating circumstance dwelling
as Outstanding Mother of the said province. Why appreciable against Joseph Marquita? NO
would she take interest in prurient matters and even
want to engage in a sexual liaison when ladies of her Held: Joseph is guilty of Homicide for killing Pampilo and
age and station in life are turning their thoughts to Murder as to family of Pampilo because of the qualifying
virtues? Accused-appellant would want this Court to circumstance of treachery since the wife was sleeping at
believe that complainant was a sex-starved old the time of the attack and was in no position to defend
woman who found accused-appellant, then 28 years herself and as to the children since they were mere
of age, so virile and irresistible that she showed children of tender years who were killed while they were
sexual aggressiveness even in their first encounter. sleeping. [Note that Treachery absorbs generic
Indeed, the picture painted of her by accused- aggravating of abuse of superior strength]
appellant is that of the equivalent of the dirty old man. Note that Alejandro Marquita was charged and
The Court cannot believe this fantasy. convicted in the lower courts but the SC acquitted him
because there was no evidence to establish conspiracy as
b) Accused claims that, other than the bare allegation it was shown that it was solely Josephs Hand which killed
that she is 70 years old and a retired public school the victims.
teacher, there is no proof that he deliberately intended
to offend or insult complainants rank or age. Dwelling Issue:
The trial court properly appreciated the existence First as to the Homicide of Pampilo - The generic
of the aggravating circumstance of insult or disregard aggravating circumstance of dwelling did not attend the
of the respect due to the offended party on account of killing of Pampilo because he gave sufficient and
her rank and age. Nerio knew that complainant was immediate provocation for the attack when he hit accused-
his Grade 1 public school teacher and was already Joseph with the tanduay bottle. Dwelling is aggravating
quite old. These facts were admitted by accused- only if the offended party has not given provocation.
appellant in the stipulation of facts embodied in the Second as to the murder of Pampilos Family
pre-trial order which he signed. As the Solicitor The SC said that considering that the killings were
General observes, accused was fully aware that he committed in the domicile of the four victims, without
was raping his old teacher. That complainant had provocation on their part, the aggravating circumstance of
already retired from the service as a teacher did not dwelling is present. Dwelling is considered an aggravating
diminish the respect due her rank as a former Grade circumstance by reason of the sanctity of privacy the law
1 teacher of accused-appellant. accords to human abode, for "he who goes to anothers
house to hurt him or do him wrong, is more guilty than he
DWELLING who offends him elsewhere."

113
CRIMINAL LAW REVIEW DIGESTS
JUSTICE ROMEO CALLEJO NOTE: = Callejo Ponente

PEOPLE vs. RIOS stabbing is not within the purview of the concept of
provocation under Article 14 (3) of the RPC. The
FACTS: Appellant Rios was charged with the crime of unrebutted facts established by the prosecution show that
murder for the fatal stabbing of deceased Ambrocio it was Rios who started the events that led to his
Benedicto. The spouses were owners of a sari-sari store unfortunate killing of Ambrocio, by stoning the latter's
in their subdivision. According Anacita (wife of the victim), house. In an apparent show of unmitigated braggadocio,
Rios, their neighbor, was hurling stones at their house the Rios even went to the victim's house on the pretext of
night of the incident. A few minutes later Rios bought buying cigarettes after the stone-throwing incident. The
cigarettes from their store and Ambrocio confronted him victim naturally confronted appellant about that incident.
about the stoning incident and an altercation between As the two engaged in heated argument, the roving tanods
them ensued. While the two were engaged in a verbal interved and two parted ways. However, a few minutes
scuffle, barangay tanods, who were then roaming the later, appellant returned to the victim's house and right at
vicinity, intervened and requested both parties to part the latter's terrace, dealt him the fatal stab wound. Under
ways. A few minutes later, Rios returned to the store. Just these circumstances, to cater to Rios' claim that the victim
then, Anacita saw her husband go to the terrace of their provoked him would amount to erasing the duly
house. Rios suddenly approached Ambrocio and stabbed established fact that by stoning the victim's house,
his right stomach. Anacita was only a meter away from the appellant himself instigated the heated argument that
antagonist; she was facing her husband's back while Rios resulted in his physical assault upon the victim.
was standing in front of Ambrocio. As Anacita started
shouting, Rios fled. The tanods saw Anacita weeping
while Ambrocio was lying lifeless in the terrace of their PEOPLE V DANIEL
house. One of the tanods assisted Ambrocio but the latter
succumbed to death even before they could reach the Facts: 13 year old Margarita Paleng filed a complaint
hospital. The postmortem certificate of death shows that against Amado Daniel alias Amado Ato for the crime of
Ambrocio died of "shock due to a stab wound at the chest rape. On September 20, 1965, Margarita, a native of Mt.
around 3 cm. penetrating the right auricle (heart)." Province, arrived in Baguio City from Tublay in a Dangwa
RTC found Rios guilty. Furthermore, it found that bus. She was then en route to her boarding house in
the killing of Ambrocio was attended by the qualifying Guisad as she was a highschool student at the Baguio
circumstance of treachery but that abuse of superior Eastern Highschool. While she was waiting inside the
strength is "comprehended" by said circumstance. It ruled bus, the accused Daniel came and started molesting her
out the presence of evident premeditation. However, it by inquiring her name and getting hold of her bag. She
considered dwelling as aggravating to the effect that even did not allow the latter and instead called the attention of
if the accused did not enter the victim's house, such as the bus driver and the conductor but was merely shrugged
when he shot the victim from under the house or when he off by them. It seemed that they were also afraid of the
fired the shot that fell the victim who was inside his house, accused. Despite the rain, she left the bus and went to
said circumstance is aggravating. ride in a jeep parked some 100 meters away. The
accused followed her and rode and sat beside her. When
ISSUE: W/N the RTC erred in considering dwelling as Margarita alighted in Guisad, she was again followed by
a generic aggravating circumstance. the accused. Reaching her boarding house, she opened
the door and was about to close it when the accused
HELD/RATIO: NO. The trial court was correct in dashed in and closed the door behind him. He pulled a
appreciating the aggravating circumstance of dwelling dagger 8 inches long and threatened her saying, If you
or morada in this case. talk, I will kill you. Because of her fear, Margarita fell
The word dwelling includes every silent. She was then forced to lie down with the accused
dependency of the house that forms an integral part placing a handkerchief in her mouth and holding the
thereof and therefore it includes the staircase of the dagger to her neck. Her attempts to flee were to no avail
house and much more, its terrace. When a crime is as she was only 4 ft and 8 inches tall and 95 lbs while
committed in the dwelling of the offended party and the Daniel was 5 ft, 7 inches tall and weighed 126 lbs. The
latter has not given provocation, dwelling may be accused was successful in having carnal knowledge of
appreciated as an aggravating circumstance. Provocation Margarita. Thereafter she lost consciousness. When she
in the aggravating circumstance of dwelling must be: (a) recovered, Daniel had already gone.
given by the offended party, (b) sufficient, and (c) For his defense, Daniel asserts that he and
immediate to the commission of the crime. Margarita have known each other since 1963 and this was
Considering these, the altercation between Rios in fact the second time he had carnal knowledge of her.
and Ambrocio that immediately preceded the latter's fatal Also, he alleges that he promised to marry Margarita and

114
CRIMINAL LAW REVIEW DIGESTS
JUSTICE ROMEO CALLEJO NOTE: = Callejo Ponente

was actually surprised that she filed the complaint against PERTINENT ISSUE: WON dwelling can be a aggravating
him. Medico-Legal report indicated that Margarita was a circumstance considering it was the house of Yolandas
virgin before the incident complained of. employer YES
The Court found Daniel guilty of the crime of rape
with the use of a deadly weapon with the aggravating RATIO: Although Yolanda was raped in a house belonging
circumstance of having been committed in the dwelling of to her employer, the same served as her residence, she
the offended party. being a stay-in laundress of Castillo. For all intents and
purposes, the same constituted a dwelling as the term
Issue: Whether or not a boarding house falls within the is used in Article 14(3) of the Revised Penal Code, it
definition of dwelling in the RPC? not being necessary, under the law, that the victim
own the place. Be she a lessee, a boarder, a bedspacer,
Held: Yes. Although Margarita was merely renting a or a maid, the place is her home, the sanctity of which the
bedspace in a boarding house, her room constituted for all law seeks to protect and uphold. Dwelling is considered an
intents and purposes a dwelling as the term is used in aggravating circumstance primarily because of the sanctity
Art. 14 (3) RPC. It is not necessary under the law, that the of privacy the law accords to human abode. One's
victim owns the place where he lives or dwells. But he a dwelling place is a sanctuary worthy of respect and that
lessee, a boarder, or a bed-spacer, the place is his home one who slanders another in the latter's house is more
the sanctity of which the law seeks to protect and uphold. guilty than he who offends him elsewhere. Cuello Calon
The correct penalty is death pursuant to Art. 335 of the says the commission of the crime in another's dwelling
RPC. However, for lack of necessary number of votes, shows greater perversity in the accused and produces
the penalty next lower in degree is to be applied. Daniel is greater alarm (People v. Monsayac, G.R. No.126787, May
sentenced to suffer the penalty of reclusion perpetua and 24, 1999).
ordered to indemnify Margarita Paleng by way of moral With the presence of one aggravating
damages of P12 K. circumstance, i.e. dwelling, the law has made it inevitable
that the greater penalty of death shall be applied.
Other crim related stuff: Three well-known
PEOPLE V SAPINOSO principles that guide the Court: (1) an accusation for rape
can be made with facility; it is difficult to prove but more
FACTS: Yolanda Partida, a 15-year old barrio lass was difficult for the person accused, though innocent, to
hired by Diosdado Castillo to work as a stay-in laundress disprove; (2) in view of the intrinsic nature of the crime of
at his residence in Tagig. Castillo's residence, rape where two persons are usually involved, the
parenthetically, also housed a shop for his stained glass testimony of the complainant must be scrutinized with
business. extreme caution; and (3) the evidence of the prosecution
At around 6 P.M. while Yolanda was lying on a must stand or fall on its own merits and cannot be allowed
folding bed located near the door of the shop, three men, to draw strength from the weakness of the evidence for
later identified as Domingo Quila and accused-appellants the defense . Likewise, when the complainant in a rape
Noel Sapinoso and David Recreo, suddenly barged in. case, more so if she is a minor, testifies that she has been
Yolanda stood up at the intrusion, only to be boxed by raped, she says in effect all that is necessary to show rape
Sapinoso, causing her to lose consciousness. has been committed , the offended party most often being
When Yolanda came to, she found Sapinoso on the only one available to prove directly the commission of
top of her. He was then inserting his penis inside her rape.
vagina, all the while poking a knife at her. Meanwhile, the
two others stood by the side of the bed and watched.
Yolanda felt pain at Sapinoso's insertion of his penis. After PEOPLE OF THE PHILIPPINES, appellee, vs.
a while, she sensed Sapinoso ejaculate, which she FRANCISCO M. SANTIAGO alias "FRANCIS," appellant.
described as "pinutok po niya yung kanya." Recreo and
Quila took their turns. The three then departed. FACTS: Spouses Francisco and Tess Santiago rented a
Yolanda immediately reported the incident to her room in the house under the care of Purita Sotero, in
employer, Castillo, when the latter arrived later that night. Baler, Aurora. They had a baby. Purita occupied another
They went to the Tagig police station to report the incident. room in the house. The neighboring house belonged to
The three accused were arrested after Yolanda identified spouses Jaime and Marissa Nisperos, who sold liquor and
them as her rapists lambanog. When Francis (accused) failed to pay rent,
Purita got had the matter placed in a police blotter, which
infuriated the accused Francis. One night, Santiago went
to the Nisperos house, eyes all red and he was angry with

115
CRIMINAL LAW REVIEW DIGESTS
JUSTICE ROMEO CALLEJO NOTE: = Callejo Ponente

Purita. He said Pare, I will kill her. Nisperos counseled the employment of means of execution that gives the
against killing her. person attacked NO opportunity to defend himself or
On Oct 11, 1995, around 5am, while Jaime went retaliate; (b) that the accused deliberately and consciously
to the store, Marissa hear Purita moaning in pain and adopted the means of execution. The prosecution failed to
shouting for help. She rushed to the house of Purita but prove that the appellant deliberately or consciously
couldnt open the door. Jaime arrived thereafter and they adopted a mode of attack to ensure the killing. There is
both tried to force open the door to no avail. Purita was even no evidence of the particulars as to how the
still moaning and shouting for help so the Nisparos aggression commenced or the events that led to the
spouses went to the side of the house where Puritas room stabbing.
was. They peeped through the wall and saw Francis As to evident premeditation, it was NOT
Santiago stabbing Purita, who was lying on the floor. The PROVEN as well. The prosecution failed to prove the
room was lighted by a lamp on the table. following: (1) the time when the accused decided to
The Nisparos went back to the store/house and commit the crime; (2) an overt act showing that the
later on Francis passed by and told Marissa that he left his accused clung to their determination to commit the crime;
baby in his room. Marissa replied: "Francis, lintek ka, bakit and (3) the lapse of a sufficient period of time, as to allow
mo sinaksak si Purita?" Santiago ignored Marissa and the accused to reflect upon the consequences of the act.
rushed to the bus terminal and boarded a bus. Meanwhile The appellant may have intended to kill the victim
Purita managed to open her door and asked for help. They even before however, there is no evidence on record that
rushed her to the hospital but she was dead on arrival. from that time on, until the victim was stabbed and killed,
She had 11 stab wounds. the appellant performed overt acts indicating his
Francis was later on found by policemen, hiding determination to commit the crime.
in a jeepney after alighting from the bus, and he admit to The aggravating circumstance of dwelling is
killing Purita. He was charged with MURDER, qualified by present. Although the appellant and the victim lived in the
treachery and evident premeditation. It was stated in the same house, the appellant and his family rented a room,
Information that he entered Puritas room, inside her while Purita lived in another. However, dwelling was NOT
house, stabbing her thereafter. alleged in the Information as mandated by Rules of
The RTC convicted him of MURDER qualified by Criminal Procedure. The rule is that when it is not alleged
treachery and evident premeditation, AGGRAVATED by in the Information, it will not be considered.
dwelling. He was sentenced to DEATH. Hence, the accused is convicted only of
HOMICIDE. Reclusion temporal.
ISSUE: W/N prosecution was able to prove the
circumstances of treachery, evident premediation and
dwelling. PEOPLE V. JOYA (1993)

HELD/RATIO: Treachery not proven. No evident AID: Rape- Store not Dwelling
premeditation. There is the circumstance of dwelling Facts: accused Romeo Joya and Joselito Arbolante were
BUT it was not alleged so it was not considered. charged for rape. The aforesaid accused, together with
Guilty of HOMICIDE only. one Ismael Cervania who was then at large, conspired
The prosecution contends that appellant killed and mutually assisted each other to have unconsented
Purita with treachery and evident premeditation beacuse carnal knowledge of 14-year old Maria Tolentino by means
Purita was asleep and defenseless when the appellant of force and intimidation such felonious sexual assault
stabbed her. And there was evident premeditation being attended by the aggravating circumstance of having
because the appellant told the Spouses Nisperos of his been committed in the dwelling of the offended party. The
intention to kill Purita. rape was done in the store of the victims mother after a
SC held that treachery was NOT PROVEN. drinking spree of the accused (Joya boxed Maria and
Treachery is a qualifying circumstance that changes the proceeded to rape her). Joyas defenses (he was not at
nature of the crime of homicide to murder. It must be the store, etc.) were denied by the court and credited
proven by the same quantum of proof as the crime itself. Marias testimony despite some inconsistencies.
There is no evidence that the victim was asleep when she
was stabbed by the appellant. Marissa testified that she Issue: WON the store of victims mom can be considered
heard Purita shouting for help. When she peeped through a dwelling to justify aggravating circumstance? (NO.)
the hole, she saw the appellant stabbing the victim.
Marissa did not know how the stabbing commenced. Held: the aggravating circumstance of dwelling can not be
For treachery to be qualifying, the prosecution considered in the case at bar. A dwelling must be a
must prove the confluence of the following requisites: (a) building or structure, exclusively used for rest and comfort.

116
CRIMINAL LAW REVIEW DIGESTS
JUSTICE ROMEO CALLEJO NOTE: = Callejo Ponente

The crime was committed in a store which was about Between seven and eight o'clock on the night of
fifteen meters away from the complainant's house. It is the 16th day of Mendoza's detention, there was a
obvious that the store cannot be considered a dwelling, or commotion in the house due to the barkings and howlings
even a dependency of complainant's home. of dogs. Soon Mendoza observed that his guards were
getting farther from him. Seizing the opportunity, he
sprinted and was able to escape. He reached at around
THE PEOPLE OF THE PHILIPPINES v. PEDRO midnight, where the Pasion was being chanted. He was
MASILUNGAN sent by the chief of police to the house of the town mayor
who told Mendoza to stay in the house.
Facts: On March 7, 1953, between eight and nine o'clock
in the evening while Jose Mendoza, then a barrio Issue: WON the crime charged falls under Art. 267 of the
lieutenant, was on the stairs of his house adjoining his, Revised Penal Code, as amended by Republic Act No. 18,
and carries with it the penalty of reclusion perpetua to
someone called out saying that he wanted to buy some
death? YES.
cigarettes. Mendoza told the prospective customer to get
them himself because he was then resting, but the latter
insisted that Mendoza himself hand the cigarettes to him Held: The concurrence of the aggravating circumstances
outside. Mendoza entered his store and got the cigarettes, of dwelling and use of motor vehicle having been shown
but upon coming out to hand them over to his customer, and proved, the Solicitor General recommends the
imposition of capital punishment, contending that the
he was grabbed by four armed men who poked their guns
penalty of reclusion perpetua imposed by the lower court
at him, warning him not to make any outcry if he wanted to
live. He was blindfolded and taken to a waiting pick-up is not in accordance with law. We find this
truck nearby. recommendations to be well taken, for besides the two
Once in the truck, Mendoza's pockets were aggravating circumstances mentioned above there also
concurred those of nocturnity and band, it having been
searched and the amount of P817.00 was taken from him.
proven that the victim was kidnapped at about nine o'clock
His blindfold was taken off, and it was then when he saw
in the evening and by four armed men.
and recognized appellant Pedro Masilungan and Arcadio
Mercado, whom he had known for a long time, they being
residents of the neighboring barrio and who used to ask Ratio: We have time and again held that alibi is the
from him petty loans of money. The pick-up truck then weakness defense that an accused can avail of, and oral
proof thereof must be clearly and satisfactorily established
made a quite long trip, and along the way appellant and
because it is so easily manufactured and usually so
Arcadio Mercado told Mendoza that he should not resent
unreliable that it cannot be given credit. In the present
the taking of his money and that which may thereafter be
taken from him because they would use it in buying case, the defense tried to established by the testimony of
firearms. The ride ended in a house situated between a appellant himself and his witness Daniel Laroza that
appellant was in the barrio in Mindoro, which involves an
rice field and a coconut grove where Mendoza was taken
11-hour trip from the province of Laguna by land and sea,
upstairs. There he saw an elderly man, said to be chief of
and that said appellant did not leave the place except
the kidnappers. They told Mendoza and made him swear
that he should not make any move without their consent, when he was arrested by Constabulary soldiers and taken
else he would be shot. They also demanded that he give to Calapan. But analyzing their testimony, we are with the
them P5,000, but he pleaded that he could not raise such lower court in not giving it any credence because of its
uncertainty and contradictory character.
a big amount, but that, if released, he was willing to give
Oral proof to establish and support an alibi must
P700.00. Appellant said that this could not be; likewise the
not be loose, vague and doubtful as in this case, but firm,
elderly man told Mendoza that he would be killed if he did
not come across with the amount. The bargaining by consistent, and trustworthy that when hurled against the
Mendoza was kept up for some days, and in the meantime evidence for the prosecution the impact must perforce
over helm the latter. In other words, such proof must not
he was being continuously guarded in shifts by armed
leave any room for doubting its accuracy, plausibility and
men.
verity. Certainly we cannot give any credit to the testimony
On the tenth day of his detention, Mendoza was
made to sign, at gunpoint poked at the opening of his ear of the appellant and his witness as to the date of
by the elderly man, a prepared note reading more or less appellant's arrival in Mindoro, for the fixing thereof is
as follows "My dear wife, send me P5,000; if you fail, it merely the result of guesswork.
The incident between appellant and Jose
means my life." This note eventually got into the hands of
Mendoza which allegedly occurred on the day prior to
Mrs. Mendoza, who found it at her doorstep.
appellant's departure for Mindoro regarding collection of
debt and which appellant attributes as the motive for

117
CRIMINAL LAW REVIEW DIGESTS
JUSTICE ROMEO CALLEJO NOTE: = Callejo Ponente

Mendoza's prosecuting him, cannot be believed, as it was The RTC found Tao guilty of robbery with rape
shown that Mendoza never bothered collecting the petty and imposed upon him the penalty of death. The RTC
debts of appellant during the course of years, and as judge appreciated dwelling as an aggravating
matter of fact, payment of such debts was left to circumstance because the incident took place supposedly
appellant's convenience. Appellant himself, on the other at the residence of private complainant's employer, "which
hand, admitted that when he last borrowed P20 from doubles as a video rental shop."
Mendoza three days prior to his departure, Mendoza did
not try to collect nor even mention the loan of P20 which ISSUES:
appellant had obtained earlier that month. Were we to 1. W/N Tao is guilty of robbery with rape GUILTY
again assume that the collection incident were true, the OF TWO SEPARATE CRIMES OF RAPE AND
same is so trifling that certainly would not cause the ROBBERY
offended party to charge the appellant with the serious 2. W/N dwelling can be appreciated as an
crime of kidnapping. aggravating circumstance NO

HELD:
PEOPLE v. TAO 1. Appellant is NOT guilty of the special complex
crime of robbery with rape. This felony contemplates a
FACTS: Amy de Guzman was tending a video rental situation where the original intent of the accused was to
shop owned by her employer and cousin Ana Marinay. take, with intent to gain, personal property belonging to
Accused Alexander Tao, a relative of Anas husband another; and rape is committed on the occasion thereof or
Gerry Marina, arrived at the shop and asked Amy what as an accompanying crime.
time Gerry and Ana would be coming home to which Amy Such factual circumstance was not present in this
replied that she did not know. Tao kept going in and out case. As related by Private Complainant Amy de Guzman,
of the shop and on the last time that he went inside the accused-appellant suddenly jumped over the counter,
shop, he jumped over the counter of the shop to where strangled her, poked a knife at the left side of her neck,
Amy was and seized the latter by placing one of his arms pulled her towards the kitchen where he forced her to
around Amys neck, while his other hand held a knife undress, and gained carnal knowledge of her against her
which he poked at her neck. Amy started shouting for help will and consent. Thereafter, he ordered her to proceed
the volume of the karaoke drowned her cries of help. upstairs to get some clothes, so he could bring her out,
Tao then dragged Amy to the kitchen of the shop where, saying he was not leaving her alive. At this point, appellant
at knife point, he ordered the latter to undress and he conceived the idea of robbery because, before they could
thereafter started raping her. However, while Tao was reach the upper floor, he suddenly pulled Amy down and
raping Amy, somebody knocked at the door of the shop started mauling her until she lost consciousness; then he
prompting the former to stop what he was doing and freely ransacked the place. Leaving Amy for dead after
ordered Amy to put on her clothes. He told her to go repeatedly banging her head, first on the wall, then on the
upstairs to the second floor to change clothes as he will be toilet bowl, he took her bracelet, ring and wristwatch. He
taking her with him. Amy then pleaded with Tao to just then proceeded upstairs where he took as well the jewelry
take anything inside the shop and to spare her life, to box containing other valuables belonging to his victim's
which Tao replied no, I will not leave you here alive. employer.
After a while and upon Amys pleading, Tao put Under these circumstances, appellant cannot be
down his knife and while he was kissing Amy, the latter convicted of the special complex crime of robbery with
got hold of the knife which she surreptitiously concealed rape. However, since it was clearly proven beyond
under the stairs. Suddenly, Tao became violent and reasonable doubt that he raped Amy de Guzman and
banged Amys head on the wall causing the latter to lose thereafter robbed her and Ana Marinay of valuables
consciousness. When she regained consciousness she totalingP16,000, he committed two separate offenses --
found herself and Tao inside the toilet of the shop and rape with the use of a deadly weapon and simple robbery
the latter again banged her head, this time on the toilet with force and intimidation against persons.
bowl, several times causing Amy to again lose 2. Dwelling aggravates a felony when the crime was
consciousness. Thereafter, Tao went upstairs and looted committed in the residence of the offended party and the
the place of valuables belonging to Amys employer, Ana. latter has not given any provocation. It is considered an
Amy, herself lost her ring, bracelet and wristwatch during aggravating circumstance primarily because of the sanctity
the incident in question. of privacy that the law accords to human abode. Ones
Tao, while admitting to robbing the shop, denied dwelling place is a sanctuary worthy of respect; thus, one
raping Amy. who slanders another in the latters house is more
severely punished than one who offends him elsewhere.

118
CRIMINAL LAW REVIEW DIGESTS
JUSTICE ROMEO CALLEJO NOTE: = Callejo Ponente

In the case at bar, the building where the two stepped back, revealing Sespene et al (3 total) armed with
offenses were committed was not entirely for dwelling a firearm. They fired four shots at Enerio. Enerio stood up
purposes. The evidence shows that it consisted of two and tried to escape but was shot. After that, he was knifed
floors: the ground floor, which was being operated as a at the nape. Just like in pinoy action movies, Enerio was
video rental shop, and the upper floor, which was used as still alive and tried to escape. But Sespene et al. caught up
a residence. It was in the video rental shop where the rape to him and killed him eventually. A complaint for murder
was committed. True, the victim was dragged to the (initially dismissed due to the absence of witnesses but
kitchen and toilet but these two sections were adjacent to subsequently revived) was filed against Sespene et al by
and formed parts of the store. Being a commercial shop the witnesses (Enerios wife and sister who witnessed the
that caters to the public, the video rental outlet was open whole thing). Sespene et al were found guilty of murder.
to the public. As such, it is not attributed the sanctity of Sespene et al then argued against convicton, claiming
privacy that jurisprudence accords to residential abodes. they were the real victims.
Hence, dwelling cannot be appreciated as an aggravating
circumstance in the crime of rape. (topic) SolGen contends that crime was attended with
aggravating circumstance of dwelling, among other
circumstances.
PEOPLE VS DACIBAR
ISSUE: W/N aggravating circumstance of dwelling
attended the crime. - NO.
Facts: Welda was sitting behind their bed near her
husband, Josue(soon to die), who was sitting at the end of
RATIO: Dwelling not applicable as enerio was only about
the bed. Suddenly, there was an explosion followed by
to step on the first rung of the ladder of the house when he
sounds of footsteps. When Welda looked through their
was assaukted by Sespene et al.
window she saw appellant coming out from under their
house, stooping and carrying a long fire arm. Seeing as
her husband was shot she shouted for help. Josue soon
died thereafter. It appears that her husband was shot from US v. IBANEZ
under the house. (Dwelling as aggravating circumstance in adultery)

Issue: WON the aggravating circumstance of dwelling is to Facts and Ruling


be considered in imputing liability Carmen Ibanez and Felix Alviola are lawfully married.
Held: yes Alviola filed a case of adultery against Ibanez and her
paramour, Pacifico Manalili. It was proven that the two
Ratio: Although the triggerman fired the shot from had intimate relations:
outside the house, his victim was inside. For the 1. They were alone together on a dry river bed in the
circumstance of dwelling to be considered, it is not shade of bamboo trees
necessary that the accused should have actually entered 2. Paramour was accustomed to frequent the home of the
the dwelling of the victim to commit the offense, it is spouses when husbandwas absent. During these visits,
enough that the victim was attacked inside his own house, the doors and windows of the house were closed
although he assailant may have devised means to 3. Wife often absented herself from her home
perpetrate the assault from outside the house. Thus, in 4. On one occasion, husband followed her and saw her
case at bar, although the attack was made from outside with paramour. The two separated ways upon perceiving
the house, below the floor of the house, nevertheless, the his presence. When he asked where his wife had been,
aggravating circumstance of dwelling may be considered she said she had been to the dressmakers
as attending the shooting. As in fact the victim was hit 5. On another occasion, husband surprised paramour
inside his own house. going down the stairs of the conjugal home and that
paramour immediately mounted his bike and rode away.
It was proven that wife and paramour had sexual
intercourse in the conjugal home twice. Husband was
PEOPLE VS SESPENE
notified and went to his house with a policeman and
surprised paramour hiding behind the kitchen door. Wife
FACTS: Enerio was went to the field to tend his carabao.
denied the presence of paramour despite being asked
When he got home and just as he was about to step on
who the bicycle outside the door belonged to.
the first rung of the ladder, Manglilog suddenly
The trial court found the defendants guilty of
appeared and attacked Enerio from the rear with a bolo,
adultery as principals by direct participation. SC affirmed
striking both his shoulders. Enerio faced Mangilog, who
but considered the aggravating circumstance of the crime

119
CRIMINAL LAW REVIEW DIGESTS
JUSTICE ROMEO CALLEJO NOTE: = Callejo Ponente

having been committed in the house of the aggrieved quarter may be assimilated to the victim's house, the
person in spite of the fact that the conjugal home was their former being an appendage of, or attachment to, the latter.
common domicile. Wife failed with her duty to be faithful
to her husband and paramour failed to respect the
sacredness of this conjugal home. PEOPLE OF THE PHILIPPINES, plaintiff-appellee, vs.
The aggravating circumstance of committing it in RODOLFO LAGARIO, ANECITO SAYONG, RODRIGO
his dwelling cannot be excused by the fact that the ENCISO and THREE JOHN DOES accused. RODOLFO
dwelling was also the home of the adultress; aside from LAGARIO, ANECITO SAYONG, and RODRIGO
the consideration that the stranger to the marriage who ENCISO, accused-appellants.
violates the law in that domicile is not a member of the
community residing there, the adultresss liability is morally FACTS:
and legally accentuated by her lack of respect for the - The Lagarios (pero hindi kasama si accused) were
domicile of the offended party. having supper. Their dogs were barking loudly so they
checked what was wrong outside. They saw 6 men.
- 3 of the 6 men rushed to the door of the house and
PEOPLE v ROEL PUNZALAN et.al. demanded that it be opened. When the residents
refused, one proceeded to the kitchen door while
FACTS: There are four accused-appellants in this case, another and an unidentified companion rammed the
namely: Roel Punzalan, Jose Besida, Marieta Mendoza main door. They eventually gained entry through a
and Domingo Mendoza (husband of Marieta). Except for hole on the door. They were armed with bolos.
Domingo, the rest of the accused are the domestic house - Hacking ensued. Matinding hacking (sa forehead, sa
helpers of the deceased-victim Mrs. Fule. They have been kamay, etc.). Arms and fingers flew. The men were
charged of the crime of robbery with homicide. after the money hidden in the trunk. (Alam na may
pera dun kasi anak nung bikitima yung isang
[note: this is how I imagine the house or compound of the akusado.) They left after.
victim. The main house is where the victim Mrs. Fule lives. - Shortly thereafter, the police arrived and investigated
Since she is old already, accused Marieta, one of the the incident.
domestic helpers, sleep near her bedroom to attend to her - The named accused above were convicted of robbery
(Mrs. Fules) medications. The helpers though have their with homicide.
own servants quarters, immediately beside the main
house of Mrs. Fule] ISSUE (this case is under the heading DWELLING ha):
W/N dwelling needs to be alleged to be appreciated.
One night, after supper, domestic helpers
Punzalan and Besida went out of the house for their HELD/RATIO: NO.
servants' quarters while the victim, Mrs. Fule and accused The generic aggravating circumstance of dwelling,
Marieta locked up all the doors to the house. Towards although not specifically alleged in the information, was
midnight, Domingo parked his jeep outside Mrs. Fules duly proved without objection on the part of the accused.
house and stayed there. Upon the signal of Marieta, the In robbery with homicide, dwelling may be properly
two other accused-appellants Punzalan and Besida appreciated as an aggravating circumstance. Dwelling is
forcibly entered the bedroom of Mrs. Lourdes Fule where not inherent in the crime of robbery with violence or
the latter was sleeping and once inside therein, jointly intimidation against persons because such crime can be
attacked or assaulted and stabbed Mrs. Fule which committed without violating or scaling the domicile of the
caused her instantaneous death and on the same victim. (Ito lang yung dwelling part.)
occasion and by reason thereof, with intent to gain, Taking into account the aggravating
ransacked the bedroom of the victim, Mrs. Fule and did circumstance of dwelling, the penalty imposable would
then take, steal and carry away cash money and pieces of have been death pursuant to the first paragraph of Article
jewelry. Accused-appellant Marieta did nothing to prevent 63 of the Revised Penal Code. In view, however, of the
the stabbing and the robbery. first paragraph of Section 19, Article III of the 1987
Constitution, which prohibits the imposition of the death
ISSUE: Whether or not the crime charged should be penalty, the penalty which must be imposed is reclusion
aggravated with the circumstance of dwelling. perpetua. The trial court imposed the penalty of life
imprisonment.
HELD: No. Dwelling should be disregarded because the
accused (except Domingo Mendoza) all resided in the ABUSE OF CONFIDENCE
servants' quarter of Mrs. Fule's residence. The servants'

120
CRIMINAL LAW REVIEW DIGESTS
JUSTICE ROMEO CALLEJO NOTE: = Callejo Ponente

In his testimony, Ostia admitted that he killed


PEOPLE V. OSTI Beverly by smashing a piece of rock bigger than the size
of his fist, about seven inches in diameter, on her head
Facts: Spouses Ponciano Onato and Edita Onato lived and chest and on the other parts of her body because, in
with their 4-year old daughter Beverly in Sto. Nio, Samar. the meantime, he lost control of himself.
Ponciano was a fisherman and a farmer but was The RTC found Ostia guilty beyond reasonable
employed by Tito Soria in his buy-and-sell of fish doubt of murder with the qualifying circumstance of
business. Roberto Ostia, a co-worker of Ponciano, evident premeditation and with the generic aggravating
resided in the poblacion of Sto. Nio. Rufo Legaspi, a circumstances of (a) abuse of confidence considering that
carpenter and a Barangay Tanod, was a neighbor of Roberto and Ponciano were co-workers, (b) nighttime
Ponciano. considering that Beverly was killed in the evening and (c)
On May 13, 1995, at about 7:00 p.m., Rufo was despoblado considering that the nearest house to the situs
seated near his house and resting before retiring for the criminis was fourteen meters.
evening. Then, Rufo saw Roberto, with Beverly on his The death penalty was imposed so the case was
right shoulder, walking towards the poblacion. Robertos automatically appealed.
left hand was holding the right hand of Mary Donoso, a 9- Note: The first ground for the appeal was Ostias
year old playmate of Beverly. The trio was in animated conviction despite his alleged improvident plea of guilty.
conversation on their way towards the poblacion. SC sided with Ostia on this issue and said the RTC judge
After an hour or so, Edita noticed that Beverly failed to adhere to the procedure accdg. to Rule 116 Sec.
had not returned to their house. She looked for her. Rufo 3 with respect to plea of guilty.
told Edita that he saw Beverly perched on the shoulder of The second ground was the qualifying
Roberto on their way towards the poblacion. Then, circumstance of evident premeditation. The SC said it was
Roberto passed by. However, Beverly was no longer with not alleged in the information and it was also not proven
him. Puzzled, Edita asked Roberto where Beverly was. by the prosecution. Hence, it could not be used.
Instead of responding, Roberto fled. The third ground was about the generic
Rufo, who witnessed the incident, advised Edita aggravating circumstances.
to report the incident to the police authorities. Edita
rushed back home and woke up Ponciano. She told her Issue: Whether or not there was abuse of confidence.
husband that Beverly had been taken by Roberto and that
Beverly had not yet returned home. The couple rushed Held: None!
from their house and reported the incident to the police
authorities. With the help of their neighbors and police Ratio: The trial court likewise erred in appreciating
officers Toribio and Espino, the couple looked for Beverly nighttime, despoblado and abuse of confidence as generic
but failed to locate her. They resumed their search the aggravating circumstances in the commission of the crime.
next day. They found Beverly sprawled in a grassy portion The prosecution failed to prove that Ostia purposely
below a copra kiln about 120 meters away from the house sought or took advantage of nighttime in killing Beverly.
of the Onato couple and about 15 meters from the nearest There is no evidence that he sought or took advantage of
house. Beverly was already dead. Pictures of Beverly the solitude of the situs criminis in committing the crime.
were taken where her body was found. Abuse of confidence could not be appreciated as generic
Since the municipal health officer was not there, aggravating circumstance because the prosecution failed
the Municipal Santiary Inspector Lorenzo Bernabe to prove that (a) Ostia enjoyed the trust and confidence of
st
conducted the autopsy. He had 4 findings: 1 , a lacerated Beverly or her parents; (b) and that even if Ostia enjoyed
nd
wound from Beverlys vaginal wall to the anus; 2 a said confidence, he took advantage of said trust or
rd
lacerated wound from the vagina to the mons pubis; 3 a confidence to kill Beverly. The barefaced fact that Ostia
th
contusion in the lumbar area and 4 , blood clots in the left and Ponciano were co-workers does not constitute
ear. evidence that the latter reposed trust and confidence in
Ponciano filed a crim complaint for rape with Ostia. In the absence of any generic aggravating or
homicide. An information for rape with homicide was then mitigating circumstances in the commission of the crime,
filed. the Ostia is meted the penalty of reclusion perpetua
On his arraignment, Ostia had no counsel so a conformably with Article 63 of the Revised Penal Code.
counsel de officio was assigned to him.
During trial, Ostia through counsel moved that he
be allowed to withdraw his plea of not guilty to rape with PEOPLE VS. ARROJADO
homicide and to enter a plea of guilty to murder. Ponciano
and the public prosecutor agreed.

121
CRIMINAL LAW REVIEW DIGESTS
JUSTICE ROMEO CALLEJO NOTE: = Callejo Ponente

Salvador Arrojado and Mary Ann Arrojado are first cousins laws are retroactive in that sense and to that extent. The
(their fathers are brothers). Alberto Arrojado, Mary Anns aggravating circumstance of abuse of confidence not
father, suffered a stroke for which reason he decided to having been alleged in the information, the same therefore
come home to Roxas City and spend the remainder of his could not be appreciated to raise accused-appellants
days there. Mary Ann and Alberto settled in a house in sentence to death.
Barangay Tanque, Roxas City where they lived on the
financial support o f Asuncion and Buenaventura (sister
and brother of Mary Ann). PEOPLE OF THE PHILIPPINES v. CAMILO
Later on, Salvador started living with Alberto and VILLANUEVA
Mary Ann. He helped care for Alberto for which he was
paid 1K salary. On December 5,1997, Reynaldo Gabuya received word
One day, Salvador went to the house of his that a kissmark was on the neck of his younger sister, 11
cousin, Erlinda Arrojado Magdaluyo, and reported that year old Nia Gabuya, who was in her fourth grade. She
Mary Ann had committed suicide. He told Erlinda that he only knew her father by name for she had been living with
was afraid he might be suspected as the one responsible her mother and her stepfather, Camilo, since she came to
for the death of Mary Ann. The matter was reported to the the age of reason. Reynaldo confronted and asked Nia
police. who planted said kissmark. He was told that it was their
The RTC found Salvador guilty of the crime of stepfather. He immediately brought his sister to the Pardo
murder with no aggravating circumstance. Police Station and had the incident blottered. Upon the
advice of the police officer, Nia was brought to the
ISSUE: Whether or not the aggravating circumstance of hospital. He further testified that Nia told her that she was
abuse of confidence is present??? YES. allegedly raped for the first time by Camilo Villanueva in
May 1997 and the last time was on December 4, 1997 at
RULING: For this aggravating circumstance to exist, it is around 12:00 midnight (but no penetration because
essential to show that the confidence between the parties according to them it was big for her). From May 1997 up
must be immediate and personal such as would give the to December 4, 1997 she has been sexually abused by
accused some advantage or make it easier for him to the appellant for seven times already.
commit the criminal act. The confidence must be a means For the defense, accused Camilo Villanueva
of facilitating the commission of the crime, the culprit denied having raped his step-daughter, Nia Gabuya. He
taking advantage of the offended partys belief that the claimed that at about 7:00 P.M. of December 4, 1997, he
former would not abuse said confidence. In this case, went to the mahjong place in order to sell the eggs which
while Mary Ann may have intimated her fear for her safety he cooked earlier in the afternoon. At 10:00 in the
for which reason she entrusted her jewelry and bank book evening, he and Felipa Gabuya, his live-in partner and
to Erlinda, her fears were subsequently allayed as shown mother of the victim went home and they arrived in the
by the fact that she took back her personal effects from house in ten minutes. Also, Camilo insisted that he did not
Erlinda. Thinking that Salvador would not do her any rape Nia and claimed that the spermatozoa found in the
harm, because he was after all her first cousin, Mary Ann victims organ was not his since he could not produce any
allowed Salvador to sleep in the same room with her after he underwent vasectomy in 1976.
father and left the bedroom doors unlocked.
The murder in this case took place after the TC: CAMILO guilty beyond reasonable doubt of
effectivity of R.A. No. 7659 on December 31, 1993 which the crime of rape as defined and penalized by Article 266-
increased the penalty for murder from reclusion A of the Revised Penal Code in relation to R.A. 7610 and
temporal maximum to death to reclusion perpetua to R.A. 8353
death. In view of the presence of the aggravating
circumstance of abuse of confidence and in accordance ISSUE:
with Art. 63(1) of the Revised Penal Code, the trial court 1. Whether the TC did not abuse its discretion in
should have imposed the penalty of death on accused- considering the fact of common law relationship
appellant. However, on December 1, 2000, the Revised between the accused and the mother of the
Rules of Criminal Procedure took effect, requiring that complainant when the information that was read
every complaint or information state not only the qualifying to the accused only accused him of rape of one
but also the aggravating circumstances. This provision NIA who was a step daughter of the accused.
may be given retroactive effect in the light of the well YES
settled rule that statutes regulating the procedure of the 2. Whether there is a generic aggravating
court will be construed as applicable to actions pending circumstance of abuse of confidence. YES.
and undetermined at the time of their passage. Procedural

122
CRIMINAL LAW REVIEW DIGESTS
JUSTICE ROMEO CALLEJO NOTE: = Callejo Ponente

SC: The trial court imposed the penalty of death after offenses as part of the civil liability when the crime was
taking into consideration the age of NIA who was then committed with one or more aggravating circumstances.
eleven years old at the time of the incident and the fact
that CAMILO is the common-law spouse of NIAs
mother. To justify the imposition of the death penalty PEOPLE VS. RAELITO LIBRANDO, LARRY
these two qualifying circumstances must be alleged in the SURDILLAS AND EDDIE PURISIMA
Information. A reading of the accusatory portion of the
information reveals that NIA is not the stepdaughter of Facts: Edwin Labandero brought his 8 y.o. daughter
CAMILO because her mother is not married to CAMILO. A Aileen to market in Barangay Bunga, Don Salvador
stepdaughter is a daughter of ones spouse by a previous Benedicto, Negros Occidental. On their way home, Edwin,
marriage or the daughter of one of the spouses by a Aileen and a relative, Fernando de los Santos, traversed a
former marriage. The relationship of stepfather hilly portion of the trail leading when they met accused-
stepdaughter presupposes a legitimate relationship. A appellants Raelito Librando, Larry Surdillas and Eddie
stepfather is the husband of ones mother by virtue of a Purisima. Raelito inquired from Edwin the whereabouts of
marriage subsequent to that of which the person spoken Fernando and without any warning hit Edwin with a piece
of is the offspring. of wood. Edwin ran but he was chased by Raelito.
This Court has consistently ruled that the Thereafter, the three men took turns hitting Edwin with
circumstances under the amendatory provisions of Section pieces of wood until the latter fell and died. Although it was
11 of Republic Act 7659, the attendance of which already dark at that time, Aileen had no trouble identifying
mandates the imposition of the single indivisible penalty of the accused-appellants since Edwin was carrying a lighted
death, are in the nature of qualifying circumstances which torch. While the men took turns in mauling the deceased,
cannot be proved as such unless alleged in the Edwin, Fernando took Aileen with him and ran to report
information, and even if proved, the death penalty cannot the incident to the Barangay Captain. The following day,
be imposed. Unlike a generic aggravating circumstance the police proceeded to the scene of the crime and saw
which may be proved even if not alleged, a qualifying the remains of Edwin Labandero lying prostrate on the
aggravating cannot be proved as such unless alleged in ground with a wooden pole on his neck. Raelito voluntarily
the information although it may be proved as a generic surrendered himself to the police while Eddie and Larry
aggravating circumstance if so included among those were invited for questioning at the police headquarters.
enumerated in the Code. The three men were asked to participate in a police line up
Since one of the twin qualifying circumstances during which Aileen positively identified them as her
aforementioned, namely, relationship, specifically that NIA fathers assailants. Raelito Librando claims that he merely
is the daughter of CAMILOs common-law wife, was not acted in self-defense and that it was Edwin who gave the
alleged in the information, CAMILO cannot be convicted of first blow. Raelito claims that his co-accused did not have
qualified rape and the death penalty cannot be imposed a hand in the killing of the deceased. The trial court
upon him, for to do so would be to deprive him of his convicted them of murder qualified by abuse of superior
constitutional right to be informed of the nature and cause strength and taking into consideration the aggravating
of the accusation. circumstances of nighttime and uninhabited place,
This notwithstanding, the fact that CAMILO is the considered only as one, and the mitigating circumstance
common-law spouse of NIAs mother and live with NIA of voluntary surrender in favor of all the accused.
may constitute the generic aggravating circumstance of
abuse of confidence there being a relation of trust and Issue: 1. WON the three accused-appellants are guilty of
confidence between her and CAMILO, whom she grew up murder -YES!
with and whom she even called papa. However, this 2. WON an 8 yr. old child is a competent witness -YES!
aggravating circumstance cannot be appreciated in 3. WON the court erred in not appreciating in Raelitos his
determining the appropriate penalty in view of the fact that favor the mitigating circumstances of a) incomplete self
the penalty prescribed for the offense of simple rape defense and b) voluntary surrender -NO!
is reclusion perpetua, an indivisible penalty. Under Article
63 of the Revised Penal Code in all cases in which the law Ratio: 1. While it is true that only Raelito Librando was
prescribes a single indivisible penalty, that penalty shall be shown to have any motive to assault the deceased,
applied by the courts regardless of any mitigating or nevertheless, it is hornbook knowledge that crimes have
aggravating circumstances that may have attended the been attributed to persons who appear to have no reason
commission of the deed. Nevertheless, this aggravating for committing them as long as they have been clearly
circumstance could provide legal basis for the award of identified as the offenders. 8 yr. old Aileen has
exemplary damages. Under Article 2230 of the Civil categorically stated that accused-appellants Larry
Code, exemplary damages may be awarded in criminal Surdillas and Eddie Purisima had a hand in the gruesome

123
CRIMINAL LAW REVIEW DIGESTS
JUSTICE ROMEO CALLEJO NOTE: = Callejo Ponente

killing of the deceased. Prosecution witness PO2 Dencing Facts: Aurelio Goze (the victim) and his wife Zenaida with
also stated that Aileen positively identified the accused- their children lived in a 3x5house with an extension called
appellants as her fathers assailants. It is basic that in the pataguab. The extension had a door apart from the door at
absence of any controverting evidence, the testimonies of the main house.
police officers are given full faith and credence as they are At 11pm, while the main house was lighted,
presumed to be in the regular performance of their official someone kicked open the door. 2persons forcibly took
nd
duties. The testimony of a single witness, if positive and Aurelio while Ernesto Dela Cruz (2 cousin of the
credible, is sufficient to sustain a conviction even in the deceased and with whom he had a land dispute) whom
absence of corroboration unless such corroboration is Zenaida recognized waited downstairs. Zenaida lighted
expressly required by law. another lamp.
While it is true that only Raelito was identified by By the moonlight, Zenaida saw these persons
name in the police blotter entry, nevertheless, it was take Aurelio to a place 30meters to the east of their house.
stated in the same police blotter that three persons had a Ernesto Dela Cruz shot Aurelio with a long firearm. The
hand in the killing of the deceased. Although Eddie and others then followed in the shooting.
Larry were not mentioned by name in the police blotter as The next morning, the body of Aurelio with
perpetrators of the crime, they were positively identified by multiple gunshot wounds was discovered around
the child, Aileen, during trial. 30meters away from their house. It was observed that
there were no trees or other structures which would
2. Any child regardless of age can be a competent witness obstruct the view of the body if you were looking from the
if he can perceive and perceiving can make known his house. The baranggay captain reported this to the SPO4
perception to others and that he is capable of relating Franklin Tagupa. Zenaida identified Dela Cruz and his
truthfully facts for which he is examined. The childs companions as the ones who killed her husband.
competence as a witness are: (a) capacity of observation; During the investigation, Dela Cruz told Tagupa
(b) capacity of recollection; and (c) capacity of that he knew who killed Aurelio. However, Tagupa testified
communication. in court that he forgot the name Dela Cruz told him. The
As noted by the trial court, Aileen during the trial was not next day, Tagupa then said that the person Dela Cruz
only a picture of innocence and honesty but was named was actually a military person and that he was
possessed with a strong power of observation and recall. afraid of retaliation.
DEFENSE STORY:
3. The mitigating circumstance of voluntary surrender was At the time the killing took place, Dela Cruz was
already appreciated for the accused appellants. To avail of sleeping the camp of the military. He was told to stay there
the mitigating circumstance of incomplete self defense, because threats from the NPA. They heard gunshots.
there must be unlawful aggression on the part of the Sgts. Evoco and Cavila arrived with Lt. Lleto and they
victim. In the case at bar, prosecution witness Aileen were told to keep quiet about what happened otherwise
testified that it was in fact the said accused-appellant who they will become the next victims. As already stated, he
delivered the first blow without any warning to the told Tagupa that he knew who killed Aurelio. (for more
deceased. The severity of the injuries inflicted on the details, pls refer to the original of the case. I dont think its
deceased as well as the fact that Raelito who admitted that important naman)
that he was of bigger built than the deceased, could hardly TC: Dela Cruz alibi was not believed. Zenaidas
present any evidence of injuries allegedly inflicted on him identification of the perpetrators were believed by the
by the deceased belie his claim of self defense. court. Guilty of Murder qualified by treachery and evident
premeditation attended by aggravating circumstance of
Extra: In the case of People vs. Santos, it has been held nighttime.
that if the aggravating circumstances of nighttime,
uninhabited place or band concur in the commission of the Issue: Whether the crime is murder or just homicide?
crime, all will constitute one aggravating circumstance only
as a general rule although they can be considered Held: Just homicide! No treachery and evident
separately if their elements are distinctly perceived and premeditation but there was abuse of superior strength.
can subsist independently, revealing a greater degree of Dela cruz assails the credibility of Zenaida as
perversity. witness. This isnt a crim issue so I wont discuss
anymore. Nevertheless, the Court believes Zenaida
NIGHT TIME because of her straightforward answers.
As to the identification of the witness, it was
PEOPLE V ERNESTO DELA CRUZ established that there was sufficient light for Zenaida to
identify the perpetrators. Just because she lighted another

124
CRIMINAL LAW REVIEW DIGESTS
JUSTICE ROMEO CALLEJO NOTE: = Callejo Ponente

lamp after her husband was taken away does not mean
that there was no light inside the house. The Court has 61. P v Goquila 428 Phil 716
held that a gasera or lampara has sufficient illumination for
identification. Moreover, Dela Cruz was known to Zenaida UNINHABITED PLACE
as her husbands cousin.
In this case, the defense belaboured the PEOPLE V. CABILES
argument that the killing took place under a moonless
night and that it was impossible for Zenaida to have seen Cabiles (uncle), Rudy Esparraguerra and his brother
what happened. The defense even submitted a Rogelio are the accused in this case. Cabiles however
certification from PAGASA as to the effect that at around remains at large.
the time of the commission of the crime, there was no Violeta was a fish vendorand is barriomates with
moon in the sky in their area. Nevertheless, moon or no the accused. One night, she visited her daughter to borrow
moon, the fact remains that Zenaida was able to identify 1k. the daughter accompanied Violeta back to her house
Dela Cruz. seeing that she was tipsy and weak. On their way to
Here, Zenaidas testimony that none of the Violetas house, Cabiles and the Esparraguerra bros
perpetrators even stopped the other from killing Aurelio appeared from nowhere and blocked their trail. The
showed that conspiracy attended the commission of the daughter recognized the 3 men, knowing them since
crime. For there to be conspiracy, it is sufficient that at the childhood.
time of the commission of the offense, all the accused has Rogelio demanded money from Violeta, who
the same purpose and were united in its execution. refused. This infuriated Rogelio, so he forcibly took the
Nevertheless, the Court agrees with Dela Cruz money from her and boxed her shoulders. Cabiles then
that the crime committed is merely homicide aggravated approached and twisted Violetas hands behind her. While
by the circumstance of abuse of superior strength but in that defenseless position, Rudy drew a bolo and hacked
which was not alleged in the information. her neck. The daughter, Salvacion was shocked and ran
Treachery was not established. There is away to go to her house. She immediately told her
treachery when the offender commits any of the crimes husband of the incident.
against persons, employing means, methods or forms in The next day, Violetas death spread in the
the execution thereof which directly and specially to insure barangay. Reynaldo, a resident thereof, apparently
its execution without risk to himself arising from the encountered the accused the night before. He had with
defense which the offended party migiht make. It must be him a bolo which he used for farming. For no reason,
proved that: 1) there was employment of means that gives Rogelio grabbed his bolo and attempted to hack him, but
the victim no opportunity to present a defense; and 2) that he got away. It turned out, that same bolo was the one
means and mode of attack must be consciously adopted. used to kill Violeta.
This the prosecution failed to prove. It appears that the All the accused put up alibis. But the trial court
decision to kill Aurelio was a spur of the moment thing found them guilty of robbery with homicide, with the
because the perpetrators were only initially looking for circumstances of disregard of sex and uninhabited place.
rice. Reclusion perpetua.
Abuse of superior strength was sufficiently
proven. What should be considered is not there are 3, 4 or Ruling: ruling affirmed, BUT THE AGGRAVATING
more assailants but whether the aggressors took CIRCUMSTANCES WERE HELD NOT PRESENT.
advantage of their combined strength to consummate the
offense. In this case, the number of attackers, the number As regards the aggravating circumstance of
and extend of gunshot wounds sustained and the manner uninhabited place (despoblado), the term uninhabited
of killing confirm the presence of this aggravating place refers not to the distance of the nearest house to
circumstance. the locus criminis. The more important consideration is
Evident premeditation was not also sufficiently whether the place of commission affords a reasonable
proven. possibility for the victim to receive some help. (note that it
NIGHT TIME PART: (super short lang sa case) was a sitio, so im assuming maraming bahay) Further,
For there to be night time as an aggravating circumstance, before it could be appreciated against the accused, it must
the accused must intentionally see the cover of the be established that solitude was purposely sought or taken
darkness for the purpose of committing the crime. If the advantage of to facilitate the commission of the crime.
place is lighted well enough for the offenders to be This circumstance was not satisfactorily proven in this
recognized, night time cannot be said to be an aggravating case
circumstance. Re: disregard of sex: That the aggravating
circumstance that the crime was committed with insult or

125
CRIMINAL LAW REVIEW DIGESTS
JUSTICE ROMEO CALLEJO NOTE: = Callejo Ponente

in disregard of the respect due the offended party on


account of his rank, age or sex, may be taken into P v Lubu 390 Phil 543
account only in crimes against persons or honor, when in
the commission of the crime, there is some insult or
disrespect shown to rank, age or sex. It is not proper to Uninhabited Place
consider this aggravating circumstance in crimes against Note: Lifted from Charm Calderinis digest under Abuse of
property. Robbery with homicide is primarily a crime confidence, but this digest will discuss more on
against property and not against persons. Homicide is a Uninhabited Place
mere incident of the robbery, the latter being the main
purpose and object of the criminal. It is thus erroneous to PEOPLE VS. RAELITO LIBRANDO, LARRY
take this aggravating circumstance into account in robbery SURDILLAS AND EDDIE PURISIMA
with homicide.
Facts:
Edwin Labandero brought his 8 y.o. daughter Aileen
PEOPLE V. LUNETA to the market in Negros Occidental. On their way
home, Edwin, Aileen and a relative, Fernando de los
FACTS: While Leon Gonzales and his wife Segunda Santos, traversed a hilly portion of the trail leading
Fuentes were walking along an uninhabited place in the when they met accused-appellants Raelito Librando,
municipality of Ivisan in Capiz, they were stopped by the Surdillas and Purisima.
appellant and one Dominador who is still at large. The Librando inquired from Edwin the whereabouts his
appellant and his companion were both armed with relative Fernando and then WITHOUT any warning,
revolvers and represented themselves as MP soldiers. hit Edwin with a piece of wood.
Leon was asked whether he was an army man to which he Edwin ran but he was chased by Raelito Librando.
answered in the negative, whereupon appellants Thereafter, the three men took turns hitting Edwin
companion gave him a fist blow on the stomach and with pieces of wood until the latter fell and died.
another blow with his revolver. In the meantime the Although it was already dark at that time, young
appellant was an onlooker with his revolver pointed at the Aileen had no trouble identifying the accused-
spouses. Dominador ordered Leon and his wife to take off appellants since Edwin was carrying a lighted torch
their clothes, a command which was obeyed. Dominador on their way home.
asked appellant to take away and kill Leon. Dominador While the men took turns in mauling the deceased,
then raped Segunda. Leon was able to escape though. Fernando took Aileen with him and ran to report the
When Dominador found out that Leon escaped, he went incident to the Barangay Captain.
out to look for him. When appellant was left with Segunda, The following day, the police proceeded to the scene
the former also raped the latter. Dominador and appellant of the crime and saw the remains of Edwin Labandero
left the scene and brought with them the clothes and other lying prostrate on the ground with a wooden pole on
stuff of the victims. After the victims reported the incident his neck.
to the police, the perpetrators were found in the house of a Librando voluntarily surrendered himself to the
lady doctor and were arrested. police while Eddie and Larry were invited for
questioning at the police HQ. The three accused were
ISSUE: W/N there was the aggravating circumstance of asked to participate in a police line up during which
commission of the crime in an uninhabited place? NO. Aileen positively identified them as her fathers
assailants.
HELD: It has not been proven that the appellant and his Librando claims that he merely acted in self-defense
companion purposely chose said place as an aid either to and that it was Edwin who gave the first blow.
an easy and uninterrupted accomplishment of their Librando further claims that his co-accused did not
criminal designs or to a surer concealment of the offense. have a hand in the killing of the deceased.
On the contrary, it is not improbable that the offended RTC convicted all of them of murder qualified by
parties were casually encountered, there being no abuse of superior strength and taking into
evidence that the accused had previously sought the consideration the aggravating circumstances of
former for any purpose whatsoever, or that said offended nighttime and uninhabited place (the two
parties were known by the accused to be habitual considered as one), and the mitigating circumstance
travellers in the vicinity. of voluntary surrender in favor of all the accused.
The offense committed is robbery with rape.
Issues:

126
CRIMINAL LAW REVIEW DIGESTS
JUSTICE ROMEO CALLEJO NOTE: = Callejo Ponente

1. WON all the three accused-appellants are guilty of GENERAL RULE. However, as an exception, they can be
murder -YES! considered separately if their elements are distinctly
2. WON an 8 yr. old child is a competent witness -YES! perceived and can subsist independently, revealing a
4. WON the court should apply the following: greater degree of perversity.
a) (mitigating) voluntary surrender - Yes
b) (mitigating) incomplete self defense - No
c) (aggravating) uninhabited place - Yes PEOPLE V. OCO
(considered as one with nighttime)
Held: Facts: Hermigildo Damuag was driving his motorcycle
2. While it is true that only Raelito Librando was shown to while Alden Abiabi was seated behind. A white tamaraw
have any motive to assault the deceased, nevertheless, FX blocked their path and thus they slowed down. Another
crimes WILL BE attributed to persons who, though motorcycle appeared and started shooting. Abiabi was
appearing to have no reason or motive, as long as they killed as a result of the gunshots while Damuag was
have been clearly identified as the offenders. Witness, wounded. There was also another motorcycle that passed
8 yr. old Aileen, categorically states that all three had a Damuag, with the driver firing shots at Damuag. Damuag
hand in the gruesome killing of the deceased. The was thrown off his bike and fell to the gutter. He saw that
testimony of a single witness, if positive and credible, is the driver of this motorcycle is Oco. He was able to identify
sufficient to sustain a conviction even in the absence of him because he was only wearing a towel around his
corroboration (unless such corroboration is expressly head, unlike the other riders who were wearing helmets.
required by law.) Damuag ran towards safety while being chased
by Oco, who was on his bike. He was brought to the
2. Any child regardless of age can be a competent hospital and he had his wounds treated. He survived
witness if he/she satisfies the minimum requirement of because of the prompt medical assistance.
the law. The he can perceive and he can make known Oco was charged of the crime of murder (Abiabi)
his perception to others and that he is capable of and frustrated murder (Damuag). The aggravating
relating truthfully facts for which he is examined. The circumstances of treachery, superior strength, motor
childs competence as a witness are: (a) capacity of vehicle, nighttime, by a band, aid of armed men, evident
observation; (b) capacity of recollection; and (c) premeditation, and unlicensed firearm.
capacity of communication. As noted by the RTC, Oco raised the defense of alibi saying that he
Aileen during the trial was not only a picture of was not there at the crime scene.
innocence and honesty but was possessed with a
strong power of observation and recall. Issue: Oco is surely guilty of murder and frustrated
murder. However, what are the aggravating circumstances
3.a) The mitigating circumstance of voluntary surrender present in the case? Which ones are absent?
was validly appreciated by the RTC.
Ruling: Aggravating circumstances present:
b) To avail of the mitigating circumstance of incomplete
self defense, there must be unlawful aggression on the 1. Treachery In this case, The unexpected and
part of the victim. Here, Aileen testified that it was in fact sudden attack on the victims, rendering them
the Librando who delivered the first blow without any unable and unprepared to defend themselves,
warning to the deceased. The number and severity of the such suddenness having been meant to ensure
injuries inflicted, the fact that Raelito Librando who admits the safety of the gunman as well as the success
that he was of bigger built than Edwin, could hardly of the attack clearly constitutes treachery. The
present any evidence of injuries allegedly inflicted on him gunshots fired against Abiabi and Damuag were
by the deceased BELIE any claim of self defense. so sudden that they were not given an
Therefore, there it cannot apply. opportunity to defend.
2. Abuse of superior strength This was absorbed
c)The fact that the attack was at nighttime, and occurred in treachery. Not only did they outnumber the
at a hilly portion of the trail leading to the three accused victims, they were also armed. The
was clearly in an uninhabited place. The Court however circumstances clearly show that the assailants
deemed the two as one aggravating circumstance. This is deliberately took advantage of their combined
in consonance with People vs. Santos, ruling that if the strength in order to consummate the crime.
aggravating circumstances of nighttime, uninhabited 3. Use of motor vehicle The motorcycles were
place or band concur in the commission of the crime, all used in going to the place of the crime, in
will constitute one aggravating circumstance AS A

127
CRIMINAL LAW REVIEW DIGESTS
JUSTICE ROMEO CALLEJO NOTE: = Callejo Ponente

rd
carrying away the effects thereof, and in restaurant where he could not pass anymore. The 3
facilitating the escape of the accused. motorcycle again fired at Damuag and then sped off.
Damuag was able to survive because of prompt medical
Aggravating circumstances absent: treatment. Abiabi on the other hand, died. Damuag was
only able to identify Oco because the dumb bastard was
1. Nighttime - This circumstance is considered only wearing a towel tied behind his forehead while the
aggravating only when it facilitated the other malefactors were wearing helmets. Oco was
commission of the crime, or was especially charged and found guilty of murder and frustrated murder,
sought or taken advantage of by the accused for imposing upon him the penalty of death.
the purpose of impunity. In this case, a lamp post
illuminated the scene of the crime. Although the Issue: What are the aggravating circumstances? (NB: In
offense was committed at night, nocturnity does Glenn Reviewer emphasis was given on commission of a
not become a modifying factor when the place is crime by a band.)
adequately lighted, and thus could no longer
insure the offenders immunity from identification Held:
or capture. Treachery
2. By a band - A crime is deemed to have been First of all, the SC found that there was treachery as the
committed by a band when more than three evidence showed that at the time the crime was
armed malefactors take part in its commission. In committed, the victims were in no position to defend
this case, the evidence on record shows that only themselves and the malefactors consciously adopted
two of accused carried firearms. particular means to ensure the execution of the crime with
3. Aid of armed men - Aid of armed men or persons no risk to themselves.
affording immunity requires that the armed men
are accomplices who take part in minor capacity, Abuse of superior strength present but absorbed by
directly or indirectly. In this case, the so-called treachery
armed men were never identified nor charged. The trial court also found that the offenses were
There was also no proof as to the participation of committed with abuse of superior strength. The
these other men. malefactors not only outnumbered the victims; at least two
4. Evident premeditation There was no direct of them were armed. The assailants deliberately took
evidence showing a plan or preparation to kill, or advantage of their combined strength in order to
proof that the accused meditated and reflected consummate the crime. Nevertheless, the aggravating
upon his decision to kill the victim. circumstance of abuse of superior strength is absorbed by
5. Unlicensed firearms - no evidence was adduced treachery.
to prove that the firearms used in the shooting
incident were unlicensed. Use of motor vehicle present
We also agree with the trial court that the generic
ARMED MEN aggravating circumstance of use of motor vehicle is
present. The appellant and his companions used motor
PEOPLE VS. OCO bicycles in going to the place of the crime, in carrying
September 29, 2003 away the effects thereof, and in facilitating their escape.

Facts: Herminigildo Damuag and Alden Abiabi were Nighttime absent


travelling on a motorcycle when a white Tamaraw FX We do not agree with the trial court, however, in its
overtook and blocked their path. Another motorcycle with appreciation of the aggravating circumstance of
2 riders on it appeared behind Damuag and Abiabi. The nighttime. This circumstance is considered aggravating
nd
2 motorcycle suddenly fired 2 shots in close succession. only when it facilitated the commission of the crime, or
Abiabi fell on the pavement. The Tamaraw sped away. was especially sought or taken advantage of by the
Another motorcycle (a 3rd one) appeared from behind accused for the purpose of impunity. The essence of this
Damuag. Damuag tried to control his motorcycle but it aggravating circumstance is the obscuridad afforded by,
zigzagged towards the gutter, throwing him off on the and not merely the chronological onset of,
ground. Damuag then saw Oco, who was riding on the nighttime. Although the offense was committed at night,
rd
back of the 3 motorcycle, firing away at him. He realized nocturnity does not become a modifying factor when the
then that he was shot on the right side of his body. place is adequately lighted, and thus could no longer
Nonetheless, Damuag was still able to run, being chased insure the offenders immunity from identification or
rd
by the 3 motorcycle, until he reached Five Brothers

128
CRIMINAL LAW REVIEW DIGESTS
JUSTICE ROMEO CALLEJO NOTE: = Callejo Ponente

capture. In this case at bar, a lamp post illuminated the unlicensed, hence, this circumstance cannot be
scene of the crime. appreciated.
In the end, the SC found that the only
By a band absent aggravating circumstance present use of motor vehicle
Likewise, we find that the offenses were not should be offset by the mitigating circumstance of
committed by a band. A crime is deemed to have been voluntary surrender. So ultimately, the penalty of death for
committed by a band or en cuadrilla when more than three the crime of murder was reduced to reclusion perpetua.
armed malefactors take part in its commission. The four And for frustrated murder, the indeterminate penalty of
armed persons contemplated in this circumstance must all prision mayor to reclusion temporal was imposed.
be principals by direct participation who acted together in
the execution of the acts constituting the crime. The Code
does not define or require any particular arms or weapons; 67. P v Viraya 400 Phil 202
any weapon which by reason of its intrinsic nature or the
purpose for which it was made or used by the accused, is FIRE ETC
capable of inflicting serious or fatal injuries upon the victim
of the crime may be considered as arms for purposes of
PEOPLE V MALNGAN
the law. In the case at bar, the prosecution alleged that the
accused and his three other co-conspirators used
Malngan was a yaya in Tondo. She burned down the
unlicensed firearms in the perpetration of the
house of her employer because she wasnt allowed to go
offenses. However, the evidence on record shows that
home to her province. Her employer told her, Sige umuwi
only two of them carried firearms. En cuadrilla, as an
ka, pagdating mo maputi ka na. Sumakay ka sa walis,
aggravating circumstance, cannot therefore be
pagdating mo maputi ka na! Incensed, she admitted that
appreciated.
Naglukot ako ng maraming diyaryo, sinindihan ko ng
disposable lighter at hinagis ko sa ibabaw ng lamesa sa
In aid of armed men absent
loob ng bahay!
There was also no evidence presented to show that
Her employers family died because of the fire (6
the offenses were committed with the aid of armed men.
of them!). and other houses in the neighborhood burned
Aid of armed men or persons affording immunity requires
down as well.
that the armed men are accomplices who take part in
She was charged with arson with multiple
minor capacity, directly or indirectly. We note that all four
homicide. The RTC convicted her of such offense.
accused were charged as principals. The remaining
suspects were never identified and charged. Neither was
Issue: Is there such a thing as arson with multiple
proof adduced as to the nature of their participation.
homicide?

Evident premeditation absent


SC: No.
There was also a paucity of proof to show that
THERE IS NO COMPLEX CRIME OF ARSON WITH
evident premeditation attended the commission of the
(MULTIPLE) HOMICIDE.
crimes. For this circumstance to be appreciated, there
The Information in this case erroneously charged
must be proof, as clear as that of the killing, of the
accused-appellant with a complex crime, i.e., Arson with
following elements: (1) the time when the offender
Multiple Homicide.
determined to commit the crime; (2) an act indicating that
Art. 320 of the RPC1, as amended, with respect
he clung to his determination; and (3) sufficient lapse of
to destructive arson, and the provisions of PD No. 16132
time between determination and execution to allow himself
respecting other cases of arson provide only one penalty
time to reflect upon the consequences of his act. Evident
for the commission of arson, whether considered
premeditation must be based on external facts which are
destructive or otherwise, where death results therefrom.
evident, not merely suspected, which indicate deliberate
planning. There must be direct evidence showing a plan or
preparation to kill, or proof that the accused meditated and
reflected upon his decision to kill the victim. No such 1 ART. 320. Destructive Arson. x x x x
evidence was presented to prove the presence of this If as a consequence of the commission of any of the acts
penalized under this Article, death results, the mandatory penalty
circumstance.
of death shall be imposed. [Emphasis supplied.]
2 Presidential Decree No. 1613:
Unlicensed firearm absent SEC. 5. Where Death Results from Arson. If by reason of or on
In the same vein, no evidence was adduced to prove the occasion of the arson death results, the penalty of reclusion
perpetua to death shall be imposed. [Emphasis supplied.]
that the firearms used in the shooting incident were

129
CRIMINAL LAW REVIEW DIGESTS
JUSTICE ROMEO CALLEJO NOTE: = Callejo Ponente

The raison d'etre is that arson is itself the end and death is As the drinking session went on, Robert and the
simply the consequence. others noticed appellants Antonio Comadre, George
When fire is used with the intent to kill a particular Comadre and Danilo Lozano (appellants) walking. The
person who may be in a house and that objective is three stopped in front of the house. While his companions
attained by burning the house, the crime is murder only. looked on, Antonio suddenly throw a hand grenade,
When the Penal Code declares that killing committed by ripping a hole in the roof of the house.
means of fire is murder, it intends that fire should be Drinking group were hit by shrapnel (fragments of
purposely adopted as a means to that end. There can be the grenade) and slumped unconscious on the floor. They
no murder without a design to take life. In other words, if were all rushed to the Hospital. However, Robert died
the main object of the offender is to kill by means of fire, before reaching the hospital. TC: appellants guilty of
the offense is murder. But if the main objective is the complex crime of murder with multiple attempted murder.
burning of the building, the resulting homicide may be
absorbed by the crime of arson. ISSUE: WON the trial court erred in convicting the
If the house was set on fire after the victims appellants?
therein were killed, fire would not be a qualifying
circumstance. The accused would be liable for the RULING: It was established that prior to the grenade
separate offenses of murder or homicide, as the case may explosion, Rey Camat, Jaime Agbanlog, Jimmy Wabe and
be, and arson. Gerry Bullanday were able to identify the culprits, namely,
Accordingly, in cases where both burning and appellants Antonio Comadre, George Comadre and Danilo
death occur, in order to determine what crime/crimes Lozano because there was a lamppost in front of the
was/were perpetrated whether arson, murder or arson and house and the moon was bright.
homicide/murder, it is de rigueur to ascertain the main
objective of the malefactor: No conspiracy. Only Antonio is liable for the crime.
(a) if the main objective is the burning of the building or When Antonio Comadre was in the act of throwing the
edifice, but death results by reason or on the occasion of hand grenade, George Comadre and Danilo Lozano
arson, the crime is simply arson, and the resulting merely looked on without uttering a single word of
homicide is absorbed; encouragement or performed any act to assist him.
(b) if, on the other hand, the main objective is to kill a Similar to the physical act constituting the crime
particular person who may be in a building or edifice, itself, the elements of conspiracy must be proven beyond
when fire is resorted to as the means to accomplish such reasonable doubt. Settled is the rule that to establish
goal the crime committed is murder only; lastly, conspiracy, evidence of actual cooperation rather than
(c) if the objective is, likewise, to kill a particular person, mere cognizance or approval of an illegal act is required.
and in fact the offender has already done so, but fire is The evidence shows that George Comadre and Danilo
resorted to as a means to cover up the killing, then there Lozano did not have any participation in the commission of
are two separate and distinct crimes committed the crime and must therefore be set free. Their mere
homicide/murder and arson. presence at the scene of the crime as well as their close
relationship with Antonio are insufficient to establish
Where then does this case fall under? conspiracy considering that they performed no positive act
Arson, the information against her stated with intent to in furtherance of the crime.
cause damage and that the deaths of her employers Neither was it proven that their act of running
were only on the occasion of the fire. away with Antonio was an act of giving moral assistance
Hence, she is being charged with the crime of to his criminal act. There being no conspiracy, only
arson. It is clear from the foregoing that her intent was Antonio Comadre must answer for the crime.
merely to destroy her employers house through the use of
fire. Antonios liability.
Treachery is present. Coming now to Antonios
liability, we find that the trial court correctly ruled that
PEOPLE vs.ANTONIO COMADRE, GEORGE treachery attended the commission of the crime.
COMADRE and DANILO LOZANO (critique this For treachery to be appreciated two conditions
decision by Tinga) must concur: (1) the means, method and form of execution
Robert Agbanlog, Jimmy Wabe, Gerry Bullanday, Rey employed gave the person attacked no opportunity to
Camat and Lorenzo Eugenio (drinking grioup) were having defend himself or retaliate; and (2) such means, methods
a drinking spree on the terrace of the house of Roberts and form of execution was deliberately and consciously
father (Jaime). adopted by the accused. Its essence lies in the adoption of

130
CRIMINAL LAW REVIEW DIGESTS
JUSTICE ROMEO CALLEJO NOTE: = Callejo Ponente

ways to minimize or neutralize any resistance, which may Act No. 7659 was already in effect. But while the case was
be put up by the offended party. pending, Rep. act No. 8294 was approved on June 6,
Antonio threw a grenade which fell on the roof of 1997.Section 2 of the latter law provides that when a
the terrace where the unsuspecting victims were having a person commits any of the crimes defined in the Revised
drinking spree. The suddenness of the attack coupled with Penal Code with the use of explosives, detonation agents
the instantaneous combustion and the tremendous impact or incendiary devices which results in the death of any
of the explosion did not afford the victims sufficient time to person or persons, the use of such explosives, etc. shall
scamper for safety, much less defend themselves; thus be considered as an aggravating circumstance:
insuring the execution of the crime without risk of reprisal Paragraph 3 of Article 248 of the Revised Penal
or resistance on their part. Treachery therefore attended Code, as amended by Rep Act No. 7659, was, thus,
the commission of the crime. amended by Section 2 of Rep. Act No. 8294. Under the
When the killing is perpetrated with treachery and latter law, the use of a hand grenade in killing the victim
by means of explosives, the latter shall be considered as a was downgraded from being a qualifying circumstance to
qualifying circumstance. a mere generic aggravating circumstance. Considering
that Section 2 of Rep. Act No. 8294 is favorable to the
RA 8294 not applicable in this case appellant, the same should be applied retroactively.
RA No. 8294 did not amend the definition of murder under Considering the factual milieu in this case, the generic
Article 248, but merely made the use of explosives an aggravating circumstance of the use of explosives is
aggravating circumstance when resorted to in committing absorbed by the qualifying circumstance of treachery.
"any of the crimes defined in the Revised Penal Code."
The legislative purpose is to do away with the use of PREMEDITATION
explosives as a separate crime and to make such use
merely an aggravating circumstance in the commission of PEOPLE OF THE PHILIPPINES vs. GABRIEL
any crime already defined in the Revised Penal Code. ANNIBONG y INGGAO
Thus, RA No. 8294 merely added the use of unlicensed
explosives as one of the aggravating circumstances FACTS:
specified in Article 14 of the Revised Penal Code. 1. Annibong, the accused, was a kitchen aide
Even if favorable to the appellant, R.A. No. 8294 assigned to the Army Camp Detachment in
still cannot be made applicable in this case. Before the Apayao. He was in the kitchen with Gabriel
use of unlawfully possessed explosives can be properly Tallong (witness), a CAFGU member. Corporal
appreciated as an aggravating circumstance, it must be Obngayan, the victim, arrived perspiring and
adequately established that the possession was illegal or thirsty. He went to get a drink and was irritated to
unlawful, i.e., the accused is without the corresponding find all the water containers empty. He went to
authority or permit to possess. Annibong, and boxed him three times in the
stomach and uttered: "Vulva of your mother, it is
CONCURRING AND DISSENTING CALLEJO, SR., J.: better that I will kill you."
I concur with the majority that the appellant Antonio 2. According to Annibong, he shot at Obngayan in
Comadre is guilty of murder for the death of Robert self defense. Obngayan allegedly took an M-16
Agbanlog, and multiple attempted murder for the injuries and aimed it at the accused, who then shot him in
sustained by the other victims. I dissent, however, from self-defense.
the ruling of the majority that the killing of Agbanlog is 3. This was contradicted by Tallong. According to
qualified by the use of explosives and not by treachery. the sole witness, the victim after boxing the
Under Section 3 of P.D. No. 1866 which took accused, was walking towards the bunkers when
effect on June 29, 1983, any person who commits any of Annibong suddenly attacked him from the back
the crimes defined in the Revised Penal Code with the use with an M-16, then later on, a garrand gun.
of explosives, detonation agents or incendiary devices Obngayan died instantaneously with his brain
which results in the death of a person shall be sentenced splattered and an eye fallen on the ground.
to suffer the death penalty. However, the imposition of the 4. The RTC found Annibong guilty of murder with
death penalty was suspended. the special aggravating circumstance of with
Under paragraph 3, Article 248 of the Revised insult or in disregard of the respect due the
Penal Code, as amended by Republic Act No. 7659, the offended party on account of his rank.
use of explosives in killing a person is a circumstance
which qualifies the killing to murder, the imposable penalty ISSUE:
for which is reclusion perpetua to death. When the crimes a) Is the self-defense theory of Annibong credible?-
were committed by the appellants on August 6, 1995, Rep. NO

131
CRIMINAL LAW REVIEW DIGESTS
JUSTICE ROMEO CALLEJO NOTE: = Callejo Ponente

b) What are the aggravating and mitigating clear proof as to when the accused
circumstances to be considered in this case? hatched the murderous plan, and the
interval of time therefrom to its
HELD: commission.
a) There was no unlawful aggression. Granting that Disregard of rank as well as respect due
the initial act of aggression came from the victim to the offended party: NO. Cannot be
when he cursed and then punched appellant appreciated as this was not alleged in
three times in the stomach, such aggression did the information.
not amount to actual or imminent threat to
appellant's life as the victim already ceased and Mitigating:
desisted thereafter. As Tallong testified, the Voluntary Surrender: YES. It is
victim was already walking slowly away towards immaterial that appellant did not
18
his bunker at the time appellant shot him immediately surrender to the authorities,
incessantly. At that point, it was no longer but did so only after the lapse of two
necessary for appellant to shoot Obngayan in days. There is voluntary surrender if
order to protect himself. In legitimate self-defense three conditions are satisfied: (1) the
the aggression must still be existing or continuing offender has not been arrested; (2) he
when the person making the defense attacks or surrendered himself to a person in
injures the aggressor. Thus when the unlawful authority or to an agent of a person in
aggression ceases to exist, the one making the authority; and (3) his surrender was
defense has no more right to kill the former voluntary. There is no dispute that
aggressor." appellant voluntarily surrendered to the
b) Aggravating: governor a person in authority, then to
Treachery: YES. (1) the means of the police, before he was arrested.
execution employed gave the person
attacked no opportunity to defend
himself or to retaliate; and (2) the means PEOPLE V. JOSE RODAS, SR. AND ARMANDO
of execution were deliberately or RODAS
24
consciously adopted. In this case, the
victim was totally unprepared and Facts: The two accused-appellants here were charged
unarmed, while appellant was carrying a together with 2 others who pleaded guilty to homicide
weapon. When shot, the victim was before the prosecution could rest its case.
already slowly turning away towards his Jose Rodas, Sr. with his 3 children, co-appellant
bunker. He was clueless of appellant's Armando Rodas, Charlito Rodas and Jose Rodas, Jr.,
sudden attack. Annibong consciously were charged for murdering victim Titing Asenda.
and purposely adopted the means of Titing Asenda was at a dance near the presence
attack to insure the execution of the of Alberto Asonda and Ernie Anggot, the two
crime without risk to himself. Such eyewitnesses to the crime. The two eyewitnesses testified
unexpected and sudden attack under that they saw the Rodas family suddenly surround Titing
circumstances that render the victim Asenda. Charlito stabbed Titing in the back. Then
unable and unprepared to defend Armando clubbed Titing with a chako [nunchucks] causing
himself constitutes alevosia. Thus, the him to fall. Then Jose Sr. handed Jose Jr. with a bolo
trial court did not err when it ruled that which Jr. used to hack Titing in the elbow.
treachery qualified the killing to murder. The eyewitnesses tried to help Titing but
Premeditation: NO. To prove this Armando pointed a gun at them. The Rodas Family left
attendant circumstance, evidence must and when the eyewitnesses approached Titing he was
show: (1) the time the offender already dead.
determined to commit the crime; (2) an The defense of Appellants Jose Sr. and Armando
act indicating that the offender had was alibi. That they were not present in the dance and it
clung to his determination; and (3) was only Charlito and Jose Jr. who killed Titing, the two
sufficient lapse of time between the who pleaded guilty.
determination to commit the crime and Despite the alibi, the RTC convicted appellant
the execution thereof to allow the Jose Sr. and Armando of murder qualified by Treachery.
offender to reflect upon the
consequences of his act. There is no

132
CRIMINAL LAW REVIEW DIGESTS
JUSTICE ROMEO CALLEJO NOTE: = Callejo Ponente

Issue: Appellants guilty of killing Titing? Aggravating immunity from identification or capture. Here, the dance
Circumstances? was adequately lighted which led to the positive
identification of the Rodas family.
Held: SC finds Jose Sr. and Armando guilty of Murder,
qualified by treachery. Denial or Alibi cannot override the D. Abuse of Superior attended the killing since there was
positive and credible testimony of two eyewitness. glaring disparity of strength between the victim and the
four accused. The victim was unarmed while the accused
NOTE Justice Callejo assigned this case under Evident were armed with a hunting knife, chako and
Premeditation but because the SC had a short discussion bolo. However, this is absorbed in Treachery.
on this, I think we should discuss other circumstances
considered by the Court.
PEOPLE vs. BALDOGO
A. For evident premeditation to be appreciated, the
following elements must be established: FACTS: Baldogo and Bermas were inmates who were
(1) the time when the accused decided to commit the serving sentence in the Penal Colony of Palawan. They
crime; were assigned as helpers of the Camacho family, who
(2) an overt act manifestly indicating that he has resides within the Penal Colony. In the evening of Feb. 22,
clung to his determination; and 1996, only siblings Jorge (14 y.o) and Julie (12 y.o.) were
(3) sufficient lapse of time between decision and left in their house together with the Baldogo and Bermas.
execution to allow the accused to reflect While Julie was studying in her room, she heard Bermas
upon the consequences of his act. calling from the kitchen ("Jul, tawag ka ng kuya mo"), but
The essence of premeditation is that the execution of the she ignored him. A few moments later, Bermas called her
criminal act was preceded by cool thought and reflection again, but Julie again ignored him. However, when Julie
upon the resolution to carry out the criminal intent during a heard a loud sound, akin to a yell (Ahh, ahh!), she got
space of time sufficient to arrive at a calm judgment. All out of the room and went to the kitchen, where she found
the SC said was that the prosecution failed to establish Jorge sprawled on the floor, lying face down and bloodied.
this circumstance. The vicinity was well lighted by a fluorescent lamp. Julie
saw Baldogo and Bermas standing over Jorge, each of
B. The essence of treachery is the sudden and them armed with a bolo. She ran back to the sala but the
unexpected attack by the aggressor on an unsuspecting two pursued her. Baldogo tied her hands at her back with
victim, depriving the latter of any real chance to defend a torn t-shirt and placed a piece of cloth in her mouth to
himself, thereby ensuring its commission without risk to prevent her from shouting for help from their neighbors.
the aggressor, and without the slightest provocation on the Baldogo dragged Julie outside the house and towards the
part of the victim. mountain. During their trek Baldogo and Bermas were
Here, Titing was completely unaware that he was able to retrieve their clothing and belongings from a trunk
going to be attacked. He was not forewarned of any which was located under a Tamarind tree. The following
danger to himself as there was no altercation or day, Bermas separated from Baldogo and Julie, who
disagreement between the accused and the victim. The continued their ascent to the mountain. The two stayed in
suddenness of the attack, the number of the accused and the mountains for a few days until Feb. 28, 1996, when
their use of weapons against the unarmed victim prevent Baldogo left Julie in the mountains to fend for herself.
the possibility of any defense or retaliation by the victim. Julie went to the lowlands & there she asked for help from
The fact that the victim was already sprawled on the Nicodemus. Nicodemus brought Julie to Balsaham where
ground and still Jose Jr. hacked him with a bolo clearly they met some personnel of the penal colony and police
constitutes treachery. officers, and Nicodemus turned Julie over for custody to
them.
C. Nocturnity cannot also be considered against the Two informations were filed accused-appellant
appellants. Nocturnity is aggravating only when it Baldogo and Bermas, one for crime of murder and the
facilitated the commission of the crime, or was especially second for kidnapping. Even before the arraignment,
sought or taken advantage of by the accused for the Bermas died. Baldogo denied killing Jorge and
purpose of impunity. The essence of this aggravating kidnapping Julie. He contends that while he was preparing
circumstance is the obscuridad afforded by, and not for sleep he was approached by Bermas, who was armed
merely the chronological onset of, nighttime. Although the with a bloodied bolo. Bermas warned him not to shout,
offense was committed at night, nocturnity does not otherwise he will also be killed. Baldogo maintained that
become a modifying factor when the place is adequately he did not intend to hurt Julie or deprive her of her liberty.
lighted and, thus, could no longer insure the offenders He averred that during the entire period that he and Julie

133
CRIMINAL LAW REVIEW DIGESTS
JUSTICE ROMEO CALLEJO NOTE: = Callejo Ponente

were in the mountain before Bermas left him, he tried to establishing when Baldogo and Bermas hid the bag under
protect her from Bermas. Baldogo asserted that he wanted the tree. The prosecution even failed to adduce any
to bring Julie back to her parents after Bermas had left evidence of overt acts on the part of Baldogo, nor did it
them and to surrender but Baldogo was afraid that Julio present evidence as to when and how he and Bermas
Sr., the father, might kill him. planned and prepared to kill Jorge and kidnap Julie and to
The RTC found Baldogo guilty of the two crimes prove that the two felons since then clung to their
charged, appreciating the qualifying aggravating determination to commit the said crimes. Although
circumstance of evident premeditation among many Baldogo and Bermas were armed with bolos, there is no
others. evidence that they took advantage of their numerical
superiority and weapons to kill Jorge.
ISSUE: W/N the RTC erred in appreciating the
qualifying circumstance of evident premeditation
despite the failure of the prosecution to prove it? PEOPLE V UBIA

HELD/RATIO: YES. Facts: Early in the evening of September 14, 1952,


The trial court convicted accused-appellant of Aureliano Carag, Mayor of Solana, Cagayan arrived on
murder with the qualifying aggravating circumstance of horseback at the house of the spouses Esteban Tambiao
evident premeditation, based on the following findings and and Teodora Quilang in the barrio of Bagag, Solana,
ratiocination: Cagayan. Upon arriving he tied his horse beside the
"The slaying of Jorge Camacho took place about 8:30 house, went up, and delivered a dead rooster which he
o'clock in the evening of February 22, 1996. It was had brought along, to Teodora Quilang to be cooked.
carried out after the accused have been through Carag, Dionisia and Esteban had conversation
tidying-up the kitchen, the dining room and the kitchen with Flora. Not long after this conversation, Teodora
wares the family of the Camachos used in their early Quilang called the three to go up for supper. Carag
dinner before 7:00 o'clock that evening. But even thought of removing the saddle off his horse, so he
before dinner, the accused have already made directed his steps towards the place where he had tied his
preparations for their flight, shown by the fact that horse. It was then that the first gunshot was heard. Carag
they already had their clothes, other personal was hit at the buttock and immediately called upon
belongings and food provisions stacked in their Teodora for help. Teodora immediately went down,
respective travelling bags then placed in a spot where accompanied by Dionisia and Esteban. Flora also heard
they can just pick them up as they take to flight." the call, so she also started to go down. It was then that
Although the SC agrees that Baldogo is guilty of further shots were heard.
murder, it does not agree with the ruling of RTC that the At that time, Proceso Ledesma, a policeman of
crime was qualified by evident premeditation. To warrant Carag, who had heard the shots and the call of Carag for
a finding of evident premeditation, the prosecution help, went down, stealthily approaching the besieged
must establish the confluence of the following house of the Tambiaos. Carag called upon Proceso to
requisites: (a) the time when the offender determined to help him, telling him to fire at his attackers and that Tomas
commit the crime; (b) an act manifestly indicating that the Ubia and his companions were on the road. It was then
offender clung to his determination; and (c) a sufficient that a voice was heard in answer, "You call for all your
interval of time between the determination and the policemen, although they are many, we are not afraid.
execution of the crime to allow him to reflect upon the The following morning, the police were called,
consequences of his act. and upon examining the premises around the house, they
The qualifying aggravating circumstance of found the dead bodies of Aureliano Carag, Dionisia
evident premeditation must be proved with certainty as the Tambiao and Esteban Tambiao.
crime itself. A finding of evident premeditation cannot be Eight witnesses testified to the facts which they
based solely on mere lapse of time from the time the saw during that night. Circumstances not denied by the
malefactor has decided to commit a felony up to the time defendants also indicate that all the other defendants must
that he actually commits it. In this case, the prosecution have participated in the commission of the crime, as
failed to prove evident premeditation. The barefaced fact pointed out by the testimonies of the witnesses for the
that Baldogo and Bermas hid the bag containing their prosecution. Appellant Tomas Ubia lost in the election for
clothing under a tree located about a kilometer or so from Mayor to the deceased Aureliano Carag. Tomas Ubia
the house of Julio Sr. does not constitute clear evidence filed a protest against him and utilized his two co-
that they decided to kill Jorge and kidnap Julie. It is defendant, Marcelo de Guzman and Loreto Mercado, as
possible that they hid their clothing therein preparatory to witnesses in his favor. These two individuals together with
escaping from the colony. There is no evidence Ruben Francisco were living at his house evidently at

134
CRIMINAL LAW REVIEW DIGESTS
JUSTICE ROMEO CALLEJO NOTE: = Callejo Ponente

Ubia's own expense. He gave them work by which to CRAFT


earn a living. The other defendants, namely, Romero
Pagulayan, Pascual Escote and Pablo Binayug, did not PEOPLE V. EMPACIS
live in Tuguegarao, but were tenants of or connected with
Tomas Ubia in his business and in politics, and when the FACTS: At about 9pm, as vicitms Fidel Saromines and his
party headed by Tomas Ubia arrived at his other house wife Camila were about to close their small store in Cebu,
in barrio Andarayan together with his co-defendants, these 2 men, Romualdo Langomez and Crisologo Empacis,
three were already there waiting for him. These are came and asked to buy some sardines and rice. After
circumstances which justify the conclusion that they they finished eating, Langomez announced a hold-up and
helped and joined Tomas Ubia in his evil design. ordered Fidel to give up his money. The latter started to
hand him PhP12K but suddenly decided to fight to keep it.
Issue: Whether or not the lower court erred in ruling that A struggle followed in the course of w/c Langomez
the commission of the crime was attended with an stabbed Fidel about 3 times. Empacis joined in and w/ his
aggravating circumstance of evident premeditation. own knife also stabbed Fidel. At this time, gunshots were
heard outside the house (im guessing the gun shots were
Held: There is no question that evident premeditation signals by the 3 other conspirators).
was present. It has been held that if a crime was planned It was only when Peter, Fidels 13-yr old son,
at 3:00 o'clock in the afternoon and carried out at 7:00 saw his father fighting for his life and rushed to his fathers
o'clock in the evening, or planned at 4:00 o'clock in the defense w/ a pinuti (a long bolo) striking Empacis and
afternoon and executed at 7:30 o'clock in the evening, the inflicting 2 wounds on him did the 2 men flee. Fidel died
aggravating circumstance of evident premeditation is from the fatal injuries, w/c penetrated his lungs and heart.
present because sufficient time has intervened between Empacis went to the clinic of Dr Eustaquio for the
the conception of the idea and the resolution to carry it out treatment of his wounds inflicted by Peter. He told the
and the fulfillment thereof This is what exactly took place doctor that he was assaulted w/o warning by a young man
in the case at bar. near the Papan Market.
The court correctly found that the aggravating The next day, police officers went looking for a
circumstance of alevosia attended the commission of the man who might have been treated for wounds from a
crime, with nighttime as having been included therein. The bladed weapon. They came to Dr Eustaquios clinic who
scene of the crime was in a remote barrio where Carag told them about Empacis. He was found at the public
must have felt secure. This, together with the suddenness market where they arrested him. He admitted going to the
of the attack and the darkness of the night, certainly store of Fidel but denied having joined Langomez in his
insured the success of the attack and shielded the attack. He asserts that he tried to stop him but the latter
conspirators from risk or danger. There was, furthermore, succeeded in stabbing Fidel. He further alleges that he
the additional circumstance of abuse of superior strength was brought by his neighbors to the clinic. The other 2
because there were no less than eight of the attackers, all men, who were accused of firing the gun from outside,
acting in concert around the besieged house, three of denied any participation in the crime. They were both
whom were armed with carbines, which are certainly absolved by the court. Langomez disappeared & could
superior in deadliness and accuracy to the only pistol with not be found.
which the victim was armed.
We, therefore, find that three murders have been Several aggravating circumstances: 1. Dwelling of the
committed, with the qualifying circumstances of treachery offended party 2. Nighttime 3. Employment of craft and
and abuse of superior strength. In the commission of fraud. 4. Advantage being taken of superior strength
these crimes, we hold that Tomas Ubia, Jose Ubia,
Loreto Mercado, and Marcelo de Guzman participated as HELD:
principals, whereas Romero Pagulayan, Pascual Escote 1. CRAFT AND FRAUD was properly appreciated against
and Pablo Binayug took part as accomplices. Empacis. Both men pretended to be bona fide customers
But as to Tomas Ubia, who conceived the plan of the victims store and on this pretext gained entry into
and utilized his influence to carry out the offense, the show the latters store and later, into another part of his
marked determination, cruelty and depravity. He did not dwelling. In previous cases, the Court held the presence
wreak vengeance on his personal enemy alone, but gave of fraud or craft when one pretended to be constabulary
vent to his anger unnecessarily murdering two other soldiers to gain entry into a residence to rob and kill the
innocent and defenseless victims. For him justice cannot residents, pretended to be needful of medical treatment
be tempered with mercy; the law must be applied in its full only killing the owner of the house, and pretended to be
force and to its full extent. wayfarers who had lost their way to enter into a house.

135
CRIMINAL LAW REVIEW DIGESTS
JUSTICE ROMEO CALLEJO NOTE: = Callejo Ponente

2. NIGHTTIME was also properly appreciated as nocturnity near the National Highway. Conductor Eduardo Valle,
was deliberately and purposely sought to facilitate the went down the minibus and allowed appellant to get inside
commission of the crime. Nighttime is not per se the bus. He observed that his clothes, particularly the right
aggravating unless shown that it was deliberately and side of his jacket and the right side of his pants, was
purposely sought to facilate or actually facilitated the soaked with blood. When asked where he was going, he
commission of the crime. The lateness of the hour didnt answer, he just paid the fare. Conductor observed a
prevented other customers from being there to defer the wad of cash in his breast pockets. Body of Bonifacio later
act or coming to the aid of the victims. found with gunshot wounds and stab founds.

3. SUPERIOR STRENGTH properly appreciated. To be ISSUE: W/N he is guilty of robbery with homicide,
deemed present, it doesnt suffice to prove superiority in aggravated by fraud and craft.
number on the part of the malefactors but that they
purposely employed excessive force, force out of HELD/RATIO: The testimonies of the prosecution witness
proportion to the means of defense available to the person was able to prove guilt beyond reasonable doubt.
attacked w/c was present in this case. Empacis & his Circumstantial evidence was able to establish the guilt of
companion took advantage of their combined strength & appellant beyond reasonable doubt. The time element of
their bladed weapons to overcome their unarmed victim & the circumstances thus proven link each chain of
assure the success of their felonious design to take the circumstances to another pointing to a reasonable
money. conclusion and no other but the guilt of appellant. From
the early morning of October 27, 1994 to 10:00 o'clock in
4. DWELLING also. The victim not having given the morning of the same day, the prosecution has
provocation. sufficiently shown that the appellant was the last person
seen with the victim before the latter was killed. About
thirty minutes later, appellant was seen speeding away.
PEOPLE OF THE PHILIPPINES,Plaintiff- Appellant's intention to rob the victim can be
Appellee, v. VIVENCIO LABUGUEN @ gleaned unerringly from the attendant circumstances.
DENCIO, accused-appellant. Obviously, robbery was the motive that impelled appellant
to convince the victim to go with him. Under the pretext of
FACTS: Appellant Labuguen is accused of the crime of selling cows to him, appellant cajoled the victim to bring a
Robber with Homicide, with the aggravating circumstance large sum of money and thereafter, lured him to a route
of fraud and craft, hence the trial court sentencing him to where appellant could divest him of his money with the
death. least danger of being caught. As aptly surmised by the trial
The victim is Bonifacio Angeles, who was court, the two inches thick of one hundred peso bills in
engaged in the business of buying cows and selling them appellant's pocket and the blood smeared on his clothes
at the public market. On Oct 27, 1994, a certain Tomas are two vital chains of circumstances that undoubtedly
Pagbigayan went to Bonifacios house, offering him 2 bespeak of the robbery with homicide appellant
cows to sell. Bonifacio said that he would see the cows committed.
when he has the time and offered to accompany him to Though not alleged in the Information, the
the crossing/terminal. They rode Bonifacios bike to go generic aggravating circumstances of fraud and craft were
there and when the latter came back to the house, the properly appreciated by the trial court. Craft involves
appellant Labuguen was riding in the back of the intellectual trickery and cunning on the part of the
motorcycle with victim. offender. When there is a direct inducement by insidious
16
While in the house, Labuguen offered 3 cows for words or machinations, fraud is present. By saying that
sale to Bonifacio. The latter wanted to inspect the cows he would accompany the victim to see the cows which the
because they were nearby so he got P40,000 cash and latter intended to buy, appellant was able to lure the victim
went with Labuguen to where the cows supposedly were. to go with him.
They rode Bonifacios motorcycle. Several witnesses saw
Bonifacio and Labuguen riding together off to the country SUPERIOR STRENGTH
side. Between 11:00 to 12:00 o'clock noon on the same
day, Geronimo Rivera was driving a Challenger, a PEOPLE V. LORETO (1993)
passenger mini-bus in Isabela. His conductor was
Eduardo Valle. AID: 3 VICTIMS- ONE of which is MURDER-ABUSE OF
At Nappaccu, Geromino Rivera (Rivera) saw at a SUPERIOR STRENGTH
distance of 200 meters, a person behind some talahibs

136
CRIMINAL LAW REVIEW DIGESTS
JUSTICE ROMEO CALLEJO NOTE: = Callejo Ponente

Facts: Samuel Loreto of murder, homicide and of slight strength over that of the victim, considering the position of
physical injuries and meting on him the penalty of death both and the employment of means to weaken the
for murder. Leah Rondena (12) died, Lettymar Rondena defense, although not annulling it.
(9) died as well, while third victim Princess Rondena (3 Note: the minority of the Leah and Lettymar was
month old baby) survived. not sufficiently proven. IF It was proven, then such act of
Loreto would have automatically been qualified to murder.
Facts are simple-- Witness Romeo (neighbor/trike driver)
witnessed the killing spree when he saw Leah Rondena
emerged from the house of Rolando (Leahs half-brother) PEOPLE V. ALARCON, TOMPONG, AND GUMAWA
and into the house of Dan (brother of accused) and Per Curiam Decision (EN BANC)
thereafter the whole incident occurred, leading sadly to
Leah (who was chased around) and Lettymars (already FACTS:
found dead) death. NOTE: This killing spree of the two Three different informations were filed against
young girls occurred while their half-brother Rolando was the accused (co-conspirators) WILFREDO
away as a tricycle driver as well, with the latter entrusting ALARCON, EDDIE TOMPONG, and EDUARDO
the babysitting of Princess to the two young ladies. NOTE GUMAWA.
also: accused Loreto was previously charged with murder o Criminal Case No. 5630 for RAPE WITH
before he came to the victims town in Puerto Princesa. HOMICIDE
o Criminal Case No. 5631 for RAPE
Issue: Aggravating circumstance of abuse of superior (resulting to the death of Aisha Dava),
strength and treachery applicable in the death of Leah committed by more than 2 persons,
Rondena? (YES only to superior strength. No to aggravated by superior strength
treachery.) o Criminal Case No. 5632 for RAPE
resulting to the death of Aisha Dava),
Held: Aside from the court affirming by circumstantial committed by more than 2 persons,
evidence of the killing of Lettymar and physical injuries on aggravated by superior strength
Princess, and more IMPORTANTLY in this case, The ALARCON, TOMPONG, GUMAWA pleaded not
victim, Leah, is a girl only (12) years of age unarmed when guilty. The three cases were heard jointly. The
brutally slain. On the contrary, the accused is a man of prosecution presented five witnesses: Dr. Irma J.
legal age and armed with an eight inches knife. Adayon, Lucia Dava, Melita Cancer, Ostimiano
Considering the gender, age, height, built, size, and agility Untalan and Amador Martinesio and rebuttal
of the accused, he is much superior in strength and witness Pedro Enque.
disposition than the hapless and innocent victim. The use The evidence of the prosecution shows:
of a bladed weapon even if already superior in physical o At 7AM of May 26, 1995, accused
strength in killing the victim is indicative of the accuseds ALARCON (17 years old) was on his
unmistakable intent of taking advantage of his superior way to Sitio Casoy, Bugasong, Antique
strength. However, the trial court further declared that to gather firewood. On his way, he met
treachery was attendant but can no longer be appreciated Lola Magang, the grandmother of
against accused-appellant because treachery absorbed the11-year old victim, Aisha Dava
abuse of superior strength. (AISHA), carrying goods to be sold at
However, the Court does not agree with the the market in Valderrama.
ruling of the trial court that although treachery absorbed o Upon reaching Sitio Casoy, ALARCON
abuse of superior strength, it is abuse of superior strength saw TOMPONG (42 years old with his
and not treachery which qualified the crime. What should wife Gloria) and GUMAWA gathering
qualify the crime is treachery as proved and not abuse of firewood and tying them in bundles.
superior strength. If treachery is not proved but abuse of AISHA was watching the group bundle
superior strength was proved by the prosecution, the firewood while TOMPONG and
crime is qualified by abuse of superior strength. (IN GUMAWA were teasing her (by telling
SHORT, treachery was not proven) her that they will court her). AISHA
Moreover, superiority does not always mean reacted by throwing pebbles at
numerical superiority. Abuse of superiority depends upon TOMPONG and GUMAWA, and left the
the relative strength of the aggressor vis-a-vis the victim. place saying that she would just look for
There is abuse of superior strength even if there is only the carabao she was tending.
one malefactor and one victim. Abuse of superiority is
determined by the excess of the aggressors natural

137
CRIMINAL LAW REVIEW DIGESTS
JUSTICE ROMEO CALLEJO NOTE: = Callejo Ponente

o After finishing her task, Gloria left followed and covered it w/ twigs of
TOMPONG, GUMAWA, and ALARCON bungargar. GUMAWA told the other 2
behind. TOMPONG approached that they will separate and nobody will
ALARCON and said that they will follow tell what happened. Untalan left his
AISHA. TOMPONG, GUMAWA, and hiding place and went home 5 minutes
ALARCON went to the place where after the 3 fled.
AISHA was grazing her carabao. Three days thereafter, the cadaver of AISHA, in a
o AISHA saw the 3 of them approaching state of decomposition, was found by the police
and stood up and looked at them. Upon in Sitio Sio. The findings in the Autopsy Report
reaching AISHA, TOMPONG pushed were:
ALARCON towards AISHA causing 1. General Appearance: Dead; lying with his
them both to fall on the ground and roll head and trunk slightly elevated from the
together. As they rolled, AISHA rest of the body; supine position; both arms
scratched the face of ALARCON to slightly flexed and perpendicular to the
defend herself but while AISHA was body; both thighs and knees flexed; knees
lying flat on her back, TOMPONG ran separated from each other 40 cms. apart;
towards her and got the knife tucked on wearing T-shirt stained with blood and rolled
her waist. AISHA shouted and cried as upward to the level just below the nipples;
the 3 ganged upon her. wearing panty and short pants stained with
o A t that instant, prosecution eyewitness blood; short pants rolled upwards to the
Melita Cancer, who was on her way inguinal area.
home after coming from AISHAs house 2. Wound, 6 cms. long, gaping, neck, right,
(w/c was situated around 80m from the involving right sternocleidomastoid muscle,
crime scene), head the cry of the victim. right common carotid artery and vein.
Cancer was supposed to collect money 3. Wound, stabbed, 2.5 cms. long, gaping, left
from AISHAs mother but left after buccinator area, running mediolaterally,
discovering that no one was home. After involving buccinator muscle thru and thru.
hearing AISHAs cry, she looked around 4. Labia majora and labia minora; Medical
and saw TOMPONG, GUMAWA, and borders not prominent because it is
ALARCON, holding and undressing markedly covered by a swollen clittoris;
AISHA. Cancer immediately fled the gaping.
area started and afraid of what she saw. 5. Clittoris: Swollen and elevated by 2.5 cms.;
o Another person, Ostimiano Untalan (68 6.5 cms. long; 3 cms. wide; oblong in
years old and a retired PC officer) saw shape; presence of somewhat fresh blood
the incident as he was on his way to the in the superior third; wound in the inferior
bamboo plantation of Salvador Dava to half.
inquire into the availability of bamboo 5. Vaginal canal: Presence of old mucus like
poled he intended to buy. Untalan heard material in the opening, left, lower quadrant;
the shouts of AISHA but felt afraid and edges irregular; rugosities not discint;
took cover and concealed himself inside admits one finger.
a thick bush around 4 ft. tall. He saw 3 6. Fourchette: Rounded base.
malefactors sexually molesting AISHA, 7. Hymen: Not clearly identified because of the
who was lying flat on her back with both extremely swollen clittoris.
hands being held by ALARCON. Version of defense:
TOMPONG laid on top of the victim o ALARCON (19 years old),
while GUMAWA covered her mouth. (testifying on his own behalf) said
Untalan saw TOMPONG, GUMAWA, that after AISHA left to look for her
and ALARCON take turns sexually carabao, TOMPONG and
abusing AISHA for about half an hour. GUMAWA approached ALARCON
o Thereafter, ALARCON struck the neck and told him to go with them to
of AISHA w/ a piece of wood and follow AISHA. He refused but
slashed further the throat and left check TOMPONG pushed him. He fell on
of the victim using a knife. GUMAWA the root of a santol tree and he
dragged the body of AISHA towards the was leaning on it when GUMAWA
canal. TOMPONG and ALARCON approached him. GUMAWA

138
CRIMINAL LAW REVIEW DIGESTS
JUSTICE ROMEO CALLEJO NOTE: = Callejo Ponente

pointed a bolo at him and come back the following Monday


threatened him that if he did not go as the house was not yet finished
with them, GUMAWA would hack but then she heard over the radio
him. Scared, ALARCON went with that the two were arrested by
TOMPONG and GUMAWA to police. As laborers, TOMPONG
follow AISHA. When they found and GUMAWA worked the whole
AISHA, TOMPONG and GUMAWA day, from 7:30 in the morning to
allegedly instructed him to hold the about 4:30 or 5:00 in the
hands of AISHA while they took afternoon, and slept in her old
turns raping her under the threat of house adjacent to the one being
being hacked with their bolos. After constructed.
TOMPONG and GUMAWA o TOMPONG and GUMAWA raise
finished raping AISHA, TOMPONG the defense of alibi. Accused
faced ALARCON and told him to TOMPONG testified that he was
take his turn, but ALARCON cried 42 years old, married, a carpenter
and said that he would not do it. and resident of Sitio Sio,
Because he did not, TOMPONG Bagtason, Bugasong, Antique. On
told GUMAWA: "We cannot do 26 May 1995, when the crimes in
otherwise." GUMAWA then took a question were allegedly
piece of wood beside him and committed, he and GUMAWA were
struck the neck of AISHA, put on working on Edna's house and they
her shorts and pulled her towards never left the site.
a hole near some shrubs. The prosecution presented Pedro Engue as
GUMAWA warned ALARCON not rebuttal witness. He testified that he had
to tell anybody about what resided in Sitio Sio for the last five years
happened. TOMPONG then and TOMPONG is his neighbor.
slashed AISHA's neck with the TOMPONG's usual work was selling
knife he took from her waist and firewood which he gathered from the land
dropped the knife beside AISHA's owned by the Davas. Engue did not know
body. TOMPONG approached whether TOMPONG had ever constructed a
ALARCON and told him not to tell house in Sitio Sio for other people. He
anybody about the incident and, if added though that GUMAWA is also a
he was caught, to just admit it. firewood gatherer in Sitio Sio.
GUMAWA and TOMPONG each Trial court convicted TOMPONG,
promised to pay ALARCON P500 GUMAWA, and ALARCON. TOPONG and
if the latter admitted to the crime. GUMAWA were sentenced to death while
o Richard Bernabe, first witness for ALARCON faces reclusion perpetua. The
TOMPONG and GUMAWA, judgment against TOMPONG and
declared that at around noon of 26 GUMAWA is before the Supreme Court on
May 1995, his neighbor ALARCON automatic review.
went to his house to ask for help
as he had raped a child. ISSUES:
ALARCON confessed to him that (1) Whether the prosecution witnesses are
he did it alone. Edna Apolinario, credible? Yes
the second witness for TOMPONG (2) Whether defense satisfactorily established
and GUMAWA, testified that on 22 the defense of alibi? No
May 1995, TOMPONG, GUMAWA
and her husband began the (1) FINDINGS OF LOWER COURT ARE
construction of her house in BINDING
Apgahan, Patnongon, Antique. After a thorough review of the evidence on
TOMPONG and GUMAWA worked record, we affirm the judgment of conviction of
continuously from 22 May up to 27 accused-appellants TOMPONG and GUMAWA. At
May 1995. At six o'clock in the the core of this petition is the credibility of
morning of 28 May 1995, the two eyewitnesses. The trial court found worthy of belief
went home. She expected them to the accounts of Melita Cancer, Ostimiano Untalan

139
CRIMINAL LAW REVIEW DIGESTS
JUSTICE ROMEO CALLEJO NOTE: = Callejo Ponente

and co-accused ALARCON. We can do no less. courts have always looked upon the defense of alibi
Appellate courts accord the highest respect to the with suspicion and have received the same with
trial court's assessment of the testimonies of caution, not only because it is inherently weak and
eyewitnesses by the trial court because of its unreliable but also because of its easy fabrication. It
unequaled opportunity to observe on the stand their cannot prevail over, and is worthless in the face of,
demeanor and manner of testifying and to detect positive identification by credible witnesses that the
whether they are telling the truth or not. This rule accused perpetrated the crime.
admits of exceptions, such as when the evaluation
was reached arbitrarily or when the trial court RULING: We are convinced beyond any doubt that
overlooked, misunderstood, or misapplied some TOMPONG, GUMAWA and ALARCON each raped
facts or circumstances of weight and substance AISHA. Since the facts adduced prove beyond doubt
which could affect the result of the case. None of the that they conspired and mutually helped each other
exceptions obtains in these cases. in committing the rapes, each should be held
TOMPONG and GUMAWA capitalize on Melita criminally liable for these rapes. Since AISHA was
Cancer's running away, not attempting to secure help killed on the occasion thereof, each should be liable
for AISHA, and not telling anybody what she had for three complex crimes of rape with homicide.
witnessed, as attributes of the falsity of her
testimony. However, Cancer has clearly explained SUPERIOR STRENGTH
that she was afraid when she saw what was Article 335 of the Revised Penal Code, as
happening. As for not telling anybody, this was amended by R.A. No. 7659, provides, inter alia:
adequately explained in her testimony. There is no when by reason or on the occasion of the rape,
accounting for the varied reactions an eyewitness homicide is committed, the penalty shall be
might have relative to what he might be seeing. death.
There is no standard form of human behavioral Fortunately for TOMPONG and GUMAWA
response when one is confronted with a strange, it was only in Criminal Case No. 5630 that they
startling or frightful experience. Fear has been were charged with ALARCON with the crime of
known to render some people immobile, if not rape with homicide. They were charged only
useless, in some life-and-death situations. As to with rape in Criminal Case Nos. 5631 and
Ostimiano Untalans failure to report the matter to the 5632. The trial court imposed on them in each
police and for inconsistencies in his testimony, it is of such cases the penalty of death because the
not uncommon for a witness to a crime to show crime in each case was "committed by more
some reluctance about getting involved in a criminal than two (2) persons, aggravated by superior
case, and in fact the natural reticence of most people strength." We do not agree with the trial
to get involved is of judicial notice. It is court on this issue.
understandable for a witness to fear for his safety While it may be true that Article 335 of the
especially when townmates are involved in the Revised Penal Code, as amended by R.A. No.
commission of the crime. Moreso in Untalan's 7659 provides, inter alia, that: whenever the
circumstances: he is old and disabled. rape is committed with the use of a deadly
weapon or by two or more persons, the penalty
(2) ALIBI shall be reclusion perpetua to death and that
TOMPONG and GUMAW claim that they could the presence of an aggravating circumstance
not have raped AISHA since they were working that would justify the imposition of the graver
day in Apgahan constructing the house of Edna penalty of death, the fact of commission "by
Apolinario. To establish alibi, an accused must show two or more persons," which partake of the
that he was at some other place for such a period of nature of a qualifying circumstance, was not
time that it was impossible for him to have been at alleged in the information in Criminal Cases
the place where the crime was committed at the time Nos. 5631 and 5632. The mere fact that three
of its commission. The trial court, after noting the were accused therein did not amount to a
distance between Apgahan to Sitio Sio in Barangay specification of the qualifying circumstance
Bagtason where the crimes were committed, held in question and was insufficient for the
that it was not physically impossible for TOMPONG purpose of complying with the
and GUMAWA to be present in Bagtason at the constitutional requirement that the accused
commission of the offenses. The burden of proving be informed of the nature and cause of the
alibi lies with TOMPONG and GUMAWA and they accusation against them.
have failed to discharge this burden. Justifiably,

140
CRIMINAL LAW REVIEW DIGESTS
JUSTICE ROMEO CALLEJO NOTE: = Callejo Ponente

Also, abuse of superior strength as a


generic aggravating circumstance, which HELD: The trial court correctly held that the qualifying
may be appreciated against the accused circumstance of abuse of superior strength was present
even if not alleged, was not proven in this since accused-appellant was taller and stronger than the
case. Mere superiority in number is not victim. An attack made by an armed man upon a woman,
enough, there must be proof of deliberate who died as result thereof, is murder, because his sex and
intent to take advantage of superior weapon gave him superiority of strength.
strength. * The SC also held the attendant circumstance of
outraging or scoffing at his person or corpse was not
SENTENCE: There being no evidence of any present in this case. As the lower court correctly held, the
modifying circumstance, the penalty to be imposed evidence fails to show this. The word "outrage" means to
pursuant to Article 63 of the Revised Penal Code, subject to gross insult. "Scoff" means to show contempt by
is reclusion perpetua, the lesser of the penalties derisive acts or language. There is no proof showing that,
prescribed by Article 335 of the Revised Penal Code in stabbing the dead body of the victim and stripping off
as amended by R.A. No. 7659. her garments, the purpose of accused-appellant was to
insult the victim or to show contempt for the dead.

PEOPLE v. OLIVO
79. P v Rivera(?) 402 Phil 547
FACTS: A body of a young Igorot woman was found in a
canal near the Athletic Bowl at Burnham Park in Baguio
City. The dead woman was identified by her granduncle, PEOPLE VS GALAPIA
Teodoro Incan, as his grandniece, Jane Lorielinda
"Lorie" Tacyo. The woman was naked from the waist FACTS: Galapia is married to Agudelo. Marriage became
down, her bloody face crushed beyond recognition, her difficult because they lived with Agudelos mom, so
mouth open in a silent scream. Her abdomen, partially Galapia left. One day, Galapia felt horny and went to see
covered by a pink shirt, had several tiny stab wounds. Agudelo to have sex. He was denied entry to the house.
There were also wounds on her neck. Scattered around Galapia waited until everyones asleep then entered the
the woman were a hairband, black shoes, white panties, house through window. Galapias advances were met with
and a pair of dark blue pants. The police recovered from Agudelos refusal along with her threat to stab him of the
the canal what appeared to be her personal effects, kitchen knife she was carrying. A commotion ensued,
together with two rough rocks stained with blood and a which resulted to Galapia getting the knife from Agudelo,
screwdriver about 10 inches long, with a black handle killing her (stabbed her heart) along with her mom and a
made of plastic or rubber. According to the police, she was nephew and injuring another nephew. Galapia afterwards
seen in the morning of June 13, 1996 at the Igorot surrendered to a policeman (dapat kay brgy captain pero
Garden, Burnham Park. She and two companions had maysakit siya). Galapia was then charged with parricide
their picture taken in the garden by a street photographer. (Agudelo), 2 Murders (mom-in-law + nephew1) and
The colored picture shows the victim smiling behind Frustrated Murder.
accused-appellant Jessie Olivo and Maybelle Sacliwen.
Lorie wore earrings, a headband, dark blue pants, and the ISSUE: W/N Galapias act of killing Agudelo can be
pink shirt which the police found on her dead body the appreciated by the aggravating circumstance of abuse of
following day. superior strength (as stated in the information). NO.
The autopsy showed that the cause of death was
neurogenic shock due to massive crushing injuries of the RATIO:
head. (Agudelo) Abuse of superior strength cannot be
Based on the testimonies of several witnesses, appreciated in this case for the reason that the said
the prosecution was able to prove that Olivo owned the circumstance is inherent in the crime of parricide where
screwdriver used to stab Lorie and several other the husband kills the wife. It is generally accepted that the
circumstances which pointed to the guilt of Olivo. The RTC husband is physically stronger than the wife.
found him guilty of murder and sentenced him to suffer the
penalty of reclusion perpretua. TREACHERY

ISSUE: W/N Olivo was guilty of murder (with the attendant


PEOPLE v. ESCOTE
circumstance of taking advantage of superior strength)
YES

141
CRIMINAL LAW REVIEW DIGESTS
JUSTICE ROMEO CALLEJO NOTE: = Callejo Ponente

(robbery wit homicide is classified as a crime against


property. Nevertheless, treachery is a generic aggravating (NOTE on the anticlimactic decision: the SC said
circumstance in said crime if the victim of homicide is treachery cannot be considered against the two because it
killed treacherously) was not alleged in the Information. Nyark.)

Facts: At past midnight, Rodolfo Cacatian (regular driver Ratio:


of Five Star passenger bus, hereinafter referred as the General Discussion on the Crime of Robbery with
driver) drove the bus from Pasay City to Pangasinan. Six Homicide
additional passengers boarded the bus in Balintawak, To warrant the conviction of robbery with violence
including Acuyan and Escote, who held up the bus (they against or intimidation of persons under Art. 294, the
had handguns) as they were passing Bulacan. Both fired prosecution was able to prove the following elements: 1)
their guns upward and accosted the passengers, divesting taking of personal property with the use of violence or
them of their money and valuables. Apparently, SPO1 intimidation against a person; 2) property taken belonged
Manio was aboard the bus. When the felons went to him to another; 3) the taking is characterized by intent to gain
and asked for his wallet and ID. When they found out his or animus lucrandi, and 4) on the occasion of the robbery
was a police officer and saw his service gun, they said: or by reason thereof, the crime of homicide was
Pasensya ka na Pare, papatayin ka namin, baril mo committed.
rinangpapataysayo. The police officer pleaded for mercy: The intent to rob must precede the taking of
Pare maawa ka sa akin. May pamilyaako. But the two human life. In robbery with homicide, so long as the
ignored his plea and shot him on the mouth, right ear, intention of the felons was to rob, the killing may occur
chest and rights side of the body. Manio sustained six before, during or after the robbery. Even if the victim of
entrance wounds. The bus driver was ordered to maintain robbery is other than the victim of the homicide, there is
the speed of the bus. He heard one of them say only one single and indivisible felony of robbery with
Ganyanlangangpumatayngtao. homicide. All the crimes committed on the occasion or by
Parangpumapatayngmanok. The other said, reason of the robbery are merged and integrated into a
Ayosnanamantayo pare. Malaki-lakiito. They alighted single and indivisible felony. All those who took part as
from the bus and instructed the driver not to report the principals in the robbery will also be held guilty as
incident (all in all, robbery was over in 25 mintues). principals of robbery with homicide although they did not
Naturally, the driver and conductor reported the take part in the homicide, unless it appears they
incident to police. Barely a month after at about midnight, endeavored to prevent it.
a team of policemen were at a checkpoint along the
national highway in Tarlac. A white taxi cab without a The Penalty of the RTC: Death (impliedly taking into
plate was stopped and asked the driver, who was Escote, account treachery)
for his ID. Escote said he was a policeman and handed Under Art. 63, par.1, the felons shall be meted
over the ID of SPO1 Manio and the money they had taken out the supreme penalty of death when the crime is
from the heist. The police became suspicious because committed with an aggravating circumstance absent any
the ID had already expired. He asked Escote if the latter mitigating. The RTC did not specify any aggravating
had a new payslip. When Escote could not produce any, circumstance in its decision. However, it is evident from
he finally confessed he was not policeman and was the facts contained in the body of the decision that it
brought to the station. He was frisked and they found five imposed the death penalty on its finding that they shot
bullets of a 9mm in his pocket. During investigation, Manio treacherously. Its elements were present: 1) at the
Escote admitted that he and Acuyan staged the robbery time of the attack, the victim was not in a position to
on the bus and killed Manio. The RTC found both of them defend himself, and 2) the accused consciously and
guilty for the crime of robbery with homicide and was deliberately adopted the particular means, methods or
sentenced to death. forms of attack employed by him. The essence of
They are now before the SC, appealing among treachery is the sudden and unexpected attack by an
others, the propriety of the both the conviction and the aggressor on the unsuspecting victim, depriving the latter
penalty. The highest penalty is meted out if there is an of any chance to defend himself and thereby ensuring its
aggravating circumstance. commission without risk to the aggressor. Treachery may
also be appreciated even if the victim was warned of the
Issues:Is the aggravating circumstance of treachery danger to his life where he was defenseless and unable to
present? - YES (But how can that be? Treachery is flee at the time of the infliction of the coup de grace. In
considered in crimes against persons and in this case, this case, the victim was shot when he was defenseless,
robbery with homicide is a crime against property. see pleading for his life, and at short range. This killing is a
discussion below)

142
CRIMINAL LAW REVIEW DIGESTS
JUSTICE ROMEO CALLEJO NOTE: = Callejo Ponente

grim example of the utter inhumanity of man to his robbery with homicide and prescribing the penalty. It is
fellowmen. neither inherent in the said crime. Hence, it should be
considered as a generic aggravating circumstance for the
Treachery as an Aggravating Circumstance: Legal Basis imposition of the proper penalty. In applying this, the law
The SC has ruled over the years that treachery is a looks at the constituent crime of homicide which is a
generic aggravating circumstance in the felony of robbery crime against persons and not at the constituent
with homicide, a special complex crime and at the same crime of robbery which is a crime against property.
time a single and indivisible offense. However, in two The crime of robbery with homicide does not lose its
cases, the SC has held that robbery with homicide is a classification as a crime against property or as a special
crime against property. Treachery is appreciated only in complex and single and indivisible crime simply because
crimes against persons and hence, should not be treachery is applied. Treachery only increased the penalty
appreciated as a generic aggravating circumstance. It in accordance with Art. 63.
held in another case that it is not appreciated in robbery
with rape precisely because it is a crime against property. SCs Ruling on the Penalty
These ruling finds support in case law that in robbery with Despite the foregoing, treachery cannot be appreciated in
homicide and rape, the latter are merely incidents of the this case because it was not alleged in the Information, as
robbery with robbery being the main purpose and object of mandated by Sec. 8, Rule 110 of the Revised Rules on
the criminal. But the SC ruled otherwise in the later case Criminal Procedure. Hence, reclusion perpetua only.
of People v. Cando when it ruled that treachery is a
generic aggravating circumstance in robbery with
homicide when the victim of homicide is killed with PEOPLE V. WILLIAM ANCHETA3 (ET.AL)
treachery. The SC opted not to apply its earlier rulings G.R. No. 143935 June 4, 2004 (431 SCRA 42)
that same year (i.e. in People v. Bariquit). (Doctrine: Treachery)

Criminal law commentators are not in agreement FACTS: There are six accused charged in this case but
as well. Aquino and Reyes said it only applies to crimes only accused Felipe Boy Ulep is appealing. The accused
against persons. But Regalado says that it can be were charges with the crime of robbery with homicide.
appreciated insofar as the killing is concerned. The accused took, robbed and carried away 30
cavans of clean palay belonging to Alfredo Roca, and in
Turning to Spanish Construction order to successfully carry out the robbery, the accused,
It must be recalled that the 1850 Penal Code of Spain, pursuant to the same conspiracy, with treachery, and with
amended by Penal Reform Code of 1870, was applied in intent to kill, fired their guns at Marjune Roca, which
the Philippines. The Penal Code of 1887 in the caused his death, shot at Benita Avendao Roca and Febe
Philippines was amended by Act 3815 (RPC), which was Roca and hurled a grenade against them and both of them
enacted and published in Spanish. In construing the Old died as consequence of the wounds they sustained; and
and Revised Penal Code, the SC had accorded respect also fired upon Alfredo Roca with their firearms, thus
and persuasive, if not conclusive, effect of the decision of performing all the acts of execution which would produce
the SC of Spain in construing the 1850 Penal Code. the crime of murder as a consequence but which,
Art. 14, par. 16 on treachery is a reproduction of nevertheless, did not produce it by reason of the timely
the 1850 Penal Code of Spain with a slight difference. In running for cover by the said Alfredo Roca. In the
the latter law, the words the persons are used whereas commission of the crime, the lower courts appreciated the
in the RPC, the words the person are used. Going by generic aggravating circumstance of treachery.
the letter of the law, treachery is applicable only to crimes
against persons as enumerated in Title Eight (Chapts. 1 ISSUES:
and 2), Book II of the RPC. However, the SC of Spain has 1. Whether or not the aggravating circumstance of
consistently applied treachery to robbery with homicide, treachery should be appreciated.
classified as a crime against property. The ratio behind it 2. Whether treachery may be appreciated in
is when robbery is coupled with crimes against persons, robbery with homicide which is classified as a
the crime is not only an assault of the property but also of crime against property.
the victims themselves. Treachery is not a qualifying
circumstance because the SC of Spain said that the word
homicide is used in its broadest and most generic sense. 3 Note: there are 3 cases entitled People v Ancheta. Two of them
Treachery is not an element of robbery with involve Treachery. But I think this is the right one, because the
homicide. Neither is it a crime specially punishable by law following website matched the SCRA number that Sir gave:
http://www.lawphil.net/judjuris/juri2006/sep2006/gr_167693_2006.
nor is it included by the law in defining the crime of html (People v Cabalquinto, [2006])

143
CRIMINAL LAW REVIEW DIGESTS
JUSTICE ROMEO CALLEJO NOTE: = Callejo Ponente

HELD: - Ka Ramil was convicted of the complex crime of


1. Yes. There was treachery as the events narrated kidnapping with murder. CA affirmed. Hence, this
by the eyewitnesses pointed to the fact that the petition.
victims could not have possibly been aware that
they would be attacked by appellant and his ISSUE: W/N accused is guilty of murder.
companions. There was no opportunity for the
victims to defend themselves as the assailants, HELD/RATIO: YES.
suddenly and without provocation, almost As regards the crime of murder, it is true that there is no
simultaneously fired their guns at them. The direct evidence of the actual killing of the victim.
essence of treachery is the sudden and Nevertheless, direct evidence of the commission of the
unexpected attack without the slightest crime is not the only matrix whereby the trial court may
provocation on the part of the person attacked. draw its conclusions and findings of guilt. It is settled that
2. Yes. Treachery is a generic aggravating conviction may be based on circumstantial evidence
circumstance to robbery with homicide although provided that the following requisites must concur: (a)
said crime is classified as a crime against there is more than one circumstance; (b) the facts from
property and a single and indivisible crime. In which the inferences are derived are proven; and (c) the
fine, in the application of treachery as a generic combination of all the circumstances is such as to produce
aggravating circumstance to robbery with a conviction beyond reasonable doubt.
homicide, the law looks at the constituent crime The evidence is replete with details to prove that
of homicide which is a crime against persons and appellant and his at-large co-accused were responsible for
not at the constituent crime of robbery which is a the abduction and death of the victim. These are:
crime against property. Treachery is applied to a) On March 26, 1992, appellant together with six
the constituent crime of homicide and not to the (6) other armed men, introducing themselves to be
constituent crime of robbery of the special members of the New Peoples Army (NPA), blocked the
complex crime of robbery with homicide. The convoy of the victim and demanded payment of a
crime of robbery with homicide does not lose its campaign fee of P50,000.00;
classification as a crime against property or as a b) When the amount was not produced right
special complex and single and indivisible crime away, they hogtied the victim with a nylon rope and
simply because treachery is appreciated as a brought him to the mountains;
generic aggravating circumstance. Treachery c) Despite payment of the ransom money, the
merely increases the penalty for the crime victim was not released and was never seen alive again;
conformably with Article 63 of the Revised Penal d) After his arrest, appellant disclosed to the
Code absent any generic mitigating authorities the place where they buried the victim at Brgy.
circumstance. In sum then, treachery is a Balao, Abra de Ilog, Occidental Mindoro, and thereat they
generic aggravating circumstance in robbery recovered the skeleton of Libertador from a shallow grave;
with homicide when the victim of homicide is and
killed by treachery. e) The victims relatives were certain that the
remains belonged to Libertador.
While the combination of said circumstances is
PEOPLE OF THE PHILIPPINES, appellee, vs. RICARDO insufficient to establish the qualifying circumstance of
SOLANGON KA RAMIL, appellant. treachery, considering the absence of eyewitness to the
actual killing of the victim; however, it is enough to sustain
FACTS: the guilt of appellant for the crime of murder qualified by
- Ka Ramil with 6 other armed men blocked the party of abuse of superior strength, which was alleged in the
Ador Vidal, a mayoralty candidate in the 1992 information and proved during trial. This qualifying
elections. circumstance is present where there is proof of gross
- Ka Ramil said he was a member of the NPA and physical disparity between the protagonists or when the
asked for campaign fees. Ador failed to give the fees force used by the assailant is out of proportion to the
so he was abducted. means available to the victim.
- Adors wife eventually handed ransom money but Ka In the case at bar, there was superiority not only
Ramil failed to give Ador back. They did not know if in strength but in number as well. The lone victim was
Ador was still alive. unarmed and was hogtied by seven (7) armed men who
- Years later, Ka Ramil was arrested. A cadaver, demonstrably abused their excessive force which was out
including a maong jacket and shorts believed to be of proportion to the defenses available to the deceased.
that of Ador were found and retrieved.

144
CRIMINAL LAW REVIEW DIGESTS
JUSTICE ROMEO CALLEJO NOTE: = Callejo Ponente

Evident premeditation cannot be considered in the 3 policemen positioned themselves outside the
the instant case. The careful selection of an ideal site compound.
wherein to block the convoy of vehicles may have been Both the big and small gates were closed. But
premeditated so that the kidnapping of the victim would be even if someone is outside the gate, the whole of the
carried out successfully; but the same cannot be said as house rented by Zaldy Garcia could still be seen.
regards the killing. It is not enough that evident Oria and Major Opina were able to enter the
premeditation is suspected or surmised, but criminal intent compound by scaling the fence. They proceeded to
must be evidenced by notorious outward acts evincing Garcias house. The pathway leading to the house is plain
determination to commit the crime. In order to be planted with Bermuda grass and is open. Aside from the
considered an aggravation of the offense, the main door of the house, there is a screen, its a double
circumstance must not merely be "premeditation"; it must opening door. If somebody is outside about one meter
be "evident premeditation." from the door, persons inside the house could be seen.
So baka yung issue of treachery dito is that dapat The door was open but the screen made of chicken wire
may eye witness to prove there was treachery? Baka lang was closed.
naman. While Oria was walking side by side with Major
Opina approaching the door, Major Opina was on his left
side, and was ahead of him. Suddenly, they were shot at.
PEOPLE V. GARCIA He was not hit but Major Opina who was about one meter
from the door was hit on the abdomen. After the shot was
Facts: Major Ines Opina and SPO4 Paterno Oria went to fired, Oria dived and positioned himself in a safety (sic)
Barangay Pugo, Bauang, La Union to serve a Warrant of place. Major Opina fell down and Oria heard the sound of
Arrest against Zaldy Garcia issued by Judge Adolfo "ehhh" from him.
Alagar. Oria then traded shots with Garcia. He called for
Garcia lived in a house inside a fenced the back-up to enter. Since they did not come, and he had
compound. The compound had two houses. One is a big run out of ammo, he had to escape from the compound
house, bungalow type while the other which was rented by leaving Opina inside. Major Lunsad then arrived and was
Zaldy Garcia is small, bungalow type made of concrete able to negotiate for Garcias surrender. Garcia asked the
hollow blocks. There is a perimeter fence, about 7 to 8 feet back-up policemen to leave. After this happened, he
high with 2 steel gates. The wider gate is about 6 to 7 feet surrendered to Major Lunsad and they were able to
wide and the other is about 5 ft. wide. Both gates were retrieve Opina.
locked at that time. The RTC found Garcia guilty of murder qualified
They stood in front of the gate that was closed, by treachery and with the special aggravating
then a woman approached them. They told her that a circumstance of "the use of unlicensed firearm" and
Warrant of Arrest was issued for the arrest of her sentenced him to death.
husband. While Major Opina and Garcias wife were
talking, Garcia came out from their house half naked. The Issue: Whether or not the murder was qualified by
wifes reaction was then normal and she told Major Opina treachery.
that they should just stay outside for she will get the key
from the caretaker and open the gate. Held: Yes! There was treachery!
When Major Opina saw Zaldy Garcia, he pointed Ratio: There is treachery when the offender commits any
his finger to Zaldy and said Zaldy you better surrender, of the crimes against persons, employing means, method
you have a warrant of arrest.(translated already) Zaldy or forms which tend directly and especially to ensure its
just waived his hands indicating as if he refuses, who was execution, without risk to the offender, arising from the
then more or less 20 meters from the gate. After waiving defense that the offended party might make.
his hands, he went inside the house. From the gate to the To constitute treachery, two conditions must
house, there were no obstruction and the ground was concur: (1) the employment of means, methods or manner
clear. of execution that would ensure the offenders safety from
At that instance, Oria told Opina to call for a any defense or retaliatory act on the part of the offended
back up. Major Opina then ordered him to do so. As they party; and (2) the offenders deliberate or conscious
were waiting for the back-up, they discussed the strategy choice of the means, method or manner of execution.
they would employ in order to arrest Zaldy Garcia. After 15 Garcia seeks to negate these elements of
to 20 minutes, 3 policemen arrived. treachery by claiming to have acted out of fear and
Upon the arrival of the 3 policemen, Oria and nervousness; he was allegedly under these stresses
Major Opina scaled the fence near the smaller gate and because persons who were armed, dressed in civilian
clothes and who did not identify themselves as members

145
CRIMINAL LAW REVIEW DIGESTS
JUSTICE ROMEO CALLEJO NOTE: = Callejo Ponente

of the police, scaled his fence. He simply reacted to the


intrusion and had no plan to shoot one of those who so ISSUE: Whether or not the RTC erred in holding that
approached his house. Hence, he concludes that there treachery attended the commission of the crime of
was no treachery and the killing could not have been murder??? -- YES
attended by this qualifying circumstance.
First, it is not disputed that the appellant went out RULING: The RTC did not explain the basis for the
of his house to see for himself the two men who came. qualification of treachery except for a terse citation that
Second, by his own testimony, he returned to his house to there was a sudden attack and that Arnulfo had no
get his gun. Third, no immediate shooting took place. The opportunity to defend himself or to retaliate.
two policemen still called for backup assistance, waited It is not only the sudden attack that qualifies a
and conferred on what to do, and only after the backup killing into murder. There must be a conscious and
came did they scale the fence. Twenty minutes must have deliberate adoption of the mode of attack for a specific
elapsed from the time the appellant went inside the house purpose. All the evidence shows that the incident was an
up to the time of the actual shooting. Fourth, Major Opina impulse killing. It was a spur of the moment crime.
was almost at the door of the appellants house when the It is not enough that the means, methods, or form
shot that killed him rang out. Fifth, the shot came from of execution of the offense was without danger to the
inside the house through a closed chicken wire screen offender arising from the defense or retaliation that might
door that effectively hid a man from inside the house from be made by the offended party. It is further required, for
someone from the outside. Sixth, the first and fatal shot treachery to be appreciable, that such means, method or
was sudden, immediately hitting Major Opina. form was deliberated upon or consciously adopted by the
We conclude from all these established facts that offender. Such deliberate or conscious choice was held
indeed treachery had attended the killing of Major Opina. non-existent where the attack was the product of an
While the original initiative originated from the police who impulse of the moment.
sought to arrest the appellant, the latters response was an
attack which showed, by its method and manner, that it did
not come at the spur of the moment. The appellant was PEOPLE vs. PERICELITO VALLESPIN alias BOBOY
duly forewarned about the identities of Major Opina and
SPO4 Oria. Not only was he forewarned, he had ample FACTS: PERICELITO VALLESPIN alias Boboy was
time to reflect on what to do. His immediate response was accused of the crime of Murder for assaulting, attacking
to arm himself and to lie in wait in ambush, literally - and and hacking with treachery RICO QUIANOLA (Quainola)
to fire from a position of concealment and relative safety at several times with the use of an axe while the latter is
the two policemen who were fully exposed and in the open sleeping, hitting and wounding him on the head which
at the time. The shooting distance of a little more than a caused his instantaneous death.
meter effectively gave Major Opina no chance. This, in our Based on the testimony of the Mogote spouses,
view, is a classic example of treachery under the definition Salvador and Juditha, on the night prior to the incident, at
of the Revised Penal Code of the term. around 9 pm, the victim Quianola, the Vallespin and a
certain Dodong Samson, all employees of the iron
workshop owned by the Mogotes, were drinking liquor in
PEOPLE VS. ANTONIO the shop. At some point, the victim Quianola stood up to
urinate and, while urinating, fell down at the rip-rap portion
Alberto Antonio and Arnulfo Tuadles decided to play of the house. He thought that the group ganged up on him
pusoy-dos. When it came to tally their scores and collect and mauled him but his boss, Salvador Mogote, explained
the winnings from the loser, an argument arose. that he was not mauled; he just fell. Vallespin suddenly
The prosecution alleged that in the course of an rushed towards the victim and strangled him, which
argument, without warning or cause, Alberto pulled his prompted Mogote and Samson to separate the two. The
gun behind his back and shot Arnulfo at close range, thus Mogotes intervened and tried to pacify their workers but
employing treacherous means to accomplish the nefarious Vallespin became unruly. For this reason, Samson
deed. On the other hand, the defense pointed out that punched him. Mogote then asked someone to fetch
Arnulfo suddenly grabbed Albertos gun from atop a Vallespins mother in order to pacify him. After that,
sidetable. Fearing for his life, Alberto reached for Arnulfos Valespin left and the drinking session apparently
hand and they grappled for the possession of the gun. As continued.
they wrestled, a single shot roared. Arnulfo fell face down At around 1am, witness Mrs. Juditha Mogote
to the floor. went to the kitchen to fix her husband a midnight snack.
The RTC found Alberto guilty of the crime of On her way to the kitchen, she noticed that Quianola was
murder qualified by treachery. asleep inside the shop. It took her husband less than 20

146
CRIMINAL LAW REVIEW DIGESTS
JUSTICE ROMEO CALLEJO NOTE: = Callejo Ponente

minutes to finish his snack. She had just gone back to relationship or conduct with the victim, and (3) nature of
sleep when she heard a sound which awakened her. She the killing or facts which show the manner of killing was so
then went to the bathroom to urinate and, while urinating, particular that defendant must have intentionally killed
heard a rasping sound. From the bathroom, which was according to a preconceived design. The following
approximately five meters from the shop, she peeped into examples were given to illustrate each category: (1) prior
the shop and saw the Vallespin hack Quianola three times possession of the murder weapon or surreptitious
with a small axe. approach of the victim; (2) prior threats to do violence to
For his part, Vallespin interposed the defense of the victim or prior conduct of the victim known to have
denial and alibi. angered the accused, and (3) evidence showing that the
TC: Guilty beyond reasonable doubt for the crime wounds were deliberately placed at vital areas of the body.
of Murder The evidence of the prosecution proved the
deliberateness of the attack made by the accused-
ISSUE: Whether the qualifying circumstance of treachery appellant Vallespin. The evidence showed the planning
is present. YES. activity of the accused-appellant the attack was carried out
surreptitiously, at two thirty oclock in the morning, after the
SC: The trial court correctly found that treachery attended accused-appellant had left the shop at least an hour
the killing of victim Quianola. There is treachery when the earlier. The evidence also showed motive as it was proven
offender commits any of the crimes against persons, that prior conduct of the victim angered accused-appellant
employing means, methods or forms in the execution the victim accused his co-workers of mauling him when he
thereof, tending directly and specially to insure its fell which angered the accused-appellant. Furthermore,
execution without risk to himself arising from the defense the testimony of the medico-legal officer and the medico-
which the offended party might make. The essence of legal report proved that three of the four hack wounds
treachery is the sudden and unexpected attack by the were inflicted on the head of the victim and were
aggressor on the unsuspecting victim, depriving the latter fatal. Based on these proven facts, it is obvious that the
of any real chance to defend himself, thereby ensuring its mode of attack was consciously adopted by the accused-
commission without risk to the aggressor and without the appellant Vallespin.
slightest provocation on the part of the victim. It can exist According to the accused-appellant, his act of
even if the attack is frontal, if it is sudden and unexpected, strangling the victim was a prior altercation that negates
giving the victim no opportunity to defend himself against treachery. We disagree. The strangling incident cannot
such attack. In essence, it means that the offended party rule out treachery for the reason that it did not serve to
was not given an opportunity to make a defense. To prove forewarn the victim about the impending danger of death.
treachery, the following must be shown: (1) the In a case where this Court ruled out treachery, the prior
employment of such means of execution as would give the altercation between victim and accused served as a
person attacked no opportunity for self-defense and warning of impending danger and alerted the victim to the
retaliation, and (2) the deliberate and conscious adoption possible aggression of the accused. In the case at bar, no
of the means of execution. altercation transpired.
We agree with the trial court that the crime was Even assuming that the accused-appellants act
committed with treachery. of strangling the victim put the latter on guard, treachery
Regarding the first element, the prosecution was may still be appreciated. This Court has ruled that
able to establish through the testimony of the spouses treachery may still be appreciated even when the victim is
Mogote that the victim was drunk and lying on his back warned of the danger to his person as long as the
when he was attacked by the accused Vallespin. The execution of the attack made it impossible for the victim to
testimonies show that the victim Quianola had no defend himself or to retaliate. The essence of treachery is
opportunity to defend himself. This Court has ruled in a the swift and unexpected attack by an aggressor on an
number of cases that treachery attends the killing of a unarmed and unsuspecting victim who does not give the
person who is drunk, unarmed, has no opportunity to slightest provocation, depriving the latter of any real
defend himself and the attack is sudden. chance to defend himself. As the evidence presented has
The second element of treachery is more difficult proven, victim Quianola was drunk and lying on his back
to determine as it involves a subjective aspect. However, when he was attacked and had no real chance to defend
Justice Reynato Punos dissenting opinion in People vs. himself against the fatal hack wounds inflicted by the
Antonio is instructive on this matter. In determining accused-appellant.
whether the mode of execution was deliberately adopted, As the attendant circumstance of treachery
three categories of evidence are considered: (1) planning qualified the killing to murder under Article 248 of the
activity or what the accused did prior to the killing; (2) Revised Penal Code, the imposable penalty is reclusion
motive or facts which show the accuseds prior perpetua to death. There being no aggravating or

147
CRIMINAL LAW REVIEW DIGESTS
JUSTICE ROMEO CALLEJO NOTE: = Callejo Ponente

mitigating circumstances, the lesser penalty of reclusion correctly relied upon by the trial court to elevate the killing
perpetua must be imposed. to murder. However, the trial court erred in appreciating
abuse of superior strength in addition to treachery. It is
well-settled that where treachery qualifies the crime to
PEOPLE VS. NESTOR GO-OD, ALEJANDRO GO-OD murder, it absorbs abuse of superior strength and the
(DECEASED), SANCHO GO-OD, RUFO GO-OD AND latter cannot be appreciated even as a generic
EMPE GO-OD aggravating circumstance. The aggravating circumstance
of abuse of superior strength, manifested by the presence
Facts: Prosecutions version: In the afternoon of the of five armed assailants against an unarmed victim,
incident, accused Alejandro Go-od, Nestor Go-od, Sancho cannot be appreciated independently because it is
Go-od, Rufo Go-od and Empe Go-od ganged up on considered absorbed in alevosia. Incidentally, evident
Aladino Ygot while the latter was looking for his goat. All premeditation can not be considered for lack of evidence
the accused after coming out from the banana plants that accused-appellant preconceived the crime.
attacked at once and simultaneously hacked Ygot with There is treachery when the offender commits
their bolos many times hitting him on different parts of the any of the crimes against the person, employing means,
body until he had fallen on the ground. Admittedly, the methods or forms in the execution thereof which tend
victim had suspected the accused of having stolen the directly and specially to insure its execution, without risk to
goat. The victim was able to get hold of a bolo which in the himself arising from the defense which the offended party
process wounded Nestor Go-od. Very near from the scene might make. The treacherous manner in which the
of the crime Anecia Monsalud and Gaudioso Suson had accused perpetrated the crime was shown by the sudden,
seen actually the killing. The victim died on the spot deliberate and unexpected attack upon the unsuspecting
suffering fifteen hack wounds. Ygot.
Accused-appellants version: He interposed Nestor Go-od was found guilty of MURDER and
denial and non-participation as his defense. He said in the was sentenced to suffer the penalty of reclusion perpetua.
afternoon of the incident, Aladino Ygot (victim) and Anecia
Monsalud (witness) were in his house looking for a goat.
He told them that he had not seen a goat. They left PEOPLE VS CHRISTOPHER AVILES
afterwards but returned immediately and asked to have his
cigarette lighted. While giving the piece of lighted wood, Facts: Novelito Contapay was driving his passenger
Ygot stabbed him hitting his right chest. Whereupon, jeepney at less than 10km because it was traffic. Danilo
Nestor Go-od ran away and fell on the ground. Then Ygot Arenas (deceased) was seated beside him. Arenas then
pursued him. When he was about to be stabbed again, his shouted apaya. Contapay turned his head towards
father Alejandro Go-od helped and parried Aladino's hand. Arenas who was then being stabbed by Aviles. Note that
Nestor Go-od was confined for two weeks. He however Aviles upper body was already inside the jeep and he had
admitted that the victim had suspected him of being the his foot on the running board. Contapay stopped the jeep
one who slaughtered the goat. and tried to help Arenas but Aviles stabbed him on the
Only Nestor and Alejandro were arraigned. The knee. Aviles ran away and Contapay was not able to
other accused, namely, Sancho, Empe and Rufo Go-od all chase him because of his wounded knee. Arenas died
remained at large. Alejandro died before judgment. The because of cardio-respiratory arrest because of
trial court relied chiefly on the positive identification of two hemorrhagic shock due to stab wound.
eyewitnesses Aviles denied stabbing Arenas and instead said
that it was his half brother Cresencia who did the stabbing
Issue: WON the guilt of the accused-appellant has been (interview of the people in the crime scene showed that
established beyond reasonable doubt. YES! Aviles and Cresencia were together on that day of the
WON stabbing).
On the other hand, Aviles version is that: he was
The acts of accused-appellant and his companions as drinking with Cresencia and some others. He left to
narrated by Anecia Monsalod and Gaudioso Suson accompany someone to the municipal hall. When he went
manifestly disclose their joint purpose and design, back, he told Cresencia that he was going him. Cresencia
concerted action and community of interest. The fact that asked him to stay and drink a little more. He then left. As
the victim, in the course of the assault against him, was he was walking towards the marked, he saw Cresencia
able to stab accused-appellant and disabled him does not running towards him with blood stains on his tshirt.
exculpate the latter from criminal responsibility. Cresencia told him that he stabbed someone.
The qualifying aggravating circumstance of TC: Aviles guilty of murder and slight physical
treachery which was alleged in the information was injuries.

148
CRIMINAL LAW REVIEW DIGESTS
JUSTICE ROMEO CALLEJO NOTE: = Callejo Ponente

CA: Affirmed. There was a commotion. Then witnesses said they just
saw Florencio fleeing for dear life, chased by the 3 men.
Issue: Whether Aviles is guilty of murder or just homicide? Florencio retreated in a vacant lot. Florencio fell to the
Was there treachery? ground and while in the process of standing up, Tamano
stabbed him at the back. Thereafter, Dela Cruz and boy
Held: Just homicide! No treachery! negro took turns in stabbing him. Florencio died.
Aviles was arguing against Contapays The trial court convicted them of murder qualified by
identification of him and the prosecutions failure to treachery and evident premeditation (reclusion perpetua)
establish a case against him. Nevertheless, the Court
believed Contapay when he positively identified Aviles as ISSUE: w/n there was treachery
the one who stabbed Arenas and him.
Trial court is correct that with regard to Contapay, SC: NO. They are liable for simple homicide only.
the crime was only slight physical injuries as the Treachery must be proved with the same quantum of
prosecution failed to prove intent to kill. Intent to kill is an evidence as the crime itself. Treachery cannot be
element of homicide. The facts show that Aviles stabbed presumed; nor can it be based on mere surmises or
Contapay only for the purpose of preventing him from speculations. In case of doubt, the same should be
helping Arenas. resolved in favor of the accused.
There is treachery when the following requisites For treachery to be appreciated, it must be
are present: 1) the employment of means, methods, or present at the inception of the attack, and if absent and
manner of execution to ensure the safety of the malefactor the attack is continued, even if present at the subsequent
from defensive or retaliatory action on the part of the stage, treachery is not considered as a qualifying or
victim; and 2) the deliberate adoption of such means or generic aggravating circumstance. The prosecution must
methods in the execution. adduce conclusive proof as to the manner in which the
The CA ruled that the fact that Arenas shouted altercation started and resulted in the death of the victim,
Apaya (perhaps a shortened form of apay aya, which is and if the prosecution fails to discharge its burden, the
more accurately translated in Filipino as bakit ba) showed crime committed is homicide and not murder.
that he was probably surprised to see Aviles trying to get In this case, the witness saw Dela Cruz, Tamano
inside the jeepney which was moving slowly because of and Boy Negro, armed with knives, chase and overtake
heavy traffic. The testimony of Contapay that after the victim in a vacant lot. The victim slipped and fell to the
hearing Arenas shout Apaya, he saw Aviles already ground. Dela Cruz, Tamano and Boy Negro forthwith took
stabbing Arenas, showed that the attack was sudden and turns in stabbing the victim as the latter tried to stand up.
unexpected. Arenas was confused as to why Aviles was The prosecution failed to adduce any evidence as to how
doing what he was doing. the aggression started and who started the same. The
For treachery to be appreciated, it must be barefaced fact that the victim was helpless when he was
present at the inception of the attack. If the attack is stabbed does not constitute proof of treachery.
continuous and treachery was present only at a In every fight it is to be presumed that each
subsequent stage and not at the inception of the attack, it contending party will take advantage of any purely
cannot be considered. accidental development that may give him an advantage
Qualifying circumstances must be proven beyond over his opponent in the course of the contest. It follows
reasonable doubt. It cannot be considered on the strength that alevosia cannot be predicated of this homicide from
of evidence which merely tends to show that the victim the mere fact that the accused overtook and slew the
was probably surprised to see the assailant try get inside deceased while the latter was endeavoring to rise from the
the jeepney. ground.
Moreover, the fact that Arenas was in between In light of the evidence, abuse of superior
Contapay and Aviles such that he had nowhere to run strength was attendant in the commission of the crime.
does not mean that there was treachery. There was no However, said circumstance was not alleged in the
evidence to prove that this situation was deliberately and Information.
consciously adopted by Aviles to protect himself from Evident premeditation was not attendant because
defensive or retaliatory action. the prosecution failed to prove the elements thereof,
namely: (1) the time when the offender determined to
commit the crime; (b) sufficient lapse of time between the
People vs Dela Cruz determination and execution to allow himself to reflect
upon the consequence of his act.
At around 12 midnight, dela cruz, Tamano, and boy negro
went to the house of deceased Florencio to confront him.

149
CRIMINAL LAW REVIEW DIGESTS
JUSTICE ROMEO CALLEJO NOTE: = Callejo Ponente

PEOPLE V. BASE Treachery even a frontal attack between determination and execution to allow reflection
can be treacherous! upon the consequences of the act.

3 men arrived in the residence of Brgy. Capt. Julianito Other crim related stuff:
Luna. 2 of these men introduced themselves as - Base argues he was not assisted by a counsel of
policemen, and said they were looking for a certain his choice when the admission from him was
Hernandez. Luna said he didnt know who that person made. This was, however, found untrue as culled
was, and 1 of the men shot him in the head. Then the men from the testimonies of the lawyers and police
ran towards their parked jeep and sped off. Luna was who interrogated him. Moreover, all the
rushed to the hospital but eventually died. constitution requires is that an accused be
Reports reached the police and they immediately assisted by a competent counsel. while the right
tracked down the assasins. The jeep they used was found to counsel is immutable, the option to secure the
in the house of a Mrs. Quizon without the men. The police services of counsel de parte is not absolute. the
thought the vehicle would eventually be recovered by the word "preferably" under Section 12 [1], Article 3
men, so they waited out. Base arrived to get the vehicle, of the 1987 Constitution does not convey the
then he was collared by a team of PC (?) soldiers and message that the choice of a lawyer by a person
brought to the camp. In a line up, several people positively under investigation is exclusive as to preclude
identified Base as one of the passengers of the jeep. other equally competent and independent
While in the camp, Base executed a written attorneys from handling his defense. If the rule
sworn statement with the assistance of a lawyer. In that were otherwise, then, the tempo of a custodial
statement, Base admitted that he knew very well that there investigation will be solely in the hands of the
was a plan to kill Luna. A week before the incident, he and accused who can impede the progress of the
his other co-accused were surveilling the residence of interrogation by simply selecting a lawyer who for
Luna. He also admitted that after the shooting, he was the one reason or another, is not available to protect
one ordered to recover the jeep. [ he alleges that he was his interest. This absurd scenario could not have
forced to make this admission, but he did not substantiate been contemplated by the framers of the charter
his claims] - There was conspiracy
Trial court: Base guilty of murder, w/ treachery
and evident premeditation. Reclusion perpetua. His co-
accuseds guilt was not proved beyond reasonable doubt 91. P v Manolo 548 s 567
and was acquitted. His other co-accused remained at
large.
PEOPLE V. DEL CASTILLO
Ruling: Murder!Evidenct premeditation and treachery
present.
Facts:
The essence of treachery or alevosia is the swift
About 6pm of November 25, 1962, Sedesias del
and unexpected attack on the unarmed victim without the
Castillo SED, Castromayor, Palencia and Patanao
slightest provocation on his part. The fact that treachery
came from a drinking session in one Badong;s house
may be shown if the victim is attacked from behind does
in Sara, Iloilo.
not mean it cannot also be appreciated if the attack is
It was slightly raining, so del Castillo, Castromayor
frontally launched. Even a frontal attack can be
and Palencia, (not Patanao though), sought shelter in
treacherous when it is sudden and the victim is
the house of Nene Emak. There they saw Pedro del
unarmed. In this case, the suddenness of the shooting
Castillo, Sr. who also took refuge from the rain.
without the slightest provocation from the victim who was
While waiting for the rain to stop a heated argument
unarmed and had no opportunity to defend himself, clearly
ensued between Castromayor and Pedro del Castillo,
qualified by the crime with treachery.
Sr. PERDO SR which resulted in a fist fight.
There was also evident premeditation. The one-
Castromayor had the upper hand but came out with a
week interval when accused-appellant and his co-
torn shirt.
conspirators first cased the victim's house up to the actual
Sedesias del Castillo offered Castromayor his T-
date of the killing underscores the presence of evident
shirt following when the group went home.
premeditation. For this aggravating circumstance to be
While walking, Sedesias del Castillo told them to stay
considered, there must be proof of the following elements
on the side of the road as a jeep (driven by Pedro del
thereof, 1.] the time the offenders determined to commit
Castillo, Jr., JUNIOR (another character), his dad
the crime; 2.] an act manifestly indicating that they clung to
PEDRO SR. was inside was well) was coming fast.
their determination; and 3.] a sufficient lapse of time

150
CRIMINAL LAW REVIEW DIGESTS
JUSTICE ROMEO CALLEJO NOTE: = Callejo Ponente

The jeep came swiftly, suddenly swerving and hit 7. WON there was conspiracy YES
SED, who was thrown about a meter away. 8. WON use of a motor vehicle can be applied? YES
PEDRO SR, with a blunt instrument, jumped out and
struck SED at the back of the head. Then followed by Held:
two more stab blows in the neck. d) The Court ruled against the appellants, mainly because
Realizing that the person he had assaulted was not of the eyewitness testimony of Virgilio Palencia. He
Castromayor, but SED, his first cousin, PEDRO SR testifies that he saw the jump drive by fast, driven by
ran after Castromayor but failed to overtake him. JUNIOR, the jeep hitting SED. Also, Palencia testifies
Meanwhile, JUNIOR (the son) upon realizing that he that he saw PEDRO SR stab SED from behind, and a
ran over his Uncle SED, carried the latter in his jeep few moments after, that PEDRO SR ran after
and brought him to Dr. Javellana, Rural Health Castromayor. Palencia even adds that after seeing this
Physician who examined the victim and found him to happen, he approached JUNIOR and said, Why do
have suffered injuries n the neck and jaw, abrasions you not pity Toto Decias (SED). To which JUNIOR
and contusions. The cause of death was determined replied, Is that Toto Decias?. When Palencia said yes,
as due to shock. JUNIOR asked for his help to bring him to the
Both PEDRO JR & SR. were charged with murder in poblacion doctor.
the Iloilo CFI, with penalties of reclusion perpetua
Their defense is denial and alibi.Though PEDRO SR So clearly there was intent and an actus reus causing the
admitted that he and Castromayor met in the house of death of SED due to the treachery of PEDRO SR and
Emak where they had heated discussion about JUNIOR. It all makes sense that when JUNIOR hit SED, it
inheritance. Castromayor, apparently, was irritated at is because Castromayor is wearing SEDs shirt, thus
PEDRO SRs intervention in the discussion and, as a causing the mistake in identity. It clearly showed the
consequence, Castromayor boxed appellant-accused treachery of their act of killing Castromayor
in the head. supposedly.Considering that appellants employed means
They grappled and PEDRO SR was able to hit back which tended directly to especially ensure its commission
Castromayor who fell on the floor. Castromayor without risk to themselves, the killing of the victim was
accused SED of trying to take sides with PEDRO SR, qualified by treachery which whenever present in the
but SED explained that he was only trying to stop the commission of the crime should be taken into account
fight. Castromayor warned SED that time will come whether or not the victim was or was not the same person
when he will pay for it. whom they have intended to kill.
Thereafter, PEDRO SR arrived home at about 7pm e) The trial court erred in considering nighttime as a
when he told his son, JUNIOR, to dress up as they generic aggravating circumstance since it is necessarily
would go to the dance at the Sara Elementary School included or absorbed in the qualifying circumstance of
with Mayor Ricardo Zerrudo. treachery.
With JUNIOR driving the jeep, they proceeded to the
mayors house who boarded the jeep and they all f) There was conspiracy between appellantsto commit the
went to Sara Elementary School crime as shown by the circumstance that immediately,
About 9pm, PEDRO SR noticed a commotion in the after Pedro Jr. had bumped the deceased with the jeep
street. He immediately notified Mayor Zerrudo about it he was driving, PEDRO SR jumped from the jeep and
and together they went to the place of the commotion with a blunt instrument stabbed his victim twice on the
and learned that something was going on inside the neck.
house of Jose del Castillo.
They went inside and saw the body of SED lying on 4. Under paragraph 20, Article 14 of the RPC, motor
top of the table. Mayor Zerrudo inquired from a peace vehicle would be an aggravating circumstance if the crime
officer which told him that Castromayor and SED had was committed by means thereof. There is no question
a quarrel which resulted in the death of the latter. that in this case this aggravating circumstance should be
Further, appellants argue that the theory of the appreciated since the defendants used a jeep and it
prosecution is highly incredible in that it was facilitated the commission of the crime.
improbable for appellant not to have recognized his Note: Though the proper penalty should be death, due to
own cousin SED and to have stabbed him TWICE on the lack of necessary votes, the Court decided to have
the neck after turning him over on his face. imposed the penalty of reclusion perpetua.

Issue:
5. WON there was treachery? YES PEOPLE V. ONG
6. WON nighttime can be applied? NO

151
CRIMINAL LAW REVIEW DIGESTS
JUSTICE ROMEO CALLEJO NOTE: = Callejo Ponente

Doctrine: The elements of treachery are (1) at the time of 2. Are the aggravating circumstances of abuse
the attack, the victim was not in a position to defend of superior strength and nighttime absorbed
himself, and (2) the offender consciously adopted the in treachery?
particular means, method, or form of attack employed by
him. Ruling: As to the first issue, yes, there was treachery.
Abuse of superior strength is absorbed in The facts show that Henry Chua's hands were
treachery. Nighttime is generally absorbed in the crime of tied and his mouth was gagged with a flannel cloth before
treachery although it was not applied in this case. he was stabbed twice with an icepick and buried in a
shallow grave near a creek. These facts portray well that
Facts: Benajmin Ong and Henry Chua used to play mah- the tied hands of the victim rendered him defenseless and
jong together. In those sessions Ong lost substantially that helpless thereby allowing the accused to commit the crime
at one time, it amounted to as much as P150,000.00. He without risk at all to their person.
suspected that he lost in unfair games and was completely There is treachery when the offender employs
cheated by Henry Chua and the his (Chuas) companions, means, methods, or forms in the execution of the crime,
who made things worse by pressing him to pay his which tend directly and specially to insure its execution
gambling debt with a threat of bodily harm upon his person without risk to himself. Chua was not given opportunity to
and that of his family. Ong was so embarrassed by Chuas make a defense in this case, and as such, there is an
pressing that he quit his job. aggravating circumstance of treachery.
One night, Chua invited Ong at the Amihan As to the abuse of superior strength, it is
Nightclub and told Ong to bring with him the money owed absorbed in the aggravating circumstance of treachery.
(P50,000.00). Ong, on the other hand, hatched a plan with The case said that it is sustained in a long line of
other men to kill Chua and avenge the embarrassment decisions.
and humiliation he suffered before the eyes of his Nighttime, however, was not absorbed in the
subordinates. aggravating circumstance of treachery in this case. The
When Chua and Ong met in the club, they had a court relied on the case of People v. Berdida, wherein
few drinks and Ong asked for patience and leniency with there was a special circumstance that showed that
regard to his indebtedness and ample time for its nighttime was not absorbed in treachery. That case had
settlement. They partied hard that night. similar facts to this case of Ong. In both the Ong and
Both men (Chua and Ong) left in Chuas car. Berdida cases, the accused took advantage of nighttime in
They were discreetly followed by Ongs men in another committing the felonies charged and had evidently chosen
car. When they reached a dark and secluded place, Ong to execute their victims under the cover of darkness, at the
urged Chua to stop the car in order to urinate, to which the dead of night, when the neighborhood was asleep.
latter obliged. Ongs men stopped their car, alighted, The Court, in Berdida, stated that Inasmuch as
poked a gun at Chua, and told him to step out of the car. the treachery consisted in the fact that the victims' hands
He was made to lie, face up. His hands were tied and his were tied at the time they were beaten, the circumstance
mouth gagged with a flannel cloth. They put him on the of nighttime is not absorbed in treachery, but can be
trunk of Ongs car. perceived distinctly therefrom, since the treachery rests
They went to Caloocan and stabbed Chua twice upon an independent factual basis. A special case
with an icepick. He was buried there with all his therefore is present to which the rule that nighttime is
belongings. absorbed in treachery does not apply.
The trial court found Ong and his men guilty of
murder. The court found qualifying circumstances of Aggravating circumstances included: nighttime,
treachery and evident premeditation. They also found uninhabited place, motor vehicle,
aggravating circumstances of superior strength, nighttime,
uninhabited place, abuse of confidence, cruelty, and use Aggravating circumstances excluded: superior strength
of a motor vehicle. (absorbed in treachery), abuse of confidence, cruelty
Ong questions the courts ruling of finding
treachery. Assuming there was treachery, he stated that Barredo, concurring and dissenting:
superior strength and nighttime should be absorbed in
treachery. He states that only evident premeditation and use of motor
vehicle should be the aggravating circumstances.
Issues:
1. Was there treachery? IGNOMINY

PEOPLE VS. FUERTES

152
CRIMINAL LAW REVIEW DIGESTS
JUSTICE ROMEO CALLEJO NOTE: = Callejo Ponente

Also, there can be no question that the crimes were


Facts: For gathering firewood and quenching their thirst committed in consideration of a price promise or reward
with coconuts gathered from a tree inside considering that Jack was hired by Fuertes for 5k to kill the
a hacienda managed by Osmundo Fuertes, Napoleon 2 victims and in fact received P200 contained in an airmail
Aldeguer, aged 14, and Mateo Aldeguer, aged 16, were envelope as down payment with the assurance that the
bound, gagged, brutally hacked to death and thrown at the balance would be paid after the job. However while this
bottom of a dried creek. Napoleon and Mateo were caught circumstance is qualifying in murder, it would merely be
gathering firewood and young coconuts inside the generic aggravating if it concurs with other qualifying
hacienda of Fuertes. During the first instance they were circumstances like treachery, as in this case.
able to escape. Fuertes called upon Salva and Gibone (I
think they are employees of Fuertes) to accompany Ignominy absent
Rolando Tano and Jack (I think these two are hired killers) Ignominy is a circumstance pertaining to the moral order
to the place where the two boys were seen. When the which adds disgrace and obloquy to the material injury
boys returned, they captured them and brought them to caused by the crime. The clause "Which add ignominy to
Fuertes. Thereafter, the boys were brought to the dead the natural effects of the act" contemplates a situation
creek, hand-tied. The shirts of the 2 boys were torn and where the means employed or the circumstances tend to
used to cover their mouths. Jack then stabbed Napoleon make the effects of the crime more humiliating or to put
with a bolo repeatedly even when the boy was already on the offended party to shame. In this case there is no
the ground. Jack gave the bolo to Rolando Tano who then showing that the offenses were perpetrated in a manner
stabbed Mateo repeatedly, after which Tano gave the bolo which tended to make its effects more humiliating to the
to Gibone and ordered him, under the threat of death, to victims.
stab Mateo. Gibone complied. Jack then ordered Gibone Neither can the act slicing the left leg of
to hand the bolo to Salva who at first refused but again, Napoleon Aldeguer's lifeless body nor the stabbing of
under the threat of being killed, succumbed to the orders Mateo Aldeguer's corpse in the stomach be considered
of Jack and eventually hacked the thigh of Napoleon. indications of ignominia because what is required is that
Thereafter, Jack took the bolo and hacked the neck of the crime be committed in a manner that tends to make its
Napoleon. Rolando Tano and Jack were handed small effects more humiliating to the victim, that is, add to his
envelopes by Fuertes after the crime was committed. They moral suffering. Thus, it was held that the fact that the
were found guilty of murder. However, Salva was used as accused sliced and took the flesh from the thighs, legs and
state witness. shoulders of the victim with a knife after killing the victim
did not add ignominy to the natural effects of the acts.
Issue: W/N there was ignominy? In the end, the SC found that several aggravating
circumstances were present, with no mitigating
Held: None! But first circumstances. Therefore, the penalty in its maximum
Abuse of superior strength present but absorbed by period which is death would be imposable. However
treachery because of the suspension thereof, the imposable penalty
SC found that there was treachery, so the aggravating is only reclusion perpetua. This penalty is single and
circumstance of abuse of superior strength had already indivisible, thus, it shall be imposed regardless of any
been absorbed. attending aggravating or mitigating circumstances.

Evident premeditation present


Evident premeditation can be presumed where, as in this PEOPLE VS VALLA
case, conspiracy is directly established. The essence of
evident premeditation is that the execution of the criminal FACTS: Accused Vincente Valle is 28 years old and
act is preceded by cool thought and reflection upon the married. He is the cousin of the victim, an 8 year old girl
resolution to carry out the criminal intent during the space named DyesebelDela Cruz. One night, a friend of
of time sufficient to arrive at a calm judgment. Its Dyesebel was walking along the road near the rice fields
requisites are: (1) the time the accused determined to when she heard and recognized the voice of Dyesebel
commit the crime; (2) an act manifestly indicating that the who was being strangled. The friend ran away out of
accused has clung to his determination: (3) a sufficient fright. Subsequently, the mother of Dyesebel approached
lapse of time between such determination and execution the barangay captain to report that Dyesebel was missing.
to allow him to reflect upon the circumstances of his act. The barangay captain immediately ordered a search party.
All of which are present in this case. The father of Dyesebel told the barangay captain that
Dyesebel was last seen with Vicente Valla. Valla was
Crime committed in consideration of a price summoned by the captain but the former did not report.

153
CRIMINAL LAW REVIEW DIGESTS
JUSTICE ROMEO CALLEJO NOTE: = Callejo Ponente

st
On the second day of searching, the search party He had intercourse with her five times. 1 ,
nd rd th
was trailed by Valla. They found the body of the victim missionary position. 2 , standing up. 3 , missionary. 4 ,
near the river with her dress on but her panties pulled doggy-style (he bent her body downwards with her hands
down until mid-thigh. The victims head was blue and her and knees resting on the ground When the latter was
vagina was bleeding. When confronted by the people, already in this position, appellant then placed himself
Valla admitted to raping and killing the victim. Valla even behind her, inserted his penis into her vagina and
offered his own daughter as payment. executed a push and pull movement in the dog's way of
th
The accused was charged with rape with sexual intercourse.) 5 , missionary.
murder.
The trial court convicted the accused of rape Issue: Is rape via doggy-style an aggravating
with homicide and sentenced him to reclusion perpetua. circumstance (ignominy)?

ISSUE: whether or not the trial court was correct in SC: Yes.
convicting the accused of rape with homicide instead of The Court held that there was ignominy because the
rape with murder - YES appellant used not only the missionary position, i.e. male
superior female inferior, but also "The same position as
HELD: The trial court did not err in convicting the dogs do" i.e., entry from behind. The appellant claims
accused of the special complex crime of rape with there was no ignominy because "The studies of many
homicide and not rape with murder as stated in the experts in the matter have shown that this 'position' is not
information. Homicide is, herein, taken in its generic novel and has repeatedly and often been resorted to by
sense. couples in the act of copulation. This may well be if the
Furthermore, The aggravating circumstance of sexual act is performed by consenting partners but not
ignominy under Article 14, No. 17 of the Revised Penal otherwise.
Code should be appreciated considering that the medico-
legal officer testified that the pubic area of the victim bore Other aggravating circumstances at issue:
blisters brought about by a contact with a lighted cigarette. Uninhabited place yes. The accused dragged the
This circumstance added disgrace and obloquy to the offended party, at the point of a dagger, to the carabao
material injury inflicted upon the victim of the crime. trail, about 10 meters from the junction, but 40 to 50
meters below to better attain his purpose without
OTHER ISSUES: interference, and to better secure himself from detection
1. Whether or not the extrajudicial confession of the and punishment. Even the junction where the two children
accused is admissible YES were left is already 400 meters from the nearest house.
a. An extrajudicial confession, to be valid, While there maybe occasional passersby, this does not
needs only the corroboration of the destroy its being an uninhabited place.
corpus delicti (other evidences which
tend to show the commission of the Superior strength No. Already absorbed in rape.
crime independent of the confession Nocturnity No. no evidence that it was sought to facilitate
itself). In this case, the testimonies of the crime.
the barangay captain, parents of the Rank No. No deliberate intent to offend the rank.
victim and others establish the corpus
delicti
2. Whether or not the statement of the accused PEOPLE OF THE PHILIPPINES vs. RENE SIAO
asking for forgiveness and offering his daughter
as payment is part of res gestae - YES Joy, Estrella and Reylan worked as house maids and
helper of Renes family. Rene ordered Reylan to dragged
Estrella to the womens quarters. Once inside, Rene
PEOPLE V SAYLAN pushed her to the bed and pointed a pistol at Reylan and
Memory aid: doggy-style rape Estrella.
Rene then asked Estrella to choose one among a
At 7 PM, accused accosted the victim Eutropia, a teacher, pistol, candle or a bottle of sprite. He also told Reylan to
(while she was with her kids) and forced her to have sex do something to Ester. Rene lighted the candle and
with him by poking her with an 8-inch dagger. (dirty dropped the melting candle on her chest. Estrella chose a
mind!) He brought her to a creek and told her to undress. bottle of sprite because she was afraid of the pistol. She
Her kids were left in a junction which was 400 meters from was made to lie down on her back on the bed with her
the nearest house. head hanging over one end. Rene then poured sprite into

154
CRIMINAL LAW REVIEW DIGESTS
JUSTICE ROMEO CALLEJO NOTE: = Callejo Ponente

her nostrils as she was made to spread her arms and the case for the prosecution. Second, accused-appellants
pointed the pistol. Estrella tried to fold her arms to cover argument that it is impossible to commit a rape in house
her breasts but Rene ordered Reylan to hold her hands. where there are many occupants is untenable. We have
Rene ordered Estrella to remove her pants and held in a number of cases that lust is no respecter of time
T-shirt and commanded her to take the initiative (ikaw ang and place. It is not impossible to perpetrate a rape even in
mauna sa lalaki.) She did not understand what Rene a small room. Rape can be committed in a house where
meant and Rene poked the pistol at her temple. there are many other occupants. Third, Ester and Reylan
Reylan was ordered to remove his shorts. Reylan could not be expected to flee or even to attempt to flee
refused but let his penis out. Rene then ordered Reylan to under the circumstances. Undoubtedly, considering that
rape Estrella. Reylan refused and Rene warned to kill Ester was only fourteen-years old and a newly employed
them both if they did not obey. housemaid, while Reylan Gimena a seventeen-year old
Estrella was made to suck the penis of Reylan at houseboy, they were easily intimidated and cowed into
gunpoint. Reylan then did the sexual act to Estrella for 10 submission by accused-appellant, who aside from being
minutes. Rene ordered to repeat the act. They were made their "amo" or employer, was menacingly threatening to kill
to lay side by side while Rene kept on pointing the pistol at them or their family with a gun if they did not do as he
them. After the side by side position, they were made to commanded them to do. Thus, it was not improbable for
assume the dog position. Reylan shouted for help. them not to attempt to escape when as accused-appellant
Teresita, sister of Rene, knocked at the door but Rene perceived they had an opportunity to do so. Moreover,
ignored. Thereafter, Rene ordered them to go to the boys while most victims will immediately flee from their
quarter and warned them: If you will tell the police, I will aggressors, others become virtually catatatonic because
kill your mothers." of the mental shock they experience. It was also not
Estrella and Joy sought permission to go home. improbable for them to report the incident to an old man
On their way home, they met an old man who saw Estrella they met on the road as there was no on else to turn to.
crying. The old man took them to his house. After the The rape was committed on May 27, 1994 or after the
incident was reported to the police, Reylan was arrested. effectivity of R.A. 7659 on December 31, 1993. The
Rene Siao and Reylan Gimena were charged with rape. governing law, Article 335 of the Revised Penal Code as
Both pleaded not guilty. amended by R.A. No 7659 imposes the penalty of
TC: Siao convicted of rape as principal by reclusion perpetua to death, if committed with the use of a
induction. Gimena acquitted because he acted under the deadly weapon.
impulse of uncontrollable fear of an equal, if not greater Accused-appellant was held guilty of rape with
injury. the use of a deadly weapon, which is punishable by
reclusion perpetua to death. But the trial court overlooked
ISSUE: WON the trial court is correct? YES. and did not take into account the aggravating
circumstance of ignominy and sentenced accused-
RULING: As to fact that Rene Siao forced and intimidated appellant to the single indivisible penalty of reclusion
at gunpoint Ester Raymundo and Reylan Gimena to have perpetua. It has been held that where the accused in
carnal knowledge of each other, we are convinced that the committing the rape used not only the missionary position,
same has been adequately proved by the prosecutions i.e. male superior, female inferior but also the dog position
evidence. Even as under settled jurisprudence, the as dogs do, i.e. entry from behind, as was proven like the
evidence for conviction must be clear and convincing to crime itself in the instant case, the aggravating
overcome the constitutional presumption of innocence, we circumstance of ignominy attended the commission
find the straightforward, consistent and candid manner in thereof.
which Ester Raymundo related her harrowing experience However, the use of a weapon serves to increase
in the hands of accused-appellant as bearing all the the penalty. Since the use of a deadly weapon increases
earmarks of verity. Not only that, the corroborative the penalty as opposed to a generic aggravating
testimony of Reylan Gimena was consistent in material circumstance which only affects the period of the penalty,
respects with that of Ester Raymundo. The testimony of said fact should be alleged in the information, because of
Ester and Reylan were assessed by the trial court to be the accuseds right to be informed of the nature and cause
credible. of the accusation against him. Considering that the
The points raised by Rene are trite and of no complaint (which was later converted into the Information)
consequence. First of all, the important consideration in failed to allege the use of a deadly weapon, specifically,
rape is not the emission of semen but the penetration of that herein accused-appellant was armed with a gun, the
the female genitalia by the male organ. Well-settled is the penalty to be reckoned with in determining the penalty for
rule that penetration, however slight, and not ejaculation, rape would be reclusion perpetua, the penalty prescribed
is what constitutes rape. Thus, this factor could not affect for simple rape under Article 335, as amended by R.A. No.

155
CRIMINAL LAW REVIEW DIGESTS
JUSTICE ROMEO CALLEJO NOTE: = Callejo Ponente

7659. Simple rape is punishable by the single indivisible Evident Premeditation: NO. There is no evidence of
penalty of reclusion perpetua, which must be applied planning or preparation to kill, much less of the time
regardless of any mitigating or aggravating circumstance when the plot was conceived.
which may have attended the commission of the deed. Ignominy: NO. For ignominy to be appreciated, it is
Hence, the penalty of reclusion perpetua imposed by the required that the offense be committed in a manner
trial court is correct. that tends to make its effect more humiliating, thus
adding to the victims moral suffering. Where the
victim was already dead when his body or a part
PEOPLE OF THE PHILIPPINES, vs. DOMINADOR thereof was dismembered, ignominy cannot be taken
CACHOLA y SALAZAR, et al. against the accused. In this case, the information
states that Victorinos sexual organ was severed after
FACTS: he was shot and there is no allegation that it was
1. 12 y.o. Jessie Barnachea was just about to leave his done to add ignominy to the natural effects of the act.
house to watch cartoons in the house next door when We cannot, therefore, consider ignominy as an
two armed men barged in his front door. They ordered aggravating circumstance.
him to drop to the floor, then hit him in the back with Dwelling: YES. As regards Carmelita and Felix, Jr.
the butt of a long gun. The intruders then shot his (mother and brother of Jessie), we appreciate the
uncle, so Jessie crawled and hid under a bed, aggravating circumstance of dwelling, since it was
whereupon the shooting continued. When the men alleged in the information and proved during the trial
left, Jessie went into the kitchen where he saw his that they were killed inside their house. Accused
mother, his brother and his cousin all slaughtered. Cachola and Amay, therefore, violated the sanctity of
2. There was a neighbor who saw men with bonnets on the said victims home.
outside the house, and neighbors also testified seeing
an owner-type jeep with El Shaddai on the front and CRUELTY
a Fruits & Vegetable Dealer at the side going toward
the house and leaving hurriedly after. A few hours PEOPLE V. SITCHON
after the incident, the jeep was intercepted at a
checkpoint. The eight accused riding the vehicle were Facts: Sitchon beat up a little 2 year old boy named Mark
brought to the police station. Anthony Fernandez to death. Victim mark spread his feces
3. Jessie positively identified two of the accused, all over the floor and this enraged Sitchon. Sitchon struck
Cachola and Amay, as the armed men who killed his him with a belt, 2x2 wood, and a hammer. He even
relatives. The OSG recommended the acquittal of the banged the little kids head on the wooden wall. This was
other 6 accused, who were just unfortunate enough to witnessed by victim-marks older brother Roberto. The
be riding in the same jeep. There was no evidence of next door neighbor Lilia also witnessed the beating
previous or simultaneous to hold them responsible as through the open door after she heard the cries of victim-
accomplices, hence the SC acquitted them. mark. Later, Sitchon brought victim-mark to the hospital
4. The death certificate of Victorino Lolarga (uncle) but he was already dead then.
revealed that his penis was excised. Would such Sitchonis the live-in partner of Marks mom. He is
circumstance amount to ignominy that can aggravate also a drug addict, he was then high from Valium 10.
the offense? Sitchonpleaded guiltyafter defense rested its
case and pleaded the defense of accident. The lower court
ISSUE: What aggravating circumstances should be convicted him of murder, qualified by treachery,
considered in this case? aggravated by cruelty and alternative circumstance of
intoxication. Sitchon sentenced to death [note: 1998 RTC
HELD: decision and 2002 SC decision].
Treachery: YES. There is no doubt that the killings
were done with treachery, considering that the Issue: What are the different aggravating and mitigating
assailants suddenly barged in and immediately went circumstances applicable in this case?
on a shooting rampage. The Court has ruled time and
again ruled that when the attack is sudden and Held: Murder, qualified by treachery, mitigated by lack of
unexpected, there is treachery.The presence of even intention to commit so grave a wrong.
this single qualifying circumstance is sufficient to [Note: Callejo assigned this case under Cruelty, thus this
qualify the killing to murder. is to be discussed first]

156
CRIMINAL LAW REVIEW DIGESTS
JUSTICE ROMEO CALLEJO NOTE: = Callejo Ponente

1. Cruelty is not present here. The test in


appreciating cruelty as an aggravating FACTS: One evening, witness Amanda Tabion was in her
circumstance is whether the accused deliberately house when she heard a motorcycle stop in front of her
and sadistically augmented the wrong by causing house and loud voices outside. One of the voices sounded
another wrong not necessary for its commission, as if someone was being tortured, so she went out to
or inhumanly increased thevictims suffering or investigate. Moonlight illuminated the area and she saw
outraged or scoffed at his person or corpse. The the 4 accused, Allan Valdez, Ludring Valdez, Itong Tabion
nature of cruelty lies in the fact that the culprit and Jose Taboac, surrounding Eusebio Ocreto, whom she
enjoys and delights in making his victim suffer knew since childhood. Ludring repeatedly hit Eusebio on
slowly and gradually, causing him moral and the head and body with large stones and boulders while
physical pain which is unnecessary for the the other accused looked on. Eusebio remained lying on
consummation of the criminal act which he the ground, unmoving. When Ludring stopped hitting
intended to commit.The sheer number of Eusebio, the 4 accused carried his body on their shoulders
wounds, however, is not a test for determining and boarded a tricycle and drove off. Amanda returned to
whether cruelty attended the commission of a her house, frightened by what she saw. The following
crime. morning, she learned that Eusebio was missing. That
The prosecution failed to show that Sitchon afternoon, the headless body of a man was found.
enjoyed the beating. The inordinate force Policemen identified the body as Eusebios. Postmortem
employed by appellant appears to have been examination of the body revealed that the victim sustained
caused not by any sadistic bend but rather by the 13 stab wounds, which the doctor opined, as being caused
drugs that diminished his capacity. by 2 or more assailants. Two days later, the decapitated
2. Treachery is evidently present here. Whenever head of Eusebio was found buried 1 foot deep, more than
you beat up a two-year old kid, its treacherous. 100m away from where his body was found. When it was
Impossible for the kid to defend himself. dug up, it was already in the state of decomposition.
3. Mitigating circumstance of plea of guilty cannot The 4 accused were charged with the crime of
be appreciated here. It is well-settled that a plea murder, with aggravating circumstances of superior
of guilty made after arraignment and after trial strength, nighttime and cruelty, by decapitating the victim.
had begun does not entitle the accused to have The RTC found Ludring Valdez and Jose Taboac guilty of
such plea considered as a mitigating the crime of murder. Only Ludring Valdez appealed.
circumstance. Here, Sitchon pleaded guilty only
after the prosecution rested his case. ISSUE: W/N the RTC erred in appreciating cruelty so
4. Lack of intention to commit so grave a wrong is as to qualify the crime from homicide to murder?
present since his only intention was to maltreat
and not to kill mark. When he realized what he HELD/RATIO: NO. The crime charged should be
did, he immediately brought mark to the hospital murder.
but it was too late. The SC agrees with the finding of the trial court that the
5. Voluntary surrender cannot be appreciated killing was attended with cruelty, because the deceased
because he failed to prove any of the was stoned, stabbed and beheaded. There is cruelty
circumstances when the culprit enjoys and delights in making his
6. The trial court appreciated intoxication as an victim suffer slowly and gradually, causing him
aggravating circumstance although it was clear unnecessary physical pain in the consummation of
that Sitchon isnt alcohol dependent but rather a the criminal act. The test is whether accused-appellant
drug addict. SC said that alternative deliberately and sadistically augmented the wrong by
circumstance of intoxication is clear when it causing another wrong not necessary for its commission
refers to alcohol and thus is not applicable to or inhumanly increased the victim's suffering or outraged
Sitchons case. Article 14 on aggravating and or scoffed at his person or corpse. In this case, evidence
article 15 on alternative do not contain an showed that the deceased was inflicted with numerous
Analogous provision unlike Article 13 on wounds before he was killed. Such acts increased the
mitigating circumstances. Criminalstatutes are to victim's suffering and caused unnecessary physical pain
be strictly construed in favor of the accused. And before his death.
no person should be brought within its terms who
is not clearly covered by it.
PEOPLE V ABDUL

PEOPLE vs. VALDEZ

157
CRIMINAL LAW REVIEW DIGESTS
JUSTICE ROMEO CALLEJO NOTE: = Callejo Ponente

Facts: That the five accused including appellant Abdul part and the other performing another part so as to
when they were invited by the victims to a salo-salo execute the crime of robbery with homicide. Annih Tanjing
without any inkling that the accused have sinister plans was deceived into loaning his gun for the purpose of
against their life and property since they were their friends. testing and examination. Once he was disarmed, he was
One of the accused got hold of the armalite carried by one immediately shot and killed. Almost simultaneously, they
of the victims asking it from him to test it, suddenly shoot the guns were grabbed from the victims and they were
at the victims composed of Abraham Annudin, Annih also shot at, killing Abraham and wounding Abdulbaser as
Tanjing, Abdulbaser Tanjiri, Idil Sahirul and Suri Jannuh a result. When the other members of Annih and Abrahams
with their firearms, thereby inflicting gunshot wounds upon group ran, the accused-appellant and his co-accused shot
their bodies which caused the death of Abraham Annudin at them. Thereafter, they smashed the faces of Annih and
and Annih Tanjing while the three other companions were Abraham to the point that their faces could no longer be
mortally wounded. Taking advantage that the two were recognized. Then, the accused-appellant and his co-
mortally wounded, the said accused, took, stole and accused left and brought with them the firearms, a watch
carried away two M-14 rifles valued at P60,000.00; One and a necklace which they took from the dead bodies. The
M-203 grenade launcher valued at P40,000.00; and One chronology of events coupled with the simultaneous
wrist watch and jewelries valued at P5,000.00, or the total execution of disarming the victims clearly shows that there
amount of P105,000.00, all belonging to the victims. was a unity of purpose and unity in the execution of the
The lower court rendered its decision finding the accused, unlawful acts to enable them to commit the crime of
Minya Abdul, guilty beyond reasonable doubt of the crime robbery with homicide.
of Robbery with double homicide and triple frustrated
homicide. 3) However the lower court erred in convicting the
And since the crime was committed with the accused of the crime of robbery with double homicide and
attendance of the aggravating circumstances of evident triple frustrated homicide. There is no crime of robbery
premeditation, treachery and by a band without any with multiple homicide under the Revised Penal Code. The
mitigating to offset any of them, hereby sentences said crime is still robbery with homicide notwithstanding the
accused to suffer the penalty of RECLUSION PERPETUA. number of homicides committed on the occasion of a
Hence, this appeal by accused-appellant Abdul. robbery since the homicides or murders and physical
injuries committed on or on occasion or by reason of the
Issue: Whether or not the accused-appellant participated robbery are merged in the composite crime of robbery with
in the commission of the crime which occurred on August homicide. However, when two or more persons are killed
19, 1988 at Langil Island. on the occasion of the robbery, the additional killings
should be appreciated as an aggravating circumstance to
Held: Yes. avoid the anomalous situation where, from the standpoint
1) Sahdiya Tanjings testimony is clear and straightforward. of the gravity of the offense, robbery with one killing would
At about five oclock in the afternoon of August 19, 1988, be on the same level as robbery with multiple killings.
she was at Langil Island, Tuburan, with the group of
Sahdiya Tanjing, Jubaira Tanjing, Annih Tanjing, Abraham 4) The court appreciated evident premeditation as an
Annudin, Asuri Jannuh, Abdulbaser Tanjiri and Idil Sahirul aggravating circumstance.
upon the invitation of Minya Abdul, Isa Abdul, Maldis We are not convinced that evident premeditation
Abdul, Inggat Doe and Jowen Appang when she was sufficiently proven. The prosecutions evidence did not
witnessed the shooting. clearly establish beyond reasonable doubt two of the three
requisites of evident premeditation, viz., a.) the time when
2) we reject accused-appellants claim that the prosecution Abdul and his co-accused determined to commit the
failed to prove the fact of death of the victims for the crime; and b.) a sufficient lapse of time between such
reason that no death certificate or testimony of an imam or determination and execution to allow him to reflect upon
Muslim priest was presented in court to prove the fact of the consequences of his act. Although there are badges of
death of Annih and Abraham. The absence of a death or premeditation in the present case, we can only speculate
burial certificate does not negate the fact of the killing as to the time elements required to appreciate evident
since corpus delicti can be proved by testimonial premeditation. Evident premeditation must be established
evidence. by clear and positive evidence and cannot be inferred nor
presumed no matter how logical and probable such
3) a conspiracy existed between the accused-appellant, inferences or presumptions might be.
Isa Abdul, Maldis Abdul, Jowen Appang, and Inggat Doe.
Evidence shows that the accused-appellant, together with 5) The lower court also appreciated treachery as an
the other accused all acted in concert, one performing one aggravating circumstance.

158
CRIMINAL LAW REVIEW DIGESTS
JUSTICE ROMEO CALLEJO NOTE: = Callejo Ponente

The accused-appellant and his cohorts, relying Ybasco when as there was information that he would
on the friendship they had with their victims, deceived deposit $250,000 for his employer. They decided tp
them into voluntarily giving their firearms to the accused- waylay Ybasco on the way to the bank.
appellant for the purpose of testing and examining said On the day of the heist, the robbers took a
firearms. Thereafter, accused-appellant together with Isa Toyota Corolla owned by Nash and went to the Makati
Abdul, suddenly, without warning, shot their victims who area to do surveillance on Ybasco. At around 6:30 p.m.,
were not aware of the danger against them and were not Ybasco emerged from the office of his employer holding a
in a position to defend themselves. The court therefore plastic bag. Momentarily, Manansala and the appellant
correctly found the presence of treachery as an confronted Ybasco and told him, May warrant of arrest
aggravating circumstance. ka. They grabbed Ybasco, handcuffed him and dragged
him to the car. Manansala and the appellant had a scuffle
6) The court also found the aggravating circumstance of with Ybasco when they grabbed the plastic bag from
band attendant in the present case. him. Roberto Acosta, a roving security guard, saw the
We do not agree. An offense is deemed incident and pulled out his gun. He sped towards the
committed by a band when more than three armed scene to investigate the incident. Del Rosario confronted
malefactors shall have acted together in the commission Acosta and grappled with him for the possession of the
thereof. This presupposes that from the onset four of the gun. As Del Rosario managed to wrest possession of the
malefactors were already armed in order to facilitate the gun from Acosta, shot him in the mouth. They boarded
commission of the crime. In the present case, only two of the car, and sped towards EDSA.
the five malefactors were armed at the start of the The robbers sped towards Laguna. When
commission of the offense. At any rate, even assuming Manansala looked inside the bag, he saw that it contained
that the aggravating circumstance of band was attendant only P5000 instead of $250,000 as he expected. He hit
in the commission of the crime, it is absorbed by Ybasco on the nape. Then the robbers Manansala,
treachery. appellant de Jesus and Del Rosario let Ybasco out in a
The crime of robbery with homicide is a special sugar farm in Laguna. They told hom that he would be
complex crime punishable under Article 294 of the allowed to go home but he shouldnt follow them because
Revised Penal Code with reclusion perpetua to death. de Jesus was a member of the NPA. But then suddenly,
Considering the presence of treachery and the additional de Jesus shot Ybasco on the head.
killing as aggravating circumstances, the maximum The robbers were later caught and one of them,
penalty of death would be imposable under Article 63 of del Rosario, confessed guilt. With respect to appellant, the
the revised Penal Code. However, since the crime was trial court found de Jesus guilty of Robbery with homicide,
committed on August 19, 1988 which is prior to the sentencing him to death.
enactment of Republic Act No. 7659 entitled An Act to
Impose the Death Penalty on Certain Heinous Crimes ISSUE: W/N the appellant de Jesus is guilty of robbery
which reimposed the death penalty, the imposable penalty with homicide, despite the qualifying circumstances
is reclusion perpetua. Reclusion perpetua is a single present that would normally qualify murder.
indivisible penalty which shall be imposed regardless of
the attending aggravating or mitigating circumstances. HELD/RATIO: Yes, robbery with homicide.
The evidence on record shows that when the
appellant and Manansala abducted Ybasco in Makati, they
102. P v Regala April 5, 2000 handcuffed the victim and transported him to a sugar field
in Cabuyao, Laguna. The appellant and Manansala
brought Ybasco out of the car. Still handcuffed, Ybasco
was shot by the appellant on the right cheek.
PEOPLE OF THE PHILIPPINES, appellee, vs.
In People v. Escote, Jr., the trial court ruled that
EDUARDO DE JESUS y ENRILE, appellant.
treachery is aggravating in robbery with homicide. The
aggravating circumstance of the use of a vehicle in
FACTS: SPO3 Ybasco was a policeman at the Makati committing robbery with homicide is also attendant in this
Police Station. He had a part time job as a money case. The appellant and his cohorts used a vehicle when
changer. Every afternoon he would deliver money for his they abducted Ybasco and transported him to Cabuyao,
employer, carrying it in a plastic bag. Laguna. However, the Information does not allege that
On February 1994, appellant De Jesus, together with a the appellant and his cohorts used a vehicle in committing
guy named Manansala and another named Del Rosario the crime charged as mandated by the Rules of Criminal
planned to stage a robbery. They had a financier, British Procedure. Also, the additional killing is not an
national Christopher Nash. They were planning to rob aggravating circumstance in robbery with homicide.

159
CRIMINAL LAW REVIEW DIGESTS
JUSTICE ROMEO CALLEJO NOTE: = Callejo Ponente

In robbery with homicide, the original criminal design occurred. The court later found out that Lorelyn was
of the malefactor is to commit robbery, with homicide deflowered by Belgar long before the current case.
perpetrated on the occasion or by reason of the
robbery. The intent to commit robbery must precede the Issue: was is the proper offense to be charged ?
taking of human life. The homicide may take place before, (considering that she was already 12 at that time) and
during or after the robbery. There is no such felony of MORE IMPORTANT: was there any aggravating
robbery with homicide through reckless imprudence or circumstance involved?
simple negligence.
It is immaterial that the death would supervene by Held: Article 335, paragraph 3, which states rape
mere accident; or that the victim of homicide is other than committed against a woman under 12 years of age. In this
the victim of robbery, or that two or more persons are case, the appellant was charged with rape through force
killed or that aside from the homicide, rape, intentional and intimidation of the complainant who is a 12-year-old
mutilation, or usurpation of authority, is committed by minor, and a retardate. And although the complainant was
reason or on the occasion of the crime. Likewise already 12 years old at the time, she was undeniably a
immaterial is the fact that the victim of homicide is one of retardate with the mentality of a 6-year old child, so that
the robbers; the felony would still be robbery with she also falls under the fourth category, for being under 12
homicide. Once a homicide is committed by or on the years of age.
occasion of the robbery, the felony committed is robbery MORE IMPORTANTLY, Under Section 17 of the
with homicide. All the felonies committed by reason of or Dangerous Drugs Act of 1972, as amended by Batas
on the occasion of the robbery are integrated into one and Pambansa Blg. 179, "when a crime is committed by an
indivisible felony of robbery with homicide. The word offender who is under the influence of dangerous drugs,
homicide is used in its generic sense. Homicide, thus, such state shall be considered as a qualifying aggravating
includes murder, parricide, and infanticide. circumstance in the definition of a crime and the
If a robber tries to prevent the commission of application of the penalty provided for in the Revised
homicide after the commission of the robbery, he is guilty Penal Code." This should serve as a sufficient deterrent if
only of robbery and not of robbery with homicide. All not a warning to those who are inclined to if not actually
those who conspire to commit robbery with homicide are habitually addicted to drugs. Their addiction will be no
guilty as principals of such crime. Homicide is said to have excuse but will aggravate any offense they commit.
been committed by reason or on the occasion of robbery
if, for instance, it was committed to (a) facilitate the
robbery or the escape of the culprit; (b) to preserve the PEOPLE V. SITCHON
possession by the culprit of the loot; (c) to prevent
discovery of the commission of the robbery; or, (d) to FACTS:
eliminate witnesses in the commission of the crime. As Emelito Sitchon (SITCHON) was charged for
long as there is a nexus between the robbery and the murder after beating to death the 2-year old son
homicide, the latter crime may be committed in a place (MARK ANTHONY Fernandez) of his common-
other than the situs of the robbery. law wife. He was convicted and sentenced to
death. The case is before the Supreme Cout on
automatic review. Initially, SITCHON pleaded not
PEOPLE V. BELGAR (1991) guilty. However, before testifying in his own
defense, he admitted killing the victim and
AID: Rape of 12 year-old retardate- Qualifying Aggravating changed his plea to guilty.
Circumstance of Drug Addiction Prosecution presented 5 witnesses: Lilia Garcia,
a neighbor; the victim's eight-year old brother
Facts: This is a case of rape of a 12-year old girl, a mental Roberto; the investigating officer, PO3 Paul
retardate with the mentality of a 6-year old. The assailant Dennis Javier; Dr. Manuel Lagonera, medico-
is a young man who is a habitual drug addict. Belgar legal officer of the National Bureau of
committed rape when he approached Lorelyn, the victim, Investigation (NBI); and Felicisima Francisco, a
while watching television. Meanwhile, Edu, a nephew of forensic chemist of the same agency.
nd
accused, four years of age, approached the mother of o SITCHON lived in the 2 floor of a 3-
Lorelyn and informed her "Lorelyn is pinapatungan by square meter house in Tondo. His
st
Kuya Boy." neighbor, Lilia, resided in the 1 floor.
Belgar claims that Lorelyn herself testified that o At about 10 AM of June 12, 1996, Lilia
she was not rape but only was kissed and fingered, and was in front of her house attending to
that he came from a pot session just before the incident her children when she heard the sound

160
CRIMINAL LAW REVIEW DIGESTS
JUSTICE ROMEO CALLEJO NOTE: = Callejo Ponente

of a boy crying. Lilia went up the and the anterior chest, which could have been
stairway and through the open door, inflicted with the use of blunt objects such as a
Lilia saw SITCHON beating MARK piece of wood or a fist. The child could have
ANTHONY. Lilia saw SITCHON hit been dead three to four hours, or not more than
various parts of MARK ANTHONGYs eight hours, prior to the postmortem
body w/ a piece of wood and banging examination. Dr. Lagonera concluded that the
the head of the boy against the wooden victim died of bilateral pneumonia secondary to
wall. The beating went on for about an multiple blunt traversal injuries or complication of
hour. After, Lilia saw SITCHON carry the lungs due to said injuries.
MARK ANTHONYs body down the Defense:
house to bring him to the hospital. o SITCHON, 40, a sidewalk vendor,
MARK ANTHONY was already black admitted killing MARK ANTHONY, the
and no longer moving. son of his live-in partner. He claimed he
o Eight-year old Roberto Fernandez is enjoyed a harmonious relationship with
the elder brother of the victim, also his partner and that he killed MARK
known as Macky. According to Roberto, ANTHONY (Macky) because he was
Macky had scattered his feces all over under the influence of shabu, marijuana,
the house. Because of this, SITCHON and Valium 10 at that time. SITCHON
beat Macky with a belt, a hammer, and professed that he began using drugs in
a 2x2 piece of wood. Roberto could 1974 and that he had also taken drugs
not do anything to help his brother two weeks before the incident.
because he was afraid SITCHON might o On June 12, 1996, SITCHON saw
also beat him up. When SITCHON Macky playing with his feces, scattering
brought Macky to the hospital, his little them all over the pillow, the bed sheets
brother, who could barely talk, was not and the curtains. SITCHON scolded the
crying anymore. boy, Putang-ina ka Macky! Bakit mo
o A certain Alice Valerio from ikinalat ng ganyan ang tae mo? Halika,
the Galang Medical Hospital informed dadalhin kita sa baba para
PO3 Paul Dennis Javier that a boy had hugasan! SITCHON got hold of Macky,
been admitted there. When PO3 Javier but the boy struggled to free himself
went to the hospital where he found the from SITCHONsgrasp. SITCHONt still
boy already dead. He observed that the reeling from the Valium 10 he had just
child had wounds on the left middle taken, became so angry that he picked
finger, the right index finger, and both up a broom with a wooden handle, and
feet, including lacerations in the upper hit the boy. SITCHON did not realize
lip and contusions all over his head and that he had hit Macky hard until he saw
body. the boy sprawled on the floor, breathing
PO3 Javier proceeded to SITCHONs house and with difficulty. He dressed Macky and
found human feces and fresh blood splattered on brought him to the
the floor. PO3 Javier recovered the broken Galang Medical Center at the corner
wooden sticks and the steel hammer, which were of Abad Santos Avenue and Tayabas
allegedly used to beat up the boy, as well as a Street, Manila. He prayed to God that
bloodstained white T-shirt. nothing serious would happen to the
SITCHON surrendered to Station 3 of their boy.
district that afternoon. o A lady doctor immediately attended to
The following day, a staff member of the Macky. SITCHON pleaded to the lady
television program Magandang Gabi doctor to do all she can to save the
Bayan turned over to PO3 Javier a brown belt child; otherwise, he would be in serious
which SITCHON allegedly also used in beating trouble. After examining the child, the
the victim. Roberto Fernandez had given the belt doctor told SITCHON that she could not
to the staff member. do anything more Macky was
Dr. Lagonera, medico-legal officer of the NBI, dead. The same day, SITCHON
conducted the postmortem examination of the surrendered to the police. He was
victims body. He found that the boy had suffered brought to the Homicide Section at 3:00
many injuries, including three wounds at the head p.m.

161
CRIMINAL LAW REVIEW DIGESTS
JUSTICE ROMEO CALLEJO NOTE: = Callejo Ponente

Explaining his change of plea, SITCHON that the prove: (a) the time the accused decided to commit the
killing of the boy was "accidental." He reiterated crime; (b) an overt act manifestly indicating that he clung
that he was under the influence of drugs, which to his determination; and (c) sufficient lapse of time
he had taken one after the other. He was a drug between the decision and the execution to allow the
[20]
dependent and, in fact, had been confined at accused to reflect upon the consequence of his act. The
the Tagaytay Rehabilitation Center. Nevertheles prosecution failed to establish any of these requisites.
s, the trial court sentenced SITCHON to suffer
the death penalty. AGGARAVATING: CRUELTY
The trial court incorrectly appreciated cruelty
ISSUE: Whether or not SITCHON is guilty of murder? against the accused. The test in appreciating cruelty as
Yes. an aggravating circumstance is whether the accused
deliberately and sadistically augmented the wrong by
GUILT causing another wrong not necessary for its commission,
The Court entertains little doubt that appellant is or inhumanly increased the victims suffering or outraged
guilty of the killing of MARK ANTHONY. SITCHONs guilt or scoffed at his person or corps. The nature of cruelty lies
was adequately established by the testimonies of Lilia in the fact that the culprit enjoys and delights in making his
Garcia and Roberto Fernandez, who both saw appellant victim suffer slowly and gradually, causing him moral and
beat Macky. These testimonies were further corroborated physical pain which is unnecessary for the consummation
by those of PO3 Paul Dennis Javier, Dr. Manuel Lagonera of the criminal act which he intended to commit. The sheer
and Felicisima Francisco, as well as the various pieces of number of wounds, however, is not a test for determining
object evidence. Indeed, appellant in open court admitted whether cruelty attended the commission of a crime The
beating the poor child, which beating resulted in the prosecution did not show that appellant enjoyed
latters death. inflicting injuries upon the victim. The inordinate
That appellant purportedly did not intend to kill the force employed by appellant appears to have been
toddler would not exculpate him from liability. Article 4(1) caused not by any sadistic bend but rather by the
of the Revised Penal Code provides that criminal liability drugs that diminished his capacity.
shall be incurred by any person committing a felony
(delito) although the wrongful act done be different from AGGRAVATING: INTOXICATION
that which he intended. The rationale of the rule is found The trial court also considered intoxication as an
in the doctrine that el que es causa de la causa es aggravating circumstance.This Court does not
causa del mal causado (he who is the cause of the cause agree. Article 13 of the Revised Penal Code provides a
is the cause of the evil caused). Thus, where the accused list of mitigating circumstances, which work to reduce the
violently kicked the sleeping victim in vital parts of the accuseds penalty. Article 13(10) allows courts to consider
latters body, the accused is liable for the supervening any other circumstance of a similar nature and analogous
death as a consequence of the injuries. Assuming, to those mentioned therein. Neither Article 14 of the
therefore, that appellant merely intended to inflict same Code on aggravating circumstances nor Article 15
physical injuries upon the boy, he is nevertheless on alternative circumstances, however, contain a provision
liable for the death of the victim caused by such similar to Article 13(10). Accordingly, the Court cannot
injuries. consider appellants drug addiction as an aggravating
circumstance. Criminal statutes are to be strictly
AGGRAVATING: TREACHERY construed and no person should be brought within their
The killing in this case was attended by terms who is not clearly within them.
treachery. There is treachery when the offender commits
any of the crimes against persons, employing means, MITIGATING: PLEA OF GUILT
methods or forms in the execution thereof which tend Appellant maintains that his plea of guilt mitigates his
directly and especially to insure its execution without risk criminal liability. To effectively alleviate the criminal liability
to himself arising from the defense which the offended of an accused, a plea of guilt must be made at the first
party might make. It is beyond dispute that the killing of opportunity, indicating repentance on the part of the
minor children who, by reason of their tender years, accused. In determining the timeliness of a plea of guilty,
could not be expected to put up a defense, is nothing could be more explicit than the provisions of the
treacherous. Revised Penal Code requiring that the offender voluntarily
confess his guilt before the court prior to the presentation
AGGRAVATING: EVIDENT PREMEDITATION of the evidence for the prosecution. It is well-settled that a
Evident premeditation is absent. For the court to plea of guilty made after arraignment and after trial had
appreciate evident premeditation, the prosecution must begun does not entitle the accused to have such plea

162
CRIMINAL LAW REVIEW DIGESTS
JUSTICE ROMEO CALLEJO NOTE: = Callejo Ponente

considered as a mitigating circumstance. As appellant companion. Not contented, they left and proceeded to the
changed his plea only after the prosecution had seashore where in a cottage there were people also
rested its case and just when he was just about to drinking. Joining the group, Anthony and Dennis again
testify, said mitigating circumstance is unavailing. drank. Later, the two and their companion transferred to
another cottage and there they again drank now with gin
MITIGATING: VOLUNTARY SURRENDER liquor except Dennis who did not anymore drink. For one
The trial court credited appellant with the mitigating reason or another, because Dennis did not drink, Anthony
circumstance of voluntary surrender. For voluntary got angry and he then bathed Dennis with gin, and boxed
surrender to be appreciated, these elements must be or mauled him and tried to stab him with a batangas knife
established: (1) the offender has not been actually but failed to hit Dennis as the latter was crawling under the
arrested; (2) he surrendered himself to a person in table. He got up and ran towards home. His family was
authority or an agent of a person in authority; and (3) his awakened, his mother shouted as Dennis was taking a
surrender was voluntary. It is sufficient that the surrender knife and appeared bloodied. Manuel Torpio woke up and
be spontaneous and made in a manner clearly tried to take the knife from Dennis but failed and, in the
indicating the intent of the accused to surrender process, wounded or cut himself in his left hand.
unconditionally, either because he acknowledges his guilt Dennis left with the knife, passed by another
or he wishes to save the authorities the trouble and route towards the seashore and upon reaching the cottage
expense which will necessarily be incurred in searching where Anthony and their companion Porboy Perez were,
for and capturing him. looked for Anthony. Anthony upon seeing Dennis sensed
Appellant has failed to adequately prove danger and he fled by taking the seashore. But Dennis,
voluntary surrender. While he claimed that he being accustomed to the place and having known the
surrendered to the police on the same day that terrain despite the darkness knew that there is only one
the victim was killed, he did not detail the circumstances exit Anthony could make and, thus, he went the other way
like the time and place of such surrender. Neither did through the nipa plantation and he was able to meet and
appellant state to whom he surrendered. He did not block Anthony. Upon seeing Dennis with a knife, Anthony
indicate if the person was a person in authority or an agent tried to evade by turning to his left and Dennis thus hit the
of the latter. PO3 Javiers testimony that he learned of back portion of Anthony. Anthony ran farther but he was
appellants alleged surrender is hearsay and does not caught in a fishing net across the small creek and he fell
serve to corroborate appellants claim. on his back. Dennis mounted Anthony and continued
stabbing the latter. He left the place went to the grassy
MITIGATING: NO INTENT TO COMMIT SO GRAVE meadow near the camp and there slept until morning. He
A WRONG then went to a certain police officer to whom he voluntarily
The Court, however, discerns no intention on the surrendered and together they went to the police
part of appellant to commit so grave a wrong against headquarters.
his victim. Appellants intention was merely to maltreat The RTC convicted Torpio of murder qualified by
the victim, not to kill him. When appellant realized the treachery or evident premeditation and appreciating in his
horrible consequences of his felonious act, he immediately favor the following mitigating circumstances: (a) sufficient
brought the victim to the hospital. Sadly, his efforts were provocation on the part of the offended party (the
for naught. deceased Anthony) preceded the act; (b) the accused
acted to vindicate immediately a grave offense committed
SENTENCE: In view of the attendance of the aggravating by the victim; and, (c) voluntary surrender.
circumstance of treachery, the killing of the victim is Torpio alleged that the RTC erred in finding that
qualified to murder, punishable under Article 248 of the treachery and evident premeditation attended the
Revised Penal Code by reclusion perpetua to death. The commission of the crime.
murder was attended by the mitigating circumstance of
lack of intention to commit so grave a wrong and there is ISSUE: W/N treachery and evident premeditation attended
no aggravating circumstance. Hence, the lesser penalty the commission of the crime NO. Only guilty of homicide
of reclusion perpetua must be imposed upon appellant. (Mitigating circumstances were properly appreciated. But
sufficient provocation and immediate vindication of a grave
RA 9165, SEC 25 offense were considered as only one mitigating
circumstance since they arose from the same incident
PEOPLE v. DENIIS TORPIO attach on Torpio by Anthony)

FACTS: Dennis Torpio and Anthony Rapas had some HELD:


round of drinks at a nearby store together with another Treachery

163
CRIMINAL LAW REVIEW DIGESTS
JUSTICE ROMEO CALLEJO NOTE: = Callejo Ponente

There is treachery when the offender employs deliberate plan but of rising tempers, or when the attack
means, methods or forms in the execution of the crime was made in the heat of anger.
which tends directly and specially to insure its execution
without risk to himself arising from the defense which the
offended party might make. There must be evidence 107. P v Navarre Nov. 18, 1998
showing that the mode of attack was consciously or
deliberately adopted by the culprit to make it impossible or
difficult for the person attacked to defend himself or
(second paragraph of Section 1)The penalty of prision
retaliate. Further, the essence of treachery is the swift and
mayor in its minimum period and a fine of Thirty thousand
unexpected attack without the slightest provocation by the
pesos (P30,000) shall be imposed if the firearm is
victim.
classified as high powered firearm which includes those
In this case, the record is barren of evidence
with bores bigger in diameter than .38 caliber and 9
showing any method or means employed by the appellant
millimeter such as caliber .40, .41, .44, .45 and also lesser
in order to ensure his safety from any retaliation that could
calibered firearms but considered powerful such as caliber
be put up by the victim. The appellant acted to avenge
.357 and caliber .22 center-fire magnum and other
Anthonys felonious acts of mauling and stabbing
firearms with firing capability of full automatic and by burst
him. Although the appellant bled from his stab wound, he
of two or three: Provided, however, That no other crime
ran home, armed himself with a knife and confronted
was committed by the person arrested.
Anthony intentionally. When the latter fled, the appellant
ran after him and managed to stab and kill the victim.
(third paragraph) "If homicide or murder is committed
Evident Premeditation
with the use of an unlicensed firearm, such use of an
To warrant a finding of evident premeditation, the
unlicensed firearm shall be considered as an
prosecution must establish the confluence of the following
aggravating circumstance.
requisites: (a) the time when the offender was determined
to commit the crime; (b) an act manifestly indicating that
the offender clung to his determination; and (c) a sufficient PEOPLE VS LADJAALAM
interval of time between the determination and the
execution of the crime to allow him to reflect upon the FACTS: Ladjaalam was charged with 4 informations, one
consequences of his act. of them was for illegal possession of firearms and another
The qualifying circumstance of evident was for multiple attempted murder with direct assault (for
premeditation requires that the execution of the criminal firing an M14 rifle to police men who were about to enter
act by the accused be preceded by cool his house to serve a search warrant). RTC found
thought and reflection upon a resolution to carry out the Ladjaalam guilty of direct assault with multiple attempted
criminal intent during the space of time sufficient to arrive homicide (not murder since no policeman was hit and
at a calm judgment. Evident premeditation needs proof of injured) and sentenced a separate offense of illegal
the time when the intent to commit the crime is possession of firearms under PD 1866, as amended by
engendered in the mind of the accused, the motive which RA 8294.
gives rise to it, and the means which are beforehand OSGs contentions: [Ladjaalam shouldnt be
selected to carry out that intent. convicted of separate offense of illegal possession]
The prosecution failed to prove the same. RTC shouldnt have applied the new law (RA
According to Manuel, the father of the appellant, the latter 8294) but PD 1866 (penalized simple illegal
told him, I have to kill somebody, Tay, because I was possession of firearms even if another crime is
boxed. This utterance is not sufficient to show that the committed at the same time) because provision
crime was a product of serious and determined does not cover specific facts of the case since
reflection. The interval between the time when the another crime - direct assault with multiple
appellant made this statement and when he actually unlawful homicides -- was committed.
stabbed Anthony was not sufficient or considerable Since there was no killing in this case, illegal
enough as to allow him to reflect upon the consequences possession cannot be deemed as an aggravating
of his act. There was no sufficient interregnum from the circumstance under the third paragraph of the
time the appellant was stabbed by the victim, when the provision.
appellant fled to their house and his arming himself with a RTCs contentions:[Should be convicted of a
knife, and when he stabbed the victim. In a case of fairly separate offense]
recent vintage, we ruled that there is no evident Second paragraphs (see top) proviso (no other
premeditation when the fracas was the result, not of a crime), refers only to homicide or murder, in both

164
CRIMINAL LAW REVIEW DIGESTS
JUSTICE ROMEO CALLEJO NOTE: = Callejo Ponente

of which illegal possession of firearms is an paragraph. Verily, where the law does not
aggravating circumstance. distinguish, neither should the SC.
In other words, if a crime other than murder or
homicide is committed, a person may still be NOTE: SC knows that this ruling exonerates Ladjaalam
convicted of illegal possession of firearms. In this from illegal possession of an M-14 rifle, an offense which
case, the other crime committed was direct normally carries a penalty heavier than that for direct
assault with multiple attempted homicide; hence, assault. It even contemplated a situation where the
the trial court found appellant guilty of illegal accused may evade conviction for illegal possession of
possession of firearms. firearms by using such weapons in committing an even
lighter offense, like alarm and scandal or slight physical
ISSUE (topic): Whose contentions are correct [W/N illegal injuries. But the Courts do not have discretion to question
possession of firearms is a separate offense when the wisdom behind the language of RA 8294. The matter
coupled with direct assault with multiple attempted should be addressed to Congress.
homicide] OR more specifically [W/N the proviso in the
second paragraph ENDING: (in relation to topic) Ladjaalam charged with
direct assault and multiple attempted homicide with the
RATIO: use of a weapon lang (no separate offense nor
aggravating circumstance)
BOTH WRONG (eyng?)
J. Panganibans opening statement:
A simple reading thereof shows that if an unlicensed Republic Act No. 8294 penalizes simple illegal possession
firearm is used in the commission of any crime, there can of firearms, provided that the person arrested committed
be no separate offense of simple illegal possession of no other crime. Furthermore, if the person is held liable
firearms. Hence, if the other crime is murder or homicide, for murder or homicide, illegal possession of firearms is an
illegal possession of firearms becomes merely an aggravating circumstance, but not a separate
aggravating circumstance, not a separate offense. Since offense. Hence, where an accused was convicted of direct
direct assault with multiple attempted homicide was assault with multiple attempted homicide for firing an
committed in this case, appellant can no longer be held unlicensed M-14 rifle at several policemen who were
liable for illegal possession of firearms. Moreover, penal about to serve a search warrant, he cannot be held guilty
laws are construed liberally in favor of the accused and of the separate offense of illegal possession of
the plain meaning of RA 8294s simple language is most firearms. Neither can such unlawful act be considered to
favorable to Lamjaalam. Since the crime committed was have aggravated the direct assault.
direct assault and not homicide or murder, illegal
possession of firearms cannot be deemed an aggravating Personal Opinion: Kalokohan yung law. (Thesis topic-
circumstance. able?)

OSGs contentions: Crime charged with Illegal


Effect
Crime was committed on September 24, 1997, Possession of Firearm
while RA 8294 took effect on July 6, 1997. Murder Aggravating
Therefore, when crime was committed, PD homicide Circumstance
1866s provision, which justified a conviction for In connection with:
illegal possession of firearms separate from any Rebellion,
other crime, was amended by RA 8294 which Sedition,
contained the specific proviso that no other Absorbed
Iinsurrection or
crime was committed. Attempted coup
The criminal case for homicide [was] not before de'etat
the SC for consideration (not those appealed) Accused will be
prosecuted only
RTCs contentions: for the committed
Limiting the proviso in the second paragraph to Any other crime
crime without
only murder and homicide is not justified. If the any effect for the
intention of the law in the second paragraph were unlicensed firearm
to refer only to homicide and murder, it should
have expressly said so, as it did in the third

165
CRIMINAL LAW REVIEW DIGESTS
JUSTICE ROMEO CALLEJO NOTE: = Callejo Ponente

2 security guards of their firearms to weaken the capability


PEOPLE v. SABADAO of the captives and then thereafter, effecting their final
mission, which was the grabbing of bundles of cash and
Facts: Defendants Sabadao, Valdez, Mayo, and Abangon the opening of the vault. This taking of the cash by the
th
were accused of robbery with homicide and illegal dead robber and then thereafter by the 4 robber and the
possession of firearm. Armed with loaded guns, they taking of the firearms completed the act of robbery.
entered RCBC in Ilocos Norte and ordered that the vault
be opened. Peace officers arrived (due to the burglar
alarm) and a shoot-out ensued. One police officer and PEOPLE v. ESTEBAN DOMACYONG (et.al)
one security guard were shot and died as a result. Aside
from the 4, there was a fifth robber (but he was shot and [Doctrine: Unlicensed firearm]
died after). The accused were able to escape with (Note: this is one of the cases which Sir would like us to
P4,200. Valdez said he was in RCBC to solicit funds for a criticize or compare with the other decisions)
seminar workshop while Sabadaoput up the defense of
alibi. FACTS: (Penned by Justice Puno). The accused in this
The RTC found them guilty of robbery with case were charged with the crime of robbery with
homicide. It found that conspiracy took place because of homicide. The information alleged that the accused were
their previously designed scheme of entry and plan of conspiring, confederating and mutually aiding one another,
operation. The crime was also attended by the with intent to gain and being then armed with guns, and by
aggravating circumstance of band, having been means of violence and intimidation, willfully, unlawfully and
perpetrated by 4 armed malefactors who acted together in feloniously take, rob and carry away cash money from the
the commission of the crime. Victoria Supermart,; that on the occasion and by reason of
Before the SC, they argue that they were not said robbery(,) and for the purpose of enabling them to
adequately identified s the perpetrators, claiming that take, steal, rob and carry away the said amount of
there was no clear and convincing proof that either of money(,) the accused willfully, unlawfully and feloniously
them caused the death of the victims, and prosecution and with intent to kill, engaged responding policemen and
was not able to establish who actually shot who. Plus, it law enforcing agents in a shootout, resulting to death and
should have been attempted robbery only because they grave injuries to some people in the vicinity. Accused
were not able to perform all the acts of execution, i.e. that Domacyong and Paleyan were also separately charged
the prosecution were not able to concretely establish that with the crime of Violation of P.D. No. 1866 (Illegal
they were able to take the money away. Possession of Firearm and Ammunition).

Issue: Guilty? YES ISSUE: Whether or not accused should be charged


separately for the crime of illegal possession of firearms?
Ratio: The rule is well-established that whenever homicide
has been committed as a consequence of or on the HELD: No.
occasion of the robbery, all those who took part as We now come to appellants guilt for the crime of
principals in the robbery will also be held guilty as illegal possession of firearms. The trial court separately
principals of the special complex crime of robbery with convicted appellants of the crime of violation of Republic
homicide although they did not actually take part in the Act No. 82944 amending Presidential Decree No. 1866.
homicide, unless it clearly appears that they endeavored The violation was also appreciated by the trial court to
to prevent the homicide. Further, conspiracy can be aggravate their penalty in the crime of robbery with
inferred from the acts of the malefactors before, during homicide.
and after the commission of the crime which are indicative Republic Act No. 8294 provides that if homicide
of a joint purpose, concerted action, and concurrence of or murder is committed with the use of an unlicensed
sentiments. In this case, their cooperative acts toward firearm, such use of an unlicensed firearm shall be
their criminal objective render them equally liable as considered as an aggravating circumstance. We have
conspirators:
Valdez gained foothold by holding the manager 4 AN ACT AMENDING THE PROVISIONS OF PRESIDENTIAL
and one of the two security guards captive. Further DECREE NO. 1866, AS AMENDED, ENTITLED "CODIFYING
deployment of reinforcement is shown by the concerted THE LAWS ON ILLEGAL/UNLAWFUL POSSESSION,
th MANUFACTURE, DEALING IN, ACQUISITION OR
entry of 2 or more of the conspirators follow by the 4 to DISPOSITION OF FIREARMS, AMMUNITION OR EXPLOSIVES
complete the strength of force that constituted conspiracy. OR INSTRUMENTS USED IN THE MANUFACTURE OF
With this in place, the implementation of the plan of action FIREARMS, AMMUNITION OR EXPLOSIVES, AND IMPOSING
STIFFER PENALTIES FOR CERTAIN VIOLATIONS THEREOF,
then started with the taking of valuables first, divesting the AND FOR RELEVANT PURPOSES."

166
CRIMINAL LAW REVIEW DIGESTS
JUSTICE ROMEO CALLEJO NOTE: = Callejo Ponente

consistently ruled that if an unlicensed firearm is used in penalty to the next higher degree. Examples are quasi-
the commission of any other crime, there can be no recidivism under Article 160 and complex crimes under
separate offense of simple illegal possession of Article 48 of the Revised Penal Code. It does not change
firearms. Thus, a simple reading thereof (R.A. No. 8294) the character of the offense charged.64 It must always be
shows that if an unlicensed firearm is used in the alleged and charged in the information, and must be
commission of any crime, there can be no separate proven during the trial in order to be appreciated.65
offense of simple illegal possession of firearms. Hence, if Moreover, it cannot be offset by an ordinary mitigating
the other crime is murder or homicide, illegal possession circumstance.
of firearms becomes merely an aggravating circumstance, It is clear from the foregoing that the meaning
not a separate offense. . . . and effect of generic and special aggravating
Moreover, penal laws are construed liberally in circumstances are exactly the same except that in case of
favor of the accused. In this case, the plain meaning of RA generic aggravating, the same CAN be offset by an
8294's simple language is most favorable to herein ordinary mitigating circumstance whereas in the case of
appellant. Verily, no other interpretation is justified, for the special aggravating circumstance, it CANNOT be offset by
language of the new law demonstrates the legislative an ordinary mitigating circumstance.
intent to favor the accused. Aside from the aggravating circumstances
In the cases at bar, the crime of robbery with abovementioned, there is also an aggravating
homicide with the use of unlicensed firearms was circumstance provided for under Presidential Decree No.
committed by appellants. In line with law and 1866, as amended by Republic Act No. 8294, which is a
jurisprudence, the use of unlicensed firearms merely special law. Its pertinent provision states:
aggravates the crime of robbery with homicide. It does not If homicide or murder is committed with the use
constitute a separate crime. Necessarily, the conviction of of an unlicensed firearm, such use of an unlicensed
appellants for illegal possession of firearms has to be set firearm shall be considered as an aggravating
aside. circumstance.
In interpreting the same provision, the trial court
reasoned that such provision is "silent as to whether it is
PEOPLE OF THE PHILIPPINES, Appellee, vs. MARLON generic or qualifying." Thus, it ruled that "when the law is
ALBERT DE LEON y HOMO, Appellant. silent, the same must be interpreted in favor of the
accused." Since a generic aggravating circumstance is
FACTS: more favorable to petitioner compared to a qualifying
- Accused was found guilty of robbery with homicide of aggravating circumstance, as the latter changes the
only one count for robbing 4 gas stations and killing nature of the crime and increase the penalty thereof by
one security guard. Modus nila ang magpa-gas then degrees, the trial court proceeded to declare that the use
say that their engine wont start so theyll ask the gas of an unlicensed firearm by the petitioner is to be
boys or the employees to help them push. After considered only as a generic aggravating circumstance.
which, theyll alight and announce a hold-up. This interpretation is erroneous, since we already held in
several cases that with the passage of Republic Act No.
ISSUE: W/N the aggravating circumstance of use of an 8294 on 6 June 1997, the use of an unlicensed firearm in
unlicensed firearm should be appreciated. murder or homicide is now considered as a SPECIAL
aggravating circumstance and not a generic aggravating
HELD/RATIO: NO. circumstance. Republic Act No. 8294 applies to the instant
Generic aggravating circumstances are those that case since it took effect before the commission of the
generally apply to all crimes such as those mentioned in crimes in 21 April 1998. Therefore, the use of an
Article 14, paragraphs No. 1, 2, 3, 4, 5, 6, 9, 10, 14, 18, 19 unlicensed firearm by the petitioner in the instant case
and 20, of the Revised Penal Code. It has the effect of should be designated and appreciated as a SPECIAL
increasing the penalty for the crime to its maximum period, aggravating circumstance and not merely a generic
but it cannot increase the same to the next higher degree. aggravating circumstance.
It must always be alleged and charged in the information, After a careful study of the records of the present
and must be proven during the trial in order to be case, this Court found that the use of unlicensed firearm
appreciated. Moreover, it can be offset by an ordinary was not duly proven by the prosecution. Although
mitigating circumstance. jurisprudence dictates that the existence of the firearm can
On the other hand, special aggravating be established by mere testimony, the fact that appellant
circumstances are those which arise under special was not a licensed firearm holder must still be established.
conditions to increase the penalty for the offense to its The prosecution failed to present written or testimonial
maximum period, but the same cannot increase the evidence to prove that appellant did not have a license to

167
CRIMINAL LAW REVIEW DIGESTS
JUSTICE ROMEO CALLEJO NOTE: = Callejo Ponente

carry or own a firearm, hence, the use of unlicensed


firearm as an aggravating circumstance cannot be Issue: Whether or not the defendants were guilty beyond
appreciated. reasonable doubt of the crime.

Held: Yes. Guilty!


PEOPLE V. ABDUL Ratio: Elements of the Crime
The following elements must be established for a
Facts: In the afternoon of Aug. 31, 1999, after Nestor conviction in the special complex crime of robbery with
Gabuya, after closing his motorcycle and bicycle spare homicide:
parts shop, headed home on his bike. Unknown to him, 1. The taking of personal property is committed
Abdul Aminola (defendant) and Alimudin Laminda were with violence or intimidation against persons;
observing him. Aminola followed Gabuya. Upon catching 2. The property taken belongs to another;
up with Gabuya, Aminola put his arms around Gabuya and 3. The taking is animo lucrandi; and
wrestled for the bag Gabuya was carrying. Gabuya 4. By reason of the robbery or on the occasion
refused to let go of his bag, whereupon Aminola pulled out thereof, homicide is committed.
a gun and shot him. Gabuya fell to the ground but still Essential for conviction of robbery with homicide
resisted, prompting Aminola to take another shot. is proof of a direct relation, an intimate connection
Mike Maitimbang (the other defendant in this between the robbery and the killing, whether the latter be
case) then approached and took something from Gabuya. prior or subsequent to the former or whether both crimes
Maitimbang shot Gabuya behind and fled towards the are committed at the same time.
direction of eyewitness Oliva. Joel, Gabuyas caretaker, The prosecution was able to establish that
gave chase but was fired upon by Maitimbang.Oliva accused-appellants committed robbery with homicide
testified seeing the incident. through the totality of their evidence. The first three
Regina, Gabuyas wife, reported the incident that elements were established when Oliva testified that he
same afternoon. Based on her information, Major Migano saw, and positively identified, accused-appellants taking
formed a team to investigate the crime. Gabuyas property by force and both shooting Gabuya.
Later that evening, an informant known as Gabuyas death resulting from their attack proves the last
"Abdul" (different from the defendant) told the police that element of the complex crime as duly confirmed by the
he witnessed what had happened to Gabuya and could tell post-mortem report.
them where the suspects could be found. True enough,
Abdul led Major Migano and his men. A blocking force was Defense of Alibi Unavailing
organized while Col. Bernido formed a team to make the Accused-appellants cannot avoid liability by way of their
arrests on the suspects. defenses. Alibi is the weakest of all defenses because it is
The following night, Major Miganos team once easy to concoct and difficult to disprove. To establish alibi,
again went to the hideout, where Abdul identified four of an accused must prove (1) that he was present at another
Gabuyas assailants. One of them, Aminola, was found in place at the time the crime was perpetrated; and (2) that it
possession of an unlicensed .45 caliber gun with one (1) was physically impossible for him to be at the scene of the
magazine and two (2) ammunitions. The following night crime. Physical impossibility "refers to the distance
Matimbang was also arrested. between the place where the accused was when the crime
Two informations were filed. One was against all transpired and the place where it was committed, as well
of them for robbing and killing Gabuya, The other one was as the facility of access between the two places."
against Abdul Aminola only for unlicensed possession of The fact that Aminolas witness, i.e., SPO2
the gun and the ammo. Lukman, corroborated Aminolas testimony about not
The defendants all denied any knowledge of the being at the situs of the crime when Gabuya was robbed
crime. They said that they were arrested without warrants and killed does not, without more, serve to strengthen
and that they only found out about the charge after the Aminolas alibi. As the appellate court aptly observed,
inquest proceedings. SPO2 Lukmans testimony did not prove the physical
The RTC found Abdul Aminola and Mike impossibility for Aminola to be at the scene of the crime.
Matimbiang guilty of robbery with homicide with the SPO2 Lukman did not categorically specify the time he
aggravating circumstance of use of unlicensed firearm, was with Aminola on the date of the incident. His
applying Section 1 of Republic Act 8294. They were testimony did not preclude the possibility of Aminola
sentenced to death. The other were acquitted. perpetrating the crime after their meeting. As the trial court
The CA affirmed but reduced the penalty to perceptively observed:
reclusion perpetua in view of the abolition of the death The time interval from Rogan Street to Bonifacio
penalty. Street is just five (5) or ten (10) minutes. Such distance

168
CRIMINAL LAW REVIEW DIGESTS
JUSTICE ROMEO CALLEJO NOTE: = Callejo Ponente

does not preclude the accused from being at the place of BURGOS-VILLAVERT, JUDGE EDGARDO SUDIAM of
the crime at the time of its commission. Hence SPO2 the RTC, Manila; RCBC and THE PEOPLE
Lukmans testimony could not be given more weight than A corporate officer cannot protect himself behind a
prosecution witness Olivas testimony. corporation where he is the actual, present and efficient
The defense of Maitimbang, likewise, cannot actor - Chief Justice Earl Warren
overcome the positive identification by Oliva. Under oath,
Oliva testified seeing Maitimbang take Gabuyas property Chingwas the Senior VP of Philippine Blooming Mills, Inc.
and shot Gabuya at the back while already prone on the (PBMI). Sometime in September to October 1980, PBMI,
ground. through Ching, applied with RCBC for the issuance of
Denial and alibi cannot prevail over the positive and commercial letters of credit to finance its importation of
categorical testimony of the witness identifying a person assorted goods. RCBC approved the application, and
as the perpetrator of the crime absent proof of ill motive. irrevocable letters of credit were issued in favor of Ching.
No reason or motive was given for Oliva to falsely testify The goods were purchased and delivered in trust to PBMI.
against accused-appellants on such a serious crime. As Petitioner signed 13 trust receipts as surety,
often noted, the trial court is in a better position to observe acknowledging delivery of various goods
the demeanor and candor of the witnesses and to decide Under the receipts, Chingagreed to hold the
who is telling the truth. goods in trust for the said bank, with authority to sell but
not by way of conditional sale, pledge or otherwise; and in
case such goods were sold, to turn over the proceeds
PEOPLE VS. MENDOZA thereof as soon as received, to apply against the relative
acceptances and payment of other indebtedness to
Cecilia Mendoza and her 10 year old daughter, Charmaine respondent bank. In case the goods remained unsold
Mendoza, attended a birthday party of a relative of Octavio within the specified period, the goods were to be returned
Mendoza at Mcdo. Later on, Octavio left and went to KFC to respondent bank without any need of demand. Thus,
and had some beer. When it was time to go home, Cecilia said "goods, manufactured products or proceeds thereof,
and Charmaine could not find the accused and so they whether in the form of money or bills, receivables, or
decided to leave. accounts separate and capable of identification" were
When they arrived home, Cecilia and Octavio respondent banks property.
proceeded to the masters bedroom. Charmaine heard When the trust receipts matured, Chingfailed to
them quarreling regarding Octavio having left the party. return the goods to RCBC, or to return their value
Suddenly, she heard 3 gunshots. Running out of her room, amounting toP6,940,280.66 despite demands. Thus, the
Charmaine saw her mother lying on the floor bleeding. bank filed a criminal complaint for estafaagainst Ching.
Cecilia bled to death. Petitioner posits that, except for his being the
The RTC found Octavio guilty of the crime of Senior Vice-President of the PBMI, there is no iota of
parricide and the crime of illegal possession of firearm and evidence that he was a participescrimines in violating the
ammunitions. trust receipts sued upon; and that his liability, if at all, is
purely civil because he signed the said trust receipts
ISSUE: Whether or not the crime of illegal possession of merely as a xxx surety and not as the entrustee.
firearm must be considered as an aggravating The RTC granted the Motion to Quash the
circumstance instead of a separate crime??? YES Informations. On February 27, 1995, respondent bank re-
filed the criminal complaint for estafa against petitioner.
RULING: Although the prosecution duly established that The City Prosecutor ruled that there was no probable
the crime of illegal possession of firearm under cause. RCBC appealed the resolution. On July 13, 1999,
Presidential Decree No. 1866 was committed, fortunately the Secretary of Justice issued Resolution granting the
for Octavio, Republic Act No 8294 which took effect on petition and reversing the assailed resolution of the City
July 7, 1997 amended the said decree and the law now Prosecutor.
merely considers the use of an unlicensed firearm as an According to the Justice Secretary, the petitioner,
aggravating circumstance in murder or homicide, and not as Senior Vice-President of PBMI, executed the 13 trust
as a separate offense. receipts and as such, was the one responsible for the
offense. Thus, the execution of said receipts is enough to
PRINCIPALS indict the petitioner as the official responsible for violation
of P.D. No. 115. Also, respondent bound himself not only
as a corporate official of PBMI but also as its surety.
ALFREDO CHING, Petitioner, vs. SECRETARY OF
JUSTICE, ASST. CITY PROSECUTOR ECILYN

169
CRIMINAL LAW REVIEW DIGESTS
JUSTICE ROMEO CALLEJO NOTE: = Callejo Ponente

CA: Ching, being the Senior Vice-President of designates an act of a corporation or a crime and
PBMI and the signatory to the trust receipts, is criminally prescribes punishment therefor, it creates a criminal
liable for violation of P.D. No. 115 offense which, otherwise, would not exist and such can be
committed only by the corporation. But when a penal
ISSUE: Whether there was sufficient babsis to continue statute does not expressly apply to corporations, it does
prosecution against Ching. not create an offense for which a corporation may be
punished. On the other hand, if the State, by statute,
SC: "x xx it is apropos to quote section 13 of PD 115 defines a crime that may be committed by a corporation
which states in part, viz: but prescribes the penalty therefor to be suffered by the
xxx If the violation or offense is committed by a officers, directors, or employees of such corporation or
corporation, partnership, association or other judicial other persons responsible for the offense, only such
entities, the penalty provided for in this Decree shall be individuals will suffer such penalty. Corporate officers or
imposed upon the directors, officers, employees or other employees, through whose act, default or omission the
officials or persons therein responsible for the offense, corporation commits a crime, are themselves individually
without prejudice to the civil liabilities arising from the guilty of the crime.
criminal offense. The principle applies whether or not the crime
There is no dispute that it was the respondent, requires the consciousness of wrongdoing. It applies to
who as senior vice-president of PBM, executed the those corporate agents who themselves commit the crime
thirteen (13) trust receipts. As such, the law points to him and to those, who, by virtue of their managerial positions
as the official responsible for the offense. Since a or other similar relation to the corporation, could be
corporation cannot be proceeded against criminally deemed responsible for its commission, if by virtue of their
because it cannot commit crime in which personal relationship to the corporation, they had the power to
violence or malicious intent is required, criminal action is prevent the act.Moreover, all parties active in promoting a
limited to the corporate agents guilty of an act amounting crime, whether agents or not, are principals. Whether such
to a crime and never against the corporation itself (West officers or employees are benefited by their delictual acts
Coast Life Ins. Co. vs. Hurd, 27 Phil. 401; Times, [I]nc. v. is not a touchstone of their criminal liability. Benefit is not
Reyes, 39 SCRA 303). Thus, the execution by respondent an operative fact.
of said receipts is enough to indict him as the official In this case, petitioner signed the trust receipts in
responsible for violation of PD 115. question. He cannot, thus, hide behind the cloak of the
Though the entrustee is a corporation, separate corporate personality of PBMI. In the words of
nevertheless, the law specifically makes the officers,
employees or other officers or persons responsible for the
offense, without prejudice to the civil liabilities of such PEOPLE VS. BULU CHOWDURY
corporation and/or board of directors, officers, or other
officials or employees responsible for the offense. The Facts: Bulu Chowdury and Josephine Ong were charged
rationale is that such officers or employees are vested with with the crime of illegal recruitment in large scale. The
the authority and responsibility to devise means necessary prosecution presented four witnesses: private
to ensure compliance with the law and, if they fail to do so, complainants Aser Sasis, Estrella Calleja and Melvin
are held criminally accountable; thus, they have a Miranda, and Labor Employment Officer Abbelyn Caguitla.
responsible share in the violations of the law. Sasis testified that he first met Chowdury when
If the crime is committed by a corporation or he applied with Craftrade Overseas Developers
other juridical entity, the directors, officers, employees or (Craftrade) for employment as factory worker in South
other officers thereof responsible for the offense shall be Korea. Chowdury, a consultant of Craftrade, conducted
charged and penalized for the crime, precisely because of the interview. During the interview, Chowdury informed
the nature of the crime and the penalty therefor. A him about the requirements and required him to undergo a
corporation cannot be arrested and imprisoned; hence, seminar. He advised him that placement would be on a
cannot be penalized for a crime punishable by first-come-first-serve basis and charged a processing fee
imprisonment.However, a corporation may be charged of P25k. Sasis completed all the requirements and paid a
and prosecuted for a crime if the imposable penalty is fine. total amount of P16k. Sasis further said that he went to the
Even if the statute prescribes both fine and imprisonment office of Craftrade three times to follow up his application
as penalty, a corporation may be prosecuted and, if found but he was informed that he would no longer be deployed
guilty, may be fined. for employment abroad. This prompted him to withdraw
A necessary part of the definition of every crime his payment but he could no longer find Chowdury. After
is the designation of the author of the crime upon whom two unsuccessful attempts to contact him, he decided to
the penalty is to be inflicted. When a criminal statute file with the POEA a case for illegal recruitment against

170
CRIMINAL LAW REVIEW DIGESTS
JUSTICE ROMEO CALLEJO NOTE: = Callejo Ponente

Chowdury. Upon verification with the POEA, he learned by his agency to deal with the applicants in its behalf.
that Craftrade's license had already expired and has not Accused-appellant in fact confined his actions to his job
been renewed and that Chowdury, in his personal description. Hence, we hold that the prosecution failed to
capacity, was not a licensed recruiter. Similar facts were prove beyond reasonable doubt accused-appellant's
given by the other witnesses, Calleja and Miranda. Labor conscious and active participation in the commission of
Employment Officer Caguitla of the Licensing Branch of the crime of illegal recruitment. His conviction, therefore, is
the POEA testified that Chowdury and his co-accused, without basis.
Ong, were not licensed recruiters nor were they connected This is not to say that private complainants are
with any licensed agency. left with no remedy for the wrong committed against them.
For his defense, Chowdury testified that he The DOJ may still file a complaint against the officers
worked as interviewer at Craftrade. As a mere employee, having control, management or direction of the business of
he only followed the instructions given by his superiors. He Craftrade so long as the offense has not yet prescribed.
never received money from the applicants and that he
already resigned from Craftrade. Citing the second
sentence of the last paragraph of Section 6 of RA 8042, PEOPLE V. ROGER TULIN
accused-appellant argues that the ones who should be
held liable for the offense are the officers having control, MT Tabangao is a cargo vessel owned by PNOC. It was
management and direction of the agency. sailing near the coast of Mindoro loaded with barrels of
The trial court found Chowdury guilty of illegal kerosene, gasoline, and diesel oil with a total value of
recruitment in large scale. 40.4M. The vessel was suddenly boarded by 7 fully armed
pirates (accused in the case Emilio Changco, Cecilio
Issue: WON accused-appellant knowingly and intentionally Changco, Tulin, Loyola, Infante, etc.). they detained and
participated in the commission of the crime charged. -NO! took control of the vessel. The name MT Tabangao and
the PNOC logo were painted over with black. Then it was
The last paragraph of Section 6 of Republic Act (RA) 8042 painted with the name Galilee. The ship crew was forced
states who shall be held liable for the offense, thus: "The to sail to Singapore.
persons criminally liable for the above offenses are the In Singapore, the ship was awaiting another
principals, accomplices and accessories. In case of vessel that did not arrive. Instead, the ship went back to
juridical persons, the officers having control, management Batangas Philippines and remained at sea. Days later, it
or direction of their business shall be liable." went back to Singapore. This time, another vessel called
the Navi Pride anchored beside it. Another accused,
he Revised Penal Code which supplements the Cheong San Hiong, supervised the Navis crew and
law on illegal recruitment defines who are the principals, received the cargo on board MT Tabangao/Galilee.
accomplices and accessories (Art. 17, 18,19). After the transfer of goods were completed, MT
An employee of a company or corporation Tabangao/Galilee went back to the Philippines and the
engaged in illegal recruitment may be held liable as original crew members were released by the pirates in
principal, together with his employer, if it is shown that he batches. The crew was ordered not to tell authorities of
actively and consciously participated in illegal recruitment. what happened.
It has been held that the existence of the corporate entity The chief engineer of the crew, however,
does not shield from prosecution the corporate agent who reported the incident to the coast guard. Afterwards, a
knowingly and intentionally causes the corporation to series of arrests were effected in different places. An
commit a crime. The culpability of the employee therefore information charging the accused with qualified piracy or
hinges on his knowledge of the offense and his active violation of the PD 532 Piracy in the Philippine Waters
participation in its commission. Where it is shown that the was filed against the accused.
employee was merely acting under the direction of his As it turns out, Navi Pride captain, Hiong, was
superiors and was unaware that his acts constituted a employed with Navi Marine Services ( a Singaporean firm,
crime, he may not be held criminally liable for an act done I think). Before the seizure of the MT Tabangon, Navi
for and in behalf of his employer. Marine was dealing for the first time with Paul Gan, a
At the time of the interview, appellant was Singaporean broker who offered to sell bunker oil to the
employed as interviewer of Craftrade which was then former. When the transaction pushed through, Hiong was
operating under a temporary authority given by the POEA assigned to supervise a ship to ship transfer. He was told
pending renewal of its license. The evidence at hand that the Galilee would be making the transfer, so Navi
shows that accused-appellant carried out his duties as Pride ship-sided with Galilee and the transfer was
interviewer of Craftrade believing that the agency was duly effected. Paul Gan received the payment. Upon arrival in
licensed by the POEA and he, in turn, was duly authorized Singapore, Hiong was asked again to transact another

171
CRIMINAL LAW REVIEW DIGESTS
JUSTICE ROMEO CALLEJO NOTE: = Callejo Ponente

transfer of oil. The same procedure was followed. Hiong prevent piracy in Philippine waters (People v. Catantan,
then went to the Philippines to arrange another transfer 278 SCRA 761 [1997]).
with Changco the pirates head. This was how Hiong was The attack on and the seizure of MT Tabangao and
arrested by the NBI agents. its cargo were committed in Philippine waters, although
All the accused put up denials and alibis. The trial the captive vessel was later brought by the pirates to
court, with ROMEO CALLEJO deciding, ruled that the Singapore, where its cargo was off-loaded, transferred
accused were all guilty. and sold. Such transfer was done under Hiongs
supervision. Although the disposition by the pirates of
ISSUE: w/n the accused are guilty of qualified piracy the vessel and its cargo was not done in Philippine
YES! waters, it is still deemed part of the same act. Piracy
falls under Title 1 of Book 2 of the RPC. It is an
RULING: [only the important part for crim] exception to the rule on territoriality in criminal law.
Hiong argues that he cannot be convicted under PD 534 The same principle applies to the case, even if Hiong
or Art 122 of the RPC as amended, since both laws punish is charged with violation of a special penal law,
piracy committed in Philippine waters. Hiong also instead of the RPC. Regardless of the law penalizing
contends that the court never acquired jurisdiction over piracy, it remains to be a reprehensible crime against
him since the crime was committed outside Philippine the whole world.
waters.
Art. 122 of the RPC (piracy in general and mutiny
in the high seas) provided that piracy must be committed PEOPLE VS ISABELO PUNO Y GUEVARRA
in the high seas by any person not a member of its
complement nor a passenger thereof. It was amended by Facts: Isabelo Puno is the driver of Mr. Socorro. While Mr.
RA 7659, which broadened the law to include offenses Socorro was allegedly in Davao for a local election,
committed in Philippine waters. PD 532 on the other hand, Isabelo arrived at Mrs. Socorros bakeshop to tell her that
embraces any person, including a passenger or member her own driver had to go to Pampanga for an emergency.
of the complement of said vessel in the Philippine waters. Hence, Isabelo will take his place.
Passenger or not, member of the complement or not, any Isabelo was driving the Mercedes Benz to bring
person is covered by the law. No conflict exists among the Mrs. Socorro home Suddenly, accused Enrique Amurao
mentioned laws, they exist harmoniously as separate (nephew of Isabelo) boarded the car beside Isabelo and
laws. poked a gun at Mrs. Socorro. An initial P7K was taken
As regards the contention that the trial court did not from her which was in her bag. They demanded P100K
acquire jurisdiction over the person of accused-appellant more. The whole time, the gun was pointed at Mrs.
Hiong since the crime was committed outside Philippine Socorros neck. They then asked her to issue a check.
waters, suffice it to state that unquestionably, the attack on After drafting 3 checks (2 checks for P30K and 1 check for
and seizure of "M/T Tabangao" (renamed "M/T Galilee" by P40K).
the pirates) and its cargo were committed in Philippine Isabelo kept on driving the car until Mrs. Socorro
waters, although the captive vessel was later brought by jumped out and then ran. Both Isabelo and Enrique were
the pirates to Singapore where its cargo was off-loaded, caught the next day when they were trying to encash the
transferred, and sold. And such transfer was done under checks they took. An information of kidnapping for ransom
accused-appellant Hiong's direct supervision. Although was filed. Defense argues that it should be simple robbery
Presidential Decree No. 532 requires that the attack and under Art294 of the RPC.
seizure of the vessel and its cargo be committed in TC: guilty of violation of PD532 (Anti Pirac and
Philippine waters, the disposition by the pirates of the Anti Highway Robbery Law of 1974). SolGen agrees with
vessel and its cargo is still deemed part of the act of this based on the observation that PD532 modified art267
piracy, hence, the same need not be committed in of the RPB.
Philippine waters.
Moreover, piracy falls under Title One of Book Two Issue: What crime was committed? Robbery under
of the Revised Penal Code. As such, it is an exception to Art.294!
the rule on territoriality in criminal law. The same principle
applies even if Hiong, in the instant case, were charged, Held: The rule in crim law is that the motive and specific
not with a violation of qualified piracy under the penal code intent of the accused in perpetrating the acts complained
but under a special law, Presidential Decree No. 532 of are invaluable aids at arriving at a correct determination
which penalizes piracy in Philippine waters. Verily, of the crime for which said accused should be held liable.
Presidential Decree No. 532 should be applied with more Thus, if murder was committed in furtherance of rebellion
force here since its purpose is precisely to discourage and then rebellion absorbs murder. Whereas, if murder was

172
CRIMINAL LAW REVIEW DIGESTS
JUSTICE ROMEO CALLEJO NOTE: = Callejo Ponente

committed because the accused has his own personal It is true that PD532 introduced some changes to
motive, rebellion and murder would constitute separate Art306 and 307 of the RPC: increase of penalties; PD
offenses. does not require that there be at least 4armed persons
In this case, there is no showing that the accused forming a band of robbers; the presumption that accused
had any motive other than to extort money under the are brigands if they use unlicensed firearms has been
compulsion of threats or intimidation. This was admitted removed in the decree. BUT, one thing has remained
when Isabelo admitted to Mrs. Socorro that he needed unchanged and that is the definition of brigandage in the
money because he had an ulcer and that he tried getting code and in the PD --- acts are committed not against a
advances from the office to no avail. specific victim but against any and potential victim on the
For the crime of kidnapping to exist, the rule is highway.
that there must be an actual intent to deprive the offended Further, it is an absurd argument of the TC that
party of her liberty. This is different from the situation just because the robbery was committed on the highway it
wherein the restraint of freedom was only incidental to the is already covered by the PD. Thus, the crime committed
commission of another offense which was primarily here is simple robbery. Accused have acted in conspiracy
intended by the offenders. Thus, as early as US vs as shown by their acts. Abuse of confidence also applied
Ancheta, it was held that, even if the victims were detained with no mitigating circumstance. No procedural obstacle to
or forcibly taken but the primary and ultimate purpose was convict even if information was kidnapping for ransom
to kill them, the incidental deprivation of liberty does not because simple robbery is necessarily included in
constitute kidnapping or serious illegal detention. In this kidnapping with ransom.
case, the testimonies of the accused show that they had
no intention to deprive Mrs. Socorro of her liberty.
There was no ransom either. Ransom is the FILOTEO VS. SANDIGANBAYAN
money, price or consideration paid or demanded for
redemption of a captured person or payment for release One morning, while the delivery mail van was traversing
from captivity. Here, the complainant readily gave the case McArthur Highway to deliver several mails in the Bulacan
and checks when demanded from her at gun point. These area, an old blue Mercedes Benz overtook their van and
were merely amounts involuntarily surrendered by Mrs. cut across its path. Eventually, armed men took over the
Socorro on the occasion of the robbery. Thus, while the van after they introduced themselves as police officers.
crime committed was indeed robbery, it is not the highway The victims were ordered to stay at the back of the van
robbery under PD532. while it was driven in circles until the van stopped in
Contrary to what the SolGen postulates, PD532 Caloocan. Eventually, petitioner and 10 others were
does not modify Art267 (Kidnapping and serious illegal apprehended and were charged with robbery-in-band
detention). Instead, what it modifies is Art306 and 307 on (hijacking).
Brigandage. This is clear form the fact that under the PD, CA: they are guilty of brigandage and not
highway robbery is synonymously used with brigandage. robbery
This is in fact consistent with the SCs earlier rulings that Accused herein are charged with the violation of
highway robbers and brigands are synonymous. PD 532. Under said decree, with respect to the highway
Brigandage is indiscriminate highway robbery robbery aspect, the offense is committed on a Philippine
(formation of a band by more than 3armed persons for the Highway which under Section 2 (c) thereof has been
purpose of committing robbery in the highway, or defined as any road, street, passage, highway and
kidnapping persons for the purpose of extortion or to bridges or any part thereof, or railway or railroad within the
obtain ransom or for any other purpose to be attained by Philippines, used by persons or vehicles, or locomotives or
means of force and violence, they shall be deemed trains for the movement or circulation of persons or
highway robbers or brigands --- art.306.) whereas, if the transportation of goods, articles or property or both, while
purpose is only a particular robbery, then the crime is only under Section 2 (e) thereof Highway
robbery. Robbery/Brigandage has been defined as the the seizure
Note also that PD532 punishes an act as of any person for ransom, extortion or other unlawful
brigandage or highway robbery only when it is perpetrated purposes or the taking away of property of another by
against any person/s indiscriminately (meaning: they dont means of violence against or intimidation of persons nor
choose who they will rob. Anyone will do. Whereas clause force upon things or other unlawful means, committed by
of PD532 says, committed upon the persons and any person on any Philippine Highway.
properties of innocent and defenceless inhabitants who The offense described in the information and
travel from one place to another) as compared to acts of established by the evidence presented by the prosecution
robbery wherein the victim is predetermined --- as in this properly falls within the ambit of the aforesaid special law.
case. Therein, it was conclusively proven that a postal van

173
CRIMINAL LAW REVIEW DIGESTS
JUSTICE ROMEO CALLEJO NOTE: = Callejo Ponente

containing mail matters, including checks and warrants, From the above, it is clear that a finding of
was hi-jacked along the national highway in Bulacan by brigandage or highway robbery involves not just the locus
the accused, with the attendant use of force, violence and of the crime or the fact that more than three persons
intimidation against the three (3) postal employees who perpetrated it. It is essential to prove that the outlaws
were occupants thereof, resulting in the unlawful taking were purposely organized not just for one act of robbery
and asportation of the entire van and its contents but for several indiscriminate commissions thereof. In the
consisting of mail matters. Also the evidence further present case, there had been no evidence presented that
showed that the crime was committed by the accused who the accused were a band of outlaws organized for the
were PC soldiers, policeman (sic) and private individuals purpose of depredation upon the persons and properties
in conspiracy with their co-accused Castro and Escalada of innocent and defenseless inhabitants who travel from
who were postal employees and who participated in the one place to another. What was duly proven in the
planning of the crime. Accordingly, all the essential present case is one isolated hijacking of a postal van.
requisites to constitute a consummated offense under the There was also no evidence of any previous attempts at
law in point are present. similar robberies by the accused to show the
indiscriminate commission thereof.
Issue: are they guilty of brigandage or robbery?

SC: they are guilty of ROBBERY and not brigandage. ASTORGA V. PEOPLE
The CA labored under the belief that because the taking or
robbery was perpetrated on a national highway (McArthur The Regional Special Operations Group (RSOG) of the
Highway), ergo, PD 532, otherwise known as the Anti- DENR of Tacloban sent a team to the island Daram to
Piracy and Anti-Highway Robbery Law of 1974, must have conduct intelligence gathering and forest protection
been the statute violated. Such reasoning has already operations regarding illegal logging. The team was
been debunked by this Court in the case of People vs. composed of 5 EEs of DENR and escorted by 2
Isabelo Puno, where it was ruled in unmistakable policemen.
language that it takes more than the situs of the robbery to They chanced upon the several yacht-like boats
bring it within the ambit of PD 532. being constructed in barangays in Daram. Astorga was
The following salient distinctions between present at one. When one of the team members (Elpidio
brigandage and robbery are succinctly explained in a Simon) approached Astorga to explain their purpose, he
treatise on the subject and are of continuing validity: was twice slapped hard on the shoulder and the Mayor
The main object of the Brigandage Law is to said in their dialect: I can make you swim back to
prevent the formation of bands of robbers. The heart of Tacloban. Dont you know that I can box? I can box. Dont
the offense consists in the formation of a band by more you know that I can declare this a misencounter? Mayor
than three armed persons for the purpose indicated in art. Astorga then ordered for reinforcements and minutes later,
306. Such formation is sufficient to constitute a violation a banca with 10 men, dressed in fatigue uniforms and
of art. 306. It would not be necessary to show, in a armed with guns, arrived. They surrounded the DENR
prosecution under it, that a member or members of the team and pointed their guns at the team members. Simon
band actually committed robbery or kidnapping or any again tried to explain their purpose and took out a
other purpose attainable by violent means. The crime is handheld radio to contact DENR Catbalogan. Mayor
proven when the organization and purpose of the band are Astorga then forcibly grabbed the radio, and said Its better
shown to be such as are contemplated by art. 306. On the if you have no radio so that your office would not know
other hand, if robbery is committed by a band, whose your whereabouts and so that you cannot ask for help. He
members were not primarily organized for the purpose of again slapped Simon hard and said If you are tough guys
committing robbery or kidnapping, etc., the crime would in Leyte, do not bring it to Samar because I will not
not be brigandage, but only robbery. Simply because tolerate it here. If you really want to confiscate anything,
robbery was committed by a band of more than three you start with the big-time. If you confiscate the boats of
armed persons, it would not follow that it was committed Figueroa I will surrender mine. (Figueroa is also an owner
by a band of brigands. In the Spanish text of art. 306, it is of several boats) When the team asked to leave, he said,
required that the band sala a los campos para dedicarse you cannot go home now because I will bring you to
a robar. Daram. We will have many things to discuss there.
In fine, the purpose of brigandage, is inter alia, The team was brought to a house where they
indiscriminate highway robbery. If the purpose is only a were fed dinner. They were allowed to go around, but not
particular robbery, the crime is only robbery, or robbery in leave the barangay. They were only allowed to leave at
band if there are at least four armed participants. 2am the next day.

174
CRIMINAL LAW REVIEW DIGESTS
JUSTICE ROMEO CALLEJO NOTE: = Callejo Ponente

The team filed a complaint, and the Ombudsman FACTS: On October 12, 1972, an information for Arbitrary
filed an information against Astorga (Mayor of Daram, Detention was filed against Juan Tuvera, Sr., Tomas
Samar) and his men for arbitrary detention. During trial, Mendoza and Rodolfo Mangsat, in the Court of First
the complainants executed an affidavit of desistance. The Instance of Pangasinan. On April 4, 1973, Tuvera filed a
Sandiganbayan still found Astorga guilty of the crime. motion to quash the information on the ground that the
facts charged do not constitute an offense and that the
Issue: Is astorga guilty? YES! proofs adduced at the investigation are not sufficient to
support the filing of the information. The information reads
Elements of arbitrary detention were all met. as follows:
1. Offender is a public officer or EE ... accused Juan Tuvera, Sr., a barrio captain,
2. He detains a person with the aid of some other private persons, namely Juan
3. The detention is without legal grounds Tuvera, Jr., Bertillo Bataoil and one Dianong, maltreated
one Armando Valdez by hitting with butts of their guns and
Clearly there was no legal ground for the detention. In fact fists blows and immediately thereafter, without legal
Astorga admitted that he was only motivated by instinct of grounds, with deliberate intent to deprive said Armando
self-preservation. Valdez of his constitutional liberty, accused Barrio captain
Was there actual detention? Yes. The prevailing Juan Tuvera, Sr., Cpl. Tomas Mendoza and Pat. Rodolfo
jurisprudence on kidnapping and illegal detention is Mangsat, members of the police force of Mangsat,
that the curtailment of the victims liberty need not Pangasinan conspiring, confederating and helping one
involve any physical restraint upon the victims another, did, then and there, willfully, unlawfully and
person. If the acts and actuations of the accused can feloniously, lodge and lock said Armando Valdez inside
produce such fear in the mind of the victim sufficient the municipal jail of Manaoag, Pangasinan for about
to paralyze the latter, to the extent that the victim is eleven (11) hours.
compelled to limit his own actions and movements in Finding that respondent Juan Tuvera, Sr. was not
accordance with the wishes of the accused, then the a public officer who can be charged with Arbitrary
victim is, for all intents and purposes, detained Detention, respondent Judge Salanga granted the motion
against his will. to quash. Tuvera contends that the elements are lacking.
Here, the restraint resulting from fear is evident. The public officers liable for Arbitrary Detention must be
In spite of their pleas, complainants were not allowed to go vested with authority to detain or order the detention of
home. This refusal was followed by the call for persons accused of a crime. Such public officers are the
reinforcements, all armed with military-issue rifles, who policemen and other agents of the law, the judges or
proceeded to encircle the team, weapons pointed at the mayors. He essentially says that he is not a public officer.
38
complainants and the witnesses. Given such It was asserted that if Armando Valdez was ever
circumstances, it was not safe to refuse Mayor Astorgas jailed and detained more than six (6) hours, Tuvera has
orders. It was not just the presence of the armed men, but nothing to do with it because he is not in any way
also the evident effect these gunmen had on the actions of connected with the Police Force of Manaoag, Pangasinan.
the team which proves that fear was indeed instilled in the Granting that it was Tuvera, Sr., who ordered Valdez
minds of the team members, to the extent that they felt arrested, it was not he who detained and jailed him
compelled to stay in the barangay. The intent to prevent because he has no such authority vested in him as a mere
the departure of the complainants and witnesses against Barrio Captain of Barrio Baguinay, Manaoag, Pangasinan.
their will is thus clear.
Re: the Joint Affidavit of Desistance executed by ISSUE: W/N Tuvera, Sr., a barrio captain is a public officer
the complainants, it is merely an additional ground to who can be liable for the crime of Arbitrary Detention?
buttress the defenses of the accused, but not the sole YES, THUS HE CAN BE HELD LIABLE FOR
consideration that can result in acquittal. There must be ARBITRARY DETENTION.
other circumstances which, when coupled with the
retraction or desistance, create doubts as to the truth of HELD: Arbitrary Detention is committed by a public officer
the testimony given by the witnesses at the trial and who, without legal grounds, detains a person. The
accepted by the judge. Here, there are no such elements of this crime are the following: That the offender
circumstances. is a public officer or employee, That he detains a person,
That the detention is without legal grounds.
Long before Presidential Decree 299 was signed
MILO V. SALANGA into law, barrio lieutenants (who were later named barrio
captains and now barangay captains) were recognized as
persons in authority. In various cases, this Court deemed

175
CRIMINAL LAW REVIEW DIGESTS
JUSTICE ROMEO CALLEJO NOTE: = Callejo Ponente

them as persons in authority, and convicted them of


Arbitrary Detention.
One need not be a police officer to be chargeable PARULAN V. DIRECTOR OF PRISONS
with Arbitrary Detention. It is accepted that other public
officers like judges and mayors, who act with abuse of Doctrine: Evasion of service of sentence is a continuing
their functions, may be guilty of this crime. A perusal of crime. As long as the crime subsists, the offender may be
the powers and function vested in mayors would show that arrested without warrant, at any place where he may be
they are similar to those of a barrio captain except that in found. He may also be tried by the courts of that place.
the case of the latter, his territorial jurisdiction is smaller.
Having the same duty of maintaining peace and order, Facts: Ricardo Parulan was serving a sentence of life
both must be and are given the authority to detain or order imprisonment, which was then commuted to 20 years, in
detention. Noteworthy is the fact that even private Muntinlupa. He was transferred to a military barracks in
respondent Tuvera himself admitted that with the aid of his Fort Bonifacio (situated in Makati). He escaped and was
rural police, he as a barrio captain, could have led the recaptured in Manila. As a result, he was prosecuted for
arrest of petitioner Valdez. the crime of evasion of service of sentence. The CFI of
From the foregoing, there is no doubt that a Manila adjudged him guilty.
barrio captain, like private respondent Tuvera, Sr., can be As a defense, Parulan argued that the court had
held liable for Arbitrary Detention. no jurisdiction over his person and over the offense
charged since he escaped from prison in Makati, but was
tried in Manila. He thus filed a petition for habeas corpus.
UMIL VS. RAMOS (Digest Online)
Issue: Did the CFI of Manila have jurisdiction to try
Facts: On 1 February 1988, military agents were Parulans case? NO.
dispatched to the St. Agnes Hospital, Roosevelt Avenue,
Quezon City, to verify a confidential information which was
received by their office, about a "sparrow man" (NPA Ruling: The Rule of Court generally provide that in all
member) who had been admitted to the said hospital with criminal prosecutions, the action shall be instituted and
a gunshot wound. That the wounded man in the said tried in the court of the municipality of province where the
hospital was among the five (5) male "sparrows" who offense was committed or any of the essential ingredients
murdered two (2) Capcom mobile patrols the day before, thereof took place.
or on 31 January 1988 at about 12:00 o'clock noon, before This, however, does not apply to continuing
a road hump along Macanining St., Bagong Barrio, crimes. There are two classes of continuing crimes,
Caloocan City. The wounded man's name was listed by namely: (1) acts material and essential to the crime occur
the hospital management as "Ronnie Javellon," twenty- in one province and some in another, and; (2) crimes
two (22) years old of Block 10, Lot 4, South City Homes, which although all the elements thereof for its
Bian, Laguna however it was disclosed later that the consummation may have occurred in a single place, yet by
true name of the wounded man was Rolando Dural. In reason of the very nature of the offense committed, the
view of this verification, Rolando Dural was transferred to violation of the law is deemed to be continuing. An
the Regional Medical Servicesof the CAPCOM, for example of the first class would include estafa and
security reasons. While confined thereat, he was positively abduction, while the second class would include
identified by the eyewitnesses as the one who murdered kidnapping and illegal detention.
the 2 CAPCOM mobile patrols. In this case, evasion of service of sentence
belongs to the second class. Such act of the escaped
Issue: Whether or Not Rolando was lawfully arrested. prisoner is continuous (or a series of acts), set on foot by a
single impulse and operated by an unintermittent force,
Held: Rolando Dural was arrested for being a member of however long it may be. It may not be validly said that
the NPA, an outlawed subversive organization. after the convict shall have escaped from the place of his
Subversion being a continuing offense, the arrest without confinement the crime is fully consummated, for, as long
warrant is justified as it can be said that he was as he continues to evade the service of his sentence, he is
committing as offense when arrested. The crimes deemed to continue committing the crime, and may be
rebellion, subversion, conspiracy or proposal to commit arrested without warrant, at any place where he may be
such crimes, and crimes or offenses committed in found. Since he was arrested in Manila, he may be tried
furtherance therefore in connection therewith constitute by the CFI of Manila.
direct assaults against the state and are in the nature of
continuing crimes.

176
CRIMINAL LAW REVIEW DIGESTS
JUSTICE ROMEO CALLEJO NOTE: = Callejo Ponente

Soria and Bista vs. Desierto (OMB) Held/Ratio: Nope!


January 31, 2005 An election day or a special holiday, should not
be included in the computation of the period prescribed by
Facts: Soria and Bista were arrested for violating the law for the filing of complaint/information in courts in cases
Omnibus Election Code (election gun ban). Soria was of warrantless arrests, it being a "no-office day." (SC citing
found in possession of a .38 caliber revolver, while Bista Medina vs. Orosco, 125 Phil. 313) Here, while it appears
was found in possession of sub-machine pistol UZI, cal. that the complaints against Soria for Illegal Possession of
9mm and a .22 cal. revolver with ammunition. Here are the Firearm and Violation of COMELEC Resolution No. 3328
pertinent dates: were filed with only on May 15, 2001 at 4:30 p.m., he had
already been released the day before or on May 14, 2001
May 13, 2001 (8:30pm) Soria and Bista were arrested. at about 6:30 p.m. by the respondents. Hence, there was
Take note that this was a Sunday no violation of Article 125 insofar as Soria was concerned.
and that the day following, May 14, In relation to Bista, there was likewise no
was election day. violation of Article 125 because the running of the 36
May 14, 2001 (4:30pm) They were brought to the hours was tolled by one day (election day). Moreover, he
residence of the Provincial has a standing warrant of arrest for Violation of B.P. Blg. 6
Prosecutor where a joint-affidavit and it was only on May 15, 2001, at about 2:00 p.m. that
was executed by the arresting he was able to post bail and secure an Order of Release.
officers. Obviously, however, he could only be released if he has
May 14, 2001 (6:30pm) Soria was released. He was no other pending criminal case requiring his continuous
detained for 22 hours. Bista was detention.
brought back to the police station The Information against Bista was filed with on
because there was a pending case May 15, 2001 but he was released from detention only on
for violation of BP6 (illegal June 8, 2001. Was there a delay in the delivery of
possession of bladed weapons ata detained person to the proper judicial authorities under the
to) against him. circumstances? The answer is in the negative. The
May 15, 2001 (2:00pm) Bista was brough before the court complaint against him was seasonably filed in the court of
where the BP6 case was pending. justice within the 36 hour period prescribed by law.
He posted bail. Remember that he was detained in May 13 and the
May 15, 2001 (4:30pm) An Information for illegal information was filed on May 15. Furthermore, 13 was a
possession of firearms and Sunday and 14 was election day.
ammunition was filed against The duty of the detaining officers is deemed
Bista. complied with upon the filing of the complaints. Citing the
June 8, 2001 Bista was finally released upon case of Agbay, the SC said that upon the filing of the
posting bail. He was detained for a complaint with the proper court, the intent behind Art. 125
total of 26 days. is satisfied considering that by such act, the detained
person is informed of the crime imputed against him and,
The petitioners filed a complaint with the OMB upon his application with the court, he may be released on
against the arresting officers for violating Article 125 of the bail.
Revised Penal Code (Delay in the delivery of detained
persons to the proper judicial authorities). The
respondents argued that Sundays, holidays and election ALBIOR v AUGUIS
days are excluded from the computation of the periods
provided in Article 125. The OMB agreed with the FACTS: Edilberto Albior is the son of the
respondents and dismissed the complaint. complainant/petitioner in this case> is the Clerk of Court of
Take note that for purposes of Article 125, the the MCTC of Talibon Bohol. 2 informations for rape was
penalty imposed by law on Soria was correccional, while filed against Edilberto before the sala of Judge Avelino
the penalty imposed by law on Bista was afflictive or Puracan of which Auguis was the clerk of court who
capital. According to Article 125, detained persons must received the complaints. Auguis immediately issued an
be delivered to judicial authorities within 18 hours for order for the detention of Edilberto. This order was
correccional penalties, and 36 hours for afflictive or capital directed to the BJMP. Edilberto was detained.
penalties. Claiming that his son was illegally detained
because no warrant was issued for his arrest and neither
Issue: W/N Article 125 was violated? was there a preliminary investigation, complainant filed a

177
CRIMINAL LAW REVIEW DIGESTS
JUSTICE ROMEO CALLEJO NOTE: = Callejo Ponente

motion for the release of Edilberto but was ignored by did not have envelopes available. CA ruled that their rights
Auguis. This forced the complainant to sue for Habeas were violated.
Corpus. During the Habeas Corpus trial, Auguis testified
that this was not the first time he ordered the commitment Issue:
of persons but said he did so in the best interest of those 1. Was their right to counsel being violated by the
persons and upon request of the Chief of Police. He regulated visiting hours?
justified his actions by saying that since PNP jails didnt 2. Was their right to privacy being violated by the
have meal provisions for detainees, he ordered their opening of their letters?
commitment to BJMP facilities.
The court, in the Habeas Corpus proceedings, Held:
found the detention illegal and ordered the release of 1. No. Section 4(b) of RA 7438 provides the
Edilberto. On the same day, a preliminary investigation standard to make regulations in detention centers
was conducted and in an Omnibus Motion, confirmed the allowable: such reasonable measures as may be
arrest of the accused. necessary to secure the detainees safety and prevent
Complainant then filed an administrative case his escape. In the present case, the visiting hours
against Auguis for usurpation of judicial functions before accorded to the lawyers of the detainees are reasonably
the Office of the Court Administrator. The OCA found connected to the legitimate purpose of securing the safety
Auguis administratively liable and imposed a fine of 3,000 and preventing the escape of all detainees.
with a warning not to repeat the violations anymore. While petitioner-lawyers may not visit the
detainees any time they want, the fact that the detainees
ISSUE: What crime is Auguii liable for? Delay in the still have face-to-face meetings with their lawyers on a
delivery of detained persons to proper judicial authorities daily basis clearly shows that there is no impairment of
detainees right to counsel. Petitioners as counsels could
HELD: The functions of the clerk of court are clearly visit their clients between 8:00 a.m. and 5:00 p.m. with a
defined in Section 5 Rule 136 of the Rules of Court. lunch break at 12:00 p.m. The visiting hours are regular
Nowhere in that provision does it provide that, absent a business hours, the same hours when lawyers normally
judge, the clerk of court has the power to issue detention entertain clients in their law offices. Clearly, the visiting
orders which is clearly judicial in nature. The good faith hours pass the standard of reasonableness. Moreover, in
defense of Auguis does not excuse his behavior. If a judge urgent cases, petitioners could always seek permission
is not available, the detaining officer must release the from the ISAFP officials to confer with their clients beyond
accused upon the expiration of the maximum period of the visiting hours. The scheduled visiting hours provide
detention allowed in Art. 125 of the RPC. In this case, reasonable access to the detainees, giving petitioners
Edilberto was detained for a period of 56 days from the sufficient time to confer with the detainees.
time he was unlawfully arrested.
2. No. (original went through a litany of US
cases, just read the original if you want to know) The
ALEJANO, ET AL V CABUAY letters alleged to have been read by the ISAFP authorities
(actually a habeas corpus case, but Justice talked about were not confidential letters between the detainees and
the limitation on lawyers visits) their lawyers. The petitioner-lawyer who received the
letters from detainees Trillanes and Maestrecampo was
FACTS: Alejano, Trillanes, etc are all AFP men detained merely acting as the detainees personal courier and not
for their participation in the 2003 Oakwood Mutiny. They as their counsel when he received the letters for mailing.
were charged with coup detat and detained in the ISAFP In the present case, since the letters were not
Detention Center under the command of General Cabuay. confidential communication between the detainees
The detainess claim that their right to counsel and their lawyers, the officials of the ISAFP Detention
was infringed upon because their counsels were only Center could read the letters. If the letters are marked
allowed to meet with them from 8 am 5 pm every day. confidential communication between the detainees
They wanted their counsels to visit them at any time of day and their lawyers, the detention officials should not
or night. read the letters but only open the envelopes for
They also claim that their right to privacy was inspection in the presence of the detainees.
infringed upon because Trillanes and Maestrecampos That a law is required before an executive
private letters were being opened and read by the ISAFP officer could intrude on a citizens privacy rights is a
officials. The letters were not sealed in envelopes (they guarantee that is available only to the public at large but
were merely folded) because the ISAFP Detention Center not to persons who are detained or imprisoned. The right
to privacy of those detained is subject to Section 4 of RA

178
CRIMINAL LAW REVIEW DIGESTS
JUSTICE ROMEO CALLEJO NOTE: = Callejo Ponente

7438, as well as to the limitations inherent in lawful searched that she relinquished possession and control of
detention or imprisonment. By the very fact of their the drug in an effort to protect herself against the
detention, pre-trial detainees and convicted prisoners have consequences of the search. Rather than indicate that
a diminished expectation of privacy rights. anyone else had knowledge of her possession of the drug,
the proofs seem to suggest that it was her effort to keep
Side note: detainees also complained of the knowledge of such possession from every other person,
living conditions of their detention cells because they were including Valeriano and his family.
boarded up and had iron bars. Their cells were dark and The fact that Valeriano refused the officers
had poor ventilation. They also complained that the iron permission to search his house for opium can not be taken
bars restricted their visits to non-contact ones. Court said against him. No public official or other person in any
it was reasonable to put these iron bars to prevent the country has the right to enter the premises of another
escape of the detainees. The the separation of the without his consent for the purpose of search or
detainees from their visitors by iron bars is merely a seizure without first being provided with the proper
limitation on contact visits. The iron bars separating the search warrant for the purpose, obtained in the
detainees from their visitors prevent direct physical contact manner provided by law.
but still allow the detainees to have visual, verbal, non- The warrant is not allowed for the purpose of
verbal and limited physical contact with their visitors. The obtaining evidence of an intended crime; but only after
arrangement is not unduly restrictive. In fact, it is not even lawful evidence of an offense actually committed. Nor
a strict non-contact visitation regulation like in Block v. even then is it allowable to invade one's privacy for the
Rutherford. The limitation on the detainees physical sole purpose of obtaining evidence against him, except in
contacts with visitors is a reasonable, non-punitive a few special cases where that which is the subject of the
response to valid security concerns. crime is supposed to be concealed, and the public or the
complainant has an interest in it for in its destruction.
Those special cases are familiar, and well understood in
US vs. VALERIANO DE LOS REYES and GABRIELA the law. Search-warrants have heretofore been allowed to
ESGUERRA search for stolen goods, for goods supposed to have been
smuggled into the country in violation of the revenue laws,
FACTS: Gabriella Esguerra visited the family of Valeriano for implements of gaming or counterfeiting, for lottery
de los Reyes when certain revenue officials came to tickets or prohibited liquors kept for sale contrary to law,
search for opium. Valeriano refused entry due to the for obscene books and papers kept for sale or circulation,
absence of a search warrant. However, due to their and for powder or other explosive and dangerous material
assertion that they were officers of the law, while not so kept as to endanger the public safety.
consenting, Valeriano offered no physical resistance to The home, therefore, can not be guaranteed as a
their entry and the search for the drug began. Later on, shelter of crime and bad faith, and, for that reason, with
one of the officers saw Gabriella threw a package (with the formalities hereinafter enumerated, the public
morphine) from the kitchen window into the grass behind authorities may enter the house of any citizen in the
the house. There is no direct evidence of any kind following cases:
showing that the accused Valeriano had any knowledge 1. To arrest any person against whom a warrant of arrest
whatever of the fact that the accused Gabriela had has been issued.
possession of the drug. 2. To capture the person of any known criminal, either
TC: by refusing entry, Valeriano had knowledge of the because of his having been caught in flagrante delicto,
drugs in his house. Otherwise, he would have offered no or because there is reasonable ground to believe that
objection to the search. he is guilty, although no warrant for his arrest has been
actually issued.
ISSUE: WON Valeriano can be held liable for the 3. To prevent the consummation of a crime the
morphine? commission of which is being planned of the same or
has already commenced.
RULING: Valeriano acquitted. Gabriella convicted. 4. To search for and seize the effects of the crime or the
TCs ruling not sufficient. The accused Gabriela was only evidence of the commission of the same and of the
a visitor in the house of Valeriano. She had been there but identity of the guilty parties.
a short time. At the time of the search the morphine was 5. To detect and seize all contraband articles which are
found exclusively in her possession and under her control. the subject of state monopolies.
It nowhere appears that any member of the family of 6. For the purpose of attaching property.
Valeriano had the slightest knowledge of its existence. It As a general rule, it may be stated that, in order
was only when the accused herself was about to be to enter a house for any purpose whatever, whether to

179
CRIMINAL LAW REVIEW DIGESTS
JUSTICE ROMEO CALLEJO NOTE: = Callejo Ponente

inspect the same, to arrest a person, or to attach property, health or morals or the separate rights of others. However,
it is necessary to first obtain the consent of the occupant right to enter one's country cannot be arbitrarily deprived.
of the same, as provided in article 6 of the constitution, It would be therefore inappropriate to construe the
and, in his absence, an order of the court will be required limitations to the right to return to ones country in the
for the preliminary inquiry in each case, upon notice to the same context as those pertaining to the liberty of abode
person affected thereby, either immediately or at the most and the right to travel.
within the twenty-four hours after the issuance of said The Bill of rights treats only the liberty of abode
order. and the right to travel, but it is a well considered view that
the right to return may be considered, as a generally
accepted principle of International Law and under our
MARCOS V MANGLAPUS Constitution as part of the law of the land.
The court held that President did not act
Facts: This case involves a petition of mandamus and arbitrarily or with grave abuse of discretion in determining
prohibition asking the court to order the respondents that the return of the Former Pres. Marcos and his family
Secretary of Foreign Affairs, etc. to issue a travel poses a serious threat to national interest and welfare.
documents to former Pres. Marcos and the immediate President Aquino has determined that the destabilization
members of his family and to enjoin the implementation of caused by the return of the Marcoses would wipe away
the President's decision to bar their return to the the gains achieved during the past few years after the
Philippines. Petitioners assert that the right of the Marcos regime.
Marcoses to return in the Philippines is guaranteed by the The return of the Marcoses poses a serious
Bill of Rights, specifically Sections 1 and 6. They threat and therefore prohibiting their return to the
contended that Pres. Aquino is without power to impair the Philippines, the instant petition is hereby DISMISSED.
liberty of abode of the Marcoses because only a court may
do so within the limits prescribed by law. Nor the President From: http://cofferette.blogspot.com/2009/02/marcos-vs-
impair their right to travel because no law has authorized manglapus-177-scra-668-gr-no.html
her to do so.
They further assert that under international law,
their right to return to the Philippines is guaranteed U.S. V. DORR
particularly by the Universal Declaration of Human Rights
and the International Covenant on Civil and Political Facts: Dorr and others were convicted of publishing a
Rights, which has been ratified by the Philippines. scurrilous libel against the Government of the United
States and the Insular Govt of the Philippine Islands.
Issue: Whether or not, in the exercise of the powers Scurrilous libels are punished under Section 8 of Act No.
granted by the constitution, the President (Aquino) may 292 of the Commission:
prohibit the Marcoses from returning to the Philippines. Every person who shall utter
seditious words or speeches, write,
Held: "It must be emphasized that the individual right publish, or circulate scurrilous libels
involved is not the right to travel from the Philippines to against the Government of the United
other countries or within the Philippines. These are what States or the Insular Government of the
the right to travel would normally connote. Essentially, the Philippine Islands, or which tend to
right involved in this case at bar is the right to return to disturb or obstruct any lawful officer in
one's country, a distinct right under international law, executing his office, or which tend to
independent from although related to the right to travel. instigate others to cabal or meet
Thus, the Universal Declaration of Human Rights and the together for unlawful purposes, or which
International Covenant on Civil and Political Rights treat suggest or incite rebellious conspiracies
the right to freedom of movement and abode within the or riots, or which tend to stir up the
territory of a state, the right to leave the country, and the people against the lawful authorities, or
right to enter one's country as separate and distinct rights. to disturb the peace of the community,
What the Declaration speaks of is the "right to freedom of the safety and order of the Government,
movement and residence within the borders of each or who shall knowingly conceal such evil
state". On the other hand, the Covenant guarantees the practices, shall be punished
right to liberty of movement and freedom to choose his [Note: According to my Dictionary scurrilous means,
residence and the right to be free to leave any country, making or spreading claims about someone with the
including his own. Such rights may only be restricted by intention of damaging their reputation. Thus perhaps its
laws protecting the national security, public order, public the same as saying its malicious libels.]

180
CRIMINAL LAW REVIEW DIGESTS
JUSTICE ROMEO CALLEJO NOTE: = Callejo Ponente

Anyway, the alleged libel was an editorial in the sedition but when you merely attack officers of the
issue of the Manila Freedom newspaper. The editorial government its under the normal crime of libel.]
basically complains about the public officers then in
position describing them as notoriously corrupt, rascals,
and men of no personal character. The editorial continued U.S. vs. ARCEO, et al.
to go on complaining about the rotten system by which the
government was being run. [Note: As I gathered from the FACTS: Alejo Tiongson, the victim, lived in his house
decision, perhaps the reason why this libelous editorial together with his wife (Alejandra) and sister-in-law
was made was because Filipinos were then being (Marcela). On the night of the commission of the crime,
appointed to government positions and these foreigners, the accused, one of whom was armed with a gun and the
that includes Dorr who most likely is an American since in other two each with a bolo, entered the victims house
another case he asked for a trial by jury, didnt want a without first getting permission. At that time, the spouses
government being run by Filipinos.] were already sleeping while Marcela was still awake,
Dorr and other defendants werent able to prove sewing. As soon as she discovered the presence of the
any of their allegations in the Trial Court. They were accused, Marcela woke the spouses. Immediately after,
convicted and now appeal. one of the accused wounded Alejo by means of a bolo.
Then they took a certain amount of money that belonged
Issue: Does the publication constitute a scurrilous libel to Alejo and fled the scene. After trial, the court found the
against the Government? accused guilty of the crime of entering the residence of
another against his will and with violence or intimidation.
Held: NO. SC acquits.
ISSUE:
Ruling: First of all, there are many ways in which libel may (1) W/N the trial court was correct in finding the
be committed such as when the libel obstructs an officer in accused guilty of the crime charged? (NOTE: The
the performance of his functions but these were not violence was committed by the accused immediately
present. The Court here focused on the issue above after their entry without the consent of Alejo.)
presented. (2) W/N express prohibition to enter the dwelling is
What does the term Government mean? Does it necessary in order to be guilty of the crime charged?
mean in a general sense the existing laws and institutions
of the Islands, or does it mean the aggregate of the HELD/RATIO:
individuals by whom the government of the Islands is, for
the time being, administered? (1) YES. Art. 491 of the (Spanish) Penal Code states
Government is the institution or aggregate of that: He who shall enter the residence (dwelling
institutions by which an independent society makes house) of another against the will of the tenant thereof
and carries out those rules of action which are shall be punished with the penalty of arresto mayor
necessary to enable men to live in a social state or and a fine of from 325 to 3,259 pesetas. x x x If the
which are imposed upon the people forming that society act shall be executed with violence or intimidation the
by those who possess the power or authority of penalty shall be prision correccional in the medium
prescribing them. Government is the aggregate of and maximum grade, and a fine of from 325 to 3,250
authorities that rule a society it is the whole political pesetas. The SC is of the belief that said provision
system. On the other hand, administration is the does not only relate to the method by which one may
aggregate of those persons in whose hands the reins of enter the residence of another without his consent,
government are for the time being. but also pertains to ones conduct immediately after
Thus, when the attack is made only on the his entry. Thus, a person armed with deadly weapons
individuals holding positions this is not a scurrilous libel who enters the residence of another in the nighttime,
against the Government but rather this is adequately without consent, and immediately commits acts of
covered by the general libel law. This is what the editorial violence and intimidation, is guilty of entering the
merely did. house of another with violence and intimidation and is
On the other hand, when libel attacks the punishable under Art. 491 of the Penal Code.
established system or form of government and its authority
then this constitutes a scurrilous libel. (2) NO. As a rule, the inviolability of the home is one of
the most fundamental of all the individual rights
[Note: I think the modern scurrilous libel is the declared and recognized in the political codes of
crime of sedition; thus when you attack the government its civilized nations. No one can enter into the house of
another without the consent of its owners or

181
CRIMINAL LAW REVIEW DIGESTS
JUSTICE ROMEO CALLEJO NOTE: = Callejo Ponente

occupants. Both the common and the civil law owned/operated by Ernesto Isip for the seizure of the
guaranteed to man the right of absolute protection to following items in violation of RA No. 8203 (IPCode):
the privacy of his home. However, under the police a. Finished or unfinished products of UNILAB,
power of the state the authorities may compel particularly REVICON multivitamins;
entrance to dwelling houses against the will of the b. Other items such as tags, labels, boxes,
owners for sanitary purposes. The government has packages, wrappers, receptacles, advertisements
this right upon grounds of public policy. It has a right and other paraphernalia used for the sale and
to protect the health and lives of all of its people. A distribution of counterfeit REVICON
man can not insist upon the privacy of his home when multivitamins;
a question of the health and life of himself, his family, c. Sales invoices, delivery receipts, official ledgers
and that of the community is involved. This private and other books of accounts used in the
right must be subject to the public welfare. recording of the manufacture and importation,
It may be argued that one who enters the sale of counterfeit REVICON multivitamins.
dwelling house of another is not liable unless he has The application was supported by an affidavit by
been forbidden i.e., the phrase "against the will of Charlie Rabe, security guard of UNILAB who alalegedly
the owner" means that there must have been an saw the manufacture and sale of fake drugs such as
express prohibition to enter. In other words, if one Revicon by Shalimar Philippines. (He was renting a room
enters the dwelling house of another without the in the Shalimar Building).
knowledge of the owner he has not entered against The search warrant was implemented at 4:30 pm
his will. This construction is certainly not tenable, on January 27, 2004 by NBI agents Besarra and
because entrance is forbidden generally under the Divinagracia in coordination with UNILAB employees. No
spirit of the law unless permission to enter is fakfe Revicon multivitamins were found; instead there
expressly given. To allow this construction would were sealed boxes at the first and second floors of the
destroy the very spirit of the law. Under the law no Shalimar Building which when opened by the NBI agents
one has the right to enter the home of another without contained bottles of Disudrin and Inoflox.
the other's express consent. Therefore, to say that Respondents herein filed an Urgent Motion to
one's home is open for the entrance of all who are not Quash the Search Warrant or to Suppress Evidence.
expressly forbidden. This is not the rule. The statute They contended that the implementing officers of the NBI
st nd rd th
must not be given that construction. No one can enter conducted their search at the 1 , 2 , 3 floors and 4
the dwelling house of another, without rendering floors of the building at No. 1524-A, Lacson Avenue, Sta.
himself liable under the law, unless he has the Cruz, Manila, where items in open display were allegedly
express consent of the owner and unless the one found. These premises were different from the address
st nd
seeking entrance comes within some of the described in the search warrant which is the 1 and 2
exceptions dictated by the law or by a sound public floors of the Shalimar Bldg. located at No. 1571, Aragon
policy. St., Stsa. Cruz, Manila. The seizure of the Disudrin and
Inoflox products which were not included in the list of
(N.B. Under the present RPC, if a public officer enters properties to be seized in the search warrant were likewise
any dwelling against the will of the owner, they will be asserted by the respondents.
liable under Art. 128 for Violation of Domicile. The RTC issued an Order sustaining that the
However, if said act was committed by a private seizing officers were only authorized to take possession of
individual, like in this case, the crime committed by finished or unfinished products of UNILAB particularly
the offender will be Trespass to Dwelling under Art. Revicon multivitamins and documents evidencing
280.) counterfeit products. No evidence was shown nor any
was given during the proceedings on the application for
search warrant relative to the seized products. THE
UNITED LABORATORIES V. SHALIMAR SEARCH WARRANT THUS SUFFERED A FATAL
PHILIPPINES/ ERNESTO ISIP INFIRMITY AND CANNOT BE SUSTAINED.
UNILAB filed the present petition for review on
FACTS: NBI Special Investigator III Rolando Besarra filed certiorari under Rule 45.
an application in the RTC of Manila for the issusance of a
search warrant concerning the first and second floors of ISSUES:
Shalimar Building, located at No. 1571, Aragon Street 1. Whether or not the petitioner is the proper party
(formerly No. 1524, Lacson Avenue, Sta. Cruz, Manila) to file the petition at bench;
occupied and/or used by Shalimar Philippines,

182
CRIMINAL LAW REVIEW DIGESTS
JUSTICE ROMEO CALLEJO NOTE: = Callejo Ponente

2. Whether or not it was proper for the petitioner to opposition/comment to UNILAB, through Modesto
file the present petition under Rule 45 of the Alejandro, Jr. The court a quo allowed the appearance of
RoC? UNILAB and accepted the pleadings filed by it and its
3. Whether or not the seized bottles of Disudrin and counsel.
boxes of Inoflox are INADMISSIBLE as evidence The general rule is that the proper party to file a
against the respondents because they constitute petition in the CA or Supreme Court to assail any adverse
the fruit of a poisonous tree or if it is justified order of the RTC in the search warrant proceedings is the
under the PLAIN VIEW DOCTRINE and thus People of the Philippines, through the OSG. However,
legally inadmissible as evidence against in Columbia Pictures Entertainment, Inc. v. Court of
respondents. Appeals, the Court allowed a private corporation (the
complainant in the RTC) to file a petition for certiorari, and
RULING: considered the petition as one filed by the OSG. The
1. We agree with the petitioners contention that a Court in the said case even held that the petitioners
search warrant proceeding is, in no sense, a therein could argue its case in lieu of the OSG.
criminal action or the commencement of a In line with this ruling, the Court gives this petition
prosecution. The proceeding is not one against due course and will allow petitioners to argue their case
any person, but is solely for the discovery and to against the questioned order in lieu of the Solicitor
get possession of personal property. It is a General.
special and peculiar remedy, drastic in nature,
and made necessary because of public 2. The general rule is that a party is mandated to
necessity. It resembles in some respect with follow the hierarchy of courts. However, in exceptional
what is commonly known as John Doe cases, the Court, for compelling reasons or if warranted by
proceedings. While an application for a search the nature of the issues raised, may take cognizance of
warrant is entitled like a criminal action, it does petitions filed directly before it. In this case, the Court has
not make it such an action. opted to take cognizance of the petition, considering the
A search warrant is a legal process which has been nature of the issues raised by the parties.
likened to a writ of discovery employed by the State to The jurisdiction of this Honorable Court is limited
procure relevant evidence of crime. It is in the nature of a to the determination of whether there is a legal basis to
criminal process, restricted to cases of public quash the search warrant and/or to suppress the seized
prosecutions. A search warrant is a police weapon, issued articles in evidence.
under the police power. A search warrant must issue in
the name of the State, namely, the People of the 2. On the validity of the seizure of the sealed boxes
Philippines. and its contents of Disudrin and Inoflox, the
A search warrant has no relation to a civil process. It Court, likewise, rejects the contention of the
is not a process for adjudicating civil rights or maintaining petitioner.
mere private rights. It concerns the public at large as A search warrant, to be valid, must particularly
distinguished from the ordinary civil action involving the describe the place to be searched and the things to be
rights of private persons. It may only be applied for in the seized. The officers of the law are to seize only those
furtherance of public prosecution. things particularly described in the search warrant. A
However, a private individual or a private corporation search warrant is not a sweeping authority empowering a
complaining to the NBI or to a government agency raiding party to undertake a fishing expedition to seize and
charged with the enforcement of special penal laws, such confiscate any and all kinds of evidence or articles relating
as the BFAD, may appear, participate and file pleadings in to a crime. The search is limited in scope so as not to be
the search warrant proceedings to maintain, inter alia, the general or explanatory. Nothing is left to the discretion of
validity of the search warrant issued by the court and the the officer executing the warrant.
admissibility of the properties seized in anticipation of a
criminal case to be filed; such private party may do so in 3. Objects, articles or papers not described in the
collaboration with the NBI or such government agency. warrant but on plain view of the executing officer may
The party may file an opposition to a motion to quash the be seized by him. However, the seizure by the officer
search warrant issued by the court, or a motion for the of objects/articles/papers not described in the warrant
reconsideration of the court order granting such motion to cannot be presumed as plain view. The State must
quash. adduce evidence, testimonial or documentary, to
In this case, UNILAB, in collaboration with the NBI, prove the confluence of the essential requirements for
opposed the respondents motion to quash the search the doctrine to apply, namely: (a) the executing law
warrant. The respondents served copies of their reply and enforcement officer has a prior justification for an

183
CRIMINAL LAW REVIEW DIGESTS
JUSTICE ROMEO CALLEJO NOTE: = Callejo Ponente

initial intrusion or otherwise properly in a position from man of reasonable caution and belief that certain
which he can view a particular order; (b) the officer items may be contrabanded or stolen property or
must discover incriminating evidence inadvertently; useful as evidence of a crime. It does not require
and (c) it must be immediately apparent to the police proof that such belief be correct or more likely
that the items they observe may be evidence of a than true. A practical, non-traditional probability
crime, contraband, or otherwise subject to seizure. In this case, Disudrin and/or Inoflox were not
The doctrine is not an exception to the warrant. It listed in the search warrant issued by the court a quo as
merely serves to supplement the prior justification among the properties to be seized by the NBI agents. The
whether it be a warrant for another object, hot pursuit, warrant specifically authorized the officers only to seize
search as an incident to a lawful arrest or some other counterfeit Revicon multivitamins, finished or unfinished,
legitimate reason for being present, unconnected with and the documents used in recording, manufacture and/or
a search directed against the accused. The doctrine importation, distribution and/or sale, or the offering for
may not be used to extend a general exploratory sale, sale and/or distribution of the said vitamins. The
search from one object to another until something implementing officers failed to find any counterfeit Revicon
incriminating at last emerges. It is recognition of the multivitamins, and instead seized sealed boxes which,
fact that when executing police officers comes across when opened at the place where they were found, turned
immediately incriminating evidence not covered by out to contain Inoflox and Disudrin.
the warrant, they should not be required to close their It was thus incumbent on the NBI agents and the
eyes to it, regardless of whether it is evidence of the petitioner to prove their claim that the items were seized
crime they are investigating or evidence of some based on the plain view doctrine. It is not enough to prove
other crime. It would be needless to require the that the sealed boxes were in the plain view of the NBI
police to obtain another warrant. Under the doctrine, agents; evidence should have been adduced to prove the
there is no invasion of a legitimate expectation of existence of all the essential requirements for the
privacy and there is no search within the meaning of application of the doctrine during the hearing of the
the Constitution. respondents motion to quash, or at the very least, during
The immediate requirement means that the the hearing of the NBI and the petitioners motion for
executing officer can, at the time of discovery of the object reconsideration on April 16, 2004. The immediately
or the facts therein available to him, determine probable apparent aspect, after all, is central to the plain view
cause of the objects incriminating evidence. In other exception relied upon by the petitioner and the NBI.
words, to be immediate, probable cause must be the direct There is no showing that the NBI and the
result of the officers instantaneous sensory perception of petitioner even attempted to adduce such evidence. In
the object. The object is apparent if the executing officer fact, the petitioner and the NBI failed to present any of the
had probable cause to connect the object to criminal NBI agents who executed the warrant, or any of the
activity. The incriminating nature of the evidence petitioners representative who was present at the time of
becomes apparent in the course of the search, without the the enforcement of the warrant to prove that the enforcing
benefit of any unlawful search or seizure. It must be officers discovered the sealed boxes inadvertently, and
apparent at the moment of seizure. that such boxes and their contents were incriminating and
The requirement of inadvertence, on the other immediately apparent. It must be stressed that only the
hand, means that the officer must not have known in NBI agent/agents who enforced the warrant had personal
advance of the location of the evidence and intend to knowledge whether the sealed boxes and their contents
seize it. Discovery is not anticipated. thereof were incriminating and that they were immediately
The immediately apparent test does not require apparent. There is even no showing that the NBI agents
an unduly high degree of certainty as to the knew the contents of the sealed boxes before they were
incriminating character of evidence. It requires opened.
merely that the seizure be presumptively In sum then, the Court finds and so hold that the
reasonable assuming that there is probable petitioner and the NBI failed to prove the essential
cause to associate the property with criminal requirements for the application of the plain view doctrine.
activity; that a nexus exists between a viewed
object and criminal activity.
Incriminating means the furnishing of PEOPLE V. HUA and LEE
evidence as proof of circumstances tending to
prove the guilt of a person. FACTS: Police operatives of the Public Assistance and
Indeed, probable cause is a flexible, Reaction Against Crime (PARAC) received word from their
common sense standard. It merely requires that confidential informant that Peter Chan and Henry Lao, and
the facts available to the officer would warrant a appellants Jogy Lee and Huang Zhen Hua were engaged

184
CRIMINAL LAW REVIEW DIGESTS
JUSTICE ROMEO CALLEJO NOTE: = Callejo Ponente

in illegal drug trafficking. Surveillance operations verified doubt kasi shes a foreigner na nag-visit lang in the
that Lao and appellant Lee were living together as Phils for 4 days pa lang and it wasnt established na in
husband and wife. those 4 days connected siya sa shabu-related
PARAC secured 2 search warrants: activities.
1. for violation of P.D. No. 1866 (illegal possession No regulated drug was found in his person or
of firearms and explosives) inside his room or in his other belongings such as
2. for violation of Rep. Act No. 6425 (dangerous suitcases, etc. Thus, he had no actual or constructive
drugs act) possession of the confiscated "shabu."
First warrant (Medyo unimportant) While no Moreover, it is not disputed that Huang Zhen Hua
persons were found inside, the policemen found two kilos had only been in the country for barely four (4) days at the
of methamphetamine hydrochloride, popularly known as time when he was arrested. The prosecution was unable
shabu, paraphernalia, and machines and tools apparently to show that in these four (4) days Huang Zhen Hua
used for the production of fake credit cards. committed acts which showed that he was in cahoots with
Thereafter, the police operatives received the drug syndicate Henry Lau and Peter Chan. It was not
information that Lao and Chan would be delivering shabu even shown that he was together with Henry Lau and
at the Furama Laser Karaoke Restaurant. They rushed to Peter Chan on any occasion. There is no direct nor
the area and 2 of them approached Chan and Lao, circumstantial evidence, of any culpability. (He was only
introduced themselves, but Chan and Lao fired shots. A staying in the condo of Lee as a guest).
shoot-out ensued where Chan and Lao were shot to death Essential elements of the crime of possession of
Second warrant (ito yung important) The regulated drugs: (a) the accused is found in possession of
policemen then proceeded to Pacific Grand Villa to a regulated drug; (b) the person is not authorized by law or
nd
enforce 2 warrant. The policemen, Pangan and two by duly constituted authorities; and, (c) the accused has
security guards of the Pacific Grand Villa proceeded to the knowledge that the said drug is a regulated drug.
condominium unit. They knocked on the door until This crime is mala prohibita, and, as such,
finally appellant Lee peeped through the window. They criminal intent is not an essential element. However, the
introduced themselves as policemen, but the appellant prosecution must prove that the accused had the intent to
could not understand them as she could not speak possess the drugs. Possession, under the law, includes
English. The policemen allowed Pangan to communicate not only actual possession, but also constructive
with appellant Lee by sign language and pointed their possession. Actual possession exists when the drug is in
uniforms to her to show that they were policemen. The the immediate physical possession or control of the
appellant then opened the door and allowed the accused. On the other hand, constructive possession exits
policemen, Pangan and the security guards into the when the drug is under the dominion and control of the
condominium unit. accused or when he has the right to exercise dominion
They searched the masters bedroom plus and control over the place where it is found. Exclusive
another bedroom where appellant Hua was sleeping. possession or control is not necessary. The accused
The policemen brought the appellants to the cannot avoid conviction if his right to exercise control and
PARAC headquarters. The following articles were found dominion over the place where the contraband is located,
and confiscated by the policemen in the condominium unit: is shared with another. In this case, the prosecution failed
2 Transparent Plastic Bags containing about one to prove that the appellant, at any time, had actual or
Kilo each of white crystalline granules later tested to be constructive possession of the regulated drug found in the
Methamphetamine Hydrochloride or Shabu, a regulated masters bedroom where appellant Lee was sleeping
drug; Transparent Plastic Baby Feeding Bottle containing AS TO LEE (important! Implementation of search
an undetermined quantity of suspected Shabu; 1 Small warrant): Guilty. His contention that the search warrant
Plastic Cannister also containing undetermined amount of was not implemented in accordance with the law was
suspected Shabu; Assorted Pieces of Shabu baseless.
Paraphernalia consisting of Improvised Tooters used for Appellant Lee avers that certain irregularities
sniffing shabu, Improvised Burners used for burning were attendant in the issuance and implementation of
Shabu, aluminum foils, etc. Search Warrant: (a) the policemen who implemented the
Lab test yielded positive result to the test for search warrant failed in their duty to show to her the said
Methamphetamine hydrochloride. Thus, the two were warrant, inform her of their authority and explain their
charged for violation of DDA. presence in the condominium unit; (b) the policemen
Trial court: both guilty gained entry into the condominium unit by force while she
was sleeping; and (c) articles and personal effects owned
RULING: AS TO HUA (not important, for DDA lang): Not by her and Lao were taken and confiscated by the
guilty. Failure to establish her guilt beyond reasonable policemen, although not specified in the search warrant.

185
CRIMINAL LAW REVIEW DIGESTS
JUSTICE ROMEO CALLEJO NOTE: = Callejo Ponente

She also contends that she was a victim of a frame-up justified in resisting, and in the process, may cause injury
because the policemen planted the regulated drug on her even to the life of the officer implementing the warrant for
bed even before they searched the bedroom. She went to which he would not be criminally liable. Also, there is a
the room of appellant Zhen Hua to find out if he was very real possibility that the police serving and
already awake, and when she returned to the bedroom, implementing the search warrant may be misinformed as
she noticed shabu on her bed. to the name or address of the suspect, or to other material
The rule against unreasonable search and affirmations. Innocent citizens should not suffer the shock,
seizure forbids every search that is unreasonable fright, shame or embarrassment attendant upon an
Section 7, Rule 126 of the Revised Rules of unannounced intrusion.
Criminal Procedure provides: Unannounced intrusion into the premises is
SEC. 7. Right to break door or window to effect search. permissible when (a) a party whose premises or is entitled
The officer, if refused admittance to the place of directed to the possession thereof refuses, upon demand, to open
search after giving notice of his purpose and authority, it; (b) when such person in the premises already knew of
may break open any outer or inner door or window of a the identity of the officers and of their authority and
house or any part of a house or anything therein to persons; (c) when the officers are justified in the honest
execute the warrant or liberate himself or any person belief that there is an imminent peril to life or limb; and (d)
lawfully aiding him when unlawfully detained therein. when those in the premises, aware of the presence of
The police officers were obliged to give the someone outside (because, for example, there has been a
appellant notice, show to her their authority, and demand knock at the door), are then engaged in activity which
that they be allowed entry. They may only break open any justifies the officers to believe that an escape or the
outer or inner door or window of a house to execute the destruction of evidence is being attempted. Suspects have
search warrant if, after such notice and demand, such no constitutional right to destroy evidence or dispose of
officers are refused entry to the place of directed search. evidence.
This is known as the "knock and announce" principle. However, the exceptions above are not
The method of entry of an officer into a dwelling and the exclusive or conclusive. At times, without the benefit of
presence or absence of such notice are as important hindsight and ordinarily on the spur of the moment, the
considerations in assessing whether subsequent entry to officer must decide whether or not to make an
63
search and/or arrest is constitutionally reasonable. unannounced intrusion into the premises. Although a
In Gouled v. The United States, it was held that a search and seizure of a dwelling might be constitutionally
lawful entry is the indispensable predicate of a reasonable defective, if the police officers entry was without prior
search. A search would violate the Constitution if the entry announcement, law enforcement interest may also
were illegal, whether accomplished by force, by illegal establish the reasonableness of an unannounced entry.
threat or mere show of force. There is no formula for the determination of
Generally, officers implementing a search reasonableness. Each case is to be decided on its own
warrant must announce their presence, identify facts and circumstances. In determining the lawfulness of
themselves to the accused and to the persons who an unallowed entry and the existence of probable cause,
rightfully have possession of the premises to be searched, the courts are concerned only with what the officers had
and show to them the search warrant to be implemented reason to believe and the time of the entry.
by them and explain to them said warrant in a language or Richards v. Wisconsin: In order to justify a "no-
dialect known to and understood by them. The knock" entry, the police must have a reasonable suspicion
requirement is not a mere procedural formality but is of the that knocking and announcing their presence, under the
essence of the substantial provision which safeguards particular circumstances, would be dangerous or futile, or
individual liberty. No precise form of words is required. It is that it would inhibit the effective investigation of the crime
sufficient that the accused has notice of the officers, their by, for example, allowing the destruction of evidence.
authority and the purpose of the search and the object to Benefield v. State of Florida: what constitutes
be seized. It must be emphasized that the notice breaking includes the lifting of a latch, turning a door knob,
requirement is designed not only for the protection of the unlocking a chain or hasp, removing a prop to or pushing
liberty of the person to be searched or of his property but open a closed door of entrance to the house, even a
also the safety and well-being of the officers serving and closed screen door. However, entry obtained through the
implementing the search warrant. Unless the person to use of deception, accomplished without force is not a
whom the warrant is addressed and whose property is to "breaking" requiring officers to first announce their
be searched is notified of the search warrant and apprised authority and purpose because the reasons behind the
of the authority of the person serving the warrant, he may rule are satisfied there was no real likelihood of violence,
consider the unannounced intrusion into the premises as no unwarranted intrusion or privacy and no damage to the
an unlawful aggression on his property which he will be residence of the accused

186
CRIMINAL LAW REVIEW DIGESTS
JUSTICE ROMEO CALLEJO NOTE: = Callejo Ponente

In this case, we rule that the policemen ISSUE: W/N Calera is guilty of dissolving or
complied with Section 7, Rule 126 of the Revised interrupting a meeting under Art 113. NO!
Rules of Criminal Procedure before entering the
condominium unit. Appellant Lee admitted, when she HELD/RATIO: The provision in Art 131 of the RPC is
testified, that the police officers were accompanied by intended to penalize the act of a public official who shall
Chuang, a Cantonese interpreter, who informed her that prohibit, stop or otherwise interrupt the holding and/or
his companions were police officers and had a search dissolve a peaceful meeting. But in order to be liable
warrant for the premises, and also explained to her that under this provision, it is necessary that the accused be
the officers were going to search the condominium a STRANGER, not a participant, of the meeting that
unit. The appellant was sufficiently aware of the authority has been interrupted and eventually dissolved.
of the policemen, who wore PARAC uniforms, to conduct In this case, the said conference was called by
the search and their purpose. the mayor and one of the officials invited to attend the
same was the appellant Calera (chief of police). So he is
not a stranger to the meeting. He was not only present in
PEOPLE V. CALERA AND CANTELA the meeting but he also took direct part in the said
proceeding.
FACTS: Juan CALINDONG was the municipal mayor of Art 131 is found under the title Crimes Against
Catarman, Samar. He issued a call to all the municipal the Fundamental Laws of the State. Its specific purpose is
officials and employees of the municipality for a to penalize the prohibition, interruption and dissolution of
conference/meeting. The purpose of the meeting was to peaceful meetings by a stranger. This is NOT that kind of
readjust the assignment of official duties of the situation.
employees to assist or harmonize with the newly
established Commonwealth government at that time. So Side issues:
the meeting was held at the municipal hall around 4pm on SC held that Calera cannot he held criminally
November 17, 1945. Among those who attended the liable under Art 144 (disturbance of proceedings) either.
meeting were municipal councilors, the mayor, vice mayor, This is because the conference in question is not a
treasurer the chief of police of the town and his assistants. meeting of the municipal council. It was in fact convened
The meeting was called to order by Mayor by the mayor. (the proceedings contemplated in this article
Calindong who explained the purpose of the conference ata are legislative and quasi-legislative meetings).
and the importance of maintaining the law and public However, the SC found Calera guilty of LIGHT
order. He then called on municipal councilor COERCIONS (art 287) instead. Evidence shows that he
CAMPOSANO to speak before the audience. Councilor was guilty of unjust vexation in causing the commotion.
Camposano asked the mayor whether he would take
responsibility for what he was about to say to the
audience. Then, the chief of police (and accused ENRILE V. JUDGE AMIN
appellant) CALERA stood up, quite agitated, telling
Camposano that the mayor cannot be responsible for Facts: Together with the filing of an information charging
whatever he was going to say. Senator Juan Ponce Enrile as having committed rebellion
Basically, Councilor Camposano was supposed complexed with murder with the Rtc of Quezon City,
to speak about a certain municipal ordinance and Calera government prosecutors filed another information charging
was disrupting him from speaking because he wont sit him for violation of P.d. No. 1829, for obstructing and
down even if ordered to by the mayor. Calera was agitated impeding the Apprehension of ex. Lt. Col. Gringo honasan
because he feels that Camposano was going to criticize by concealing him in his house.
and speak against the police force. As a result, there were Enrile assails this, stating that: The alleged
exchange of words, shouting between the mayor, the harboring or concealing of Col. Honasan in a supposed
councilor and the chief of police. meeting on 1 December 1989 is absorbed in, or is a
Due to the confusion, there was disorder in the component element of, the "complexed" rebellion
municipal hall and the audience rushed out of the hall and presently charged against Sen. Enrile as alleged co-
the meeting was DISSOLVED. The justice of peace conspirator of Col. Honasan on the basis of the same
present in the meeting tried to break up the fight between meeting on 1 December 1989. (it was alleged that
the 3 but failed. SO the meeting was never finished. Now, Honasan and some 100 rebel soldiers attended the mass
the Calera faces charges of violation of Art 131 of the RPC and birthday party held at the residence of the petitioner in
(prohibition interruption, and dissolution of peaceful that evening) .
meetings).

187
CRIMINAL LAW REVIEW DIGESTS
JUSTICE ROMEO CALLEJO NOTE: = Callejo Ponente

Issue: whether or not the petitioner could be separately scheduled to proceed along Espaa Avenue in front of the
charged for violation of PD No. 1829 notwithstanding the UST and going towards Mendiola bridge. Police officers
rebellion case earlier filed against him.(considering that blocked them along Morayta Street and prevented them
such presidential decree is a special law) from proceeding further. They were then forcibly
dispersed, causing injuries on one of them. Three other
Held: petition granted. Judge enjoined from holding further rallyists were arrested.
proceedings against Enrile. Offense mentioned in PD All petitioners assail Batas Pambansa No. 880
1829 abZZZorbed in rebellion! The Public Assembly Act of 1985, some of them in toto
and others only Sections 4, 5, 6, 12, 13(a), and 14(a), as
Ratio: as the Hernandez case states: The rejection of both well as the policy of CPR. They seek to stop violent
options shapes and determines the primary ruling of the dispersals of rallies under the no permit, no rally policy
Court, which that Hernandez remains binding doctrine and the CPR policy.
operating to prohibit the complexing of rebellion with any Petitioners Bayan, et al., contend that BP 880 is
other offense committed on the occasion thereof, either as clearly a violation of the Constitution and the International
a means to its commission or as an unintended effect of Covenant on Civil and Political Rights and other human
an activity that commutes rebellion. rights treaties of which the Philippines is a signatory. They
This doctrine is applicable in the case at bar. If a argue that B.P. No. 880 requires a permit before one can
person can not be charged with the complex crime of stage a public assembly regardless of the presence or
rebellion for the greater penalty to be applied, neither can absence of a clear and present danger. It also curtails the
he be charged separately for two different offenses where choice of venue and is thus repugnant to the freedom of
one is a constitutive or component element or committed expression clause as the time and place of a public
in furtherance of rebellion. assembly form part of the message for which the
The petitioner is now facing charges of rebellion expression is sought.
in conspiracy with the fugitive Col. Gringo Honasan. Petitioners Jess del Prado, et al., in turn, argue
Necessarily, being in conspiracy with Honasan, petitioners that B.P. No. 880 is unconstitutional as it is a curtailment
alleged act of harboring or concealing was for no other of the right to peacefully assemble and petition for redress
purpose but in furtherance of the crime of rebellion thus of grievances because it puts a condition for the valid
constitute a component thereof. it was motivated by the exercise of that right. It also characterizes public
single intent or resolution to commit the crime of rebellion. assemblies without a permit as illegal and penalizes them
In the light of the Hernandez doctrine the and allows their dispersal. Thus, its provisions are not
prosecution's theory must fail. The rationale remains mere regulations but are actually prohibitions. Regarding
the same. All crimes, whether punishable under a the CPR policy, it is void for being an ultra vires act that
special law or general law, which are mere alters the standard of maximum tolerance set forth in B.P.
components or ingredients, or committed in No. 880, aside from being void for being vague and for
furtherance thereof, become absorbed in the crime of lack of publication.
rebellion and can not be isolated and charged as KMU, et al., argue that the Constitution sets no
separate crimes in themselves. limits on the right to assembly and therefore B.P. No. 880
cannot put the prior requirement of securing a permit. And
even assuming that the legislature can set limits to this
BAYAN v. ERMITA right, the limits provided are unreasonable: First, allowing
the Mayor to deny the permit on clear and convincing
FACTS: Rallies of September 20, October 4, 5 and 6, evidence of a clear and present danger is too
2005 is at issue. BAYANs rally was violently dispersed. 26 comprehensive. Second, the five-day requirement to
petitioners were injured, arrested and detained when a apply for a permit is too long as certain events require
peaceful mass action they was preempted and violently instant public assembly, otherwise interest on the issue
dispersed by the police. KMU asserts that the right to would possibly wane. As to the CPR policy, they argue
peaceful assembly, are affected by Batas Pambansa No. that it is preemptive, that the government takes action
880 and the policy of Calibrated Preemptive Response even before the rallyists can perform their act, and that no
(CPR) being followed to implement it. KMU, et al., claim law, ordinance or executive order supports the policy.
that on October 4, 2005, a rally KMU co-sponsored was to Furthermore, it contravenes the maximum tolerance policy
be conducted at the Mendiola bridge but police blocked of B.P. No. 880 and violates the Constitution as it causes
them along C.M. Recto and Lepanto Streets and forcibly a chilling effect on the exercise by the people of the right
dispersed them, causing injuries to several of their to peaceably assemble.
members. They further allege that on October 6, 2005, a ISSUES:
multi-sectoral rally which KMU also co-sponsored was (1) Is BP 880 constitutional YES.

188
CRIMINAL LAW REVIEW DIGESTS
JUSTICE ROMEO CALLEJO NOTE: = Callejo Ponente

(2) Is the CPR policy valid? NO. which case the rally may be peacefully dispersed following
HELD: the procedure of maximum tolerance prescribed by the
(1) B.P. 880 is not an absolute ban of public law.
assemblies but a restriction that simply regulates the time, The Court directed the Secretary of the Interior
place and manner of the assemblies. It refers to all kinds and Local Governments to take all necessary steps for
of public assemblies that would use public places. The the immediate compliance with Section 15 of Batas
reference to lawful cause does not make it content- Pambansa No. 880 through the establishment or
based because assemblies really have to be for lawful designation of at least one suitable freedom park or plaza
causes, otherwise they would not be peaceable and in every city and municipality of the country. After thirty
entitled to protection. Maximum tolerance is for the (30) days from the finality of this Decision, subject to the
protection and benefit of all rallyists and is independent of giving of advance notices, no prior permit shall be required
the content of the expressions in the rally. There is, to exercise the right to peaceably assemble and petition in
likewise, no prior restraint, since the content of the speech the public parks or plazas of a city or municipality that has
is not relevant to the regulation. not yet complied with Section 15 of the law.

(2) The Court ruled that in view of the maximum


tolerance mandated by B.P. No. 880, CPR serves no valid PEOPLE V. ASUNCION (en banc case)
purpose if it means the same thing as maximum tolerance
and is illegal if it means something else. Accordingly, what FACTS: Respondents (Paterna Ruiz, Noli Narca, Fr. Nick
is to be followed is and should be that mandated by the Ruiz, Lydia Narca, Rodolfo Corteza, and Tomas
law itself, namely, maximum tolerance, which means the Dominado) were charged with subversion under R.A.
highest degree of restraint that the military, police and 1700. It was alleged that they were conspiring together,
other peace keeping authorities shall observe during a confederating with and mutually helping one another by
public assembly or in the dispersal of the same. The policy overt acts with the common objective to overthrow the duly
of "calibrated preemptive response" is in consonance with constituted government of the Republic of the Philippines.
the legal definition of "maximum tolerance" under Section They were also members of the Communist Party of the
3 (c) of B.P. Blg. 880, which is the "highest degree of Philippines/National Democratic Front and/or its successor
restraint that the military, police and other peacekeeping or of any subversive association in violation of said law.
authorities shall observe during a public assembly or in the Another information was filed against them for violation of
dispersal of the same." P.D. 1866 (Illegal Possession of Firearms).That they had
It should be emphasized that the policy of unlicensed firearms being used in support and furtherance
maximum tolerance is provided under the same law which of the crime of subversion or rebellion.
requires all pubic assemblies to have a permit, which Respondents argued that the filing of 2 separate
allows the dispersal of rallies without a permit, and which informations for each of the accused violates the rule on
recognizes certain instances when water cannons may be double jeopardy, and that there being only a single
used. This could only mean that "maximum tolerance" is criminal intent, the other offense of illegal possession of
not in conflict with a "no permit, no rally policy" or with the firearms, ammunition and explosives should be absorbed
dispersal and use of water cannons under certain in the charge of violation of R.A. 1700, following the
circumstances for indeed, the maximum amount of doctrine in People v. Hernandez.
tolerance required is dependent on how peaceful or unruly The lower court agreed with the contention and
a mass action is. Our law enforcers should calibrate their held that applying by analogy the doctrine laid down in the
response based on the circumstances on the ground with case of People v. Hernandez (99 Phil. 515), the
the view to preempting the outbreak of violence. possession of firearms, ammunition and explosives to
Furthermore, there is need to address the which all the accused are charged is a constitutive
situation adverted to by petitioners where mayors do not ingredient of the crime of subversion and, hence,
act on applications for a permit and when the police absorbed by the same and cannot be punished
demand a permit and the rallyists could not produce one, separately. Deadly weapons are needed and necessary to
the rally is immediately dispersed. In such a situation, as a generate the kind of force and violence to accomplish the
necessary consequence and part of maximum tolerance, purpose of subversion. The elements of force, violence
rallyists who can show the police an application duly filed and other illegal means mentioned in R.A. 1700 may be
on a given date can, after two days from said date, rally in done with the use of violence, explosives and ammunition
accordance with their application without the need to show or the possession thereof.
a permit, the grant of the permit being then presumed
under the law, and it will be the burden of the authorities to
show that there has been a denial of the application, in

189
CRIMINAL LAW REVIEW DIGESTS
JUSTICE ROMEO CALLEJO NOTE: = Callejo Ponente

ISSUE: Whether the crime of illegal possession of for bail (previous one denied) for his conviction of rebellion
firearms, ammunition and explosives, punishable under complexed with murders, arsons and robberies. The
P.D. 1866 is absorbed by the crime of subversion prosecution said to deny this again because the capital
punishment may be imposed. The defense however
HELD/ RATIO: No. Subversion does not absorb crimes contends that rebellion cannot be complexed with murder,
under P.D. 1866 arson, or robbery. The information states that the
The case of People v. Hernandez and other murders, arsons and robberies allegedly perpetrated by
recent cases on the matter involve the crime of rebellion in the accused as a necessary means to commit the crime
which it cannot be complexed with a violation of common of rebellion, in connection therewith and in furtherance
crimes, since force and violence are already necessary thereof.
ingredients of the same. Applying by analogy rebellion to
subversion because both are political offenses intended to ISSUE: W/N rebellion can be complexed with murder,
destabilize and overthrow the government with the use of arson, or robbery. NO! (deemed abzorbed, este
force, violence or other illegal means is untenable. absorbed) Bail granted.
The Court held that to espouse such theory that
force and violence are the very essence of subversion, RATIO: Under the allegations of the amended
then it loses its distinction from rebellion. Subversion is a information, the murders, arsons and robberies described
crime distinct from that of actual rebellion. The crime of therein are mere ingredients of the crime of rebellion
rebellion is committed by rising publicly and taking up allegedly committed by HERNANDEZ, as means
arms against the Government for any of the purposes necessary for the perpetration of said offense of rebellion
specified in Article 134 of the Revised Penal Code; while and that the crime charged in the amended information is,
the Anti-Subversion Act (Republic Act No. 1700) punishes therefore, simple rebellion, not the complex crime of
affiliation or membership in a subversive organization as rebellion with multiple murder, arsons and robberies.
defined therein. In rebellion, there must be a public Under Article 1346 and 1357, these five (5)
uprising and taking of arms against the Government; classes of acts constitute only one offense, and no more,
whereas, in subversion, mere membership in a subversive and are, altogether, subject to only one penalty. One of
association is sufficient and the taking up of arms by a the means by which rebellion may be committed, in the
member of a subversive organization against the words of said Article 135, is by engaging in war against
Government is but a circumstance which raises the the forces of the government and committing serious
penalty to be imposed upon the offender. violence in the prosecution of said war. These
Furthermore, subversion, like treason, is a crime expressions imply everything that war connotes. Since
against national security, while rebellion is a crime against Article 135 constitute only 1 crime, Article 48 doesnt apply
public order. Rising publicly and taking arms against the since it requires the commission of at least 2 crimes.
Government is the very element of the crime of rebellion.
On the other hand, R.A. 1700 was enacted to outlaw the DISSENT: Montemayor
Communist Party of the Philippines (CPP), other similar The murders, robberies and arsons are not necessary or
associations and its successors because their existence indispensable in the commission of rebellion and so are
and activities constitute a clear, present and grave danger
to national security.
The crime of subversion cannot absorb crimes 6 The crime of rebellion or insurrection is committed by rising
publicly and taking arms against the Government for the purpose
under P.D. 1866. In fact, the legislature provided for 2
of removing from the allegiance to said Government or its laws,
distinct offenses: (1) illegal possession of firearms the territory of the Philippine Islands or any part thereof, of any
qualified by subversion (P.D. 1866) and (2) subversion body of land, naval or other armed forces, or of depriving the
qualified by the taking up of arms against the Government Chief Executive or the Legislature, wholly or partially, of any of
their powers or prerogatives.
(R.A. 1700). 7 any person, merely participating or executing the commands of
The Supreme Court remanded the case to the others in a rebellion shall suffer the penalty of prision mayor in its
lower court for further proceedings and trial. minimum period.
The penalty is increased to prision mayor and a fine not to exceed
P20,000 for any person who promotes, maintains or heads a
rebellion or insurrection or who, while holding any public office or
PEOPLE V. HERNANDEZ employment, takes part therein:chanroblesvirtuallawlibrary
1. engaging in war against the forces of the government,
2. destroying property, or
FACTS: Amado HERNANDEZ5 (member of the CPP and 3. committing serious violence,
President of the Congress of Labor Organizations) re-filed 4. exacting contributions or
5. diverting public funds from the lawful purpose for which they
have been appropriated
5 Future National Artist

190
CRIMINAL LAW REVIEW DIGESTS
JUSTICE ROMEO CALLEJO NOTE: = Callejo Ponente

not ingredients or elements of the latter. When a crime is a Ajibun, two men they suspected were responsible for the
necessary means to commit another, he said, there is a disappearance of two of Kamlons followers. They
complex crime; but when it is indispensable, there is only chanced upon and abducted the two, who claimed they
one crime. From this premise, one can commit rebellion by had no knowledge of such disappearance. They were
rising publicly and taking arms against the government detained overnight. The next day, they were brought to a
without firing a single shot. store in the market place and were made to sit with their
hands tied to the roof. Kamlon fired his automatic carbine
NOTE: Doctrine REVERSED. Rebellion can be complexed at Alling, who died instantly. He ordered one of his
with common crimes now. followers, Ulluh, to cut off the dead mans head. Ulluh
Reason: Article 135 was amended by the brought the head and body to his vinta and dropped these
Republic Act No. 6968 (An Act Punishing the Crime of into the sea. Kamlon spared Ajibun and tried him
Coup Detat). Prior to its amendment by Republic Act No. instead for his alleged participation in the disappearance.
6968, Article 135 punished those who while holding any Kamlon merely fined him and set him free.
public office or employment, take part therein by any of Kamlons version, which the courts did not
these acts: engaging in war against the forces of believe, was that Alling was shot to death not by him but
Government; destroying property; committing serious by some relatives of a woman who, on that occasion, he
violence; exacting contributions, diverting funds for the and Ajibun were attempting to abduct.
lawful purpose for which they have been appropriated. In this petition for appeal before the SC, Kamlon
Since a higher penalty is prescribed for the crime alleged, among others, that the CFI erred in convicting him
of rebellion when any of the specified acts are committed for kidnapping with murder in spite of the fact that the said
in furtherance thereof, said acts are punished as acts of violence were committed in furtherance of sedition
components of rebellion and, therefore, are not to be and therefore absorbed in the latter crime.
treated as distinct crimes. The same acts constitute
distinct crimes when committed on a different occasion Issues: Was the crime of kidnapping with murder
and not in furtherance of rebellion. In short, it was because absorbed in the crime of sedition? NO
Article 135 then punished said acts as components of the
crime of rebellion that precludes the application of Article Ratio: The cited cases of Hernandez and Geronimo are
48 of the Revised Penal Code thereto. In the eyes of the inapplicable since these two cases involved the crime of
law then, said acts constitute only one crime and that is rebellion, not sedition. There is neither law nor
rebellion. jurisprudence that would allow the SC to uphold Kamlons
To reiterate, before Article 135 was amended, a claim.
higher penalty is imposed when the offender engages in The SC adheres to the rule of stare decisis. It
war against the government. "War" connotes anything cannot disregard its ruling in the case of Cabrera where it
which may be carried out in pursuance of war. This implies held that sedition is not the same offense as murder, the
that all acts of war or hostilities like serious violence and former being a crime against public order and the latter
destruction of property committed on occasion and in that against persons. Sedition is a crime directed against
pursuance of rebellion are component crimes of rebellion the existence of the State, the authority of the government,
which is why Article 48 on complex crimes is inapplicable. and the genera public tranquility. Murder is a crime
In amending Article135, the acts which used to be against the lives of individuals. The offenses charged in
component crimes of rebellion, like serious acts of the two informations for sedition and murder are perfectly
violence, have been deleted. These are now distinct distinct in point of law, however nearly they may be
crimes. The legal obstacle for the application of Article 48, connected in point of fact. In the case of Umali, the SC
therefore, has been removed. convicted the accused separately of sedition, multiple
murder, etc.
Conclusion: The common offenses, such as
PEOPLE v. KAMLON murder, are distinct and independent acts separable
from sedition.
Facts: Kamlon was convicted by the CFI of Sulu for being
the leader of a sedition (others were also convicted, but Additional: The SC said that in citing the cases of
not important). In another criminal case, he was Hernandez and Geronimo, Kamlon missed a very
sentenced to death for the kidnapping of Alling and Ajibun significant point. In those two cases, murder and other
complexed with the murder of Alling. acts of violence were absorbed by rebellion, the common
Heres what happened (version which the CFI crimes alleged to have been committed in furtherance of
believed): Two years prior to the trial, Kamlon together rebellion were specifically charged in the information. For
with three armed companions set out to look for Alling and this reason, they were necessarily alleged to have been

191
CRIMINAL LAW REVIEW DIGESTS
JUSTICE ROMEO CALLEJO NOTE: = Callejo Ponente

committed for political ends. In the case at bar, the disaffection among the people and a state of feeling
information made no allegation of political motivation. The incompatible with a disposition to remain loyal to the
evidence showed that the killing had no political or social Government and obedient to the laws.
color, but was purely motivated by personal vengeance. In the words of the law, Perez has uttered
seditious words. He has made a statement and done an
act which tended to instigate others to cabal or meet
PEOPLE v. PEREZ together for unlawful purposes. He has made a statement
FACTS: Leonard Wood was the Governor-General of the and done an act which suggested and incited rebellious
Philippines. One time, while holding a discussion with conspiracies. He has made a statement and done an act
several persons on political matters, including the which tended to stir up the people against the lawful
administration of Governor-General Wood, accused authorities. He has made a statement and done an act
Perez, shouted a number of times: "The Filipinos, like which tended to disturb the peace of the community and
myself, must use bolos for cutting off Wood's head for the safety or order of the Government. All of these various
having recommended a bad thing for the Filipinos, for he tendencies can be ascribed to the action of Perez and
has killed our independence." may be characterized as penalized by Act No. 292.

ISSUE: Is accused Perez liable of any crime? (Yes! He


violated Act No. 292, the Treason and Sedition Law) PEOPLE V. RECTO

HELD: In criminal law, there are a variety of offenses FACTS:


which are not directed primarily against individuals, but - There was a report about rice being stolen from a
rather against the existence of the State, the authority of bodega. The barangay captain and a barangay
the Government, or the general public peace. The kagawad went to the bodega to investigate. The chief
offenses created and defined in Act No. 292 are distinctly barangay tanod passed by and asked what they
of this character. Among them is sedition, which is the were doing in the bodega.
raising of commotions or disturbances in the State. It is a - The appellant and his group arrived and was begged
revolt against legitimate authority. Though the ultimate by the barangay captain not to start trouble. Despite
object of sedition is a violation of the public peace or at this, the appellant brought out a balisong which made
least such a course of measures as evidently engenders the barangay captain retreat. The barangay kagawad
it, yet it does not aim at direct and open violence against approached and asked the appellant and his group to
the laws, or the subversion of the Constitution. surrender their weapons (may parang baril pa silang
It is of course fundamentally true that the dala) but the latter shot him instead. At this time, the
provisions of Act No. 292 must not be interpreted so as to chief tanod was hiding in an old kubeta, where he saw
abridge the freedom of speech and the right of the people appellants group kill the kagawad.
peaceably to assemble and petition the Government for - The chief tanod and the barangay captain jumped out
redress of grievances. Criticism is permitted to penetrate from the window of the kubeta and ran but the former
even to the foundations of Government. Criticism, no was shot on his thigh while the latter was shot on his
matter how severe, on the Executive, the Legislature, and elbow. Still, they were able to escape.
the Judiciary, is within the range of liberty of - Several cases were filed against Recto and his group,
speech, unless the intention and effect be seditious. But one of which concerned the injury inflicted upon the
when the intention and effect of the act is seditious, the chief tanod (which I think is the one important for the
constitutional guaranties of freedom of speech and press class). Based on that, appellant was charged with
and of assembly and petition must yield to punitive QUALIFIED DIRECT ASSAULT.
measures designed to maintain the prestige of constituted
authority, the supremacy of the constitution and the laws, ISSUE: W/N appellant should be charged with qualified
and the existence of the State. direct assault for the injury of the chief tanod.
Here, the person maligned by the accused is the
Chief Executive of the Philippine Islands. His official HELD/RATIO: NO.
position, like the Presidency of the United States and other Direct assault, a crime against public order, may be
high offices, under a democratic form of government, committed in two ways: first, by any person or persons
instead, of affording immunity from promiscuous comment, who, without a public uprising, shall employ force or
seems rather to invite abusive attacks. But in this instance, intimidation for the attainment of any of the purposes
the attack on the Governor-General passes the furthest enumerated in defining the crimes of rebellion and
bounds of free speech was intended. There is a seditious sedition; and second, by any person or persons who,
tendency in the words used, which could easily produce without a public uprising, shall attack, employ force,

192
CRIMINAL LAW REVIEW DIGESTS
JUSTICE ROMEO CALLEJO NOTE: = Callejo Ponente

or seriously intimidate or resist any person in are a double crosser. One who cannot keep his promise.
authority or any of his agents, while engaged in the Justo then grabbed a lead paper weight from the table of
performance of official duties, or on occasion of such Caridad and challenged De La Cuesta to go out.
performance. The first mode is tantamount to rebellion or Justo left Caridads office, followed by De la
sedition, without the element of public uprising. The Cuesta. When they were in front of the table of one Carlos
second mode, on the other hand, is the more common Bueno, a clerk in the division office, De la Cuesta asked
form of assault, and is aggravated when: (a) the Justo to put down the paper weight, but instead
assault is committed with a weapon, or (b) when the Justo grabbed the neck and collar of De La Cuestas polo
offender is a public officer or employee, or (c) when shirt and it was torn. Carlos Bueno separated them, but
the offender lays a hand upon a person in authority. not before De La Cuesta had boxed Justo several times.
An agent of a person in authority is any person
who, by direct provision of law or by election or by The CF found Justo guilty of the crime of assault
appointment by competent authority, is charged with the upon a person in authority. The CA affirmed.
maintenance of public order and the protection and
security of life and property, such as barrio councilman, Issue: Whether or not there was still direct assault
barrio policeman and barangay leader, and any person considering De La Cuesta agreed to fight.
who comes to the aid of persons in authority. In the case
at bar, the barangay chief tanod of Ambulong, Held: Yes! Even if at the time of the assault the officer
Magdiwang, Romblon -- was clearly an agent of a was not performing his duties, as long as the attack was
person in authority. However, contrary to the findings by reason of his official duties, or past official duties, there
of the trial court, he was not engaged in the is direct assault. (This sentence is from an online Callejo
performance of his official duties at the time he was Reviewer made by 4A-2009)
shot. Neither was he attacked on the occasion of such
performance. Ratio: The character of person in authority is not
It must be emphasized that the chief tanod was assumed or laid off at will, but attaches to a public
on his way home when he happened to pass by official until he ceases to be in office. Assuming that De
the bodega. During trial, the chief tanod explained that La Cuesta was not actually performing the duties of his
when appellants group arrived, it was the Barangay office when assaulted, this fact does not bar the existence
Captain and the Kagawad who talked to the group. of the crime of assault upon a person in authority, so long
Melchor did not do anything to avert the tension. He only as the impelling motive of the attack is the
watched what was transpiring and later hid himself when performance of official duty. This is apparent from the
the first shot was fired. phraseology of Article 148 of our Revised Penal Code, in
Unquestionably, he was a barangay penalizing attacks upon person in authority while
chief tanod; however, at the crime scene he was a engaged in the performance of official duties or on
mere bystander. Apparently, he was not acting and occasion of such performance, the words on occasion
had no occasion to act in the performance of his signifying because or by reason of the past
official duties that afternoon. Thus, the attack on him performance of official duty, even if at the very time of the
did not amount to direct assault. assault no official duty was being discharged.
The evident purpose of the law is that public
officials and their agents should be able to discharge their
JUSTO V. COURT OF APPEALS official duties without being haunted by the fear of being
assaulted or injured by reason thereof.
Facts: Nemesio B. de la Cuesta was a district supervisor The argument that De la Cuesta, cannot claim to
of the Bureau of Public Schools, stationed in Ilocos Norte. have been unlawfully attacked because he had accepted
On the morning of Oct. 16, 1950, he was in the division the Justos challenge to fight, overlooks the circumstance
office in Laoag, Ilocos Norte. At 11:25 am., as he was that as found by the CA, the challenge was to go out,
leaving to eat, he saw Severino P. Justo talking with i.e., to fight outside the building, it not being logical that the
Severino Caridad, the academis supervisor. fight should be held inside the office building in the plain
Justo asked De la Cuesta to go with him and view of subordinate employees. Even applying the rules in
Caridad to the office of the latter. They did and in the office duelling cases, it is manifest that an aggression ahead of
of Caridad, Justo asked about the possibility of the stipulated time and place for the encounter would be
accommodating Miss Racela as a teacher in the district of unlawful; to hold otherwise would be to sanction
De la Cuesta. Caridad said that there was no vacancy, unexpected assaults contrary to all sense of loyalty and
except that of the position of shop teacher. Upon hearing fair play. In the present case, assuming that De la Cuesta
Caridads answer, Justo said to De La Cuesta: cShet, you accepted the challenge of the accused, the facts clearly

193
CRIMINAL LAW REVIEW DIGESTS
JUSTICE ROMEO CALLEJO NOTE: = Callejo Ponente

indicate that he was merely on his way out to fight the HELD: No. To decide this question, it is first necessary to
accused when the latter violently lay hands upon him. The determine whether Cruz was authorized to search the
acceptance of the challenge did not place on him the person of the accused. We are of the opinion that after the
burden of preparing to meet an assault at any time even customs authorities have permitted the accused to land in
before reaching the appointed place for the agreed Manila, the terminus of his voyage, he ceased to be a
encounter, and any such aggression was patently illegal. passenger within the meaning of said section 1338 of the
Administrative Code. The fact that the accused returned to
pier No. 1 to get the baggage that he had left there the day
PEOPLE VS RELLIN before does not subject him to the operation of said
section.
Doctrine: In all forms of assault, resistance or The Jones Law provides That the right to be
disobedience, it is required that (a) the accused knew the secured against unreasonable searches and seizures
identity of the victim and (b) the victim was then acting in shall not be violated. That foreigners in the Philippines
the due and lawful performance of his duties, or the are entitled to the benefits of the individual rights secured
reason for the attack against him was his performance of by the Philippine Bill is undeniable. It was too late to look
such official duties for any contraband.
Commenting on the meaning and score of
resistance and disobedience, as elements of the crimes
against public authority and its agents, Groizard, among
PEOPLE vs. CHAN FOOK.
other things, says:
A person in authority, his agent or a public
FACTS: Chan Fook (accused/appelant), a Chinese subject,
officer who exceeds his power can not be said to
was a passenger of the US Military Transport South Bend,
be in the exercise of the functions of his office.
which arrived in Manila on April 6, 1920. Having been
The law that defines and establishes his powers
allowed by the immigration authorities to land, he left the
does not protect him for anything that has not
boat on the same day. At about 3 or 4 pm of the following
been provided for.
day, he went to pier no. 1 to get his baggage. After the
The scope of the respective powers of
search of the baggage, postcards of an indecent character
public officers and their agents is fixed, If they go
were found. Eugenio M. Cruz, a custom agent, attempted
beyond, it and they violate any recognized rights
to search the accused to which the Chan Fook objected.
of the citizens, then the latter may resist the
The agent seized Chan Fook by the arm with intent to
invasion, specially when it is clear and manifest.
search his body, after showing him his police badge. The
The resistance must be coextensive with the
accused resisted and struck the secret agent on the
excess, and should not be greater than what is
stomach. The latter in turn struck him on the neck. Here
necessary to repel the aggression.
the customs inspector, Anastacio Jacinto, intervened, and
The invasion of the prerrogatives or rights
explained to the accused that Cruz was a customs secret
of another and the excess in the functions of an
service agent and had the right to search him. Then the
office, are the sources that make for legitimate
appellant made no further resistance and allowed himself
resistance, especially, in so far as it is necessary
to be searched.
for the defense of the persons or their rights in
Appellant Chan Fook was prosecuted for the
the manner provided for in article 8 of the Penal
crime of resistance and disobedience to the public
Code. (3 Groizard, pp. 456, et seq.)
authority. The prosecution alleges that under section 1338
In the case at bar the action of the accused in
of the Administrative Code all persons coming into the
laying his hands on the agent Cruz is, in our opinion, an
Philippine from foreign countries shall be liable to
adequate defense to repel the aggression of the latter,
detention and search by the customs authorities under
who had seized him by the arm for the purpose of
such regulations as may be prescribed relative thereto.
searching him. In accordance with the repeated decisions
The defense, however, contends that once the accused
of the supreme court of Spain, the gravity of a
has arrived at the point of his destination by being allowed
disobedience to an order of a person in public authority is
to leave the boat and to land he was beyond the
measured and graded by the circumstances surrounding
jurisdiction of the customs authorities, and, therefore, not
the act, the motives prompting it, and the real importance
liable to search without judicial warrant.
of the transgression rather than by the source of the order
disobeyed. And, taking into consideration the
ISSUE: Whether the accused committed the crime of
circumstances of the present case, wherein the agent
resistance and disobedience to the public authority.
Cruz had exceeded his functions, and wherein the
accused acted in defense of the most highly esteemed of

194
CRIMINAL LAW REVIEW DIGESTS
JUSTICE ROMEO CALLEJO NOTE: = Callejo Ponente

individual rights the constitutional right to be secured Article 250 says: The penalty for assaults falling
against unreasonable searches we are of the opinion within the next preceding article shall be . . . when the
that there is no ground for finding the accused guilty of the offense is committed under any of the following
crime defined in article 252 of the Penal Code. circumstances:
3. When the offenders lay hands upon any
person in authority.
US V. GUMBAN According to the above provisions of law, in order
that the crime of assault punishable by these articles may
FACTS: Accused is Nicomedes Gumban. He is charged exist, it is sufficient that there be an assault upon a person
with the crime of assault upon agents of authority. in authority committed by laying hands upon him. The fact
On Aug 13, 1917, Petronilo Gumban is the of giving a slap to a person in authority should necessarily
municipal president of Jaro, Iloilo. He was with municipal be qualified in the sense of laying hands upon the same
councilor Magdaleno Suliano who was reporting about the person.
condition of his animals. Suddenly, Gregorio Ismana, a The facts proved in this case involve all the
tenant of councilor Suliano arrived and reported to Mayor necessary elements that constitute the crime of assault,
P. Gumban an incident. inasmuch as the offended party, being a municipal
Ismana related that he had surprised a carabao president, was a person in authority, and was in the
belonging to Policarpio Gumban and as a result, the performance of his official duties.
carabao destroyed the planted area belonging to councilor In the present case, the crime involved is that of
Suliano. So Ismana seized the said carabao and brought it assault upon a person in authority, in which the force
to the police station in the barrio, which was within the necessary to constitute this crime is specifically defined by
zone affected by the quarantine. the law and consists in laying hands upon the person. In
Thereafter, Epifanio Gumban and Nocomedes this case, it is not necessary to ascertain what force the
Gumban (accused), who were brothers Policarpio, of the law requires in order to constitute an assault, since the law
owner of the carabao, arrived to where the municipal itself defines concretely this force in providing that it
president Petronilo Gumban was to protest the taking of consists in laying hands upon the person. The law simply
their carabaos. After hearing the protests Petronilo mentions the laying hands without making any distinction
(municipal president) said that in his opinion, Ismala had as to the different cases, and it would not be just to make
the right to take the carabao to the police station. But he that distinction when the law does not make it. It is to be
promised that the following day, he was going to intervene noted that the same provision of the law with regard to
in the matter and telephone the man in charge of the intimidation or resistance is not intended to be applied to
quarantine so that the said carabao would not be co- the case of laying hands.
mingled with the other carabaos in quarantine. Upon The information qualifies the crime charged as an
hearing this statement of the president, the accused assault upon an agent of authority. Inasmuch as the
insulted the said president and gave him a slap on the offended party, as municipal president, is a person in
face which struck his left ear. authority and not a mere agent of authority, the
TC convicted him of assault upon an AGENT of designation of the crime given by the fiscal is erroneous.
authority.

ISSUE: W/N Nicomedes Gumban is guilty of direct PEOPLE VS LADJAALAM


assault upon an AGENT of authority or did the TC (the case is really long but the relevant part is only 2
mean PERSON IN AUTHORITY. paragraphs short)

HELD/RATIO: The facts proved at the trial constitute the A search warrant was obtained to search the house of
crime of assault with the hands upon a person in authority Ladjaalam because there was information that the same
as defined in paragraph 2 of article 249 in connection with was being used as a drug den. After the warrant has been
paragraph 3 of article 250 of the Penal Code. issued, 30 police officers went to the house of Ladjaalam
The offense of assault (atentado) is committed but a few meters before reaching the house, Ladjaalam
by: was already informed about the raid so when the police
2. Any person who shall attack, employ force officers got to the front of the house, Ladjaalam started
against, or seriously resist or intimidate, any firing his M-14 Armalite at the police officers. Eventually,
person in authority, or the agents of such person, he was arrested together with other suspects for firing at
while engaged in the performance of his official the police officers. Upon the search, several foils of
duties, or by reason of such performance. shabu, M-14 armalites and magazines, among others,
were discovered.

195
CRIMINAL LAW REVIEW DIGESTS
JUSTICE ROMEO CALLEJO NOTE: = Callejo Ponente

LC: search warrant was void because it was issued for Escano was still alive, he fired at the latters chest while he
more than one offense BUT since accused open fired at was sprawled face down.
the police officers, the subsequent arrest and search Hindi pa nakuntento, he locked the room where
became valid Escano was, and entered the faculty room.he found some
teachers and students and ordered them to lock doors and
Issue: w/n accused is guilty of direct assault with multiple close windows, holding them as hostages. Buti na lang the
counts of attempted homicide police came with his family who pleaded to give himself
up, which he did.
SC: YES He was convicted of qualified illegal possession
The trial court was correct in convicting appellant of direct of firearm and ammunition and murder.
assault with multiple counts of attempted homicide. It
found that the act of the accused of firing an M-14 rifle at Issue: guilty? Yez!
the police officers, who were about to enter his house to Was crime committed in contempt of or with insult to
serve a search warrant constituted such complex crime. public authorities?
We note that direct assault with the use of a weapon
carries the penalty of prision correccional in its medium (There were issues with regard to self-defense, double
and maximum periods, while attempted homicide carries jeopardy and certain aggravating circumstances here. Im
the penalty of prision correccional. Hence, for the present skipping them and discussing only the part related to the
complex crime, the penalty for direct assault, which HW.)
constitutes the most serious crime, should be imposed
and applied in its maximum period. The trial court held that since the crime was committed in
front of teachers, then it was done in contempt or with
insult to public authorities because RA 1978 provides that
PEOPLE V. RENATO TAC-AN a public school teacher is a person in authority.

Tac-An was 18 years and 7 months, while his victim, SC: The trial court erred in finding such aggravating
rd circumstance. Art. 152 of the RPC as amended defines
Escano was 15. They were then classmates in 3 year
HS. They were good friends, both being members of the who a person in authority is.
Bronx Gang. Tac-An had been to Escanos house a few A careful reading of the last paragraph of Article
times, and the latters mother noticed that Tac-An carried 152 will show that while a teacher or professor of a public
a handgun. Escanos mom told her son to stay away from or recognized private school is deemed to be a "person in
Tac-An, so Escano withdrew from the Bronx Gang. This authority," such teacher or professor is so deemed only
caused their relations to turn sour. Tac-An and Escano for purposes of application of Articles 148 (direct
quarreled with each other, they were sent to the principals assault upon a person in authority), and 151
office. Their fights worsened after that. (resistance and disobedience to a person in authority
One day, when Tac-An left his math project on or the agents of such person) of the Revised Penal
his chair because he had to ask their teacher something, Code. In marked contrast, the first paragraph of Article
Escano sat on the project. Angered by what he saw, Tac- 152 does not identify specific articles of the Revised
An kicked Escano out of the chair. A fist fight would have Penal Code for the application of which any person
ensued if not for the timely intervention of 2 teachers. "directly vested with jurisdiction, etc." is deemed "a
Then their class continued, at which point Tac-An went person in authority." Because a penal statute is not to be
home and got his got. He got back to the classroom given a longer reach and broader scope than is called for
15mins later. by the ordinary meaning of the ordinary words used by
When their math class started, Tac-An suddenly such statute, to the disadvantage of an accused, we do
burst in the room and fired at Escano. He missed and hit a not believe that a teacher or professor of a public or
desk instead. The students rushed towards the teacher for recognized private school may be regarded as a "public
protection. Tac-An fired another shot and hit the authority" within the meaning of paragraph 2 of Article 14
31
th
blackboard. Third time hit the concrete wall. The 4 shot of the Revised Penal Code, the provision the trial court
hit Escano who was on his way to the only door of the applied in the case at bar.
room to escape. He was hit on the head and fell bleeding. Tac-An is guilty, but sentence was changed.
Tac-An left the room.
Outside, Tac-An was spotted by another teacher,
who had no idea that he caused the commotion. That SENATE V. ERMITA
teacher asked Tac-An to help Escano since he was still (Note that this is a Consti case, so the Crim Law aspect of
alive. Tac-An reentered the room and upon confirming that this case was not thoroughly discussed, not even a single

196
CRIMINAL LAW REVIEW DIGESTS
JUSTICE ROMEO CALLEJO NOTE: = Callejo Ponente

mention of the RPC. Hehe. So please just read in relation HELD: The testimony of the person summoned must be
to Art. 150 of the RPC which punishes, among others, the upon matters, into which the National Assembly/Congress
refusal to answer any legal inquiry, or to produce books, has jurisdiction to inquire. (FROM REYES BOOK 2, 2008.
papers, documents or records in his possession, when THIS IS NOT FROM THE CASE.)
required by them to do so in exercise of their functions.) The Congress power of inquiry is expressly
recognized in Section 21 of Article VI of the Constitution
FACTS: On September 21 to 23, 2005, the Committee of which reads:
the Senate as a whole issued invitations to various SECTION 21.The Senate or the House of
officials of the Executive Department for them to appear Representatives or any of its respective
on September 29, 2005 as resource speakers in a public committees may conduct inquiries in aid of
hearing on the railway project of the North Luzon Railways legislation in accordance with its duly published
Corporation with the China National Machinery and rules of procedure. The rights of persons
Equipment Group (hereinafter North Rail Project). The appearing in or affected by such inquiries shall be
public hearing was sparked by a privilege speech of respected.
Senator Juan Ponce Enrile urging the Senate to Citing Arnault v. Narazeno, the Court said: That
investigate the alleged overpricing and other unlawful this power of inquiry is broad enough to cover officials of
provisions of the contract covering the North Rail Project. the executive branch may be deduced from the same
On September 28, 2005, the President issued case. The power of inquiry, the Court therein ruled, is co-
E.O. 464, ENSURING OBSERVANCE OF THE extensive with the power to legislate. The matters which
PRINCIPLE OF SEPARATION OF POWERS, may be a proper subject of legislation and those which
ADHERENCE TO THE RULE ON EXECUTIVE may be a proper subject of investigation are one. It
PRIVILEGE AND RESPECT FOR THE RIGHTS OF follows that the operation of government, being a
PUBLIC OFFICIALS APPEARING IN LEGISLATIVE legitimate subject for legislation, is a proper subject for
INQUIRIES IN AID OF LEGISLATION UNDER THE investigation.
CONSTITUTION, AND FOR OTHER PURPOSES, which, Since Congress has authority to inquire into the
pursuant to Section 6 thereof, took effect immediately. operations of the executive branch, it would be
Also on September 28, 2005, Senate President incongruous to hold that the power of inquiry does not
Drilon received from Executive Secretary Ermita a copy of extend to executive officials who are the most familiar with
E.O. 464, and another letter informing him that officials of and informed on executive operations.
the Executive Department invited to appear at the meeting As discussed in Arnault, the power of inquiry,
[regarding the NorthRail project] will not be able to attend with process to enforce it, is grounded on the necessity
the same without the consent of the President, pursuant to of information in the legislative process. If the information
[E.O. 464] and that said officials have not secured the possessed by executive officials on the operation of their
required consent from the President. On even date which offices is necessary for wise legislation on that subject, by
was also the scheduled date of the hearing on the alleged parity of reasoning, Congress has the right to that
wiretapping, Gen. Senga sent a letter to Senator Biazon, information and the power to compel the disclosure
Chairperson of the Committee on National Defense and thereof.
Security, informing him that per instruction of [President In fine, the oversight function of Congress may
Arroyo], thru the Secretary of National Defense, no officer be facilitated by compulsory process only to the extent that
of the [AFP] is authorized to appear before any Senate or it is performed in pursuit of legislation. This is consistent
Congressional hearings without seeking a written approval with the intent discerned from the deliberations of the
from the President and that no approval has been Constitutional Commission.
granted by the President to any AFP officer to appear Ultimately, the power of Congress to compel the
before the public hearing of the Senate Committee on appearance of executive officials under Section 21 and the
National Defense and Security scheduled [on] 28 lack of it under Section 22 (of the Art. VI of the 1987
September 2005. Constitution) find their basis in the principle of separation
of powers. While the executive branch is a co-equal
ISSUE: W/N the Congress had the power to compel their branch of the legislature, it cannot frustrate the power of
attendance? YES, AS LONG AS IT IS IN AID OF Congress to legislate by refusing to comply with its
LEGISLATION. (Remember that ultimately, in this case, demands for information.
the SC nullified Sections 2(b) and 3 of EO 464 which When Congress exercises its power of
provide who are covered by executive privilege and saying inquiry, the only way for department heads to exempt
that such officials must obtain the consent of the President themselves therefrom is by a valid claim of privilege.
prior to appearing in either House of Congress. ) They are not exempt by the mere fact that they are
department heads. Only one executive official may be

197
CRIMINAL LAW REVIEW DIGESTS
JUSTICE ROMEO CALLEJO NOTE: = Callejo Ponente

exempted from this power the President on whom was there any allegation to show how he made use of his
executive power is vested, hence, beyond the reach of position as mayor to facilitate the commission of the
Congress except through the power of impeachment. It is crimes charged. The information merely alleges that
based on her being the highest official of the executive petitioner falsified the disbursement voucher by
branch, and the due respect accorded to a co-equal counterfeiting therein the signature of Mejorada. For the
branch of government which is sanctioned by a long- purpose of determining jurisdiction, it is this allegation that
standing custom. is controlling, not the evidence presented by the
prosecution during the trial.
Under Article 172 of the RPC, the offender
must be a private individual or maybe a public officer,
ADAZA v SANDIGANBAYAN (From 4A Yas Sanchez) employee or notary public who does not take
advantage of his official position.
Facts: The DPWH awarded to the PTA of Manawan Under Article 171, an essential element of the
National High School a contract for the construction of a crime is that the act of falsification must be committed
school building consisting of 2 classrooms. Although the by a public officer, employee or notary who takes
project was completed, the PTA failed to receive the last advantage of his official position.
installment payment for such. The offender takes advantage of his official
Later, PTA President Mejorada was informed that position in falsifying a document when (1) he has the duty
the check had been released to petitioner Adaza, who was to make or to prepare or otherwise intervene in the
the municipal mayor at that time of Jose Dalman. preparation of the document; or (2) he has the official
Petitioner went to the Office of the Auditor of the custody of the document which he falsifies.
DPWH and requested that he be furnished with certified It is thus apparent that for purposes of
true copies of the relevant documents pertaining to the acquisition of jurisdiction by the Sandiganbayan, the
contract, including the disbursement voucher and the requirement imposed by R.A. 8249 that the offense be
corresponding check representing the last payment made committed in relation to the offenders office is
by the DPWH for the project. entirely distinct from the concept of taking advantage
Confronted with Disbursement Voucher, of ones position as provided under Articles 171 and
Mejorada detected that the signature above his printed 172 of the Revised Penal Code.
name acknowledging receipt of the check was not his and R.A. 8249 mandates that for as long as the
that petitioners signature was affixed on the voucher. offenders public office is intimately connected with the
Mejorada saw that there were 2 signatures at the dorsal offense charged or is used to facilitate the commission of
portion, his forged signature and another which he found said offense and the same is properly alleged in the
to be that of Aristela Adaza (Aristela), wife of petitioner. information, the Sandiganbayan acquires jurisdiction.
A case for falsification by taking advantage of Indeed, the law specifically states that the Sandiganbayan
public office was filed against Adaza with the has jurisdiction over all other offenses or felonies whether
Sandiganbayan. Adaza now contends that the simple or complexed with other crimes committed by the
Sandiganbayan has no jurisdiction. public officials and employees mentioned in subsection a
of Section 4 in relation to their office. Public office, it
Issues/Held: bears reiterating, need not be an element of the offense
W/n the Sandiganbayan has jurisdiction - No charged.
W/n the offender can be prosecuted for the crime of On the other hand, the element of taking
falsification by a public officer- Yes advantage of ones position under the RPC becomes
relevant only in the present case, not for the purpose of
Ratio: For the Sandiganbayan to have exclusive determining whether the Sandiganbayan has jurisdiction,
jurisdiction, it is essential that the facts showing the but for purposes of determining whether petitioner, if he is
intimate relation between the office of the offender held to be liable at all, would be legally responsible under
and the discharge of official duties be alleged in the Article 171 or Article 172.
information. While the Sandiganbayan is declared bereft of
Although the petitioner was described in the jurisdiction over the criminal case filed against petitioner,
information as a public officer being then the Mayor with the prosecution is not precluded from filing the appropriate
salary grade 27 of Jose Dalman, Zamboanga del Norte, charge against him before the proper court.
there was no allegation showing that the act of falsification
of public document attributed to him was intimately
connected to the duties of his office as mayor to bring the SIQUIAN V. PEOPLE
case within the jurisdiction of the Sandiganbayan. Neither

198
CRIMINAL LAW REVIEW DIGESTS
JUSTICE ROMEO CALLEJO NOTE: = Callejo Ponente

Facts: Jesusa Carreon went to the office of Manuel He also took advantage of his official position in
Siquian, the municipal mayor of Isabela, to apply for a job falsifying the document. Abuse of public office is
in the office of the mayor. Siquian then appointed her as a considered present when the offender falsifies a document
clerk in the office of the municipal secretary and even said in connection with the duties of his office which consist of
that her salary would be included in the budget. either making or preparing or otherwise intervening in the
Accompanying her appointment is the certification, among preparation of a document. In this case, Siquian was
others, of the availability of funds through a form issued by charged with the duty of issuing the certification necessary
Siquian and addressed to the CSC, pursuant to the for the appointment of Carreon.
requirements of the latter. Lastly, the existence of a wrongful intent to injure
It should be noted that the Municipal council of a third person is not necessary when the falsified
Isabela, failed to enact the annual budget for the document is a public document. The SC relied on the Go
municipality for the Fiscal Year 1975-76. As such, the Tiok case in stating that wrongful intent on the part of an
annual budget for the previous Fiscal Year 1974-75, was accused to injure a third person is not an essential
deemed re-enacted. No such position existed then. element of the crime of falsification of public document.
Carreon worked for five months and was This is because the principal thing punished in falsifying
supposed to receive her salary of P120. She approached public documents is the violation of the public faith and the
the municipal treasurer to ask for the money but the latter destruction of truth as therein solemnly proclaimed.
said that there was no money yet. She then sued Siquian Siquian cannot raise the defense of good faith.
for falsification of a public document. He presides at all meetings of the municipal council and
The RTC and CA ruled in favour of Carreon. signs all ordinances and resolutions passed by the
Siquian interposed the defense of a lack of criminal intent. municipal council. He was also aware that there was no
budget and no such position (clerk of municipal secretary)
Issue: Was Siquian guilty of falsification of public existed.
documents?

Ruling: Yes. SAMSON V. CA


He was found guilty under par 4 of art 171,
making untruthful statements in a narration of facts; the FACTS: Lascano and his wife are entitled to some money
elements of which are: (a) That the offender makes in a because their son was a soldier for the USAFFE during
document untruthful statements in a narration of facts; (b) nd
the 2 world war.
that he has a legal obligation to disclose the truth of the Amado Cruz was with 2 persons purporting to be
facts narrated by him; and (c) That the facts narrated by the Lascano spouses. Cruz asked the help of his friend
the offender are absolutely false. Rufino Samson, the appellant in this case, in order for the
In this case, all the elements for falsification were Lascano spouses to get their checks. Samson verified
met especially when Siquian stated that funds were the identity of the purported Lascanos by examining their
available for the position to which Jesusa Carreon was residence certificates. True enough, Samson had a friend
appointed when he knew that, in reality, the position itself with the Finance Dept of the AFP, Lt. Valencia, and they
did not even exist and no funds had been appropriated. It were able to get their checks that amounted to 12K.
is further bolstered by the fact that when the budget was The group then went to the Treasury Dept to
deemed re-enacted, there is no such position as Clerk to cash the checks. Samson also knew the teller and
the Municipal Secretary, the position to which Carreon represented that the Lascanos were the real Lascanos.
was appointed. And there is also no appropriation made in Mrs. Lascano placed her thumbprint on the back of the 2
the Annual Budget for the Fiscal Year 1974-75 for such checks while Mr. Lascano signed his name. SAMSON, on
position, thus rendering Siquian's statement in his the other hand, SIGNED ON THE BOTTOM AS THE
certification utterly false. LAST INDORSER of the CHECK. The 12K was released.
Siquian also had the legal obligation to disclose The group went to Aristocrat with about 11 other persons.
the truth of such facts. Under the civil service rules and They had their lunch and Samson was given P310 as
regulations, a certification of the availability of funds for the gratitude money. Thereafter, the purported Lascanos
position to be filled up is required to be signed by the head were never seen again.
of office or any officer who has been delegated the Days later, Samson was informed that the
authority to sign. As an officer authorized by law to issue Lascanos were not the real Lascanos. Samson
the certification, Siquian has a legal obligation to disclose investigated and found that real Mr. Lascano could barely
the truth of the facts narrated by him in said certification walk and the real Mrs. Lascano was a teacher who denied
which includes information as to the availability of the receiving any money.
funds for the position being filled up.

199
CRIMINAL LAW REVIEW DIGESTS
JUSTICE ROMEO CALLEJO NOTE: = Callejo Ponente

Samson was charged together with Cruz, a Mr. FACTS:


Vergara and two john does for estafa through falsification Armed Forces and Police Savings and Loan Association
of a commercial document. Inc. (AFPSLAI) is a non-stock non-profit corporation
RTC convicted all of them. On appeal to the CA, rendering savings and loan services to its members.
Samson was only found guilty of Reckless Imprudence. Petitioner Noe Andaya was its president.
Noe Andaya proposed to increase the capitalization of
ISSUE: Can falsification be committed through the AFPSLAI to boost its lending capacity to its
Negligence? YES. members. Pursuant to this, the Board of Trustees
passed a resolution creating the Finders Fee Program
HELD: Samson was definitely grossly negligent in which provided that any officer, member or employee of
assuring the identity of the impersonators to Lt. Valencia AFPSLAI who can solicit at least 100,000 pesos worth
and the Teller as a result of which the govt lost 12K. of investment is entitled to a Finders Fee equivalent to
Samson should not have relied on the mere 1% of the solicitation.
representations of his friend and the residence certificates Thereafter, the Central Bank notified AFPSLAI that its
of the impersonators. financial position was precarious because of
Samson cooperated in the commission of estafa mismanagement. The Board ordered an investigation
through falsification of a commercial document without which resulted to the filing of criminal information
which the estafa would not have been accomplished.8 against Andaya for ESTAFA THROUGH
Insofar as the falsification is concerned the act of FALSIFICATION OF COMMERCIAL DOCUMENTS.
endorsing the check constituted a written representation What happened was Ernesto Hernandez was able to
that the true payees were the ones who indorsed and solicit from an outsider an investment worth 2.1M. Since
cashed the checks, when in truth and fact the true payees Hernandez did not want his Finders Fee worth 21,000
had no part in the endorsement.9 reflected in his Income Tax Return, he asked Andaya if
Falsification can be committed through the latter could find someone who can receive the cash
negligence since intent to cause damage is not an in his behalf. Thus Andaya told Guilas (who was a clerk
element of falsification because what the law seeks to of AFPSLAI) to receive the Fee and turn it over to him
ensure that the public has confidence in these (Andaya) so that the latter may give it to Hernandez. To
documents.10 this end, Andaya allegedly substituted the name of
SC affirmed the conviction for Reckless Hernandez for Guilas in the disbursement voucher.
Imprudence resulting in falsification under Art.365.11 The Trial Court convicted Andaya saying that all the
DOCTRINE FALSIFICATION CAN BE COMMITTED BY elements of the crime are present. First, Andaya caused
NEGLIGENCE. to it appear in the disbursement voucher that Guilas,
instead of Hernandez, was entitled to the Fee and
Second, that this was done with criminal intent to cause
ANDAYA v PEOPLE damage to the government in the form of evading taxes.

ISSUES: Whether or not Andaya is guilty of the crime


8 Note: J. Reyes dissented on this point such that, liability as a charged - NO
principal by indispensable co-operation requires knowledge of the
criminal act and since Samson was ultimately made responsible
HELD: Petitioner must be acquitted on reasonable doubt
for his reckless negligence, its fallacious to argue that he
cooperated to the falsification through his negligence. Petitioner should not have been charged with estafa
9 Article 171 Paragraph 2 Causing it to appear that persons through falsification of commercial document but only
have participated in any act or proceeding when they did not in estafa through falsification of private document whose
fact so participate.
10 Please be careful with this statement because I think the SC elements are as follows: (1) offender commits any of
made a mistake here since in Reckless Imprudence what is the acts of falsification mentioned in 171 (2)
punished is the negligence and the result merely determines the falsification was committed on a private document
penalty imposed. If intent to falsify was not an essential element
of falsification then it would be mala prohibita and we all know that and (3) the falsification caused damage or there was
falsification is mala in se. Remember the case about tampering of intent to cause damage to a third person.
1 element is satisfied. Prosecution was able to
Election Results, court said that its mala in se, I think the same st
rule applies here. OR is intent to cause damage different from
intent to falsify? I dont know.
establish the participation of Andaya in causing the
11 Note: Strong Dissent was made by Justice Reyes that Samson voucher to be named after Guilas rather than
cannot be convicted of Reckless Imprudence resulting in Hernandez. Although Andaya did not personally cause
Falsification because this is not necessarily included in the
the substitution, he is nevertheless a principal by
intentional crime of Estafa through Falsification, the crime
charged in the information. In the words of Justice Reyes malice induction for having ordered so. In this case, the act of
or intent cannot co-exist with negligence

200
CRIMINAL LAW REVIEW DIGESTS
JUSTICE ROMEO CALLEJO NOTE: = Callejo Ponente

falsification done by Andaya is under 171(2) or causing The City Prosecutor of Tacloban resolved that 3
it to appear that a person has participated in something informations of falsification (under Art 172, Par 1 in
but in fact has not. relation to Art 171, Par 2) be filed against Silvina and
2
nd
element is satisfied. However, the prosecution Camenforte (a lawyer who conspired with Lastrilla).
wrongfully classified a disbursement voucher as a However, it did not find any probable cause to file against
commercial document. It is not. it is a private document. Lastrilla. Granda appealed to the DOJ but the DOJ just
A commercial document is an instrument commonly affirmed.
used by merchants to facilitate trade and credit On appeal to the CA, the CA finally resolved that
transactions. They are regulated by the Code of 3 informations of falsification (under Art 172, Par 1 in
Commerce. On the other hand, a private document is relation to Art 171, Par 1, 2 and 5) be filed against
an instrument executed by a private person without the Lastrilla. In essence, the CA said that Lastrilla falsified the
intervention of a notary by which some disposition or documents in three ways imitating the signature of the
agreement is proved. Again, a voucher is not a grandparents, causing it to appear that persons have
commercial document. participated when in fact they did not (since the
3
rd
element was not satisfied. This is because grandparents were dead), and altering true dates.
irrespective of who really has the right to claim the Fee Lastrilla claimed that the CA committed error in
Guilas or Hernandez the fact remains that AFPSLAI finding probable cause against him. Lastrilla also claimed
is indebted to the extent of 21,000. AFPSLAI suffered that there was no damage done because he allegedly paid
no damage because in the first place it was under P18m for the land, hence he cant be held liable for
obligation to make such payment. The ration of the Trial falsification.
Court that it was the Government which stands to suffer
because of tax evasion is untenable. Why? Because the Issue: was there probable cause to engender the belief
information alleged damaged to AFPLSAI and not that petitioner is one of the authors of the falsification?
damage to the Government. If you sustain the finding of
the TC you violate Andayas right to be properly Held: Yes, there was probable cause.
informed of the crime he is charged with. Lastrilla attested to the fact that the grandparents signed
Note however that in the book theres no such thing daw the deeds in his presence in 1985. However, he also
as estafa through falsification of private document. Pero admitted that the negotiations for the sales only started in
the court did not even mention this. 1998 (even when the documents said 1985) so how
could the negotiations happen only after the deeds were
executed? The PNP Crime Lab also reported that the
LASTRILLA V GRANDA signatures of the grandparents did not match specimen
signatures of the spouses.
FACTS: Granda was the grandson of the lot owners To his claim that no damage was done, the Court
(Rafael and Aurora). The lot owners died in 1989 and said that in falsification of public or official documents, it is
2000, respectively. Granada claims that Lastrila, et al, not necessary that there be present the idea of gain or the
falsified three deeds of sale covering numerous parcels of intent to injure a third person for the reason that in these
land in favor of the latter and the latters relatives. cases, the principal thing punished is the violation of the
Because of the 3 deeds of sale, the parcels of land were public faith and the destruction of the truth as therein
registered to Lastrilla and his relatives. solemnly proclaimed.
The first deed covered 2 parcels of land. The The case teaches us that there are as many
second deed covered 2 parcels of land. While the third counts of falsification as to documents falsified, regardless
deed covered 3 parcels of land. of the parcels of land contained in each deed. In this case,
Granda claims that there were 3 deeds of sale falsified thats why 3 counts
1) the signatures on the 3 deeds of sale were falsified and of falsification were charged to Lastrilla and his cohorts.
were not of his grandparents, Moreover, it also teaches us that even if there are multiple
2) the 3 deeds of sale were antedated (the deeds said that modes of falsifying a single document, it will still be
the transactions took place on Dec 1985, but in fact they considered as 1 crime thats why even if Lastrilla falsified
took place in 1999 or 2000) each document in three ways (1, 2 and 5), he was still
3) the witness to the deed (Grandas sister Silvina) could charged of 1 count of falsification per document.
not have possibly signed the 3 deeds in 1985 because she With regard to Art 171, Par 2, the case teaches
was cloistered in a convent at that time. us that the provision applies even if those who were made
4) the subject deeds were only registered with the RD only to appear to participate are already dead.
on 2000, or 15 years after the purported sales

201
CRIMINAL LAW REVIEW DIGESTS
JUSTICE ROMEO CALLEJO NOTE: = Callejo Ponente

GOMA VS CA. 4. That such person or persons did not in fact so


participate in the proceeding.
FACTS: The Ombudsman filed an information for The first two elements clearly obtain, petitioners,
FALSIFICATION OF PUBLIC DOCUMENT under during the period material, being local government elected
ARTICLE 171 (2) against Goma and Umale. It was alleged officials who, by reason of their position, certified, as
that Brgy. Chairperson Goma and Secretary Umale Umale did, as to the holding of a barangay session and
falsified a brgy. Resolution by allocating the amount of falsely attested, as Goma did, as to the veracity of a
P18,000 as disbursement for a seminar of 2 officials. In resolution supposedly taken up therein. The other two
the said Resolution, it was indicated that it was passed on elements are likewise present. As correctly observed by
motion of Kgwd. Dizon and seconded by Kgwd. Dela Cruz the CA:
when in truth and in fact no meeting was held as there [Petitioners] made it appear in the Barangay
was no quorum. The said Resolution was even signed by resolution that all members of the Sangguniang Barangay
Goma and Umale and with the official seal of the brgy. deliberated upon and unanimously approved the
Goma and Umale denied the allegations, claiming that the questioned resolution, when in fact no such deliberation
said Resolution was a mere draft and it is not sufficient to and approval occurred. The non-participation of the
disburse funds. members of the Sangguniang Barangay in the passage of
RTC: GUILTY. The Resolution have all the the resolution was established by the 15 October 1995
appearance of a complete and "true and genuine resolution issued by 7 of the 8 members of the
document," sealed and signed by the Sanggunian Sangguniang Barangay denying that the challenged
secretary.CA: affirmed. resolution was passed upon and approved by the council.
Indeed, the contents and appearance of the
ISSUE: WON the Resolution is a public document? WON Resolution argue against the very idea of its being merely
petitioners can be held liable? a proposal or a draft barangay enactment.
Falsification of a public document is
RULING: consummated upon the execution of the false document.
1. Resolution is a Public Document. And criminal intent is presumed upon the execution of the
Revised Rules on Evidence, public documents include criminal act. Erring public officers failure to attain their
"[t]he written official acts, or records of the official acts of objectives, if that really be the case, is not determinative of
the sovereign authority, official bodies and tribunals, and their guilt or innocence. The simulation of a public
public officers, whether of the Philippines, or of a foreign document, done in a manner so as to give it the
country." appearance of a true and genuine instrument, thus,
There can be no denying that the public money-disbursing leading others to errors as to its authenticity, constitutes
and seemingly genuine Resolution, in the preparation of the crime of falsification.
which petitioners, in their official capacity, had a hand, is, In fine, the element of gain or benefit on the part
in context, a public document in a criminal prosecution for of the offender or prejudice to a third party as a result of
falsification of public document. And it bears to stress that the falsification, or tarnishing of a documents integrity, is
in falsification under Art. 171(2) of the RPC, it is not not essential to maintain a charge for falsification of public
necessary that there be a genuine document; it is enough documents. What is punished in falsification of public
that the document fabricated or simulated has the document is principally the undermining of the public faith
appearance of a true and genuine document or of and the destruction of truth as solemnly proclaimed
apparent legal efficacy. therein. In this particular crime, therefore, the controlling
consideration lies in the public character of a document;
2. GUILTY of Falsification (Causing it to appear that and the existence of any prejudice caused to third persons
persons have participated in any act or or, at least, the intent to cause such damage becomes
proceeding when they did not in fact so immaterial.
participate).
The elements of the crime of falsification of public
documents, as above defined and penalized, are: MONTEVERDE V. PEOPLE - ??
1. That the offender is a public officer, employee, there is no complex crime when (1) two or more
or notary public. crimes are committed, but not by a single act; or
2. That he takes advantage of his official position. (2) committing one crime is not a necessary
3. That he falsifies a document by causing it to means for committing the other (or others)
appear that persons have participated in any act If falsification of a public and/or a commercial
or proceeding. document were not necessary to commit estafa

202
CRIMINAL LAW REVIEW DIGESTS
JUSTICE ROMEO CALLEJO NOTE: = Callejo Ponente

and neither were the two crimes the result of a Leonilas act falls under par2 of Art. 171 causing it to
single act, then there is no complex crime. appear that persons have participated in any act or
Since not complexed, acquittal in one does not proceeding when they did not in fact so participate.
mean acquittal in other as they are considered
separate crimes. Even if it is complexed, acquittal Re: third element
in component crime will not lead to acquittal of PCCI only grants loans to its bona fide members with no
other offense/s. subsisting loans. As mentioned earlier, 3 out of the 4
Sales Invoice is also a commercial document. persons were not members. The remaining one had
If prosecution doesnt prove that second actually settled the loan but only for the purpose of
document is falsified, as it didnt present original, avoiding legal prosecution, with the understanding
no crime as the second document might be an however that she will be reimbursed once the money is
original as well. collected from Leonila.

Re: second element


LEONILA BATULANON VS. PEOPLE The vouchers were indeed private documents because
they were not documents used by merchants or
Facts: Leonila was employed as a cashier/manager of businessmen to promote or facilitate trade or credit
Polomolok Credit Cooperative Incorporated (PCCI). She transactions, nor are they defined and regulated by the
was in charge of receiving deposits from and releasing Code of Commerce or other commercial laws. Rather they
loans to the members of the cooperative. During an audit are private documents, which have been defined as deeds
in 1982 certain discrepancies were discovered in relation or instruments executed by a private person without the
to the release of loans. 4 informations for estafa through intervention of a public notary or of other persons legally
falsification of commercial documents were filed against authorized, by which some disposition or agreement is
Leonila. In summary, these informations stated that proved, evidenced, or set forth.
Leonila falsified cash/check vouchers in the name of 4
different persons, thereby making it appear that these No complex crime of estafa through falsification of private
persons were granted loans when in fact they did not even document!
apply for them, and moreover, they did not sign any of the If the falsification is done as a means to commit estafa,
said vouchers. then the crime would be falsification. On the other hand, if
The witnesses testified that 3 out of the 4 estafa could have been committed without the necessity of
persons were not even members of PCCI and that one of falsifying the document, the proper crime would be estafa.
them [the non-members] was the son of Leonila who was, NB: SC didnt say why but it cited Gregorio, citing Cuello
at that time, only 3 years old. Eventually, the TC found her Calon. And according to J. Callejo (feeling ko ancestor
guilty of estafa through falsification of commercial niya si Cuello Calon Cuello Calon = Callejo), both
documents. On appeal however, the CA affirmed with crimes share a similar element damage or intent to
modification, finding her guilty instead of falsification of cause damage.
private documents (Art. 172, par2).
So what crimes were committed?
Issue: What crimes were committed? As to the 3 persons falsification of private documents (3
counts)
Held/Ratio: 3 counts of falsification of private documents As to Leonilas 3-year-old child estafa!
and one estafa.
Why estafa???
Elements of the crime: Because Leonila did not falsify the signature of her son. In
1) Offender committed any of the acts of falsification fact, it appeared in the voucher that she wrote by:
enumerated in Art. 171, except par7; Batulanon, indicating that she received the proceeds of
2) Falsification was committed in any private the loan in behalf of her son. Such act does not fall under
document; and any of the instances of falsification enumerated in Art. 171.
rd
3) Falsification caused damage to a 3 party OR at Nonetheless, such representation caused damage to
least the falsification was committed with intent to PCCI which makes her liable for estafa!
cause such damage.

Re: first element R.F. NAVARRO CO. AND HEIRS OF R. NAVARRO VS.
VAILOCES AND HEIRS OF E. RODRIGUEZ

203
CRIMINAL LAW REVIEW DIGESTS
JUSTICE ROMEO CALLEJO NOTE: = Callejo Ponente

FACTS: Petitioners (Laura Navarro12 and other heirs of In any case, assuming, ex gratia argumenti, that
R. Navarro) instituted an action for annulment of the Deeds of Sale with Assumption of Mortgage were
documents, titles and/or reconveyance against the heirs of spurious, petitioners are already barred by laches.
E. Rodriguez and Luzon Surety Co. Petitioners alleged
that they are the owners of certain lots by virtue of TCT
61619 registered under the name of Raymundo Navarro THE PEOPLE OF THE PHILIPPINES v PO GIOK TO
(deceased) and R.F. Navarro Co13. Raymundo entrusted
said property to E. Rodriguez, but it was discovered by FACTS: In the Court of First Instance of Cebu, the
petitioners that after Raymundos death, E. Rodriguez was defendant appellee Po Giok To was charged with the
able to transfer the property in his name. Petitioners claim crime of falsification. He misrepresented to the City
that the transfer of property in E. Rodriguez name was Treasurer of Cebu that his name is Antonio Perez, that his
fraudulent as it was done without any consideration and place of birth is Jaro, Leyte and that his citizenship is
knowledge of the petitioners, and that the signature of Filipino. From such misrepresentation of facts the City
Raymundo on the Deeds was forged. Upon discovery of Treasurer issued him a residence certificate.
such fraudulent transfers, petitioners demanded that from The accused filed a motion to quash on the
private respondents to return the properties. ground that the information does not allege sufficient facts
On the other hand, private respondents (heirs of to constitute the crime of falsification. The City Fiscal
E. Rodriguez and Luzon Surety Co.) alleged that the opposed the motion to quash claiming that the information
transfer of the property was done through a Deed of Sale alleges all the integral elements of the offense charged as
with Assumption of Mortgage executed by Raymundo in defined by the statute. The lower Court, however, found
favor of E. Rodriguez. Thus, TCT 61619 was cancelled the motion to quash meritorious and ordered the
and a new one (TCT 62411) was issued in the name of E. amendment of the information.
Rodriguez. Hence, this appeal by the Government.
The RTC found the transfer as fictitious, thus
ruling in favor of petitioners. On appeal, the CA reversed ISSUE: Whether or not the information in question should
the RTC decision, ruling in favor of private respondents allege the following facts in order to be sufficient to convict
the defendant of the crime of falsification:
ISSUE: W/N the Deed of Sale is spurious since it was 1) That the accused had the obligation to
not signed by Raymundo Navarro. NO. disclose the truth in the document allegedly
falsified;
HELD/RATIO: Petitioners claim that the aforementioned 2) That the accused had the wrongful intent to
deed of sale is spurious as the same was not signed by injure a third
Raymundo F. Navarro. Petitioners' attempts to show that
the Deeds of Sale with Assumption of Mortgage executed HELD: We agree with the Solicitor-General that the first
by Navarro and Rodriguez are false do not impress the element allegedly lacking in the information, that is, the
SC. The bare assertions on the part of Laura Navarro, obligation on the part of the accused to disclose the truth
wife of Raymundo, that the signature appearing on the as to the facts that should appear in a residence
Deeds of Sale is not that of her husband is not enough. certificate, is inherent in the very nature and purpose of
Forgery is not presumed; it must be proven by clear, said document.
positive and convincing evidence. Those who make Section 3 Commonwealth Act 465 provides:
the allegation of forgery have the burden of proving it "that the residence certificate for persons shall
since a mere allegation is not evidence. In the case at contain the full name, place and date of birth,
bar, where the alleged forged signature was that of a citizenship, civil status, length of residence in the
President of a Corporation, petitioners could have easily city or municipality where the certificate is
presented other documents bearing the true signature of issued, occupation or calling.
R.F. Navarro Sr., to substantiate their claim. Not having Needless to say, this provision implies that the
done so, Lauras uncorroborated claim cannot be given person to whom the certificate is issued must state to the
much weight. This is so especially in light of the fact that officer who issues the same, the true facts, required to
Laura was one of the plaintiffs and stood to gain by having appear therein, the latter having merely the ministerial
the deeds of sale and the transfer certificate of title in the function of recording thereon the facts as supplied by this
name of Luzon Surety Company declared void. person. And to guarantee that the facts given correctly and
truly identify the holder of the certificate, he is also
required by Sec. 3 above to sign the document and affix
12 Wife of the deceased Raymundo Navarro.
his right hand thumb mark thereon.
13 Raymundo Navarro was the President of R.F. Navarro Co.

204
CRIMINAL LAW REVIEW DIGESTS
JUSTICE ROMEO CALLEJO NOTE: = Callejo Ponente

There is, therefore, no question that the The reason for the distinction is given in a
accused had the duty to disclose the true facts about decision of the Supreme Court of Spain in the case
his name, place of birth, and citizenship to the officer of People vs. Pacana that in the falsification of public or
or employee who issued his residence certificate and official documents, whether by public officials or by private
such duty being inherent in the transaction, there was persons, it is unnecessary that there be present the idea
no need for the criminal charge to allege that the of gain or the intent to injure a third person, for the reason
accused had such duty. that, in contradiction to private documents, the principal
Anent the second element allegedly lacking in the thing punished is the violation of the public faith and the
information in question, the law is clear that wrongful destruction of the truth as therein solemnly proclaimed.
intent on the part of the accused to injure a third Moreover, the acts charged, if true, would result
person is not an essential element of the crime of in confusion in the government records, since the
falsification of public document. fingerprint of the accused would not correspond to that of
Article 172, par. 1, in connection with Art. 171, the person whose personal circumstances are recited in
par. 4, of the Revised Penal Code provides as follows: the certificate. Such confusion in its records evidently
ART. 171. Falsification by the public officer, operates to the Government's prejudice. Being the natural
employee or notary or ecclesiastic minister. and direct result of the criminal act charged, the accused
The penalty of prision mayor and a fine not to must be presumed to have intended it.
exceed 5,000 pesos shall be imposed upon any
public officer, employee, or notary who, taking Side issue: It is argued for the defendant that there being
advantage of his official position shall falsify a a special law with respect to residence certificates
document by committing any of the following expressly punishing their falsification (Commonwealth Act
acts: No. 465), this special law, and not the provisions of the
4. Making untruthful statements in a narration of Revised Penal Code, should apply in this case. RPC can
facts. still apply since under Art. 10 of the RPC has
ART. 172. Falsification by private individuals and supplementary application to all special laws, unless the
use of falsified documents. The penalty latter should provide the contrary, and CA No. 465 makes
of prision correccional in its medium and no provision that it exclusively applies to all falsifications of
maximum periods and a fine of not more than residence certificates.
5,000 pesos shall be imposed upon: Thus the information was sufficient, and its
1. Any private individual who shall commit any of dismissal for insufficiency by the Court below was
the falsifications enumerated in the next improper and erroneous.
preceding article in any other kind of commercial
document.
On the other hand, Art. 172, par 2, defining the DAVA v. PEOPLE
crime falsification of private document, provides:
2. Any person who, to the damage of a third FACTS: Michael Dava bumped pedestrians Bernadette
party, or with intent to cause such damage, shall Roxas Clamor and Dolores E. Roxas, causing death to
in any private document commit any of the acts former and physical injuries to the latter. As a
of falsification enumerated in the next preceeding consequence, his driver's license was confiscated and he
article. was charged with homicide and serious physical injuries.
The distinction made by the law between One day, the brother of Bernadette and the father of
falsification by private persons of PUBLIC DOCUMENTS Dolores, saw Dava driving a Volkswagen. Knowing that
AND PRIVATE DOCUMENTS IS CLEAR. Dava's driver's license was used as an exhibit in court and
The first (FALSIFICATION OF PUBLIC that no traffic violation receipt had been issued to Dava,
DOCUMENTS BY PRIVATE INDIVIDUALS) is committed they had Dava apprehended for driving without a license.
by the mere performance of any of the acts of falsification When he was apprehended, he showed the police officers
enumerated in Art. 171 a non-professional driver's license No. 2706887 with
While the second (FALSIFICATION OF PRIVATE official receipt No. 0605870 issued by Agency Pampanga
DOCUMENTS BY PRIVATE INDIVIDUALS) is committed in the name of Michael T. Dava. When asked about the
not only by the performance of any of the acts of source of his license, Dava informed them that his
falsification enumerated in Art. 171 but it must likewise be officemate (Manalili) had secured it for him. He was
shown that such act of falsification was committed to the brought to the police station and charged w falsification of
damage of a third party or with intent to cause such a public document.
damage. Prosecution witnesses: Caroline Vinluan of the
Angeles City branch of the Bureau of Land Transportation

205
CRIMINAL LAW REVIEW DIGESTS
JUSTICE ROMEO CALLEJO NOTE: = Callejo Ponente

(BLT). He testified that the license was earlier brought to public faith and the destruction of the truth proclaimed
him and he was asked whether it was fake or genuine. He therein.
examined it and found that it was "fake or illegally issued"
because form No. 2706887 was one of the 50 forms which People vs. Sendaydiego,
had been reported missing from their office sometime in The rule is that if a person had in his possession a
November, 1976 and that it was never issued to any falsified document and he made use of it (uttered it),
applicant for a license. He added that any license that was taking advantage of it and profiting thereby, the
not included their office index card was considered as presumption is that he is the material author of the
"coming from illegal source' and "not legally issued by any falsification In the absence of a satisfactory
agency." Although the form used for the license was explanation, one who is found in possession of a
genuine, the signature of the issuing official was fake. forged document and who used or uttered it is
Defense witness: Manalili. He said he obtained the license presumed to be the forger
by paying fixers We agree with the petitioner that the presumption
enunciated in the Sendaydiego case is not absolute as it is
ISSUE: WON Dava can be convicted of falsification? subject to the exception that the accused should have
YES a satisfactory explanation why he is in possession of a
false document. His explanation, however, is
RATIO: Elements of the crime of using a falsified unsatisfactory as it consists mainly in passing the buck to
document in transaction (other than as evidence in a his friend, Manalili.
judicial proceed penalized under the last paragraph of
Article 172) are following:
(a) the offender knew that a document was falsified by FLORES VS LAYOSA
another person;
(b) the false document is embraced in Article 171 or in any Facts: On Dec 1991, private respondent Benigno
of subdivisions Nos. 1 and 2 of Article 172; Montera, employee of the National Food Authority (NFA),
(c) he used such document (not in judicial proceedings), filed a complaint in the Office of the Ombudsman (OMB)
and against the petitioners Flores, Elizon and Soloria, and 2
(d) the use of the false document caused damage to others (Dansal and Vallada). OMB filed an Information
another or at last it was used with intent to cause such charging the petitioners with the offense of Estafa through
damage. Except for last, all of these elements have been Falsification of Public Documents.
proven beyond reason doubt in this case. It was alleged that Dansal and Flores were dept
manager and asst manager respectively while Elizon,
a. Petitioner himself requested officemate Manalili to get Soloria and Vallada were security personnel of NFA.That
him a license. He misrepresented to Manalili that he has the petitioners conspired in falsifying the DAILY TIME
not at any time been issued a driver's license. Through RECORD of Vallada, making it appear that the latter
this misrepresentation petitioner was ableto induce reported for work as a security guard at the NFA when in
Manalili to deal with "fixers" in securing the subject driver's fact, he never reported for work. And because of this,
license. Vallada was still able to collect his salary.
b. A driver's license is a public document. The blank form Prosecutors filed a motion to suspend the
of the drivers license becomes a public document the petitioners pendent lite, which was granted by the court,
moment it is accomplished. Thus, when driver's license
suspending them for 90 days. Trial court applied Sec 13 of
No. 2706887 was filled up with petitioner's personal data RA 3019 (Anti Graft and Corrupt Practices Act) which
and the signature of the region of the San Fernando LTC mandates that a public officer charged for an offense
agency was affixed therein, even if the same was involving fraud upon govt or public funds or property shall
simulated, the driver's license became a public document. be suspended pending case in court. Petitioners filed an
c. When petitioner was apprehended he presented the MR which was denied. A certiorari to the Sandiganbayan,
license to the officer. Because he was a detailman who did which only affirmed the TCs issuance of order of
his job with the use of a car, it is probable that hes been suspension pendente lite because it falls under Sec 13 of
using the license. RA 3019.
d. The driver's license being a public document, proof of While petitioners concede that
the fourth element of damage caused to another person or the Information sufficiently alleges the elements of the
at least an intent to cause such damage has become offense of falsification of public document, they assert that
immaterial. In falsification of public or official documents, it does not contain an averment of fraud or deceit on their
the principal thing being punished is the violation of the part. Hence, they claim that the Information does not

206
CRIMINAL LAW REVIEW DIGESTS
JUSTICE ROMEO CALLEJO NOTE: = Callejo Ponente

charge them with estafa but only falsification of public the Information (which states that petitioners committed
document. And that they shouldnt be suspended. Also, estafa under Article 315), or as claimed by the People in
that it was private res Montera who filed for suspension, their Comment (that petitioners committed estafa under
not prosecutors. Article 318) and the absence of the words "fraud" or
Respondent People argue that the "deceit" in the Information, the Court agrees with the
Information sufficiently alleges the elements of estafa Sandiganbayan and the RTC that the factual allegations
through falsification of public document under Article 318 therein sufficiently inform petitioners of the acts
in relation to Article 171 of the RPC. According to the constituting their purported offense and satisfactorily
People, this complex crime is an "offense involving fraud allege the elements of estafa in general committed
upon government or public funds or property" under through the offense of falsification of public document.
Section 1314, R.A. 3019. Thus, the suspension pendente In the case at bar, although the word "deceit"
lite of petitioners is justified. Also, although Montera filed or "fraud" was not specifically alleged in the
the motion to suspend, it was with conformity of the public information, nonetheless, the same alleges the
prosecutor. manner by which deceit or fraud was committed. It
was committed by falsifying the daily time record of
Issue: W/N the offense charged is a complex crime of accused Vallada; and that it was committed by using
estafa through falsification of public docs. said falsified daily time record to collect the
W/N offense charged falls within the coverage of Section corresponding salary of Vallada to the damage and
13 of R.A. No. 3019. YES! prejudice of the National Food Authority. To our mind
W/N suspension of petitioners pendente lite is proper. these allegations are sufficient to maintain the validity of
YES! the information.
It must be noted that the crime for which the
Held/Ratio: The Information alleges that petitioners took accused are charged is the complex crime of estafa
advantage of their positions and conspired and falsified through falsification of public document wherein the
the DTR of Valleda to make it appear as if he reported for falsification of the public document is a necessary means
work that month but he in fact didnt and in the processes to commit the estafa.
appropriated Valladas salary to the prejudice of NFA. It bears stressing that the words "fraud" or
Still, any error in the Information, with regard to "deceit" need not be used in an information for the
the specification of the particular mode of estafa, allegedly allegations therein to sufficiently allege the offense of
committed by petitioners will not result in its invalidation estafa. It is enough that acts constituting abuse of
because the allegations therein sufficiently inform confidence or deceit, which are indispensable to estafa,
petitioners that they are being charged with estafa through are averred in the information.
falsification of public document. The court determines whether the information
Crimpro rules state that the Information should charging a public officer with an offense that falls under
contain the designation of the offense and the criminal Sec 13 of R.A. No. 3019, such as to warrant suspension
acts/omissions complained as constituting the offense. pendent lite. If it does, the court is bound to issue an order
BUT the SC has clarified in several cases that the of preventive suspension of the accused public officer as a
designation of the offense is NOT controlling because matter of course. The order of suspension pendente lite,
what actually determines that nature and characted of the while mandatory in nature, is by no means automatic or
crime charged are the facts alleged in the information. self-operative. Before such suspension is imposed, a
Thus, notwithstanding the discrepancy between determination as to the validity of the information must first
the mode of commission of the estafa as alleged in be made in a pre-suspension hearing. Accused must be
given an opportunity to challenge the validity of the
criminal proceedings against him.
14 Section 13 of R.A. No. 3019 provides:
Suspension and loss of benefits.Any incumbent public officer In this case, petitioners were afforded the
against whom any criminal prosecution under a valid opportunity to challenge the validity of the charges against
information under this Act or under Title 7, Book II of the them, and whether it falls under Sec 13 of RA 3019 when
Revised Penal Code or for any offense involving fraud upon
government or public funds or property whether as a simple
they filed a opposition to Monteras motion.
or as a complex offense and in whatever stage of execution
and mode of participation, is pending in court, shall be
suspended from office. Should he be convicted by final
judgment, he shall lose all retirement or gratuity benefits under CAUBANG V. PEOPLE
any law, but if he is acquitted, he shall be entitled to
reinstatement and to the salaries and benefits which he failed Doctrine: possessor of falsified document is deemed as
to receive during suspension, unless in the meantime
administrative proceedings have been filed against him.
the forger/ immaterial if falsified document contains no
(Emphasis supplied) false contents, so long as signature has been forged!

207
CRIMINAL LAW REVIEW DIGESTS
JUSTICE ROMEO CALLEJO NOTE: = Callejo Ponente

Facts: Caubang is charged with falsification of public EUGELIO G. BARAWID vs. PEOPLE OF THE
document, as a private individiual, by forging the signature PHILIPPINES,
of treasurer Pagaduan contained in the statement of
assets and liabilities of the BANGANGA consolidated FACTS: Benardino was the former Municipal Mayor and
arrastre stevedoring services (BCASS), making it appear Chairman of the Pre-Qualification Bid and Awards
that said PAGADUAN participated in said registration of Committee (PBAC); Barawid a Municipal Treasurer was a
the newly formed corporation. member of the PBAC; Tomas was the PBACs acting
Petitioner and Pagaduan's stevedoring Secretary.
companies were merged after an agreement by both The Minutes of the opening of bids show that
parties, which was called the aformentioned BCASS. on December 8, 1997, the PBAC members convened and
Caubang was designated as the representative to register assessed the qualifications of the four bidders who
the merged company with the SEC, although thereafter participated in the bidding for the construction of the
Pagaduan stated that said merger shall not push through. extension of the public market which was later awarded to
Pagaduan was then surprised when said BCASS was MASCOM. The Minutes was signed by petitioner Tomas in
already registered with the SEC and wondered why his her capacity as the acting Secretary of the PBAC.
signature was affixed in the STatement of assets and Bernardino and Barawid and the other PBAC members
liabilities as treasurer thereof. also signed a Prequalification Bid and Award
Petitioner's defense is that he brought to the SEC Committee, Abstract of Bidding and Abstract of
only the AOI, treasurer's affidavit, and the 2,5oo paid up Proposal which reflected the names of the four bidders
capital, and denied having presented the statement of and their respective bids.
assets and liabilities. A new mayor was elected before the
commencement of the said project. He thereafter
ISSUE: WON guilty of falsification of document by forging conducted a public bidding for the same project and it
the signature of Pagaduan, even though he denied having was awarded to KYRO Builder as the lowest bidder.
brought or made such statement of assets and liabilities. In a case filed by MASCOM against the new
mayor Dizon, he contended that the award to KYRO is
HELD: guilty. proper because the project could not be validly given to
MASCOM as there was in fact no competitive public
RATIO: the arguments of petitioner are mere denials bidding held on December 8, 1997. Such allegation of
which, if weighed agaonst documentary evidence and Dizon was supported by affidavits of the former members
testimony of witnesses, do not convince the court to of PSAC which states that no public bidding was held in
reverse its decision. connection with the construction of the public market
Moreover, having been the one responsible for extension nor was the local PBAC convened on December
the filing of registration papers, the accused must likewise 8, 1997. Affiants also declared that the documents in
be accountable therefore. As the authorized rep, he is connection with the alleged bidding were delivered to their
deemed to be the one in custody and possession of the residence/office; and that they signed the same upon the
statement of assets and liabilities. Hence, the court is representation of MASCOMs representative that the
convinced that the possessor and user of a falsified documents were necessary for the PNB loan application of
document is presumed to be the forger thereof. the municipality in connection with the construction of the
Also equally important, it is immaterial whether or public market.
not the contents set forth in the forged statement were Thus, an Information was filed charging
false, so long as the signature of another was petitioners and the PBAC members of falsification by
counterfeited. Lastly, in this crime of falsification, the making it appear in the Minutes of the opening of bids,
princal thing punished is the PUBLIC FAITH AND Prequalification Bid and Award Committee, Abstract of
DESTRUCTION OF TRUTH as therein proclaimed. The Proposal, and Abstract of Bidding, that they and COA
act of forging Pagaduan's signature is the act punishable, representative conducted a public bidding on December 8,
and so INTENT TO CAUSE DAMAGE OR GAIN IS 1997, participated in by four bidders, when no such
IMMATERIAL, moreso actual damage. ( as opposed to bidding was in fact conducted.
private documents, wherein intent to cause damage or
gain is necessary). ISSUE: W/N the petitioners are guilty of the crime.

HELD: Bernardino and Barawid: NO. Tomas: YES.


NESTOR A. BERNARDINO and CELEDONIA N. In the instant case, petitioners were charged with
TOMAS, vs PEOPLE OF THE PHILIPPINES, falsification under paragraph 2, Article 171 of the Revised

208
CRIMINAL LAW REVIEW DIGESTS
JUSTICE ROMEO CALLEJO NOTE: = Callejo Ponente

Penal Code, by causing it to appear that persons have petitioner Noe S. Andaya of falsification of private
participated in any act or proceeding when they did not in document.
fact so participate. Its elements are: (1) that the offender Armed Forces and Police Savings and Loan
is a public officer, employee or notary public; (2) that he Association, Inc. (AFPSLAI), a non-profit
takes advantage of his official position; (3) that he falsifies association engaged in savings and loan
a document by causing it to appear that a person or transactions filed a complaint against Andaya for
persons have participated in any act or proceeding when estafa through falsification of commercial
they did not in fact so participate. document.
The evidence presented by the prosecution to Andaya was elected president and general
establish that no bidding was conducted on December 8, manager of AFPSLAI and during his term. During
1997 could not be considered for purposes of determining his term, he sought to increase the capitalization
whether a public bidding was indeed held on that day of AFPSLAI to boost its lending capacity to its
because of the affiants admission that they do not have members. Consequently, the Board of Trustees
personal knowledge whether or not said bidding was of AFPSLAI approved a Resolution setting up a
indeed conducted. The affiants declared that no public Finders Fee Program whereby any officer,
bidding was held on December 8, 1997. However, said member, or employee (except investment
declaration is merely an expression of an opinion and not counselors) of AFPSLAI who could solicit an
a fact considering that like prosecution witnesses investment of not less than P100k would be
Ronquillo and Mayor Dizon, they also have no personal entitled to a finders fee equivalent to 1% of the
knowledge as to whether or not a bidding was indeed amount solicited.
conducted. Such declaration cannot be admitted as On Sept. 1991, the Central Bank wrote Gen.
evidence in the court being an opinion of witness which is Abadia (Chairman of the Board of Trustees)
inadmissible as evidence pursuant to the Rules of Court. regarding the financial position of AFPSLAI due
And while the Information alleged conspiracy such that the to its alleged flawed management. Gen. Abadia
acts of the affiants may be attributed as well to petitioners requested the NBI to conduct an investigation on
Bernardino and Tomas, the same cannot be considered alleged irregularities in the operations of
against said petitioners inasmuch as no evidence was AFPSLAI.
presented by the prosecution to establish conspiracy. One of the cases filed pursuant to the
Conspiracy must be established by positive and investigation is this criminal complaint filed
conclusive evidence. It cannot be based on mere against Andaya based on alleged fraudulent
conjectures but must be established as a fact. implementation of the Finders Fee Program.
On the part of Tomas, even if we assume that all On Oct. 1991, an information for estafa through
the PBAC members attended the bidding, including those falsification of commercial document was filed
who executed an affidavit to the contrary, she is still liable against Andaya. Andaya pleaded not guilty.
for falsification. Note that she was the only one who Prosecution presented 2 witnesses (1)
signed the Minutes of the opening of bids which stated, Diosdado Guillas testified that upon learning of a
among others, that COA representative Ronquillo finders fee of P21,000.00 in his name, Andaya
attended the public bidding on December 8, 1997. As instructed him to collect the money and turn it
acting Secretary of the PBAC she has the duty to prepare over to Andaya. According to him, there was no
or intervene in the preparation of the Minutes of the prohibition in placing the finders fee under the
meetings of the PBAC which should be recorded pursuant name of a person who did not actually solicit the
to Section 37 of the Local Government Code. In making it investment. (2) Judy Balangue testified that
appear that COA representative Ronquillo attended the Andaya instructed him to prepare a Certificate of
bidding when the latter categorically testified that he never Capital Contribution in the name of Rosario
attended a public bidding on December 8, 1997, Tomas Mercader for an investment in AFPSLAI in the
took advantage of her official position, rendering her liable amount of P2.1M and to inform Guilas that the
for falsification under Article 171 paragraph 2 of the finders fee will be placed under Guilas name
Revised Penal Code. (Note that the P2.1M was actually deposited in
AFPSLAI).
Defense presented 3 witnesses (1) Arevalo,
PEOPLE V. ANDAYA secretary of AFPSLAI, explained that the finders
fee was for P2.1M investment solicited by
Facts: Hernandez from Mercader but placed under the
This is a petition for review on certiorari from the name of Guilas upon request of Hernandez so
CA which affirmed the RTC decision convicting

209
CRIMINAL LAW REVIEW DIGESTS
JUSTICE ROMEO CALLEJO NOTE: = Callejo Ponente

that the P21k would not be reflected in his the govt because it was meant to lower the
income tax return. (2) Hernandez testified that he tax base of Hernandez and thus evade
solicited the P2.1M and that he asked Andaya to payment of taxes on the finders fee.
place the finders fee in the name of one of his CA affirmed the trial court but stated in certain
employees so that he would not have to report a portions of its decision that Andaya was guilty of
higher tax base. (3) Andaya testified. estafa through falsification of commercial
In denying the charge against him, Andaya document.
claimed that the P21k finders fee was in fact
payable by AFPSLAI and denied Issue: Whether the CA contradicted the ruling of the
misappropriating the P21k for his personal trial court when the CA contradicted the ruling of the
benefit since it was turned over to Hernandez, trial court when it used estafa through falsification of
who was the true solicitor of the investment. commercial document whereas in the trial court,
Thus, since AFPSLAI was liable for P21k, then Andaya was convicted for the crime of "falsification of
no damage was done to the association. In private document.
addition, Andaya alleged that Guilas consented
to the entire set-up. Held: Andaya ACQUITTED based on reasonable
In sum, Andaya claimed that he merely approved doubt upon a review of the evidence.
the substitution of the name of Hernandez w/ that The alleged points of contradiction were the
of Guilas in the disbursement voucher upon the result of inadvertence in the drafting of the CAs
request of Hernandez. Andaya brushed aside the decision. The CA actually affirmed the conviction
imputation of condoning tax evasion by claiming of Andaya for the crime of falsification of private
that the issue in the proceeding was whether he document and not of estafa through falsification
defrauded AFPSLAI and not his alleged of commercial document.
complicity in tax evasion. Even if Andaya did not specify the factual and
Trial court found him guilty and charged him for legal aspects of his conviction that should be
falsification of private document. Andaya filed a reversed, the SC deems it proper to review any
motion for new trial but was denied. Instead, the question including those not raised under the
trial court reopened the case in order to avoid a principle that an appeal in criminal cases opens
miscarriage of justice. Andaya submitted the whole action for review.
documentary evidence to prove that AFPSLAI did Elements of falsification of a private document
not suffer any damage from the payment of the under Article 172, paragraph 2 in relation to
P21k finders fee. The trial court still convicted Article 171 of the Revised Penal Code are: (1)
Andaya for falsification of private document the offender committed any of the acts of
based on the following facts: falsification under Article 171 which, in the case
o Hernandez solicited from Mercader an at bar, falls under paragraph 2 of Article 171, i.e.,
investment of P2.1M for AFPSLAI causing it to appear that persons have
o Hernandez requested Andaya to place participated in any act or proceeding when they
the finders fee in the name of another did not in fact so participate; (2) the falsification
person was committed on a private document; and (3)
o Andaya caused it to appear in the the falsification caused damage or was
disbursement voucher that Guilas committed with intent to cause damage to a third
solicited the 2.1M investment party.
o The voucher served as the basis for the Although the public prosecutor designated the
issuance of the check for P21k offense charged in the information as estafa
o Guilas encashed check and turned over through falsification of commercial document,
the money to Andaya who gave it to petitioner could be convicted of falsification of
Hernandez private document, had it been proper, under the
The trial court ruled all the elements of well-settled rule that it is the allegations in the
falsification of private document were present: information that determines the nature of the
1. First, petitioner caused it to appear in the offense and not the technical name given by the
disbursement voucher, a private document, public prosecutor in the preamble of the
that Guilas, instead of Hernandez, was information.
entitled to the P21k finders fee. The facts alleged in the information are sufficient
2. Second, the falsification of the voucher was to constitute the crime of falsification of private
done w/ criminal intent to cause damage to document - (1) petitioner caused it to appear in

210
CRIMINAL LAW REVIEW DIGESTS
JUSTICE ROMEO CALLEJO NOTE: = Callejo Ponente

Disbursement Voucher No. 58380 that Guillas suffered no damage because it really owed the
was entitled to a finders fee from AFPSLAI in the P21k finders fee to Hernandez albeit the sum
amount of P21k when in truth and in fact no was initially paid to Guilas and only later turned
finders fee was due to him; (2) the falsification over to Hernandez. Clearly then, the third
was committed on Disbursement Voucher No. essential element of the offense as alleged in the
58380; and (3) the falsification caused damage to information, i.e., the falsification caused damage
AFPSLAI in the amount of P21k. to AFPSLAI in the amount of P21k, was not
The 1st element of the offense charged in the proven by the prosecution.
information was proven by the prosecution. The finding of the trial court that the falsification
Significantly, petitioner admitted his participation was done with criminal intent to cause damage to
in falsifying the voucher when he testified that he the government by lowering the tax base of
authorized the release of the voucher in the Hernandez, the Court does not agree because it
name of Guilas upon the request of Ernesto violates the constitutional right of Andaya to be
Hernandez. While petitioner did not personally informed of the nature and cause of the
prepare the voucher, he could be considered a accusation against him. Note that the information
principal by induction, had his conviction been is valid BUT there is a variance between the
proper, since he was the president and general allegation in the information and proof adduced
manager of AFPSLAI at the time so that his during the trial with respect to the third essential
employees merely followed his instructions in element of falsification of private document , i.e.,
preparing the falsified voucher. the falsification caused damage or was
The 2nd element of the offense charged in the committed with intent to cause damage to a third
information, i.e., the falsification was committed party. We find this variance material and
in Disbursement Voucher No. 58380, a private prejudicial to petitioner which, perforce, is fatal to
document, is likewise present. The public his conviction in the instant case.
prosecutor erroneously characterized the If Andaya were to be convicted based on his
disbursement voucher as a commercial intent to cause damage to the government, this
document but as correctly ruled by the trial court, would be unconstitutional, because Andaya was
the subject voucher is a private document only; it not forewarned that he was being prosecuted for
is not a commercial document because it is not a intent to cause damage to the government. It
document used by merchants or businessmen to would be simply unfair and underhanded to
promote or facilitate trade or credit convict petitioner on this ground not alleged while
transactions nor is it defined and regulated by the he was concentrating his defense against the
Code of Commerce or other commercial ground alleged. Thus, Andaya was deprived of
law. Rather, it is a private document, which has his opportunity to object to the evidence
been defined as a deed or instrument executed presented by the prosecution on the ground that
by a private person without the intervention of a the evidence did not conform to the allegations in
public notary or of other person legally the information for the simple reason that no such
authorized, by which some disposition or evidence was presented by the prosecution to
agreement is proved, evidenced or set forth, begin with.
because it acted as the authorization for the
release of the P21k finders fee to Guilas and as
the receipt evidencing the payment of this finders GALEOS V. PEOPLE & ONG V. PEOPLE / Feb. 9, 2011
fee.
However, as to the 3rd element, the same was FACTS: Galeos and Ong were charged and found guilty
not proven. In the information, it was alleged that by the Sandiganbayan of falsification of public documents
petitioner caused damage in the amount of under Article 171, Paragraph 415 of the RPC; Galeos with
P21,000.00 to AFPSLAI because AFPSLAI did 4 counts and Ong with 8 counts.
not owe P21k to Guilas. However, contrary to
these allegations, Andaya was able to prove that
AFPSLAI owed a finders fee in the amount of 15 Art. 171. Falsification by public officer, employee or notary or
ecclesiastic minister. The penalty of prision mayor and a fine
P21k although not to Guilas but to Ernesto not to exceed 5,000 pesos shall be imposed upon any public
Hernandez. Considering that Hernandez was officer, employee, or notary who, taking advantage of his official
able to solicit a P2.1M investment from Mercader, position, shall falsify a document by committing any of the
following acts:
it follows that he was entitled to receive the
4. Making untruthful statements in a narration of
finders fee in the amount of P21k. AFPSLAI facts;

211
CRIMINAL LAW REVIEW DIGESTS
JUSTICE ROMEO CALLEJO NOTE: = Callejo Ponente

Ong was the Mayor of the Municipality of Naga, what is punished is the violation of the public faith and the
Cebu. Ong extended permanent appointments to Galeos destruction of the truth as therein solemnly proclaimed.
and Federico T. Rivera for the positions of Construction The first element was proven. The question of
and Maintenance Man and Plumber I, respectively, in the whether or not persons are related to each other by
Office of the Municipal Engineer. The prosecution alleged consanguinity or affinity within the fourth degree is one of
that on several occasions, Galeos and Rivera (River later fact. Contrary to petitioners assertion, statements
died so the charges were dropped as to him) falsified their concerning relationship may be proved as to its truth or
individual Statements of Assets, Liabilities and Net Worth falsity, and thus do not amount to expression of opinion.
(SALN). On 4 occasions, Galeos either answered No or When a government employee is required to disclose his
left blank the boxes pertaining to the question of whether relatives in the government service, such information
th
he was related within the 4 degree of consanguinity or of elicited therefore qualifies as a narration of facts
affinity to anyone working in the government. The SALNs contemplated under Article 171 (4) of the Revised Penal
were filed by Galeos and subscribed and sworn to before Code, as amended. Further, it bears to stress that the
Ong. It was later found out that Galeos and Ong were first untruthful statements on relationship have no relevance to
degree cousins, as their mothers were sisters. the employees eligibility for the position but pertains
Galeos claimed that he was not being untruthful rather to prohibition or restriction imposed by law on the
when he merely left the box blank (unanswered), while appointing power.
Ong argued that the subject SALN do not contain any The second element is likewise present. Legal
untruthful statements containing a narration of facts and obligation means that there is a law requiring the
that there was no wrongful intent of injuring a third person disclosure of the truth of the facts narrated. Permanent
at the time of the execution of the documents, and that he employees employed by local government units are
cannot be held liable for falsification for merely required to file the following: (a) sworn statement of
administering the oath in a document since it is not among assets, liabilities and net worth (SALN); (b) lists of
the legal obligations of an officer administering the oath to relatives within the fourth civil degree of consanguinity or
certify the truthfulness and/or veracity of the contents of affinity in government service; (c) financial and business
the document. Both accused claimed that they had no interests; and (d) personal data sheets as required by
knowledge that they were in fact related and there was no law. A similar requirement is imposed by Section 8 (B) of
intent on their part to make the untruthful statements. Republic Act No. 6713 otherwise known as the Code of
Conduct and Ethical Standards for Public Officials and
ISSUE: Whether both accused were guilty of falsification Employees,
YES.
The third element was also satisfied.
HELD: All the elements of falsification of public documents As to Ong:
by making untruthful statements have been established by As chief executive and the proper appointing
the prosecution. authority, Ong is deemed to have issued the certification
The elements of falsification under Article 171, recommending to the CSC approval of Galeos
par. 4 are as follows: appointment. Since Ong was duty bound to observe the
(a) the offender makes in a public prohibition on nepotistic appointments, his certification
document untruthful statements in a stating compliance with Section 79 of R.A. No. 7160
narration of facts; constitutes a solemn affirmation of the fact that the
(b) he has a legal obligation to disclose appointee is not related to him within the fourth civil
the truth of the facts narrated by him; and degree of consanguinity or affinity. Having executed the
the facts narrated by him are certification despite his knowledge that he and Rivera
absolutely false. were related to each other within the fourth degree of
It must also be proven that the public officer or affinity, as in fact Rivera was his cousin-in-law because
employee had taken advantage of his official position in the mother of Riveras wife is the sister of Ongs mother,
making the falsification. In falsification of public document, Ong was guilty of falsification of public document by
the offender is considered to have taken advantage of his making untruthful statement in a narration of facts. He also
official position when (1) he has the duty to make or took advantage of his official position as the appointing
prepare or otherwise to intervene in the preparation of a authority who, under the Civil Service rules, is required to
document; or (2) he has the official custody of the issue such certification.
document which he falsifies. Likewise, in falsification of
public or official documents, it is not necessary that there As to Galeos
be present the idea of gain or the intent to injure a third As to Galeos contention that leaving the boxes in blank
person because in the falsification of a public document, cannot be considered as untruthful, the Court held that

212
CRIMINAL LAW REVIEW DIGESTS
JUSTICE ROMEO CALLEJO NOTE: = Callejo Ponente

one is guilty of falsification in the accomplishment of his


information and personal data sheet if he withholds RATIO:
material facts which would have affected the approval of The elements of the crime of falsification under Article 171
his appointment and/or promotion to a government (6) of the Revised Penal Code are:
position. By withholding information on his relative/s in the (1) that there be an alteration (change) or intercalation
government service as required in the SALN, Galeos was (insertion) on a document;
guilty of falsification considering that the disclosure of such (2) that it was made on a genuine document;
relationship with then Municipal Mayor Ong would have (3) that the alteration or intercalation has changed the
resulted in the disapproval of his permanent appointment meaning of the document; and
pursuant to Article 168 (j) (Appointments), Rule XXII of the (4) that the changes made the document speak something
Rules and Regulations Implementing the Local false.
Government Code of 1991 (R.A. No. 7160). When these are committed by a private individual on a
private document the violation would fall under paragraph
2, Article 172 of the same code, but there must be, in
CONCERNED EMPLOYEES VS GENEROSO - ?? addition to the aforesaid elements, independent evidence
of damage or intention to cause the same to a third
person. Given the admissions of GARCIA that she altered
the receipt, and without convincing evidence that the
GARCIA VS CA
alteration was with the consent of private complainant, the
Court holds that all four (4) elements have been proven
FACTS: GARCIA was charged with Falsification of a
beyond reasonable doubt. As to the requirement of
Private Document by being altering a receipt for P5000 to
damage, this is readily apparent as it was made to appear
make it appear to be P55k. The receipt was the product of
that Alberto had received P50,000 when in fact he did not.
a verbal agreement between QUIJADA and GARCIA for
Hence, GARCIA's conviction.
the sale of formers house. A partial payment of P5k was
made by GARCIA, and he prepared two handwritten, one
for each of them. The deal went sour and GARCIA filed a
complaint for estafa against QUIJADA for his failure to PANUNCIO v. PEOPLE
execute a deed of sale and deliver the subject property.
Among the evidence she submitted was the copy of the Facts: Operatives of the LTO and the Special Mission
receipt she prepared. However, the receipt appeared to Group Task Force Lawin of the Presidential Anti-Crime
have been altered, by inserting fifty before five and the Commission (PACC) led by PNP Superintendent Panfilo
number 5 was inserted before 5,000 (basically made it Lascon and Senior Inspector Ouano, Jr. raided the
appear that P5k P55k) plus other changes.16 QUIJADA residence of Panuncio, who was a jeepney operator.
noticed the alterations and instituted criminal action They were armed with a search warrant and confiscated
against GARCIA. GARCIA admitted the alteration, but LTO documents, 17 pieces of private vehicle plates, a
countered that it was done in the presence and at the copy machine, typewriters, etc. One of the LTO docs
request of QUIJADA (no signature of QUIJADA coz he confiscated was MVRR No. 63231478 (Official Receipt
was in a hurry daw). GARCIA added that this was ata) issued to Manlite Transport Corporation. Panuncio
retaliation to the estafa case filed against QUIJADA. was arrested and brought to the PACC.
RTC: Against GARCIA. If she made changes in The LTO filed a complaint against Panuncio and
the receipt while Alberto was counting the money it would an Information was filed against him for violating Art. 172
not have taken more than five (5) seconds to affix his (1)17 in relation to Art. 17118 of the RPC. It was alleged
signature thereon even if he was in a hurry to leave.
Elements of Article 172 (2) in relation to Art 171(6) proven 17 Art. 172. Falsification by private individuals and use of falsified
BRD. documents. The penalty of prision correccional in its medium
and maximum periods and a fine of not more than P5,000 shall be
CA: Affirm conviction
imposed upon:
1. Any private individual who shall commit any of the
ISSUE: W/N GARCIA falsified the receipt (a private falsifications enumerated in the next preceding article in any
document) in violation of Art 172 in relation to Art 171. public or official document or letter of exchange or any other kind
of commercial document.
YES. 18 Art. 171. Falsification by public officer, employee or notary or
ecclesiastic minister. - The penalty of prision mayor and a fine
not to exceed 5,000 pesos shall be imposed upon any public
16 Inserted additional words 'Now covered by T.C.T. # 3998 R.D. officer, employee, or notary who, taking advantage of his official
Mandaluyong MM. the parties agree to execute of [sic] valid deed position, shall falsify a document by committing any of the
of conveyance covering the same sale, changed the date of the following acts:
receipt and put P55k on top of the receipt 1. Counterfeiting or imitating any handwriting, signature or rubric;

213
CRIMINAL LAW REVIEW DIGESTS
JUSTICE ROMEO CALLEJO NOTE: = Callejo Ponente

that the accused falsified the vital information as changed the meaning of the document within the context
appearing on the LTO Official Receipt. of Art. 171 (6).
Panuncio denied that she was the source of the Panuncio argues that the MVRR was not found in
falsified documents. She alleged that Manlite, which she her possession and that it was not proved that she had
used to co-own with her late husband, had already participated in the criminal act. The SC disagrees. The
stopped operating and the new business was under her falsified copy was found during a valid search conducted
name. She alleged that she was not home when the raid in her residence. It was issued in the name of Manlite,
took place and when she did get home, the authorities had which she admitted was owned by her and her late
already emptied her shelves. She was forced to sign the husband. Thus, there is a presumption that she falsified it
warrant, inventory receipt and certificate of orderly search. and she was using it for her benefit. The falsified
She claimed that she was charged with falsification document could be used for another vehicle operated by
because she refused the police authorities demand for Manlite to make it appear that it was validly registered with
money. the LTO.
The RTC founder her guilty for the crime of
falsification of a public document under Arts. 171 and 172. ARTICLE 203
The CA affirmed with modification as to the sentence.
PEOPLE v. SANDIGANBAYAN
Issue: Whether the elements of falsification of a public
document under Art. 172 (1) in relation to Art. 171 of the FACTS: Efren Alas was the President and Chief
RPC have been established - YES Operating Officer of the Philippine Postal Savings Bank
(PPSB). The charge against him was due to the alleged
Ratio: anomalous advertising contracts he entered with Bagong
The elements of the crime of falsification of a public Buhay Publishing Company which purportedly caused
document under these provisions are: damage and prejudice to the government.
1) That the offender is a private individual or a public It is a given fact that the Sandiganbayan has
officer or employee who did not take advantage of his jurisdiction only over public officers unless private persons
official position; are charged with them in the commission of the offenses.
2) That he committed any of the acts of falsification A public officer has been ruled, as a person whose duties
enumerated in Art. 171; and involve the exercise of discretion in the performance of the
3) That the falsification was committed in a public, official function of government.
or commercial document. Alas now challenges the jurisdiction of the
In this case, Panuncio is a private individual. The Sandiganbayan. Alas contends that while PPSB is a
MVRR is an official document issued by the LTO. It is the subsidiary of the Philippine Postal Corporation (a
owners copy of the OR of the payment of the vehicles government owned corporation), PPSB is not created by a
registration fee. Panuncio falsified the owners copy by special law. It was organized and incorporated under the
making it appear that it was issued to a vehicle of Manlite Corporation Code. Since the PPSB was not created by
with a plate number different from the vehicle for which it special law, then, its officers did not fall within the
was issued in the LTOs files. The alteration she made jurisdiction of the Sandiganbayan.

ISSUE: Does the Sandiganbayan have jurisdiction over


2. Causing it to appear that persons have participated in any act presidents, directors or trustees, or managers of
or proceeding when they did not in fact so participate; government-owned or controlled corporations organized
3. Attributing to persons who have participated in an act or and incorporated under the Corporation Code for
proceeding statements other than those in fact made by them;
4. Making untruthful statements in a narration of facts; purposes of the provisions of RA 3019, otherwise known
5. Altering true dates; as the Anti-Graft and Corrupt Practices Act? (YES, it has
6. Making any alteration or intercalation in a genuine document jurisdiction!)
which changes its meaning;
7. Issuing in an authenticated form a document purporting to be a
copy of an original document when no such original exists, or HELD: Congress enacted RA 8249 (An Act Further
including in such a copy of a statement contrary to, or different Defining The Jurisdiction Of The Sandiganbayan.
from, that of the genuine original; or Amending For The Purpose Presidential Decree No. 1606,
8. Intercalating any instrument or note relative to the issuance
thereof in a protocol, registry, or official book. As Amended, Providing Funds Therefor, And For Other
The same penalty shall be imposed upon any ecclesiastical Purposes) which states:
minister who shall commit any of the offenses enumerated in the Section 4, Jurisdiction. The Sandiganbayan shall
preceding paragraphs of this article, with respect to any record or
document of such character that its falsification may affect the exercise exclusive original jurisdiction in all cases
civil status of persons. involving:

214
CRIMINAL LAW REVIEW DIGESTS
JUSTICE ROMEO CALLEJO NOTE: = Callejo Ponente

a. Violations of Republic Act No. 3019, as (respondents) for violation of RA 3019, violation of RA
amended, otherwise known as the Anti- 6713 (Code of Conduct and Ethical Standards of
Graft and Corrupt Practices Act, Public Officials and Employees), and violation of SC
Republic Act No. 1379, and Chapter II, Resolution 2-9-2002.
Section, Title VII, Book II of the Revised - Cortes said the proceedings were irregular because
Penal Code, where one or more of the (1) he was not cross-examined to rebut the direct
accused are officials occupying the testimony of Reyes; (2) the lawyer daw of Reyes
following positions in the government, negotiated with the justices to reverse their denial of
whether in a permanent, acting or the Demurrer to Evidence.
interim capacity, at the time of the
commission of the offense, ISSUE: W/N the justices are liable for the charges against
(1) Officials of the executive them. NO.
branch occupying the positions of regional
director, and higher, otherwise classified HELD/RATIO: It must be stressed that as a matter of
as grade 27 and higher, of the policy, the acts of a judge in his judicial capacity are not
Compensation and Position Classification subject to disciplinary action. He cannot be subjected to
Act of 1989 (Republic Act No. 6758) liability civil, criminal or administrative for any of his
specifically including: official acts, no matter how erroneous, as long as he acts
xxx xxx xxx in good faith. Only judicial errors tainted with fraud,
(g) Presidents, dishonesty, gross ignorance, bad faith or deliberate intent
directors or trustees, or to do an injustice will be administratively sanctioned.
managers of government- It is also worth mentioning that the provisions
owned or controlled of Article 204 of the Revised Penal Code as to
corporations, state universities rendering knowingly unjust judgment refer to an
or educational institutions or individual judge who does so in any case submitted
foundations. (Italics ours) to him for decision and has no application to the
members of a collegiate court such as the
The legislature, in mandating the inclusion of Sandiganbayan or its divisions, who reach their
presidents, directors or trustees, or managers of conclusions in consultation and accordingly render
government-owned or controlled corporations within the their collective judgment after due deliberation. It also
jurisdiction of the Sandiganbayan, has consistently follows, consequently, that a charge of violation of the
refrained from making any distinction with respect to the Anti-Graft and Corrupt Practices Act on the ground
manner of their creation. that such a collective decision is unjust cannot
The deliberate omission, in our view, clearly reveals prosper.
the intention of the legislature to include the presidents, An administrative complaint is not an appropriate
directors or trustees, or managers of both types of remedy where judicial recourse is still available, such as a
corporations within the jurisdiction of the Sandiganbayan motion for reconsideration, an appeal, or a petition for
whenever they are involved in graft and corruption. Had it certiorari, unless the assailed order or decision is tainted
been otherwise, it could have simply made the necessary with fraud, malice, or dishonesty.
distinction. But it did not. In the case at bar, the record is bereft of any
showing of wrongful, improper or unlawful conduct on the
ARTICLE 204 part of the respondent justices. The complainant failed to
substantiate his claim of corruption and bribery, and
CORTES V. CHICO-NAZARIO merely relied on mere conjectures and suppositions.
Charges based on mere suspicion and speculation cannot
FACTS: be given credence.
- Leonides Cortes filed a case for violation of RA 3019 Finally, as correctly pointed out by the Court
(Anti-Graft) against Dolores Reyes who was the Administrator, the respondents cannot be held liable for
municipal treasurer of Samal, Bataan. Cortes alleged violation of Supreme Court Resolution No. 2-9-2002, as
that Reyes maliciously caused the sale of the formers the same does not define nor punish an offense, but
property and did not even notify him about the auction merely defines the extent of the consequence of an
sale. This case was submitted to the Sandiganbayan. administrative complaint if filed against Justices of the
- Cortes now assails the regularity of the proceedings Court of Appeals, the Sandiganbayan, Judges of the
in the Sandiganbayan. Hence, he filed a case against Regular and Special Courts and court officials who are
the Sandiganbayan Justices 5
th
Division lawyers.

215
CRIMINAL LAW REVIEW DIGESTS
JUSTICE ROMEO CALLEJO NOTE: = Callejo Ponente

failure on the part of the complainant to prove the same


warrants the dismissal of the administrative complaint.
EDUARDO DIEGO V. JUDGE CASTILLO Anent the charge of gross ignorance of the law:
The error must be gross or patent, malicious, deliberate
Facts: In 1965 Lucena Escoto, adopting the name of or in evident bad faith. It is only in this latter instance,
Crescencia Escoto (Cresencia), married Jorge de Perio, when the judge acts fraudulently or with gross ignorance,
Jr. (Jorge) before Mayor Reyna of Dagupan city. In 1978, that administrative sanctions are called for as an
Jorge was able to secure a Decree of Divorce with the imperative duty of this Court.
Family District Court of Harris County, Texas. In 1987, As a matter of public policy, the acts of a judge in
Crescencia married Manuel P. Diego before the Rev. Fr. his official capacity are not subject to disciplinary action,
Clemente T. Godoy in Dagupan City. The marriage even though such acts are erroneous. Good faith and
contract showed that Cresencia used and adopted the absence of malice, corrupt motives or improper
name Lucena Escoto, with a civil status of single. considerations are sufficient defenses in which a judge
Eduardo Diego, brother of Manuel Diego, filed a charged with ignorance of the law can find refuge. It does
criminal case for bigamy against Crescencia. Judge not mean, however, that a judge, given the leeway he is
Castillo ruled that Cresencia married Manuel on the accorded in such cases, should not evince due care in the
honest belief that she was free to do so by virtue of the performance of his adjudicatory prerogatives.
decree of divorce; this mistake of fact is sufficient to acquit In this case, a respondent Judge had been less
Cresencia of the charges. Eduardo filed an administrative than circumspect in his study of the law and jurisprudence
complaint against Respondent Judge for allegedly applicable to the bigamy case. The SC had previously
knowingly rendering an unjust judgment in a criminal case held in People v. Schneckenburger, that the accused who
and/or rendering judgment in gross ignorance of the law. secured a foreign divorce, and later remarried in the
Philippines, in the belief that the foreign divorce was valid,
Issue: W/N Judge Castillo knowingly rendered an unjust is liable for bigamy.
judgement in the criminal case of bigamy and/or rendered The error committed by respondent Judge being
judgement in gross ignorance of the law. gross and patent, the same constitutes ignorance of the
law of a nature sufficient to warrant disciplinary action.
Held and Ratio: Judge not guilty of knowingly rendering an
unjust judgment but found guilty of rendering judgment in * Judge Castillo rendered the decision before the
gross ignorance of the law. effectivityof A.M. No. 01-8-10-SC which classified gross
Knowingly rendering an unjust judgment is a ignorance of the law as a serious charge and penalized
criminal offense defined and penalized under Article the offense with a fine of not less than P20,000 but not
204 of the RPC. For conviction to lie it must be proved more than P40,000. As such SC ruled that the sanction be
that the judgment is unjust and that the judge knows that it a fine in the amount of P10,000.
is unjust. Knowingly means consciously, intelligently,
willfully or intentionally. It is firmly established in this
jurisdiction that for a judge to be held liable for knowingly ANG V. JUDGE ASIS
rendering an unjust judgment, it must be shown that the
judgment is unjust as it is contrary to law or is not Facts: Gina Ang ran for Mayor in Kawayan, Biliran in the
supported by the evidence, and that the same was made 1998 elections. She lost to Caridad Atok who was the one
with conscious and deliberate intent to do an injustice. declared mayor. Ang then filed an election protest which
The law requires that (a) the offender is a judge; was assigned to Judge Asis.
(b) he renders a judgment in a case submitted to him for While the case was pending, Judge Asis
decision; (c) the judgment is unjust; (d) he knew that said allegedly told Angs lawyers that hed decide in favour of
judgment is unjust. In order to hold a judge liable, it must Ang in exchange for money. Unknown to Ang, her father,
be shown that the judgment is unjust and that it was made on three separate dates (Oct 98, Jan 99, April 99), gave
with conscious and deliberate intent to do an Judge Asis a total of P 140,000.
injustice. That good faith is a defense to the charge of In Dec 98, Angs lawyers told her that Asis had
knowingly rendering an unjust judgment remains the law. requested assistance in the promotion of his brother, then
Even assuming that a judge erred in acquitting an Examiner II at the Bureau of Customs, through Angs
accused, she still cannot be administratively charged cousin, Atty. Ramon Salazar, Jr., who was the Chief of
lacking the element of bad faith, malice or corrupt Staff of the Customs Commissioner. Ang refused. Her
purpose. Malice or bad faith on the part of the judge in family contacted Salazar though and the brother was
rendering an unjust decision must still be proved and promoted.

216
CRIMINAL LAW REVIEW DIGESTS
JUSTICE ROMEO CALLEJO NOTE: = Callejo Ponente

Ang already wanted to file an administrative in fact, adduced any proof to show that impropriety
complaint against Asis because the protest had not yet attended the issuance of the subject decision. To reiterate,
been resolved despite the lapse of the 60 day period. She bad faith is not presumed and he who alleges the same
then learned that Asis asked for her help in order for his has the onus of proving it. In view of the fact that Angrelied
son to be admitted for training at the Philippine Heart mainly on second-hand information to prove her charges,
Center. Ang initially refused but ended up following her her complaint is reduced into a bare indictment or mere
lawyers request her lawyers request to bring Asis son to speculation.
Manila and even paid for the latters plane fare and Indeed, the Rules, even in an administrative
accommodation until his application was granted by the case, demand that if the respondent judge should be
Philippine Heart Center. disciplined for grave misconduct or any graver offense, the
In Jan. 2000 Asis allegedly called Ang, asking for evidence against him should be competent and should be
P 4,000 for his son. Ang didnt have cash at that time. She derived from direct knowledge. The judiciary to which
asked a friend to write a check for Judge Asis. respondent belongs demands no less. Before any of its
In March of 2000, Judge Asis finally decided the members could be faulted, competent evidence should be
case in favour of Caridad Atok. presented, especially since the charge is penal in
Ang then charged Judge Asis with Bribery, character.
Extortion and Violation of the Anti-Graft and Corrupt To hold a judge liable for knowingly rendering an
Practices Act with the Office of the Court Administrator. unjust judgment or order, it must be shown beyond
Judge Asis denied asking for help. He said his brother was reasonable doubt that the judgment or order is unjust and
promoted through the recommendation of his superiors that it was made with a conscious and deliberate intent to
and of Mayor Jinggoy Estrada. Regarding his son, he do an injustice.
alleged that Enrique, Jr. went to Manila to apply for work In short, this Court cannot give credence to
at the Heart Center upon the advice of a family friend. charges based on mere suspicion or speculation. While
While in Manila, his son stayed with his uncle, Nestorio this Court will never tolerate or condone any act, conduct
Asis, who works at the Bureau of Customs. Asis also said or omission that would violate the norm of public
that his son was prepared to pay for training in cash but accountability or diminish the peoples faith in the judiciary,
that he suddenly received a check for P4,000.00 from neither will it hesitate to shield those under its employ from
Angs friend. unfounded suits that only serve to disrupt rather than
The case was referred to CA Justice Perlita Tria- promote the orderly administration of justice.
Tirona. She recommended that the bribery and extortion While the bribery and extortion cases were
charges be dismissed. She however suggested severe dismissed, Asis was reprimanded for his incompetence in
reprimand with respect to Asis delay in deciding the case. not acting speedily on the election protest. He was also
Tirona basically said that while Ang learned of admonished to be more circumspect and to act with more
Asis request through her lawyers, she never presented dispatch in the performance of his judicial functions.
these lawyers (Lee, Matriano) to corroborate Angs
allegations. It was also never proven whether Salazar,
could have really helped in the promotion. Judge Asis on IN RE BORROMEO
the other hand was able to show Jinggoy really helped.
With respect to the check, while it was indeed made by FACTS: Joaquin Borromeo is not a lawyer, but has
Angs witness, it was never proven that Ang solicited the apparently read some law books and come to possess
check. awareness of some substantive legal principles and
procedural rules. For 16 years, he has been instituting and
Issue: Whether or not Judge Asis was guilty of bribery prosecuting legal proceedings in various courts. (just in
and extortion. case Sir asks who the hell this guy is)

Held: No! NOTE: The significance of this description is that


Borromeo was held guilty for contempt for litigating issues
Ratio: For administrative liability to attach it must be already declared to be without merit, rendered adversely
established that Asis was moved by bad faith, dishonesty, to him in many suits and proceedings, rulings which had
hatred or some other motive. Evident bad faith connotes a become final and executory, obdurately and unreasonably
manifest deliberate intent on the part of the accused to do insisting on the application of his own individual version of
wrong or cause damage. the rules, founded on nothing more than his personal (and
The record does not show that Asis was moved quite erroneous) reading of the Constitution and the law;
by ill-will or bad faith in rendering the adverse judgment, or he has insulted the judges and court officers, including the
that his ruling was manifestly unjust. Complainant has not, attorneys appearing for his adversaries, needlessly

217
CRIMINAL LAW REVIEW DIGESTS
JUSTICE ROMEO CALLEJO NOTE: = Callejo Ponente

overloaded the court dockets and sorely tried the patience judge would be inconsistent with the possession of this
of the judges and court employees who have had to act on freedom, and would destroy that independence without
his repetitious and largely unfounded complaints, which no judiciary can be either respectable or useful.
pleadings and motions.
Borromeo obtained loans or credit
accommodation from 3 banks: Traders Royal Bank (TRB), LINDA M. SACMAR vs. JUDGE AGNES REYES-
United Coconut Planters Bank (UCPB), and Security Bank CARPIO
and Trust Co. (SBTC). He constituted mortgages over
immovables belonging to him, or members of his family, or FACTS: In the case filed by complainant Sacmar against
third persons. He failed to pay. When demands were Zoren Legaspi in the MTC of Pasig City, the latter was
made upon him, he imposed his own terms and conditions convicted for grave threats and was sentenced to arresto
which were quite inconsistent with those agreed upon with mayorand and to pay complainant moral damages of
his obligees or those prescribed by law. When the banks twenty thousand pesos (P20,000.00). Upon appeal by
refused, Borromeo brought suits right and left. He sued Legaspi, RTC Judge Reyes-Carpio (herein respondent)
the banks, its officers, and even the lawyers who modified the decision, finding the accused guilty only of
represented the banks. He also sued the public Other Light Threats under Article 265 of the RPC,
prosecutors, the judges of trial courts, CA, and SC who reducing the penalty to arresto menor and to pay moral
rendered judgments adverse to him, as well as the damages of ten thousand pesos (P10,000).
clerks of court and court employees. Complainant claims that respondent judge
wittingly afforded unwarranted benefits to the accused
ISSUE: WON a party can file an administrative or criminal which caused undue injury to her as private complainant in
complaint against the judge for rendition of unjust the case. She likewise avers that respondent judge
judgment??? NO. exhibited manifest partiality towards the accused when
she disregarded the evidence on record in modifying the
NOTE: Borromeo was alleging that the judges or justices decision of the MTC by downgrading the conviction of
rendered manifestly unjust judgments or interlocutory Legaspi from Grave Threats to Other Light Threats
orders (not in accord with evidence, with law or thereby reducing the criminal and civil liabilities of Legaspi.
jurisprudence, tainted by grave abuse of discretion). Respondent vehemently denied all the charges,
claiming that she rendered her decision in good faith,
RULING: In the case of In Re Wenceslao Laureta, the SC without malice, and without any conscious and deliberate
ruled that the provisions of Article 204 of the Revised intent to favor a movie actor whom she does not even
Penal Code as to "rendering knowingly unjust judgment," know.
refer to an individual judge who does so "in any case In her Reply, complainant pointed out that
submitted to him for decision" and even then, it is not the respondent judge, in her Comment, failed to explain why
prosecutor who would pass judgment on the "unjustness" she unilaterally downgraded the conviction of accused
of the decision rendered by him but the proper appellate Legaspi. In effect, respondent judge has impliedly
court with jurisdiction to review the same, either the Court admitted the charges against her when she failed to
of Appeals and/or the Supreme Court. The said penal specifically challenge these charges. Complainant
article has no application to the members of a collegiate assailed the claim of respondent judge that the
court such as this Court or its Divisions who reach their downgrading of the offense was rendered in good faith
conclusions in consultation and accordingly render their and without malice.
collective judgment after due deliberation.
To subject to the threat and ordeal of ISSUE: Whether or not Respondent Judge rendered an
investigation and prosecution, a judge, more so a member unjust judgment pursuant to Art. 204, RPC and for
of the Supreme Court for official acts done by him in good violation of Section 3(e) of RA 3019, the Anti-Graft and
faith and in the regular exercise of official duty and judicial Corrupt Practices Act. NO.
functions is to subvert and undermine that very
independence of the judiciary, and subordinate the HELD: As a rule, the acts of a judge which pertain to his
judiciary to the executive. "For it is a general principle of judicial functions are not subject to disciplinary power
the highest importance to the proper administration of unless they are committed with fraud, dishonesty,
justice that a judicial officer in exercising the authority corruption or bad faith. To hold otherwise would be to
vested in him, shall be free to act upon his own render judicial office untenable, for no one called upon to
convictions, without apprehension of personal try the facts or interpret the law in the process of
consequences to himself. Liability to answer to everyone administering justice can be infallible in his judgment.
who might feel himself aggrieved by the action of the

218
CRIMINAL LAW REVIEW DIGESTS
JUSTICE ROMEO CALLEJO NOTE: = Callejo Ponente

A perusal of the records, particularly the assailed claims that the judge should have considered the fitness of
decision of respondent judge, hardly show that respondent Rosana first).
judge has indeed knowingly and deliberately rendered an According to the complainant, the actuations of
unjust judgment. Complainant failed to satisfactorily show respondent judge showed abuse of authority and
that respondent judge acted in bad faith, with malice or in ignorance of the law. In his comment, respondent judge
willful disregard of her right as a litigant. Although the denied the complainants allegations and maintained that
application and interpretation of the law by respondent the questioned order finds support in law and
judge differed from that of the judge of the Metropolitan jurisprudence. The OCA submitted its
Trial Court, complainant cannot sweepingly claim that report recommending the dismissal of the complaint for
respondent judge knowingly rendered an unjust judgment. lack of merit. The Court finds the recommendation of the
For a charge of knowingly rendering an unjust judgment to OCA to be well-taken.
prosper, it must be shown that the judgment was unjust,
and not that the judge merely committed an error of Issues: 1. WON the order of respondent judge issuing the
judgment or took the unpopular side of a controversial writ constitutes abuse of authority -NO! 2. WON the order
point of law. He must have known that his judgment was of respondent judge ordering the provisional custody of
indeed unjust. The failure of a judge to correctly interpret the four-year old child to her mother constitutes ignorance
the law or to properly appreciate the evidence presented of the law -NO!
does not necessarily render him administratively liable. (Side issue)Section 5, Rule 102 of the Rules of Civil
After several exchanges of pleadings between Procedure on Habeas Corpus, shows that a court may
the parties, Court Administrator Presbitero J. Velasco, Jr. grant the writ if it appears upon presentation of the petition
rendered the opinion that this administrative matter is not that the writ ought to be issued. Clearly therefore,
a proper subject of an administrative investigation. He respondent judge was well within his authority when he
pointed out that, pursuant to the rule in Wingarts v. Mejia, issued the writ as no hearing is required before a writ may
complainant failed to show beyond reasonable doubt that be issued.The law grants the mother the custody of a child
the assailed judgment was unjust and that respondent under 7 years of age. In the case at bar, it is
judge consciously and deliberately intended to do injustice uncontroverted that the child subject of the habeas
to her by rendering such unjust judgment. Accordingly, on corpus case is only four years old, thus, the custody
October 31, 2001, the Office of the Court Administrator should be given to the mother.
recommended the dismissal of the instant administrative
complaint against respondent judge. (Main issue)The acts of a judge which pertain to his
judicial functions are not subject to disciplinary power
unless they are committed with fraud, dishonesty,
CHARLTON TAN V. JUDGE ADRE corruption or bad faith. As a matter of policy, in the
absence of fraud, dishonesty or corruption, the acts of a
FACTS: This is an administrative complaint for gross judge in his judicial capacity are not subject to disciplinary
ignorance of the law filed by Charlton Tan against Judge action even though such acts are erroneous. Otherwise, a
Adre. judicial office would be untenable, for no one called upon
Complainant Charlton Tan was the respondent in to try the facts or interpret the law in the administration of
a habeas corpus case filed by his wife Rosana Reyes- justice can be infallible.
Tan. Respondent judge issued the writ prayed for and He cannot be subjected to liability - civil, criminal,
ordered complainant to bring before the court the body of or administrative - for any of his official acts, no matter
their daughter, Charlene Reyes Tan. On the scheduled how erroneous, as long as he acts in good faith. In such a
date of hearing, the court provisionally turned over the case, the remedy of the aggrieved party is not to file an
custody of the child to the mother. A motion for administrative complaint against the judge but to elevate
reconsideration praying for the return of the child to the error to the higher court for review and correction,
complainant or a shared custody be given to the parents because an administrative complaint is not an appropriate
was filed. Allegedly sensing the partiality of respondent remedy where judicial recourse is still available. The court
judge, complainant filed a motion to inhibit him, but the has to be shown acts or conduct of the judge clearly
same was denied. indicative of arbitrariness or prejudice before the latter can
Complainant alleged that respondent judge acted be branded the stigma of being biased and partial.
with grave abuse of authority when he at once issued the In the case at bar, the questioned orders were
Order granting the issuance of a writ of habeas corpus issued after considering the pleadings filed by the parties.
without first conducting a hearing for that purpose and There is not a scintilla of evidence that the respondent
when he hurriedly turned over the custody of their judge, in issuing the questioned orders, was impelled by
daughter to his wife Rosana on the day of the hearing ( he ill-will, malice, revenge, personal animosity, impulse to do

219
CRIMINAL LAW REVIEW DIGESTS
JUSTICE ROMEO CALLEJO NOTE: = Callejo Ponente

injustice, greed, corrupt consideration or other similar interlocutory order must have the elements: 1) that the
motive. offender is a judge; 2) that he performs and of the
It must be stressed that an administrative following acts: a) he knowingly renders unjust interlocutory
complaint against a judge cannot be pursued order or decree; or b) he renders a manifestly unjust
simultaneously with the judicial remedies accorded to interlocutory order or decree through inexcusable
parties aggrieved by his erroneous order or judgment. negligence or ignorance.
Administrative remedies are neither alternative nor OCA perceived no evidence that Gabo issued the
cumulative to judicial review where such review is questioned order knowing it to be unjust; and neither is
available to the aggrieved parties and the same has not there proof of conscious and deliberate intent to do an
been resolved with finality until there is a final declaration injustice. SC said that to make the judge liable, proof is
by the appellate court that the challenged order or beyond reasonable doubt which was not established in
judgment is manifestly erroneous, there will be no basis to this case.
conclude whether respondent judge is administratively Gross ignorance of the law --- Murder is
liable. Gross ignorance of the law is a serious accusation, punishable by reclusion to death. Thus, being a capital
and a person who accuses a judge of this very serious offense, Judge Gabo should been mindful that bail cannot
offense must be sure of the grounds for the accusation. be allowed as a matter of right. Summary proceeding must
have been conducted to determine the strength of the
ARTICLE 206 evidence against the accused as to entitle him to post bail.
Side note --- As regards the charge of violating
LUCIA LAYOLA VS JUDGE BASILIO GABO Section 3 (e) of the Anti-Graft and Corrupt Practices Act,
the OCA stressed that the important element of the
Facts: Layola filed a complaint charging Gabo of violation offense, which is damage or injury to the complainant, or
of the Anti-Graft and Corrupt practices act for issuing an manifest partiality shown to any party, lacks evidentiary
unjust interlocutory order and with gross ignorance of the support. There is no allegation of any injury suffered by
law. Office of the Court Administrator found the judge not the complainant as a result of the conduct or actuation of
liable. the respondent judge, nor was there any showing of
It appears that Layola filed a complaint with the undue benefit or advantage given to the adverse party
Ombudsman charging SPO2 Leopoldo German and PO2 under the orders complained of.
Tomasito Gagui of Sta. Maria, Bulacan with homicide.
Allegedly, these police officers killed Layolas son. ARTICLE 208
Information for murder was drafted and the case was
indorsed to the prosecutor of Bulacan. The case was US V. MENDOZA
docketed in Judge Gabos sala. (one page case)
Chief of Police of Sta. Maria filed a petition to
take custody of German. The petition was based on FACTS: Mangunay mounted a carabao with a torch in
PD971, PD1184 and EO106 wherein it is provided that a hand. He went to the house of Mateo to ask for food.
police officer charged with any crime may be placed under However, his carabao went too close to the house that the
the custody of the immediate superior officers upon lighted torch he was carrying touched the roof, catching
request and these superior officers shall be responsible for fire. The house was almost burned down if not for the
the appearance the accused during trial. Also, under the timely intervention of neighbors.
provisions of the PDs and EO, the case is service The owner of the house reported the incident to
connected because the offense stemmed from the death Mendoza, the lieutenant of the said barrio. However, he
of Layolas son, a violator of the law, who was then inside did not do anything and did not even reported the incident.
the municipal jail. He instructed Mangunay to go home. Because of this,
Judge Gabo granted the petition. MR was filed Mendoza was charged and convicted as an accessory to
for that order but was denied. the crime of arson.

Issue: Whether Judge Gabo knowingly rendered an unjust SC: acquitted.


judgment? NO!
Whether there was gross ignorance of the law in Had the accused incurred responsibility by his conduct, he
ordering release of the accused? YES! should have been charged with the crime of 'prevaricacion'
for neglect of the duties of his office by maliciously failing
Held: Knowingly rendering an unjust judgment --- SC to move the prosecution and punishment of the
sustained the findings of the Office of the Court delinquent.
Administrator in the case. Knowingly rendering an unjust

220
CRIMINAL LAW REVIEW DIGESTS
JUSTICE ROMEO CALLEJO NOTE: = Callejo Ponente

However, he was charged as an accessory. In ARTICLE 210


the case at bar, since the principal was acquitted since it
was proven that the burning was merely incidental, then GARCIA V. SANDIGANBAYAN
the accessory must likewise be acquitted. (I think the important part here is the identification of those
government positions which would be within the scope of
ARTICLE 209 the R.A. 1379.)

US V. LARANJA Facts: Petitioner Major General Carlos F. Garcia was the


(magulong case!) Deputy Chief of Staff for Comptrollership of the AFP. Atty.
Maria Olivia Elena A. Roxas from the office of the
Laranja, with other men, went to the house of Candoy. ombudsman, after due investigation, filed a complaint
They began singing and drinking, and later a quarrel against petitioner with the Ombudsman, for violation of
ensued which resulted in the death of Candoy and one Sec. 8, in relation to Sec. 11 of Republic Act (R.A.) No.
Ando. Complaints for homicide were filed Laranja killed 6713,19 violation of Art. 183 of the RPC, and violation of
Candoy, Iyon killed Ando. Section 52 (A)(1), (3) and (20) of the Civil Service Law.
When Laranjas case was called, an agreement Based on this complaint, a case for Violations of R.A.
was entered into between his counsel de officio (Mr. No. 1379,20 Art. 183 of the Revised Penal Code, and
Lozano) and the fiscal. This agreement was to the effect Sec. 8 in relation to Sec. 11 of R.A. No. 6713, was
that the testimony of Iyon, whose case was tried first, will filed against petitioner.
be admitted in the case of Laranja also. Laranja was later Petitioners wife Clarita Depakakibo Garcia, and
convicted of homicide. their three sons, Ian Carl, Juan Paolo and Timothy Mark,
Laranja, now with a different counsel insists that all surnamed Garcia, were impleaded in the complaint for
the trial court erred in allowing the agreement between the violation of R.A. No. 1379 insofar as they acted as
fiscal and the counsel de officio. He also raised the conspirators, conduits, dummies and fronts of petitioner in
argument that the counsel assigned to him by the court receiving, accumulating, using and disposing of his ill-
was not of his choice he wants a new trial. The court gotten wealth.
said that the only way for the case to be remanded for a The Republic of the Philippines, acting through
new trial is if the counsel de officio was disqualified. To the Ombudsman, filed before the Sandiganbayan, a
know whether he was disqualified, it must be asked, did Petition for the Issuance of a Writ of Preliminary
he change positions/ sides? Attachment against petitioner, his wife, and three sons,
Iyon killed Ando. Laranja killed Candoy. Candoy seeking the forfeiture of unlawfully acquired properties
is the father of Iyon. Ando is the bro-in-law of Laranja. The under Sec. 2 of R.A. No. 1379.
counsel de officio assisted the prosecution in Iyons case It was alleged that the Office of the Ombudsman,
to show that Iyon was guilty of killing Ando. In defending after conducting an inquiry similar to a preliminary
Laranja, it was his duty to show that Candoy the victim, investigation in criminal cases, has determined that a
was the aggressor. (RECAP: the same lawyer was for prima facie case exists against Maj. Gen. Garcia and the
prosecution in the Iyon case, Defense for Laranja) other respondents therein who hold such properties for,
with, or on behalf of, Maj. Gen. Garcia, since during his
Issue: is Laranja entitled to a new trial? Yes! incumbency as a soldier and public officer he acquired
huge amounts of money and properties manifestly out of
Ruling: There was no clear showing when counsel de proportion to his salary as such public officer and his other
officio was appointed for Laranja whether before or after lawful income, if any.
the trial of Iyon. Either way, there was opportunity for the Petitioner (as respondent a quo) filed a Motion to
lawyer to act in a double capacity and this must be Dismiss on the ground of lack of jurisdiction of the
avoided. Courts will give no approval for Lozanos Sandiganbayan over forfeiture proceedings under R.A.
conduct. He should have called the attention of the court No. 1379. On even date, petitioner filed the present
to these facts, and the court would definitely relieve him of Petition, raising the same issue of lack jurisdiction on the
as counsel de officio. Public policy prohibits him from part of the Sandiganbayan. Petitioner argues in this
defending Laranja under the circumstances because the Petition that the Sandiganbayan is without jurisdiction over
relation between attorney and client is one of trust and the civil action for forfeiture of unlawfully acquired
confidence of the highest degree. He learns all the facts of
his clients case both the strong and the weak points. No 19 Code of Conduct of Ethical Standards for Public Officials and
opportunity must be given to lawyers to take advantage of Employees; 20 February 1989.
20 An Act Declaring Forfeiture In Favor of the State Any Property
the secrets of his clients. Found to Have Been Unlawfully Acquired By Any Public Officer or
Employee and Providing for the Proceedings Therefor

221
CRIMINAL LAW REVIEW DIGESTS
JUSTICE ROMEO CALLEJO NOTE: = Callejo Ponente

properties under R.A. No. 1379, maintaining that such the Compensation and Position Classification Act
jurisdiction actually resides in the Regional Trial Courts as of 989 (R.A. No. 6758), specifically including:
provided under Sec. 2 of the law,21 and that the (a) Provincial governors, vice-governors,
jurisdiction of the Sandiganbayan in civil actions pertains members of the sangguniang panlalawigan, and
only to separate actions for recovery of unlawfully provincial treasurers, assessors, engineers, and
acquired property against President Marcos, his family, other city department heads;
and cronies as can be gleaned from Sec. 4 of Presidential (b) City mayor, vice-mayors, members
Decree (P.D.) No. 1606, as amended, and Executive of the sangguniang panlungsod, city treasurers,
Orders (E.O.) Nos. 14and 14-A assessors, engineers, and other city department
Essentially, petitioner maintains that heads;
Sandiganbayan is just a criminal court with no jurisdiction (c) Officials of the diplomatic service
over civil actions. occupying the position of consul and higher;
(d) Philippine army and air force
ISSUE FOR THE PROVISION UNDER WHICH THIS colonels, naval captains, and all officers of higher
CASE WAS ASSIGNED, the issue is W/N Sandiganbayan rank;
had jurisdiction over this case? YES. (The main issue was (e) Officers of the Philippine National
W/N the Ombudsman had the power to investigate, file Police while occupying the position of provincial
and prosecute petitions for forfeiture under R.A. No. 1379? director and those holding the rank of senior
YES.) superintended or higher;
(f) City and provincial prosecutors and
HELD The Court in Republic v. Sandiganbayan, deduced their assistants, and officials and prosecutors in
that jurisdiction over violations of R.A. No. 3019 and 1379 the Office of the Ombudsman and special
is lodged with the Sandiganbayan. Under R.A. No. prosecutor;
8249,22 the Sandiganbayan is vested with exclusive (g) Presidents, directors or trustees, or
original jurisdiction in all cases involving violations of R.A. managers of government-owned or controlled
No. 3019, R.A. No. 1379, and Chapter II, Sec. 2, Title VII, corporations, state universities or educational
Book II of the Revised Penal Code, where one or more of institutions or foundations;
the accused are officials occupying the following positions (2) Members of Congress and officials thereof
whether in a permanent, acting or interim capacity, at the classified as Grade '27' and up under the
time of the commission of the offense: Compensation and Position Classification Act of
(1) Officials of the executive branch occupying 1989;
the positions of regional director and higher, (3) Members of the judiciary without prejudice to
otherwise classified as Grade '27' and higher, of the provisions of the Constitution;
(4) Chairmen and members of Constitutional
Commission, without prejudice to the provisions
of the Constitution; and
21 Sec. 2. Filing of petition.Whenever any public officer or (5) All other national and local officials classified
employee has acquired during his incumbency an amount of
property which is manifestly out of proportion to his salary as such as Grade '27' and higher under the
public officer or employee and to his other lawful income and the Compensation and Position Classification Act of
income from legitimately acquired property, said property shall be 1989.
presumed prima facie to have been unlawfully acquired. The
The Sandiganbayan is vested with jurisdiction
Solicitor General, upon complaint by any taxpayer to the city or
provincial fiscal who shall conduct a previous inquiry similar to over violations of R.A. No. 1379, entitled An Act Declaring
preliminary investigations in criminal cases and shall certify to the Forfeiture In Favor of the State Any Property Found to
Solicitor General that there is reasonable ground to believe that Have Been Unlawfully Acquired By Any Public Officer or
there has been committed a violation of this Act and the
respondent is probably guilty thereof, shall file, in the name and Employee and Providing For the Proceedings Therefor.
on behalf of the Republic of the Philippines, in the Court of First What acts would constitute a violation of
Instance of the city or province where said public officer or such a law? A reading of R.A. No. 1379 establishes
employee resides or holds office, a petition for a writ commanding
that it does not enumerate any prohibited acts the
said officer or employee to show cause why the property
aforesaid, or any part thereof, should not be declared property of commission of which would necessitate the
the State: Provided, That no such petition shall be filed within one imposition of a penalty. Instead, it provides the
year before any general election or within three months before procedure for forfeiture to be followed in case a public
any special election.
22 RA 8249, AN ACT FURTHER DEFINING THE JURISDICTION officer or employee has acquired during his
OF THE SANDIGANBAYAN, AMENDING FOR THE PURPOSE incumbency an amount of property manifestly out of
PRESIDENTIAL DECREE NO. 1606, AS AMENDED, proportion to his salary as such public officer or
PROVIDING FUNDS THEREFOR, AND FOR OTHER
PURPOSES
employee and to his lawful income and income from

222
CRIMINAL LAW REVIEW DIGESTS
JUSTICE ROMEO CALLEJO NOTE: = Callejo Ponente

legitimately acquired property. Section 12 of the law As a result, Merencillo was convicted under RA
provides a penalty but it is only imposed upon the public 3019 (Anti-Graft and Corrupt Practices Act) and Art. 210 of
officer or employee who transfers or conveys the the RPC (Direct Bribery). Merencillo contends that the
unlawfully acquired property; it does not penalize the convictions placed him under double jeopardy.
officer or employee for making the unlawful acquisition. In
effect, as observed in Almeda, Sr. v. Perez, it imposes Issues: Can the accused interpose the defense of double
the penalty of forfeiture of the properties unlawfully jeopardy when he is convicted under RA 3019 and Direct
acquired upon the respondent public officer or Bribery?
employee. It is logically congruent, therefore, that
violations of R.A. No. 1379 are placed under the Ruling: No.
jurisdiction of the Sandiganbayan, even though the Sec. 3 of RA 3019 states that [i]n addition to acts
proceeding is civil in nature, since the forfeiture of the or omissions of public officers already penalized by
illegally acquired property amounts to a penalty. The existing law, the following [acts] shall constitute corrupt
soundness of this reasoning becomes even more practices of any public officer and are hereby declared
obvious when we consider that the respondent in unlawful This means that one may be charged with
such forfeiture proceedings is a public officer or violation of RA 3019 in addition to a felony under the RPC
employee and the violation of R.A. No. 1379 was for the same delictual act, either concurrently or
committed during the respondent officer or subsequent to being charged with a felony under the
employees incumbency and in relation to his office. Revised Penal Code. There is no double jeopardy here.
This is in line with the purpose behind the creation of the Remember, the test in double jeopardy is
Sandiganbayan as an anti-graft courtto address the (1)whether one offense is identical with the other or is an
urgent problem of dishonesty in public service. attempt to commit it or a frustration thereof; or (2)whether
Following the same analysis, petitioner should one offense necessarily includes or is necessarily included
therefore abandon his erroneous belief that the in the other. An offense charged necessarily includes that
Sandiganbayan has jurisdiction only over petitions for which is proved when some of the essential elements or
forfeiture filed against President Marcos, his family and ingredients of the former, as alleged in the complaint or
cronies. information, constitute the latter; and an offense charged
is necessarily included in the offense proved when the
essential ingredients of the former constitute or form a part
MAMBA VS JUDGE of those constituting the latter.
In this case, there is neither identity nor
necessary inclusion between the two offenses. The
provision in sec. 3(b) of RA 3019 punishes unlawful acts
MERENCILLO V. PEOPLE
such as Directly or indirectly requesting or receiving any
gift, present, share percentage or benefit, for himself or for
Doctrine: Conviction under both RA 3019 and Direct
any other person, in connection with any contract or
Bribery is NOT double jeopardy
transaction between the Government and any other party,
wherein the public officer in his official capacity has to
Facts: Lucit Estillore went to the BIR office in Tagbilaran
intervene under the law.
City to ask for the computation of taxes due on the sale of
The elements under such section in RA 3019 are:
real property to Ramasola Superstudio, Inc (owned by Ma.
(1) the offender is a public officer; (2) he requested or
Angeles Cesar) and to apply for a certificate authorizing
received a gift, present, share, percentage or benefit; (3)
registration (CAR). The revenue examiner assessed the
he made the request or receipt on behalf of the offender or
taxes and Estillore paid them. The examiner, however,
any other person; (4) the request or receipt was made in
advised Estillore that the CAR will be released in 7 days
connection with a contract or transaction with the
pending approval of Merencillo (evil supervisor).
government and (5) he has the right to intervene, in an
Merencillo called Cesar wanting to meet her in
official capacity under the law, in connection with a
his office. Upon their meeting, Merencillo demanded P20,
contract or transaction has the right to intervene.
000 in exchange for the CAR. Cesar said that she will
On the other hand, the elements for direct bribery
confer with her business partners first. Merencillo soon
are: (1) the offender is a public officer; (2) the offender
made several demands for the money and refused to
accepts an offer or promise or receives a gift or present by
release the CAR. An entrapment scheme was set up after
himself or through another; (3) such offer or promise be
Cesar complained to the PNP. As such, Merencillo was
accepted or gift or present be received by the public officer
caught red-handed!
with a view to committing some crime, or in consideration
of the execution of an act which does not constitute a

223
CRIMINAL LAW REVIEW DIGESTS
JUSTICE ROMEO CALLEJO NOTE: = Callejo Ponente

crime but the act must be unjust, or to refrain from doing handed the envelope to Tad-y who opened the same and
something which it is his official duty to do and (4) the act upon seeing it asked whether it was safe. He then handed
which the offender agrees to perform or which he it under the table to Velez. When Tad-y and Velez left,
executes is connected with the performance of his official they were apprehended by the police. Forensics found
duties. ultraviolet powder on Tad-ys shirt and forearm.
The violation of Section 3(b) of RA 3019 is
neither identical nor necessarily inclusive of direct bribery. According to Defense
Despite having common elements, not all the essential Obviously, Tad-y denied asking for P4k from Encabo.
elements of one offense are included among or form part Also, Tad-y said that when they went to inspect the
of those enumerated in the other. buildings, it had some defects in its construction, which
was the reason why he didnt want to sign the final
Why? Because of the following reasons: inspection certificate. He also insists that he is not
1. The mere request or demand of a gift, authorized to sign and issue an occupancy certificate
present, share, percentage or benefit is under the law because only a building official can do so.
enough to constitute a violation of Section Furthermore, Tad-y also argued that after they inspected
3(b) of RA 3019. In direct bribery, the building, he was supposed to go bowling but only
acceptance of a promise or offer or receipt of reluctantly went with Encabo to the bakeshop because he
a gift or present is required. was hungry. He also said that the ultraviolet powder was
2. RA 3019 is limited only to contracts or found on his forearm only because when he was arrested,
transactions involving monetary the arresting officers forced him to touch the money which
consideration where the public officer has he parried away with his forearm.
the authority to intervene under the law.
Direct bribery, on the other hand, involves Tad-y was convicted! Velez was not because Encabo
performance of an act constituting a crime; testified that he [Velez] didnt know shit.
execution of an unjust act which does not
constitute a crime; and agreeing to refrain or Issue: W/N Tad-y is guilty of direct bribery?
refraining from doing an act which is his
official duty to do. Held/Ratio: NO!
Elements of Direct Bribery
1. Offender is a public officer;
TAD-Y VS. PEOPLE (2005) Check!
SC said that Tad-y was no doubt a public
Facts: officer. He was working in the OCE.
According to the Prosecution 2. Offender accepts an offer or promise or receives
Julio Encabo was employed by for the construction of a a gift or present by himself or through another;
building in Bacolod City. After the construction was No check
finished, he went to the Office of the City Engineer (OCE) J. Callejo cited the case of Formilleza vs.
to secure the final inspection certificate as well as the Sandiganbayan where it was held that, there
occupancy certificate. Tad-y was one of the engineers must be clear intention on the part of the
working in the OCE. Tad-y allegedly intervened during the public officer to take the gift and consider the
processing of the certificates and told Encabo that his same as his own property, such as when he
[Encabos] employers still owed him [Tad-y] P4k, and that puts it away for safekeeping. Mere physical
the certificates would not be signed without paying the receipt unaccompanied by any other
said amount. Encabo allegedly complained to the mayor of circumstance to show such acceptance is
Bacolod but he did not tell the mayor that Tad-y was trying not sufficient to conclude that the crime has
to extort him because according to him, Tad-y was his been committed (Note that this came from
kumpadre and he didnt want to tarnish his reputation. So an indirect bribery case but was applied to
Encabo went to the police and an entrapment was direct bribery by J. Callejo here).
organized. Encabo set up a meeting with Tad-y where Facts showing that Tad-y had no intention to
brought an envelope with him containing the P4k which keep the money or consider it as his own:
was marked with ultraviolet powder. Tad-y, together with o If Tad-y really intended to accept the
Nestor Velez (also an employee in the OCE), met up with bribe, he would not have brought Velez
Encabo. After they had conducted the final inspection of with him.
the building, they went to a bakeshop. There, Tad-y o After the inspection, Tad-y was on his
signed the final inspection certificate. After which, Encabo way to his bowling tournament. He only

224
CRIMINAL LAW REVIEW DIGESTS
JUSTICE ROMEO CALLEJO NOTE: = Callejo Ponente

joined them in the bakeshop because Tad-y was made to incriminate himself when he
Encabo had invited him. was forced to touch the marked money.
o When the envelope was handed to Tad-
y, he asked what it was for. When he Some other note-worthy stuff
opened it in full view of Velez and saw Agreement between the public officer and the
its contents, he handed it to Velez bribe-giver may be express or implied. It may be
instead of keeping it. proved by direct or circumstantial evidence. Proof
3. Acceptance is with a view to committing some of such an agreement may rest upon relevant
crime, or in consideration of the execution of an and competent circumstantial evidence.
act which does not constitute a crime but the act It is not necessary that the money is received
must be unjust, or to refrain from doing before or at the time the agreement was made. It
something which it is his official duty to do so; is sufficient that it was received afterwards.
and
Not discussed
4. The act which the offender agrees to perform is PELIGRINO V PEOPLE
connected with the performance of his official
duties. To convict the accused in a prosecution for the violation of
No check Section 3(b) of the Anti-Graft Law, mere receipt of a gift or
Under the National Building Code, only the any other benefit is enough, even without any express
building official can issue an occupancy demand for it. The duration of the possession is not
certificate. Tad-y is not a building official. So controlling. Important are the appellants words, action
the issuance of the said certificate was not and reactions showing acceptance thereof.
connected with the performance of his
official duties as an engineer in the OCE. Peligrino was an Examiner for the BIR, hence, a public
officer. His co-accused was Atty. Buenafe. Atty. Buenafe
Additional Facts as to why Tad-y was acquitted delivered a letter to the complainant Dr. Feliciano saying
There was no evidence to prove that Tad-y and that Peligrino was going to examine his (Feliciano) books.
Encabo met before the day when Encabo went to However, Peligrino never came to examine the books.
the OCE. No evidence that Tad-y had previously A few weeks later, Peligrino and Atty. Buenafe
asked for P4k. went to the doctor and told him that his tax deficiencies
Testimony of Encabo not entitled to full probative amounted to P500k. Flabbergasted since his books were
weight because it was evasive and never examined, the doctor figured he was being extorted.
chameleonic. He negotiated for a smaller amount, and they agreed that
o He told the lower courts that Tad-y 200k would instead be paid P50k going to the BIR, P150
demanded the P4k before he would sign going to the pockets of Peligrino and Buenafe.
the final inspection certificate. And this Feliciano told the NBI and they arranged that
was contrary to what he told the police marked money would be given to the accused during the
which was that Tad-y demanded the pay-off. On the day of the pay-off, only Peligrino appeared.
P4k for the issuance of the occupancy He received the envelope with marked money, looked at it
certificate. and then placed it on the table. Then the NBI agents
o He said that Tad-y was his kumpadre arrested him.
which is why he didnt tell the mayor that He was charged under Sec 3 (b) of the Anti-Graft
Tad-y tried to extort him but he went to Law23. Sandigan found him guilty, but acquitted Atty.
the police anyway and filed the criminal Buenafe.
case. Contradictory!
o Encabo claimed that months after the Issue: Is he guilty?
occupancy certificate was issued, Tad-y
was still conducting inspections of the
building. The thing is, during this time a 23 SEC. 3. Corrupt practices of public officers. -- In addition to
criminal charge was already filed acts or omissions of public officers already penalized by existing
law, the following shall constitute corrupt practices of any public
against Tad-y. SC said it is incredible officer and are hereby declared to be unlawful:
that Tad-y would continue inspections (b) Directly or indirectly requesting or receiving any gift,
and demand the money during such present, share, percentage, or benefit, for himself or for any other
person, in connection with any contract or transaction between
time. the Government and any other party, wherein the public officer in
his official capacity has to intervene under the law.

225
CRIMINAL LAW REVIEW DIGESTS
JUSTICE ROMEO CALLEJO NOTE: = Callejo Ponente

A person found in possession of a thing taken


Held: Yes. from the recent execution of a wrongful act is presumed to
be both the taker and the doer of the whole act
The elements of this offense are (1) the offender is a
public officer (2) who requested or received a gift, a
present, a share, a percentage, or a benefit (3) on behalf LAURO G. SORIANO, JR. vs. THE HONORABLE
of the offender or any other person (4) in connection with a SANDIGANBAYAN AND THE PEOPLE OF THE
contract or transaction with the government (5) in which PHILIPPINES
the public officer, in an official capacity under the law, has
the right to intervene. FACTS: Thomas N. Tan was accused of qualified theft.
The case was assigned to Petitioner Soriano, who was
1, 4, and 5 are obviously present. then an assistant city fiscal. Soriano then demanded 4,000
from the accused Tan for dismissing the case. Tan
Claim 1: Peligrino contends that he never demanded the reported the demand to NBI, which set up an entrapment.
money and that he merely informed the doctor of his tax The entrapment succeeded and an information was filed in
deficiencies, and that it was the latter who requested the the SB.
reduction of the amount claimed. Soriano was charged of violating Section 3b of
Peligrino wrong. Section 3(b) penalizes three RA 3019 for demanding 4,000 from Tan and receiving
distinct acts -- (1) demanding or requesting; (2) receiving; 2,000. SB: Soriano guilty as Principal. MR filed by Soriano
or (3) demanding, requesting and receiving -- any gift, was also denied.
present, share, percentage, or benefit for oneself or for
any other person, in connection with any contract or ISSUE: WON the preliminary investigation of a criminal
transaction between the government and any other party, complaint conducted by a Fiscal is a "contract or
wherein a public officer in an official capacity has to transaction" so as to bring it within the ambit of Section
intervene under the law. These modes of committing the 3(b) of RA 3019?
offense are distinct and different from each other. Proof of
the existence of any of them suffices to warrant conviction. RULING: NO, the investigation conducted cannot be
The lack of demand is immaterial. After all, Section 3(b) of considered as a contract or transaction. Soriano is liable
RA 3019 uses the word or between requesting and NOT of RA 3019 but of Bribery under Article 210.
receiving. SEC. 3. Corrupt practices of public officers. In
addition to acts or omissions of public officers
Claim 2: He also denies that he received payoff money already penalized by existing law, the following
from complainant. According to him, receive, as shall constitute corrupt practices of any public
contemplated in the offense charged, connotes a voluntary officer and are hereby declared to be unlawful:
act coupled with knowledge. Hence, where the giving of (b) Directly or indirectly requesting or receiving
the money affords the accused no opportunity either to any gift, present, share, percentage, or benefit,
refuse or to return it to the giver, no punishable offense for himself or for any other person, in
ensues. He claims that the 40 seconds or less that the connection with any contract or transaction
boodle money was in his hands was merely a momentary between the Government and any other party,
possession that could not prove receipt, which the law wherein the public officer in his official capacity
requires for the offense charged to be consummated. has to intervene under the law.
Hes wrong. The duration of the possession is not It is obvious that the investigation conducted by
the controlling element in determining receipt or the Soriano was not a contract. Neither was it a
acceptance. Here, he opened the envelope containing the transaction because this term must be construed as
boodle money, looked inside, closed it and placed the analogous to the term which precedes it. A transaction,
envelope beside him on the table. Such reaction did not like a contract, is one which involves some consideration
signify refusal or resistance to bribery, especially as in credit transactions and this element (consideration)
considering that he was not supposed to accept any cash is absent in the investigation conducted by the petitioner.
from the taxpayer. The proximity of the envelope relative In the light of the foregoing, We agree with the
to petitioner also belies petitioners contention that he petitioner that it was error for the Sandiganbayan to have
refused the bribe.24 convicted him of violating Sec. 3 (b) of R.A. No. 3019.

24 Court also quoted Cabrera v. Pajares where acceptance was


established because the accused judge placed the bribe money protestations that the money bills landed on the open pages of his
between the pages of his diary or appointment book, despite his diary, only after he had flung them back to the complainant.

226
CRIMINAL LAW REVIEW DIGESTS
JUSTICE ROMEO CALLEJO NOTE: = Callejo Ponente

Soriano also claims that he cannot be convicted Held: Marifosque cannot feign ignorance and profess good
of bribery under the RPC because to do so would be faith since all the indicia point to his guilt and malicious
violative of as constitutional right to be informed of the intent.
nature and cause of the accusation against him. Wrong. A
reading of the information clearly makes out a case of Marifosque did not introduce his asset to Sy at the time
bribery so that Soriano cannot claim deprivation of the of the transaction. Neither did he attempt to present or
right to be informed. justify his reciept of the marked money when he was
arrested and interrogated. Instead, he accepted his
arrest with an air of resignation characteristic of a
MARIFOSQUE V. PEOPLE culprit who is caught red-handed. Capt. Salvo testified
that Marifosque even attempted to give back the money
Facts: This is a petition for review on certiorari of the to Sy. This was a clear showing that he was well aware
Sandiganbayan resolution finding Marifosque, a member of the illegality of his transaction. Had he been engaged
of the police force of Legaspi City, guilty of the crime of in a legitimate deal, he would have courageously faced
direct bribery. the arresting officers and indigantly protested, which is
The spouses Sy went to the office of Captain Salvo the normal reaction of an innocent man. Instead, he
of the PNP to report the robbery of Shellane tanks at their meekly submitted to the indignity of arrest with the
gasoline station, and the alleged extortion attempt made docility of a man at a loss for a satisfactory explanation.
by Police Sergeant Marifosque, in exchange for the Marifosques solicitous and overly eager conduct in
recovery of the lost items. Thus, Capt. Salvo set up a plan pursuing the robbery incident betrays an intention not
to entrap him -Sy would pay off Marifosque, using marked altogether altruistic. It denotes a corrupt desire on his
money (P4,800) wrapped in newspaper, at the Golden part to obtain pecuniary benefits from an illegal
Grace Dept. Store while the police would position transaction. Moreover, at the time he got the tip, he was
themselves strategically outside. Marifosque arrived via no longer on duty. He was too overzealous to meet with
tricycle, went inside, and demanded the money from Sy Sy despite that another investigator was already
who then handed him the money. Upon Sys signal, the assigned to the case. (Ayan, eager beaver kasi.) His
police operatives swooped down on their comrade and justification that he wanted to encourage the victim to
arrested him. Sy later on testified that Marifosque pursue the case rings hollow and untrue. It is clearly an
demanded P7,200 but she bargained to lower it down to afterthought -the main reason he met them was for no
4.8k. reason other than to demand money.
Marifosques defense (which an assets testimony While the asset corroborated he received P1k from
corroborates): a police asset came to his house to tip him Marifosque, it was revealed during cross examination
off about a robbery he witnessed at a gasoline station. that: it was not the usual practice for Marifosque to give
They went to the police station to report the incident, then incentives for tips; and that the asset did not complain
the asset asked if he could get P350 per cylinder tank as a that the P1k was not enough.
reward. So Marifosque relayed that message to Sy, who The conduct of Marifosque during recovery of the
agreed if that was the only way to recover the tanks and stolen goods is suspicious. He did not arrest or invite
apprehend the robbers. Based on the assets info, the Arnaldo for questioning. That he was promised
police then went to the house of Arnaldo where they found additional information is a flimsy excuse. He did not
the stolen gas tanks. Arnaldo arrived but he was not follow the standard procedure.
arrested bec he allegedly promised to lead them to the Curiously, Arnaldo turned out to be the brother of the
other stolen tanks. The police went back to the station and police asset. This strange coincidence indicates a
made a report. He said that: 1) he was not the one who conspiracy between Marifosque and the thief to steal
asked for a reward for Sy but his asset; 2) There was no from the victim and later on cash in on the recovery of
evidence to prove that he intended to appropriate the the lost items.
amount; 3) He merely relayed the assets request for a As Marifosque is a police officer, a public officer,
reward to Sy who was agreeable to the same. and that his act of receiving the money was connected
Despite this defense, the Sandiganbayan convicted with his duties, the case falls within the 2nd paragraph of
Marifosque of direct bribery. Art. 213 of the RPC: Direct Bribery. He is ordered to pay a
fine 3x the amount he extorted, in addition to prison term
ISSUE: W/N the act of Marifosque in recieving sums of and special temporary disqualification.
money for delivery to his asset constitutes Direct Bribery -
YES.
ACEJAS V. PEOPLE; HERNANDEZ V. PEOPLE

227
CRIMINAL LAW REVIEW DIGESTS
JUSTICE ROMEO CALLEJO NOTE: = Callejo Ponente

FACTS: Hernandez is an intelligence agent of the Bureau The circumstances here show the guilt of
of Immigration and Deportation (BID). He served a mission Hernandez, a BID agent. He was the one in possession of
order to Mr. Takao Aoyagi in his home and informed him the passport. He was present at several meetings. He
that he was wanted in Japan as a Yakuza big boss, a drug asked for the 300K downpayment. He directed Aoyagi to
dependent and an overstaying alien. Takao Aoyagi give the money to Acejas.
promised to appear at an investigation in the BID and as Acejas guilt as a conspirator was also shown by
guarantee he gave his passport. the circumstances. At the time when the money was
The wife of Takao Aoyagi, Bethel Pelingon- demanded, he kept silent although he knew that his
Aoyagi, called up Expedito Perlas. Perlas referred them to supposed clients were being extorted. He was also the
a lawfirm, of which Atty. Acejas was tasked to take on the person who received the 1M. Lawyers must report
case. attempts to extort to the authorities and must advise their
The wife Bethel Pelingon-Aoyagi informed her clients not to perform an illegal act. Moreover, lawyers
brother, Jun, of her husbands problem. Jun Pelingon should not participate in the illegal act.
contacted BID Comissioner Respicio. Commissioner There was also no instigation here since the
Respicio told Jun to contact him in his office. criminal intent came from the conspirators who demanded
The Aoyagis and Jun Pelingon met with Atty. for the money and arranged the payoff.
Acejas and Perlas at Aristocrat in Roxas Blvd where Although the Prosecution did not present Takao
Acejas told Pelingon that he was preparing a lawsuit Aoyagi, the person extorted, it was able to prove all the
against the BID agents who took the passport of Aoyagi. elements of the crime. The decision on who to present as
Later, Hernandez with a certain Conanan arrived at the witness lies with the Prosecution.
restaurant. The prosecution proved the elements of direct
The group again met at the Hotel Nikko where bribery. First, there is no question that the offense was
Hernandez said that he would give back the passport if committed by a public officer. BID Agent Hernandez
Takao Aoyagi paid him 1M, but he said that 300K D/P extorted money from the Aoyagi spouses for the return of
would do in the meantime. Pelingon said that Takao the passport. Acejas was his co-conspirator. Second, the
Aoyagi could pay in full. In this meeting Acejas didnt do offenders received the money as payoff, which Acejas
anything when the demand was made. received for the group and then gave to Perlas. Third, the
Pelingon then contacted BID Commissioner money was given in consideration of the return of the
Respicio and informed him of what transpired. BID Comm. passport, an act that did not constitute a crime. Fourth,
Respicio referred the matter to NBI who together with both the confiscation and the return of the passport were
agents of the BID set up an entrapment operation. made in the exercise of official duties. A conspiracy exists
The group again met, this time in the Diamond even if all the parties did not commit the same act, if the
Hotel. Aoyagi had the 1M inside an envelope and handed participants performed specific acts that indicated unity of
it to Hernandez. Hernandez however waived his hand and purpose in accomplishing a criminal design.
pointed to Acejas. Aoyagi thus handed it to Acejas. Acejas
handed the money to Perlas. Hernandez returned the
passport to Aoyagi. Thereafter, the group was SAZON vs. SANDIGANBAYAN
apprehended by the NBI.
Acejas, Hernandez, Perlas and Conanan were FACTS: Petitioner Zenaida Sazon was a Senior Forest
charged in the Sandiganbayan for Direct Bribery under the Management Specialist of the DENR. A travel order was
second mode. The Sandiganbayan held that the elements issued by the DENR directing Zenaida to proceed to
as well as the conspiracy was shown by evidence of the Karuhatan and Navotas to investigate alleged illegal
prosecution. operations. While looking for the office of Vifel Shipyard,
which was the subject of the travel order, they chanced
ISSUE: Guilt of Acejas and Hernandez? YES! Guilty of upon R&R Shipyard and asked the guard for a certain Mr.
Direct Bribery. Opena, the operations manager. Since Zenaida knew Mr.
Opena, she wanted to ask directions to Vifel. While
HELD: The second mode of direct bribery acceptance Zenaida was talking with the guard, her team spotted
of a gift in consideration of the execution of an act that squared logs (dungon logs) piled at the R&R compound.
does not constitute a crime has the following elements: Since dungon logs were banned species, they asked for
1) offender was a public officer, 2) who received the gifts documents pertaining to the logs. R&R failed to do so,
or presents personally or through another, 3) in thus they decided to return 5 days later. Upon their return,
consideration of an act that did not constitute a crime, and R&R still could not produce documents. Zenaida insisted
4) that act related to the exercise of official duties. that the logs were banned species and threatened Mr.
Opena that he could be arrested and that the logs could

228
CRIMINAL LAW REVIEW DIGESTS
JUSTICE ROMEO CALLEJO NOTE: = Callejo Ponente

be confiscated. Mr. Opena however claimed that the logs persons, or by using force upon things. Taking is
were yakal and tangile, not dungon. considered complete at the time the offender
Two days later, Atty, Agbi, lawyer of R&R, met gains possession of the thing, even if he has no
with Zenaida to talk about the logs. Zenaida instructed opportunity to dispose of it. When Zenaida took
Atty. Agbi to meet her at a bakeshop near her office. Atty. possession of the brown envelope containing the
Agbi told Zenaida that she had the receipts covering the money, it already consisted of taking. As a
logs, but Zenaida said that they were insufficient. Zenaida public officer employed with DENR, Zenaida had
initially demanded P300K if no papers would be submitted, the power to make reports on forestry violations
P200K if incomplete, and P100K if the papers were which could result to possible confiscation of logs
complete. Days later, Zenaida made a final demand of if possession thereof is not justified, and the
P100K in exchange for the favor of fixing the papers of prosecution of violators of forestry laws. Thus,
the alleged hot logs. She even offered Atty. Agbi P25K as Zenaida could not demand and eventually
her share in the amount. Atty. Agbi reported the matter to receive any amount from private persons as a
the police. An entrapment operation against Zenaida was consideration for the formers non-performance of
planned. The operation proved to be successful. Zenaida her lawful task. In this case, Zenaida threatened
denied the accusations against her and alleged that it was R&R with possible confiscation of the logs and
in fact Atty. Agbi who proposed the settlement. She stated prosecution if they would not accede to her
that during the entrapment operation, when Atty. Agbi demand for P100K. This made her eventual
offered her a brown envelope containing money, she receipt of the amount as an unlawful taking.
stood up to leave, but a man came from nowhere and (3) that the taking was with intent to gain - intent
immediately handcuffed her while another man took to gain, or animus lucrandi, is an internal act;
pictures. hence, presumed from the lawful taking of things.
She was charged for Robbery Extortion. After Actual gain is irrelevant as the important
trial, Sandiganbayan rendered a decision finding her guilty consideration is the intent to gain. Having
of the crime. The court found that the elements of robbery established that the money was unlawfully taken
with intimidation were established by the prosecution. by Zenaida for her personal benefit, intent to gain
Zenaida appealed, alleging that the elements of the crime was likewise proven.
were absent. (4) that there is violence against or intimidation
of persons or force upon things - Intimidation
ISSUE: W/N the SB erred in concluding that the is defined in Blacks Law Dictionary as unlawful
elements of the crime of robbery with intimidation are coercion; extortion; duress; putting in fear. In
present. robbery with intimidation of persons, the
intimidation consists in causing or creating fear in
HELD/RATIO: NO! SB decision is affirmed. the mind of a person or in bringing a sense of
mental distress in view of a risk or evil that may
The crime charged was simple robbery under Art. 293 and be impending, real or imagined. Such fear of
294(5) of the RPC. Its elements are: (1) that there is injury to person or property must continue to
personal property belonging to another, (2) that there is operate in the mind of the victim at the time of the
unlawful taking of the property, (3) that the taking was with delivery of the money. From these principles, the
intent to gain, and (4) that there is violence against or SC finds that the P100K grease money; was
intimidation of persons or force upon things. The SC found taken by Zenaida through intimidation. By using
that the prosecution adequately established these her position as Senior Management Specialist of
elements. the DENR, she succeeded in coercing the
complainants to choose either to part with their
(1) that there is personal property belonging to money or suffer the burden and humiliation of
another - it is undisputed that Zenaida prosecution and confiscation of the logs.
demanded and eventually received P100K, a
personal property belonging to R&R.
(2) that there is unlawful taking of the property - GREGORY JAMES POZAR V. CA
It was also proven that the money was unlawfully
taken by Zenaida from R&R, with intent to gain FACTS: Gregory James Pozar was convicted of the crime
and through intimidation. In robbery, there must of Less Physical injuries and the crime of Oral Defamation.
be unlawful taking, which is defined as the taking After his conviction he applied for probation. Pozar and
of items without the consent of the owner, or by his lawyer Atty. Suarez went to the probation office to
means of violence or intimidation against inquire for the requirements needed for the application.

229
CRIMINAL LAW REVIEW DIGESTS
JUSTICE ROMEO CALLEJO NOTE: = Callejo Ponente

Unfortunately, they did not reach Probation Officer Danilo It is well to note and distinguish direct bribery
Ocampo and was asked by Mr. Manalo, clerk of the from indirect bribery. In both crimes, the public officer
Probation Office to come back since Mr. Danilo Ocampo receives a gift. While in direct bribery, there is an
was out. Later, Atty. Suarez called the Probation Office agreement between the public officer and the giver of the
and was able to speak with Probation Inspector Mrs. gift or present. In indirect bribery, usually no such
Primitiva Francisco and informed him of all the necessary agreement exists. In direct bribery, the offender agrees to
requirements to be submitted for the application for perform or performs an act or refrains from doing
probation. Subsequently, all these requirements were something, because of the gift or promise while in indirect
submitted by Mr. Pozar either to Mrs. Francisco or Mr. bribery, it is not necessary that the officer should do any
Manalo. particular act or even promise to do an act, as it is enough
On December 10, 1979, Pozar was able to speak that he accepts gifts offered to him by reason of his office.
with Mr. Danilo Ocampo in a meeting where he was The record shows that herein accused while
asking permission from the Probation Officer to go to applying for probation made more or less 12 visits in the
Baguio. The Probation Officer required him to submit a office as he was directed to report every Monday at 10:00
photocopy of his visa, I.D. picture. o'clock in the morning. He reported for 6 to 7 consecutive
On December 17, Mr. Pozar went to the weeks and there were times that he went there
Probation Office looking for Mr. Ocampo to submit the unscheduled for conference and clarification of the various
requirements but since he was not there, handed to Mr. requirements he needed. During all the time he went
Manalo an envelope addressed to the Probation Officer there, he met Manalo, Mrs. Francisco and Mr. Ocampo
and requested to give the same to Mr. Ocampo. When himself asking for different requirements.
Mr. Ocampo opened the envelope in the presence of Mr. From the foregoing, We can fairly deduce that the
Manalo, it contained a photocopy of the applicants procedure for processing petitioner's application for
passport, copy of his visa and attached to the documents probation in the Probation Office at Angeles City was not
is a P100.00 peso bill. precise, explicit and clear cut And since the accused
Mr. Ocampo upon the suggestion of the presiding petitioner is a foreigner and quite unfamiliar with
judge of City Court of Angeles who learned about the probation rules and procedures, there is reason to
P100 given to the Probation Officer in an envelope by the conclude that petitioner was befuddled, if not
applicant (judge who heard Mr. Pozars application for confused so that his act of providing and advancing
probation), filed an Information Sheet against the accused. the expenses for whatever documentation was needed
RTC found Mr. Pozar guilty of Corruption of Public Officer. further to complete and thus hasten his probation
application, was understandably innocent and not
ISSUE: Whether or not Mr. Pozar is guilty of Corruption of criminal. law library
Public Officer The Government's own evidence as indicated in
the Post-Sentence Investigation Report that the giving of
HELD: No he is not guilty of Corruption of a Public officer. the P100.00 was done in good faith, is vital for it belies
Decision of the CA reversed. petitioner's criminal intent. There being no criminal
1) RTC made an error of convicting accused of the intent to corrupt the Probation Officer, the accused
crime of Corruption of Public Officer as a petitioner is entitled to acquittal of the crime charged.
consummated offense since the facts show that We hold and rule that the prosecution has not proved
the public officer did not accept the P100.00 bill. the guilt of the accused beyond reasonable doubt.
Hence the crime should have been attempted There is no moral certainty required to convict him.
corruption of public officer.
However the crucial point is whether
the prosecution was able to establish beyond FORMILLEZA V SANDIGANBAYAN
reasonable doubt that the P100 bill was meant
to bribe and corrupt the City Officer or to FACTS: Leonor Formilleza was the personnel supervisor
defray the photocopying expenses for of the regional office of the National Irrigation
documents pertinent for the application for Administration (NIA) in Tacloban City, Leyte. Her duties
probation. include the processing of the appointment papers of
In the case at bar, the Information employees.
against the petitioner charged the accused for On the other hand, a certain Mrs. Estrella Mutia
willfully and feloniously giving Mr. Danilo Ocampo was an employee of the NIA whose term was coterminous
the sum of P100.00 under the circumstances that with a project. When her appointment was terminated, she
would make the said officer liable for bribery. nonetheless continued working for the NIA pursuant to the

230
CRIMINAL LAW REVIEW DIGESTS
JUSTICE ROMEO CALLEJO NOTE: = Callejo Ponente

verbal instructions of the regional director of the officers by simply putting within their physical custody
Administration. some gift, money or other property.
Mrs. Mutia testified that she took steps to obtain The Sandiganbayan noted that the photographs
either a permanent or at the least a renewed appointment; of the entrapment show that the petitioner was accosted
that the regional director advised her to see the petitioner by the PC soldiers after she accepted the marked money.
Formilleza who was to determine the employees to be Against the evidence of the defense that it was when
appointed or promoted; and that Formilleza refused to Formilleza stood up that Mrs. Mutia suddenly placed
attend to her appointment papers unless the latter were something in her hand which she did not know to be
given some money. money and when she saw that it was money she threw it
Mutia reported the matter to the Philippine away. An examination of the seven photographs that
Constabulary (PC) authorities in the province (her were allegedly taken immediately after the passing of the
husband was in the PC). Entrapment was set. money shows that the petitioner was standing up when the
st
1 attempt. The plan did not materialize as the PC agents apprehended her. This corroborates petitioner's
petitioner did not show up at the designated rendezvous at story. There was no picture showing petitioner to be
the NIA building canteen. seated which should be her position immediately after the
nd
2 attempt, in canteen. Mrs. Mutia maintains that money was handed to her under the table, which should
after they had finished taking their snacks, she handed the be the case according to the version of the
marked money bills under the table with her right hand to prosecution. None of the photographs show the petitioner
the petitioner who received the same with her left hand. At in the process of appropriating or keeping the money after
that moment, one of the PCs approached the petitioner it was handed to her.
and held her hand holding the money bills. Photographs There were other persons in the canteen like the
were taken of Formilleza standing up with her hands held undercover PC agents and their colleagues. Under the
by the PC. circumstances and in such a public place it is not probable
At the PC crime laboratory she was found that petitioner would have the nerve to accept bribe money
positive for ultra-violet powder dusted on the marked from Mrs. Mutia even under the table. If the petitioner
money. She was charged before the Sandiganbayan for knew and was prepared to accept the money from Mrs.
direct bribery. Mutia at the canteen, the petitioner would not have invited
In the proceedings before the Sandiganbayan, her officemate Mrs. Sevilla to join them. Mrs. Sevilla stated
the petitioner maintains her innocence that there was she did not see the alleged passing of the money.
no entrapment; the scenario was but a scheme set up by Guilt has not been proved beyond reasonable doubt.
Mrs. Mutia and her husband's colleagues in the PC. The ACQUITTED.
petitioner denies having accepted the supposed bribe
money. ARTICLE 217
Sandiganbayan decision: guilty of indirect
bribery, not direct bribery. (case did not say how SB got BALUYOT v. HOLGANZA
to this conclusion)
Formilleza elevated the case to SC via Petition
FACTS: Petitioner Francisca Baluyot was the chapter
for Review, alleging that the findings of SB is not
administrator of Philippine National Red Cross BOHOL
supported by evidence
chapter. On March 21, 1977, the PNRC headquarters sent
a team of auditors to conduct a spot audit on the Bohol
ISSUE: WON Formilleza is guilty of indirect bribery? NO
chapter. It was found that there was a cash shortage of
P154,350. Baluyot as the chapter administrator was held
RULING: The essential ingredient of indirect bribery as
accountable for the shortage.
defined in Article 211 of the Revised Penal Code is that
Thereafter, respondent Holganza, in his capacity
the public officer concerned must have accepted the gift or
as member of the board of directors of the Bohol chapter,
material consideration. There must be a clear intention on
filed an affidavit-complaint for malversation under Art 217
the part of the public officer to take the gift so offered and
of RPC, with the Office of the Ombudsman. However,
consider the same as his own property from then on, such
upon recommendation of other respondent Militante (Graft
as putting away the gift for safekeeping or pocketing the
Investigation Officer), an administrative case for
same. Mere physical receipt unaccompanied by any other
dishonesty was also filed against Baluyot.
sign, circumstance or act to show such acceptance is not
The OMB ordered Baluyot to file a counter-
sufficient to lead the court to conclude that the crime of
affidavit, which she did. She raised as a defense that the
indirect bribery has been committed. To hold otherwise will
Ombudsman did not have jurisdiction over the controversy
encourage unscrupulous individuals to frame up public

231
CRIMINAL LAW REVIEW DIGESTS
JUSTICE ROMEO CALLEJO NOTE: = Callejo Ponente

because the OMB had authority only over GOCCs, which province, loaned out 56M of the 100M for the livelihood
she claimed the PNRC is NOT. projects of Lingkod Tarlac Foundation Inc. (LTFI), being
OMB denied Baluyots motion to dismiss and the represented by its Exec. Director Andres Flores in the
subsequent MR. OMB maintains that it has jurisdiction Memorandum of Agreement (MOA). How this 56M was
over the case of malversation. utilized is the subject of the 25 criminal cases filed against
herein petitioners, with 2 out of the 25 cases being the
ISSUE: W/N the Office of the Ombudsman has matter being appealed herein. (Prosecution relied mainly
jurisdiction over the malversation case, where the on COAs report of the LTFI disbursement of such funds).
office is a chapter of PNRC. SB thus ruled that Ocampo is guilty of malversation of
public funds by consenting, thru negligence, that another
HELD/RATIO: Yes, has jurisdiction. person misappropriate such funds. Flores was likewise
Baluyot contends that the OMB has no held guilty, having received such funds and deposited in
jurisdiction over the controversy because PNRC is his bank account.
allegedly a private voluntary org. And because it does not
receive budgetary support from the government, and the Issue: won PETITIONERS guilty of malversation,
funds actually come from fund campaigns, private considering that the public funds alleged to have been
contributions, and that it is not audited by the COA, etc. malversed were loaned by the Province of Tarlac to LTFI;
Basically, she contends that PNRC is not a GOCC, hence hence, LTFI acquired ownership of the funds which thus
not under OMBs jurisdiction. shed their public character and became private funds?
SC cited Camporendondo v. NLRC where it was
held that PNRC is a GOCC with an original charter under Held: Petition meritorious. Both acquitted. Ocampo not
RA 95 as amended. The test to determine whether a found negligent (Art.217) as he made proper safeguards
corporation is a GOCC, or private in nature is simple. Is it for the handling of funds of LTFI.
created by its own charter for the exercise of a public
function, or by incorporation under the general corporation Ratio: The MOA shows that LTFI is allowed to borrow
law? Those with special charters are government funds directly from the Provincial Government to fund
corporations subject to its provisions, and its employees Lingkod Tarlac Foundation projects provided the projects
are under the jurisdiction of the CSC, and are compulsory are livelihood projects. Moreover, the MOA stipulates
members of the GSIS. under the Conditions for Release of Funds that the
The PNRC was not "impliedly converted to a Province of Tarlac shall release in lump sum the
private corporation" simply because its charter was appropriate funds for the approved projects covered by
amended to vest in it the authority to secure loans, be individual loan documents upon signing of the
exempted from payment of all duties, taxes, fees and respective loan agreement.
other charges of all kinds on all importations and Based on the foregoing, it is clear that the funds
purchases. released by the Province of Tarlac, including the money
SC holds that PNRC is clearly a GOCC, hence allegedly malversed by petitioners, were in the nature of a
OMB has jurisdiction over the matter pursuant to the loan to LTFI. (Art. 1953 of the Civil Code provides that a
Ombudsman Act of 1989, which gives the office person who receives a loan of money or any other fungible
jurisdiction over complaints against employees of the thing acquires the ownership thereof, and is bound to pay
government, including GOCCs. to the creditor an equal amount of the same kind and
quality.)Hence, petitioner Ocampo correctly argued that
the funds shed their public character when they were
OCAMPO III V. PEOPLE lent to LTFI as it acquired ownership of the funds with
an obligation to repay the Province of Tarlac the amount
Doctrine: One cannot be guilty of malversation of public borrowed. (Thus, the relationship between the Province of
funds when such funds became private in character due to Tarlac and the LTFI is that of a creditor and debtor).
a loan agreement, which in effect transfers ownership of Lastly, Gov. Ocampo cannot be said to be
said funds. negligent (Art.217) in permitting Flores to misappropriate
such funds because he set proper safeguards before
Facts: Tarlac Gov. Ocampo and Andres Flores were releasing them to LTFI, and therefore he was neither
found guilty of malversation in the Sandiganbayan. accountable nor in custody thereof. Flores cannot be guilty
During Cory admin, Tarlac was one of four either because these funds became private in character
provinces chosen to test the decentralization of LGUs, so due to the said loan agreement.
the Dept of Budget and Management (DBM) allocated
100M to Tarlac. Ocampo, then the Governor of said

232
CRIMINAL LAW REVIEW DIGESTS
JUSTICE ROMEO CALLEJO NOTE: = Callejo Ponente

RENE P. PONDEVIDA V. SANDIGANBAYAN (c) That those funds or property


were public funds or property for
FACTS: The Sandiganbayan rendered judgment which he was accountable.
convicting petitioner Pondevida, the Municipal Treasurer (d) That he appropriated, took,
of Badiangan, Iloilo, of three counts of the complex crime misappropriated or consented or,
of malversation of public funds through falsification of through abandonment or
commercial documents, and sentencing him to suffer the negligence, permitted another
penalty of reclusion perpetua and perpetual special person to take them.
disqualification for each count.
The petitioner questioned the validity of his A public officer may be liable for malversation even
conviction alleging that he is not criminally liable for if he does not use public property or funds under his
malversation under Article 217 of the RPC because (a) the custody for his personal benefit, but consents to the taking
prosecution failed to prove that, before he was charged thereof by another person, or, through abandonment or
with malversation complexed with falsification of negligence, permitted such taking.
commercial documents in the Office of the Ombudsman, The prosecution is burdened to prove beyond
the Office of the Provincial Auditor had demanded the reasonable doubt, either by direct or circumstantial
refund of the amounts of the three checks; and (b) the evidence, that the public officer
Sandiganbayan ignored the cash deposit slips issued by appropriated, misappropriated or consented or through
the Land Bank (LBP) showing that he deposited an abandonment or negligence, permitted another person to
amount which included the total amount of the three take public property or public funds under his custody.
checks. Absent such evidence, the public officer cannot be held
The petitioner further avers that the charges criminally liable for malversation. Mere absence of funds
against him were barred by the decision of the RTC of is not sufficient proof of conversion; neither is the mere
Iloilo City in Criminal Case convicting him of malversation failure of the public officer to turn over the funds at any
of P1,176,580.59 which included the amount of the 3 given time sufficient to make even the prima facie case. In
checks subject of the cases before the Sandiganbayan. fine, conversion must be proved. However, an
He also assert that the prosecution failed to prove that he accountable officer may be convicted of malversation
misappropriated the said amount for his own personal use even in the absence of direct proof of
and benefit. The petitioner posits that had the prosecution misappropriation so long as there is evidence of
proved that he received a demand to account for or refund shortage in his account which he is unable to explain.
the said amounts, the burden could have shifted on him to Demand to produce public funds under a public
prove that he did not misappropriate or take away the said officers custody is not an essential element of the
amounts for his personal use or benefit. felony. The law creates a prima facie presumption of
connivance if the public officer fails to produce public
ISSUE: Is the petitioner guilty? YES funds under his custody upon demand therefor. However,
the presumption may be rebutted by evidence that the
Ratio: Malversation is defined and penalized in Article 217 public officer had fully accounted for the alleged cash
of the Revised Penal Code. shortage.
Malversation may be committed by appropriating In the present case, the petitioner does not dispute
public funds or property; by taking or misappropriating the the fact that, by his overt acts of drawing and issuing the
same; by consenting, or through abandonment or checks to the order of Grande, Celis and Tiu, they were
negligence, by permitting any other person to take such able to encash the checks. Even if the petitioner
public funds or property; or by being otherwise guilty of the received P893,860.67 from them a day after the checks
misappropriation or malversation of such funds or were encashed, by then, the felonies of malversation had
property. already been consummated. Case law has it that the
The essential elements common to all acts of individuals taking of funds is completed and is
malversation under Article 217 of the Revised Penal Code consummated even if the severance of the funds from
are the following: the possession was only for an instant. Restitution of
the said amount after the consummation of the crimes
(a) That the offender be a public is not a ground for acquittal of the said crimes.
officer. On the petitioners claim that he deposited the
(b) That he had amount of P893,890.67 with the LBP on June 15, 1995 as
the custody or control of funds or evidenced by the deposit slips cannot be considered. The
property by reason of the duties of petitioner was burdened to prove that the said amount was
his office.

233
CRIMINAL LAW REVIEW DIGESTS
JUSTICE ROMEO CALLEJO NOTE: = Callejo Ponente

part of the deposit he made with the LBP on June 15, OMB issued Order (described above). Manhit
1997, but he failed to do so. filed petition for review before the CA. CA held
Manhit was aware of the P12M donation since he
was of the Executive Committee and he knew the
MANHIT V. OMBUDSMAN (2007) existence of the Special Account under the
Facts: control of Usec Valdes. Further, he knew that the
Manhit seeks reversal of the CA Decision check he was signing was going to be drawn
affirming the Order of the Ombudsman (Order) against said Special Account and procurement of
finding Manhit guilty of Conduct Prejudicial to the vehicles was made w/o public bidding. Manhit
Best Interest of the Service, aggravated by the appeals to SC.
offense of Simple Misconduct. The Order
imposed a penalty of fine equivalent to 6 months Issue: w/n Manhit should be charged (particularly he says
of his salary. that he is not specifically obliged by law to report the
On Dec. 1998, Andrew Gonzales (then Secretary receipt of the funds, nor of the special account, and that as
of DECS) requested financial assistance from the a non-official signatory to the special account, he cannot
Land Bank of the Phil (LBP), allegedly for the be held liable for signing a check from said account which
purchase of office equipment. The request was was eventually accepted by the depository bank).
approved and LBP allocated P12M as donation
to DECS. In connection to this donation, then Held: Yes he should, CA affirmed. Manhits liability does
DECS Usec Valdes wrote LBP-Pasig Manager, not really lie on his failure to report the matter but on his
requesting the opening of a DECS special his direct participation and complicity in the illegal
account. disbursement of public funds.
LBP opened a Special Account for DECS to Donations in cash or in kind to the Government
which the P12M donation and interest from other or any of its instrumentalities or agencies become
DECS accounts were credited. The account is a government funds or property. Even the
checking account w/o an account name and it proceeds of donations cannot be disbursed or
was not reflected in the DECS Book of Accounts. disposed of except in accordance with law. Some
Subsequently, DECS purchased vehicles worth of the governing rules are found in the GAA, the
P21.5M, with funding sourced from this special Administrative Code of 1987, and the
account. Procurement of vehicles was made w/o Government Accounting and Auditing Manual.25
any public bidding and w/o authority from the Manhit admitted knowledge of the existence of
Office of the President. the donation but denies knowledge of the special
On Sept. 2000, the Fact Finding and Intelligence account. His denial cannot overcome the
Bureau of the Office of the Ombudsman (OMB) common finding of fact of the OMB and the Court
filed an Admin Complain against Manhit and of Appeals that he was aware of the existence of
some DECS officials for (1) failure to observe the special account.
guidelines on government accounting in
connection w/ the receipt and use of donations
from LBP; (2) allowing the use of the proceeds of
25 Receipts from donations shall be accounted for in the
the donation for a purpose other than that for w/c books of the government in accordance with pertinent
it was granted; (3) failure to comply w/ rules on accounting and auditing rules and regulations. Such donations,
procurement of government vehicles; and (4) whether in cash or in kind, shall be deemed automatically
appropriated for purposes specified by the donor. The receipts
taking part in the illegal disbursement of donated from cash donations and sale of donated commodities shall
funds through conspiracy by silence having been be remitted to the National Treasury and recorded as a
the principal beneficiaries of said illegal Special Account in the General Fund and shall be available to
the concerned implementing agency through a Special Budget
transactions. pursuant to Section 35, Book VI of E.O. No. 292. The agency
OMB found Manhit did not even attempt to cause concerned shall submit to the Department of Budget and
the submission of a report to the Dept. of Budget Management, the Senate Committee on Finance, the House
Committee on Appropriations, and to the Commission on
and Mgt and to the COA relative to the receipt of
Audit a quarterly report of all donations whether in cash or in
the donation and/or disbursement thereof. kind, as well as expenditures or disbursements of the amount
Further, it concluded that Manhit directly released. In case of violation of this Section, the erring
participated in the disbursement of the donation officials and employees shall be subject to disciplinary
action under the provisions of Section 43 and 80 of Book VI,
when he signed a check payable to Autohand E.O. No. 292 and to appropriate criminal action under
(supplier of vehicles). existing penal laws

234
CRIMINAL LAW REVIEW DIGESTS
JUSTICE ROMEO CALLEJO NOTE: = Callejo Ponente

In any event, the inescapable truth is that witness, who testified that she and her team, in the course
petitioner signed a check drawn against the of their audit examination, discovered that FRIAS, on
special account. Indeed, he does not deny that behalf of the Municipality of Capas, made cash advances
he signed Check No. 00003048009. The check amounting to P50,000 and P950,000. These cash
was honored by the LBP and its proceeds were advances were allocated for the maintenance of economic
used to purchase a vehicle. The signing of the enterprises and the augmentation of the general
check is an indication of his awareness of the fund, respectively. However, Abesamis and her team and
existence of the special account and his disallowed them for lack of a specific legal purpose.
recognition that his signature could pave the way On Dec. 19, 1997, Abesamis notified FRIAS,
for the encashment of the check. municipal treasurer Panganiban and municipal accountant
Definition of gross and prejudicial in connection Domingo of the disallowance of the cash advances and
with the offense of conduct prejudicial to the best directed them to settle the P1M immediately. Panganiban
interest of the service. and Domingo failed to return the amount, while FRIAS
o Gross connotes something out of refused for the reason that he gave the proceeds of the
measure; beyond allowance; not to be cash advances to Panganiban. Abesamis, mindful of
excused; flagrant; shameful FRIAS's predicament, pointed out that the cash advances
o Prejudicial detrimental or derogatory were made under his (FRIAS') authority. Moreover, the
to a party; naturally, probably, or checks were payable to him (as payee) and he admitted
actually bringing about a wrong result receipt thereof. Thus, even if he gave the proceeds to
Manhits acts have prejudiced public service. Panganiban, he was still required to return the P1M.
Public office is a public trust. Public officers must Notwithstanding Abesamis' demand, FRIAS did
at all times be accountable to the people, serve not account for the cash advance. Thus, Abesamis
them with utmost degree of responsibility, recommended the filing of this criminal complaint against
integrity, loyalty, and efficiency. Any act or FRIAS.
omission on the part of those involved in the In his defense, FRIAS argued that he was not
public service which would violate the norm of liable for the cash advances because he did not derive
public accountability and diminish the faith of the any benefit from them. Panganiban alone benefited from
people in the public office cannot be the cash advances as she used the P1M to settle her
countenanced. existing deficiencies with the COA. FRIAS pointed out that
In sum, petitioners signing the check, with full the COA, upon Abesamis' recommendation, also filed a
knowledge that it is to be drawn, as it in fact was, criminal complaint against Panganiban.
th
from the special account sourced from the The 4 Div. of the Sandiganbayang found FRIAS
donation, and that the proceeds of the check are guilty due to the concurrence of the ff elements: 1) FRIAS
to be used, as they in fact were, for the was a public officer; 2) he was an officer accountable for
procurement of a vehicle, coupled with the fact public funds or property; 3) he was required by law or
that the purchase was effected without public regulation to render accounts to the COA or provincial
bidding, renders him liable for conduct prejudicial auditor and 4) he failed to render an account for the period
to the best interest of the service, aggravated by of two months after such accounts should have been
simple misconduct. rendered.

ISSUE: Whether FRIAS was guilty under Article 218


YES!
FRIAS v. PEOPLE

HELD:
FACTS: Hermes FRIAS was charged with violation of
PETITIONER IS AN ACCOUNTABLE PUBLIC OFFICER
Article 218 of the Revised Penal Code. It was alleged that
According to the Local Government Code, municipal
FRIAS, an accountable officer, being then the Municipal
mayors are chief executives of their respective
Mayor of Capas, Tarlac, after being required by the
municipalities. Section 102 of the Government Auditing
Commission of Audit to settles his disallowed cash
Code of the Philippines provides that The head of any
advances amounting to P1M, failed to render accounts for
agency of the government is immediately and
a period of two months after such accounts should have
primarily responsible for all government funds and
been rendered, to the damage and prejudice of the
property pertaining to his agency.
government.
In Barriga v. Sandiganbayan, the Court held that
The prosecution presented Tarlac Provincial
public officers are accountable if they, as part of their
Auditor Abesamis of the Commission on Audit as its sole

235
CRIMINAL LAW REVIEW DIGESTS
JUSTICE ROMEO CALLEJO NOTE: = Callejo Ponente

duties, receive public funds or property which they are


bound to account for but fail to do so. QUERIJERO VS PEOPLE
FRIAS never denied that he received the checks
representing the disallowed cash advances. He in fact FACTS: Accused was an accountable public officer, a
admitted that the disallowed cash advances were made cashier of the Provincial Heal Office, and as such was
under his authority, that he was the payee of the checks accountable for public funds. She misappropriated the
and that he received them. Thus, it is clear that he, as amount of 165,000. It appears that a COA team went to
municipal mayor, received and had possession of (and the office of the accused and opened a safety deposit box
consequently was accountable for) the cash advances. assigned to her, she was summoned on the opening but
FRIAS was undeniably an accountable officer. alleged that she has been on sick leave a number of days
prior to the inspection. When the COA team found the
SECTION 347 OF THE LOCAL GOVERNMENT CODE shortage, accused denied having any unliquidated market
AND SECTION 5 OF COA-CIRCULAR 97-002 REQUIRE and miscellaneous expenses since it has already been
ACCOUNTABLE OFFICERS TO RENDER AN ACCOUNT paid but not reimbursed.
Because FRIAS was an accountable officer, he was
obliged to liquidate the cash advances. However, Issue: What would sustain a conviction for malversation of
considering that the cash advances were disallowed, public funds, and was the audit properly made.
FRIAS was bound to return the P1,000,000 to the
Government. Section 347 of the Local Government Code Held: audit not properly made, cannot sustain conviction
provides that Local treasurers, accountants and other
local accountable officers shall render their accounts Ratio: Under Art 217 of RPC conviction requires proof that
within such time, in such form, style, and content and 1) offender is public officer 2) has custody or control of
under such regulations as the COA may prescribe. funds 3) funds involved are public funds for which he is
Likewise, Section 5 of COA Circular 97-002 provides that accountable 4) he has misappropriated by himself or has
All cash advances shall be fully liquidated at the end consented to through negligence or abandonment, the
of each year. taking by another of such funds. It appears that the
FRIAS had until January 20, 1998 to settle the accused at the time was the cashier of the health office,
disallowed cash advances. To escape liability for violating she was found guilty on art 217 which provides that the
Article 218 of the Revised Penal Code, despite his failure failure of a public officer to have duly forthcoming any
to timely return the P1M, FRIAS should have settled the public funds or property with which he is chargeable, upon
cash advances on or before March 20, 1998. When demand by and duly authorized officer, shall be prima
Abesamis demanded an accounting of the cash advances facie evidence that he has put such missing funds to
from FRIAS, he could have compelled Panganiban to personal use
render the necessary accounting or he could have simply Such presumption was overcome by petitioner
prepared it himself. FRIAS, however, did neither. When because the presumption cannot arise in this case
asked why he did not liquidate the P1M, FRIAS because the audit conducted was irregular, incomplete
nonchalantly gave the excuse that he was too busy and inaccurate and hence the shortage was not
preparing for the forthcoming elections and for his defense established. The records showed that there were other
in a murder case. He never settled the disallowed cash accountable officers in the cashiers office who were
advances. Consequently, he was guilty of violating Article responsible for the same funds as the accused. However
218 of the Revised Penal Code. only the cash and accounts of accused were examined by
the COA team, this would lead to certain errors and
PETITIONER IS LIABLE TO RETURN P1M TO THE inaccuracies. The prima facie presumption arises only if
GOVERNMENT there is no issue as to the accuracy and regularity of the
Inasmuch as the cash advances were disallowed audit and the fact that the funds are missing is duly
and FRIAS failed to return the P1M within the allowable established.
period, the funds were deemed illegally or improperly used An accountable officer is defined in the
or applied. Section 342 of the Local Government Code Government Auditing Code of the Phil, he is every officer
makes him jointly and severally liable with the Panganiban of any government agency whose duties permit or require
and Domingo, being their department head/superior, for the possession or custody of government funds and who
the sum illegally and improperly used. shall be accountable therefor and for the safekeeping
FRIAS was held liable to restitute the P 1M to the thereof in conformity with law. It is the nature of the duties
Government without prejudice to his right to recover it from which he performs and not the nomenclature of the
persons who were solidarily liable with him. position held. It should be emphasized then at this
juncture that the audit team should have examined all

236
CRIMINAL LAW REVIEW DIGESTS
JUSTICE ROMEO CALLEJO NOTE: = Callejo Ponente

accountable officers in the same place as there is a SB denied BARRIGAs contentions, following
possibility that one might be borrowing from another and a other jurisprudence27 where it held that an offense is
simultaneous cash count of all officers in the vicinity committed in relation to public office when there is a direct,
should have been conducted to guarantee accuracy. not merely accidental, relation between the crime charged
Instead it was only the accused who was subjected to the and the office of the accused such that, in a legal sense,
examination despite the fact that there were at least three the offense would not exist without the office. This means
other accountable officers in the office. The audit is the office must be a constituent element of the crime as
demmed incomplete and irregular, the shortage of funds defined in the statute. SB said that the offices of a mayor
which was attributed to the accused was not indubitably and municipal accountant were constituent elements of
established. ACQUITTED. the felonies of malversation and illegal use of public funds.
What theyre trying to point out is that the
jurisprudence relied on by BARRIGA apply only where the
BARRIGA VS SANDIGANBAYAN office held by the accused is not a constituent element of
the crimes charged and in such cases, the Info must
WARNING: MAHABA! contain specific factual allegations showing that the
commission of the crimes charged is intimately connected
FACTS: BARRIGA and VILLAMOR, Municipal accountant with or related to the performance of the accused public
and mayor respectively of a municipality in Cebu, were officers public functions. (So GR: is Montilla and EX: lang
charged with malversation and illegal use of public yung Montejo).
funds (in relation to water projects [ex. construction of
pipelines and deep wells] in their municipality). Three ISSUE: W/N SB committed GADALEJ in Denying
informations were filed against them (Note: Info also BARRIGAs contentions. NO! SB has original jurisdiction
alleged that BARRIGA conspired with Mayor). BARRIGA over the crimes of malversation and illegal use of public
filed a motion to quash Info on numerous grounds: funds.
Under Section 4 of Republic Act No. 8294, the
Sandiganbayan has no jurisdiction over the RATIO:
crimes charged because the Info failed failed to Coverage of SB Original Jurisdiction
allege and show the intimate relation between the RA 8249 provides that the Sandiganbayan has original
crimes charged and her official duties as jurisdiction over crimes and felonies committed by public
municipal accountant, which are conditions sine officers and employees, at least one of whom belongs to
qua non for the graft court to acquire jurisdiction any of the five categories enumerated at the time of the
over the said offense. commission of such crimes. There are two classes of
Her position as municipal accountant is classified public office-related crimes under subparagraph (b) of
as salary grade 24, not an accountable officer. Section 4 of Rep. Act No. 8249:
1. those crimes or felonies in which the public
SB has no jurisdiction in felonies of malversation
office is a constituent element as defined by
and illegal use of public funds.
statute and the relation between the crime and
although SB has jurisdiction over offenses
the offense is such that, in a legal sense, the
committed by public officials and employees in
offense committed cannot exist without the office;
relation to their office, the mere allegation in the
2. such offenses or felonies which are intimately
Info that she committed the offenses charged in
connected with the public office and are
relation to her office is not sufficient as the
perpetrated by the public officer or employee
phrase is merely a conclusion of law and
while in the performance of his official functions,
controlling are the specific factual allegations that
through improper or irregular conduct.
would indicate the close intimacy between the
discharge of her official duties and the
SB Orig Juris on #1 even if without factual allegations
commission of the offenses charged. And to
The Sandiganbayan has original jurisdiction over
support her contentions, she cited
criminal cases involving crimes and felonies under
jurisprudence26.
the first classification. Considering that the public office
The conspiracy was not proved as they failed to
of the accused is by statute a constituent element of the
show her exact participation.
crime charged, there is no need for the Prosecutor to state
The funds were not under her control or
in the Information specific factual allegations of the
administration.
intimacy between the office and the crime charged, or that

26 People vs Montejo and lacson vs Exec Sec 27 Montilla vs Hilario

237
CRIMINAL LAW REVIEW DIGESTS
JUSTICE ROMEO CALLEJO NOTE: = Callejo Ponente

the accused committed the crime in the performance of his SC upheld SBs ruling that Mayors office is a constituent
duties. element of malversation and illegal use of public funds or
property. Mayors position is classified as SG 27. And
SB has Orig Juris on #2 only if with factual allegations since Info alleged that BARRIGA conspired with her co-
However, the Sandiganbayan likewise has original accused, the municipal mayor, in committing the said
jurisdiction over criminal cases involving crimes or felonies, the fact that her position as municipal accountant
felonies committed by the public officers and is classified as SG 24 and as such is not an accountable
employees enumerated in Section (a) (1) to (5) under officer is of no moment; the Sandiganbayan still has
the second classification IF the Information contains exclusive original jurisdiction over the cases lodged
specific factual allegations showing the intimate against her. A public officer who is not in charge of
connection between the offense charged and the public public funds or property by virtue of her official
office of the accused, and the discharge of his official position, or even a private individual, may be liable for
duties or functions - whether improper or irregular. The malversation or illegal use of public funds or property
requirement is not complied with if the Information merely if such public officer or private individual conspires
alleges that the accused committed the crime charged in with an accountable public officer to commit
relation to his office because such allegation is merely a malversation or illegal use of public funds or property.
conclusion of law. The determinative fact is that the position of her co-
accused, the municipal mayor, is classified as SG 27, and
#1 covers: (1) malversation & (2) illegal use of public under the last paragraph of Section 2 of Rep. Act No.
funds/ property 7975, if the position of one of the principal accused is
Two of the felonies that belong to the first classification are classified as SG 27, the Sandiganbayan has original and
malversation defined and penalized by Article 217 of the exclusive jurisdiction over the offense.
Revised Penal Code, and the illegal use of public funds or
property defined and penalized by Article 220 of the same Name of position not controlling factor but nature of
Code. The public office of the accused is a constituent duties
element in both felonies. Even though a municipal accountant is not obliged to
receive public money or property, nor is she obligated to
Elements of Malversation account for the same; hence, she is not an accountable
The prosecution must prove the following essential officer within the context of Article 217 of the Revised
elements: Penal Code, it cannot thereby be necessarily concluded
1. The offender is a public officer; that a municipal accountant can never be convicted for
2. He has the custody or control of funds or property by malversation under the Revised Penal Code. The name
reason of the duties of his office; or relative importance of the office or employment is
3. The funds or property involved are public funds or not the controlling factor. The nature of the duties of
property for which he is accountable; and the public officer or employee, the fact that as part of his
4. He has appropriated, taken or misappropriated, or duties he received public money for which he is bound to
has consented to, or through abandonment or account and failed to account for it, is the factor which
negligence, permitted the taking by another person of, determines whether or not malversation is committed
such funds or property. by the accused public officer or employee. Hence, a
mere clerk in the provincial or municipal government may
Elements of Illegal use of public funds or property be held guilty of malversation if he or she is entrusted with
The prosecution is burdened to prove the following public funds and misappropriates the same.
elements:
1. The offenders are accountable officers in both crimes.
2. The offender in illegal use of public funds or property CASTILLO v. BUENCILLO
does not derive any personal gain or profit; in
malversation, the offender in certain cases profits Facts: Castillo was the complainant in a criminal case for
from the proceeds of the crime. estafa against Zabella before the RTC San Pablo. During
3. In illegal use, the public fund or property is applied to trial, Zabella offered P70k as settlement for the civil aspect
another public use; in malversation, the public fund or of the case. Castillo was reluctant to accept the amount
property is applied to the personal use and benefit of for being insufficient, prompting the counsel of Zabella to
the offender or of another person. suggest that the money be left with the court. Presiding
judge agreed and ordered OIC Branch Clerk of Court,
Even if position not covered, if in conspiracy with defendant Buencillo (hereinafter CLERK), to receive the
position which is = also guilty of malversation amount. Fearful that the money might be lost if left in the

238
CRIMINAL LAW REVIEW DIGESTS
JUSTICE ROMEO CALLEJO NOTE: = Callejo Ponente

office steel cabinet which had no lock, Buencillo deposited misappropriatio of funds. There is a distinction between
the same in her personal bank account. property in custody and property in custodia legis.
The civil aspect of the case was settled with Custody of things means to have them in charge of
Zabella giving Castillo postdated checks as payment for safekeeping and implies temporary control; it does not
his obligation, including the P70k already deposited with connote domination of supremacy of authority. On the
the court. Castillo requested for the clerk to give her the other hand, for property to be in custodia legis, it must
money but the latter only remitted P50k, retaining in her have been lawfully seized and taken by legal process and
possession the amount of P20k. It is this P20k that authority, and placed in the possession of a public officer.
triggered the instant administrative complaint filed by In the case at bar, the P70k was voluntarily deposited and
Castillo against the clerk for alleged serious misconduct not pursuant to a seizure order by the court. Although in
and dishonesty. the custody of the court, it was not in custodial egis and
The clerk contends that the P20k was voluntarily never became public fund. There was no
left by Castillo along with the postdated checks to answer misappropriation.
for the latters monetary obligation in connection with a
paluwagan system she participated in. Castillo, however, 2. Whether clerk may retain the P20k and apply the
contends that the clerk refused to give the 20k, which she same to offset the paluwagan debt - NO
tried collecting several times. Letters sent by her lawyer Castillo admitted that she owed the clerk for her
were ignored, hence she has no other recourse but to filed participation in the paluwagan but contends that she did
an administrative case. not want to use the P20k for payment, instead giving the
In addition, Castillo filed a supplemental clerk three postdated checks. However, the clerk should
complaint alleging that the clerk has been operating a be reminded that her private dealings and businesses
canteen within the premises of the Hall of Justice and should not be mixed with her public duties. While private
succeeding in stealing electricity and water from the city persons may normally offset their obligations upon
government through illegal connections. These acts agreement, public officials hold a different position in
violated a SC Admin. Circular prohibiting the use of the society and must always bear in mind that their actions
Halls of Justice for residential or commercial purposes. reflect their status as such. Public official should always
The clerk denied that she owned the canteen, since it was uphold public interest over and above personal interests.
registered under the name of her son-in-law. She claimed SC ordered the P20k returned.
that the admin complaint was filed to harass and malign
her, Castillo hoping to secure leverage in the two criminal 3. Whether paluwagan is gambling - NO
complaints filed by clerk against her for violation of BP22. It is not a game of chance and does not involve
wagering, gambling, or betting penalizd under the RPC.
Issue: Whether the clerk is guilty of misconduct and Hence, the clerk did not violate any law.
dishonesty YES
4. Whether the operation of the canteen is violative of
Ratio: the SC Circular SC said no evidence it belonged to
To answer this question, the SC considered 4 sub-issues: the clerk
1. Whether clerk can validly deposit the P70k in her However, the SC has enough reason to believe
personal bank account NO that the clerk should have refrained from pursuing
SC said that even if it were done in GF, still not activities which interfered with her official functions.
appropriate and without justification. Every public officer is Whether she is owner or merely helping manage the
bound to exercise prudence and caution in the discharge canteen is irrelevant. Her frequent absence from her post
of his duties, acting primarily for the benefit of the public. during office hours so as to attend to personal matters
If the steel cabinet had no lock, she should have informed undermines her efficiency as a court employee. As OIC
the judge so that proper arrangements could be made. If and legal researcher whose duties are essential to the
it were at all necessary to deposit the money, it should speedy administration of justice, she is obligated to devote
have been made in a bank account in the name of the her time and full attention that her position demands.
court, the amount being in the nature of a fiduciary fund. In all these allegations, the clerk does not seem
The interest should have accrued to the general fund of to be directly liable for the violation or irregularities.
the government instead of accruing to the personal However, she cannot avoid responsibility for her acts and
account of the clerk. still needs to be disciplined. Public officer is a public trust.
They are servants of the people, not their rulers. Every
However, her act does not constitute misappropriation. official or employee connected with an office charged with
Castillo contends that the P70k given to the court turned the dispensation of justice is circumscribed with a heavy
into property in custodia legis, making the clerk liable for burden of responsibility, their conduct above suspicion,

239
CRIMINAL LAW REVIEW DIGESTS
JUSTICE ROMEO CALLEJO NOTE: = Callejo Ponente

and must be examples of integrity, uprightness & honesty. the crime of malversation is that a public officer must take
They must at all time be accountable to the people. The public funds, money or property, and misappropriate it for
clerk failed to live up to these high ethical standards. She his own private use or benefit. In this case, there is no
undermined the integrity of the service and jeopardized proof that respondent appropriated the amount.
the publics faith in the courts. But considering that it is Clerks of Court are the chief administrative officers of
her first admin case in her 37 years of service, the SC did their respective courts. With regard to the collection of
not accept the recommendation of the OCA that she be legal fees, they perform a delicate function as judicial
suspended from office without pay for a year. Instead, officers entrusted with the correct and effective
found guilty of simple misconduct for the following acts: 1) implementation of regulations thereon. Hence, as
depositing amount entrusted to her by the court in her custodians of court funds and revenues, they have always
personal bank account and 2) attending to personal been reminded of their duty to immediately deposit the
matters during court hours, which prevented her from various funds received by them to the authorized
performing her functions. She was fined P5k. government depositories for they are not supposed to
keep funds in their custody. For those who have fallen
short of their accountabilities, we have not hesitated to
AQUINO v. OLIVARES impose the ultimate penalty. Even undue delay in the
remittances of amounts collected by them at the very least
FACTS: Judge Aquino reported that he issued a constitutes misfeasance. This Court has never and will
Memorandum to Ricardo Olivares, Clerk of Court of MCTC never tolerate nor condone any conduct which would
Davao del Norte, directing Olivares to explain in writing violate the norms of public accountability and diminish, or
under oath why he should not be administratively charged even tend to diminish, the faith of the people in the justice
for violation of Supreme Court Circular No. 50-95 and/or system. Indeed, as an officer of the court, respondent was
malversation through falsification of public document for duty-bound to use reasonable skill and diligence in the
keeping in his possession for five (5) months the cash performance of his officially designated duties. He has
bond in the amount of P12,000.00, posted by an accused. fallen short of this standard. Thus, we find him
In his written explanation, Clerk of Court Olivares administratively liable for simple neglect of duty.
acknowledged that he incurred delay in depositing the bail
bond of P12,000.00 to the Municipal Treasurer (now City
Treasurer) but raised as a defense that the delay was due PEOPLE V. ENFERMO
to oversight considering that he is old and sometimes
forgetful. He noticed the bail bond contained in an FACTS:
envelope only when he was checking his records in - Office of the Ombudsman filed 12 informations
preparation for his retirement. Immediately, he deposited against Enfermo and Entienza, both former
the amount with the Municipal Treasurer. He vehemently employees of the National Research Council of the
denied that he misappropriated the same. Philippines (NRCP). Several of the cases filed were
dismissed for repeated failure of the prosecution to
ISSUES: present evidence. As a result of the dismissal of those
1. W/N Olivares may be held liable for malversation cases, the only ones that survived and were 2 cases
through falsification of public document? (No) for Malversation through Falsification of Public
2. W/N Olivares may be held administratively liable? Documents and 2 other cases for Estafa through
(Yes) Falsification of Public Documents in which only
Enfermo was charged.
HELD: Supreme Court Circular No. 50-95 dated October - Enfermo, held the position of Disbursing Officer II,
11, 1995 states that all collections from bailbonds, rental Accounting Section, Finance and Administrative
deposits, and other fiduciary collections shall be deposited Division of the NRCP. His duties and responsibilities
within twenty-four (24) hours by the Clerk of Court are to:
concerned, upon receipt thereof, with the land Bank of the o assist the cashier in the preparation and the
Philippines. release of the checks covering the financial
Clearly, respondent Olivares violated the above transaction of the NRCP; and
Circulars. He should have deposited the cash bond o assist the cashier in encashing the checks
immediately or within twenty-four (24) hours with the for salaries of the employees of NRCP.
Municipal Treasurer. Instead, it took him about five (5) - It was discovered that the debit and credit records of
months to do so. the checks issued and paid by its depositary bank,
The charge of malversation through falsification of Land Bank of the Philippines (LBP), did not balance.
public document has no basis. One essential element of An investigation disclosed that there were double

240
CRIMINAL LAW REVIEW DIGESTS
JUSTICE ROMEO CALLEJO NOTE: = Callejo Ponente

issuances of checks, which were covered by only one Forest Ranger Mandia scaled and made notches on most
disbursement voucher each. After investigation, of the lumber before issuing a seizure receipt and turning
appellant was confronted and was asked to pay debt over its custody to petitioner Brgy. Captain Arriola in the
but he refused. presence of petitioner Chief Tanod Radan. Arriola
- Appellant was convicted by the Regional Trial Court acknowledged receipt thereof and signed accordingly.
on all four charges. CA affirmed Hence this petition. Mandia subsequently discovered that the lumber,
stockpiled at the backyard of Radan, was missing.
ISSUE: W/N he is guilty of malversation. Accompanied by several police officers and Foresters, he
requested petitioners to turn over custody of the
HELD: YES. confiscated lumber but the latter claimed it was taken
- The elements of malversation, essential for the away without their knowledge. Subsequently, petitioners
conviction of an accused, under the above penal produced lumber and claimed that these were the ones
provision are that: they recovered. Upon closer inspection however, Mandia
1. the offender is a public officer noted that the lumber produced by petitioners were
2. he has the custody or control of funds or different from those previously confiscated.
property by reason of the duties of his office; A subsequent investigation showed that the
3. the funds or property involved are public missing lumber was actually hauled to and used in the
funds or property for which he is Magdiwang Cockpit where petitioner Arriola is a
accountable; and stockholder. As a result, both petitioners were charged
4. he has appropriated, taken or with the crime of Malversation of Public Property thru
misappropriated, or has consented to, or Negligence or Abandonment.
through abandonment or negligence Arriola asserts that none of the powers, duties
permitted, the taking by another person of, and functions of a Barangay Captain enumerated in the
such funds or property. LGC directly or by inference suggests he is an
In the case at bar, We find that the prosecution has accountable officer with respect to the custody of illegally
satisfactorily proved all the elements of the crime of sawn lumber. He insists the confiscated lumber was
malversation under Art. 217 of the Revised Penal placed in his custody "not by reason of the duties of his
Code. Appellant did not give any explanation as to the office" as Barangay Captain, thus he is not legally
shortage in the funds which have been traced to the accountable to answer for its loss so as to make him liable
double issuance of checks, the responsibility for for Malversation under Art. 217 of the RPC.
which fell on the cashier (Entienza) and the disbursing Petitioners claim that the confiscated lumber were stolen.
officer (appellant). With respect to the replacement lumber they subsequently
- Also, one of the contentions of Enfermo na related sa produced, petitioners believed in good faith that the
malversation was that the funds taken by him in the various lumber found scattered in a nearby creek were the
cases for malversation were private na, not public. He missing confiscated lumber left by the thieves who failed
claims that Avanzado and Manalo signed their to transport them across.
names, which signified that they had already received
the money and therefore the money had already Issue: W/N petitioners Arriola and Radan are accountable
become private in character. From the testimony of officers in relation to the confiscated items.
Avanzado and Manalo, it is plain that appellant took
the money prior to their signing the payrolls. The SC Held and Ratio: SC held Arriola accountable but acquitted
said this was untenable citing local and foreign Radan.
jurisprudence which basically states that so long as To find an accused guilty of malversation, the prosecution
the money is in the hands of the disbursing officer, it must prove the following essential elements: a.] The
is still the money of the State. Hence, the money was offender is a public officer; b.] He has the custody or
public in nature since it had not yet been received by control of funds or property by reason of the duties of his
Avanzado and Manalo when appellant took it. office; c.] The funds or property involved are public funds
or property for which he is accountable; and d.] He has
appropriated, taken or misappropriated, or has consented
ARRIOLA V. SANDIGANBAYAN to, or through abandonment or negligence, permitted the
494 SCRA 44 taking by another person of, such funds or property.
An accountable officer under Article 217 is a
Facts: DENR Forest Rangers and PNP officers public officer who, by reason of his office is accountable
confiscated 44 pieces of illegally sawn lumber totaling for public funds or property. Sec. 101 (1) of the
1,174 board feet w/ an estimated value of P17,611.20. Government Auditing Code of the Philippines (PD No.

241
CRIMINAL LAW REVIEW DIGESTS
JUSTICE ROMEO CALLEJO NOTE: = Callejo Ponente

1455) defines accountable officer to be every officer of any period February 8, 1989 to September 20, 1989. As a
government agency whose duties permit or require the result of the audit, it was assumed that petitioner had a
possession or custody of government funds or property cash shortage of P107,299.02. A report was prepared.
and who shall be accountable therefor and for the Rueda signed on the certification on the dorsal portion of
safekeeping thereof in conformity with law. the report to the effect that his accountability for the funds
In the determination of who is an accountable of the municipal government of Tigaon, Camarines Sur
officer, it is the nature of the duties which he performs was correctly stated.
and not the nomenclature or the relative importance the In his defense, petitioner Rueda disclaimed any
position held which is the controlling factor. criminal liability on the ground that the assumed shortage
In the instant case, Arriola knowingly and willingly was the result of unliquidated cash advances made by
signed the seizure receipt for the confiscated articles. By several municipal officials and employees as evidenced by
affixing his signature, he undertook to safeguard the various chits or vales and expenses of the municipal
lumber on behalf of the Government. Although his usual government of Tigaon as evidenced by several
duties as Barangay Captain do not ordinarily include the disbursement vouchers.
receipt of confiscated articles on behalf of the Rueda declared that the municipal officials and
Government, by virtue of the DENR Primer on Illegal employees took the cash advances from the cash
Logging, which had for its basis Section 68 of PD No. collections of the municipal collectors before the cash
705, he may be called on to take custody thereof as the collections, in the total amount of P41,234.71, were turned
need arises. Furthermore, by affixing his signature in the over to him as municipal treasurer. What they turned over
seizure receipt which clearly enumerates his obligations to him were the chits and vales evidencing such cash
as a custodian therein, he effectively becomes an advances. Although he never tolerated the practice and
accountable officer therefore. had verbally warned the municipal officials and employees
Records show that prior to its confiscation by the from making those cash advances, they continued to do
DENR officers, the lumber was previously apprehended by so
Arriola. Thus, even without the seizure receipt where he Petitioner Rueda stressed that the cash
signed as custodian for the said lumber, Arriola was advances were made with the consent of the municipal
accountable therefore because he was the one who mayor, and had been the practice in the municipality of
originally took possession of it on behalf of the Tigaon long before he assumed office as municipal
government. Moreover, he admitted that he already knew treasurer. He would later on deduct the cash advances
about the missing lumber long before the DENR officers made from their respective salaries in installment, and
came back to get it but he did not inform them about its after they were paid, he would turn over the amount to the
loss because "somebody advised me not to report office of the municipal treasurer. With respect to the
because the one who got the lumber might panic and subject chits and vales, petitioner Rueda declared that
tuluyan na ang lumber." after the same were paid, he turned over the amount to
As for Radan, SC held the trial court erred in the office of the municipal treasurer who then credited
judging him liable as an accessory. The evidence adduced those payments as restitution of the shortage on his total
by the prosecution to prove Radans liability as an cash accountability. Thus, the debtors themselves
accessory were neither clear nor convincing. His presence liquidated the cash advances and petitioners
during the time when the DENR officers turned over the accountabilities had been fully restituted before the start of
custody of the seized items to Arriola is not enough proof the preliminary investigation in the office of the
of complicity, nor the fact that the confiscated lumber was Ombudsman.
placed behind his fathers house. The assertion that he Consequently, petitioner Rueda stated that
was responsible for the alleged transport of the before the start of the preliminary investigation in the
confiscated articles to the cockpit in Dulangan was a mere Office of the Ombudsman, all his financial accountabilities
conjecture. had been fully restituted. The cash advances, in the form
*SC affirmed no subsidiary imprisonment imposed of chits and vales amounting to P41,234.71, had been
because the principal penalty imposed is higher than wholly paid or redeemed by their respective debtors. The
prision correccional. disbursement vouchers of P53,700.00 representing
various legitimate expenses of the municipality of Tigaon,
Camarines Sur and the collection deposits in the amount
RUEDA VS SANDIGANBAYAN of P12,384.06 were all liquidated.
A certification dated July 11, 1990, signed by Mr.
Facts: Rueda was the municipal treasurer of Tigaon, Francisco N. Briguera, in-charge-of office of the municipal
Camarines Sur. An audit examination of the treasurer of Tigaon, Camarines Sur, and verified and
accountabilities of Rueda was conducted covering the found correct by Melanio C. Alarcon, state auditing

242
CRIMINAL LAW REVIEW DIGESTS
JUSTICE ROMEO CALLEJO NOTE: = Callejo Ponente

examiner (Exh. 9), showed that petitioner Rueda had permitted, the taking by another person of, such funds or
fully restituted the cash shortage discovered during the property.
cash examination. As such, petitioner claimed innocence Concededly, the first three elements are present
and therefore must be acquitted. in this case. It is the last element,whether or not petitioner
The Sandiganbayan found him guilty of really has misappropriated public funds, where the instant
malversation petition focuses itself. In convicting petitioner, the
His MR was denied, hence this appeal. Sandiganbayan cites the presumption in Article 217 of the
Revised Penal Code that the failure of a public officer to
Issue: Whether or not Rueda was guilty of malversation of have duly forthcoming any public funds with which he is
public funds under Art. 217 par. 4 of the RPC. chargeable, upon demand by any duly authorized officer,
shall be prima facie evidence that he has put such missing
Held: No. The taking of public funds for personal use was funds or property to personal uses. The presumption is, of
never proved, course, rebuttable. This Court has repeatedly said that
when the absence of funds is not due to the personal use
Ratio: The Sandiganbayan found that petitioner admitted thereof by the accused, the presumption is completely
his accountability and failed to have duly forthcoming his destroyed; in fact, the presumption is deemed never to
cash shortage in the amount of P107,299.02 with which he have existed at all.
is chargeable, and that he did not tender the required An accountable officer under Article 217 of the
written explanation as to why the shortage was Revised Penal Code must receive money or property of
incurred. His failure to do so, the Sandiganbayan the government which he is bound to account for. It is
said, instantly created aprima facie evidence pursuant to the nature of the duties of, not the nomenclature used for,
the last paragraph of Article 217 of the Revised Penal or the relative significance of the title to, the position,
Code that he had put such missing funds to personal use. which controls in that determination.
The SC disagrees. Petitioner did not admit any Based on this definition, to be held accountable
shortage. The mere fact that he signed the dorsal side of the public officer must receive the money or property,
the report of cash examination is not an admission of and later fails to account for it. When a public officer is
shortage. His signature was only evidence that asked to account for the cash in his accountability, this
he received a copy of the report. Thus, it is incorrect to necessarily means that he has to produce the cash in bills
say that petitioner admitted his shortage when he signed and coins and other cash items that he received. It does
the audit report prepared by the audit team. For one thing, not include collectibles and receivables or even
he was made to sign it right away; for another, his promissory notes.
signature only meant an acknowledgment that a demand Petitioner Rueda did not receive the money
from him to produce all his cash, money and paid (cash), which he was supposed to produce or account for
vouchers had been made. It did not mean that he at the time of the audit. The money did not reach the
admitted any shortage. In fact, subsequent events hands of petitioner. Therefore, it is not part of his cash
showed that he had fully explained his accountability. accountability.
Thus, he satisfactorily explained the shortage. In other
words, there was no direct evidence or proof that he put
public funds to personal use. When absence of funds was QUIZO VS. SANDIGANBAYAN
not due to personal use, the presumption is completely
destroyed. The taking or conversion of public funds for FACTS: Arturo Quizo works as a Money Order Teller of
personal use must be affirmatively proved. When there is Cagayan de Oro Post Office. He was found to have
no shortage, taking, appropriation, conversion or loss, incurred a shortage in his cash and other accounts of
there is no malversation. 17,421.74, after an audit was conducted by the
The elements of malversation, essential for the Commission on Audit (shortgage was only 1.74).
conviction of an accused, under the above penal provision However, Quizo was able to reimburse the full amount
are that: within a reasonable time.
(a) the offender is a public officer; Notwithstanding full restitution, an information for
(b) he has the custody or control of funds or property by malversation of public funds was instituted by the
reason of the duties of his office; Tanodbayan against Quizo before the Sandiganbayan. On
(c) the funds or property involved are public funds or a motion for reconsideration/reinvestigation, the
property for which he is accountable; and Tanodbayan filed a motion to dismiss on the ground that
(d) he has appropriated, taken or misappropriated, or has no damage was inflicted on the government since there
consented to, or through abandonment or negligence was full restitution of the malversed funds within a
reasonable time. But this was denied by the

243
CRIMINAL LAW REVIEW DIGESTS
JUSTICE ROMEO CALLEJO NOTE: = Callejo Ponente

Sandiganbayan, ruling that damage to the government is 27, 1995. On December 27, 1995, the auditor from the
not an essential element of the crime of malversation of COA assigned to the NBI, conducted a routine audit
public funds and that restitution is not a defense that examination of the accountability of Chan. The auditor
would exempt the offender from criminal liability. found that all collections for the period beginning
December 7, 1995 up to the date of the audit, December
ISSUE: WON there is sufficient evidence to show a prima 27, 1995, were accounted for.
facie case against Quizo??? NO. On January 24, 1996, the same auditor conducted
another audit examination. However, the auditor found a
RULING: Article 217 of the Revised Penal Code provides shortage of P290,228.00. The auditor thus issued a
that the failure of a public officer to have duly forthcoming demand letter to petitioner to restitute the missing funds
any public funds or property with which he is chargeable, and explain the shortage. In a parallel move, she sent a
upon demand by any duly authorized officer, shall be memorandum to the Regional Director requesting that
prima facie evidence that he has put such missing funds Chan be immediately relieved of her assignment. The
or property to personal uses. Hence, an accountable Regional Director issued a Special Order replacing Chan
public officer may be convicted of malversation even if with Gloria Alvarez, effective March 1, 1996.
there is no direct evidence of misappropriation and the As a standard operating procedure, the COA
only evidence is that there is a shortage in his accounts conducted an audit examination and found that Chan had
which he has not been able to explain satisfactorily. This is a cumulative shortage of cash accountability in the amount
because the law establishes a presumption that mere of P333,360.00. The COA Region VII thus filed a
failure of an accountable officer to produce public funds complaint against petitioner for Malversation of Public
which have come into his hands on demand by an officer Funds.
duly authorized to examine his accounts is prima Petitioner filed a Motion for Reconsideration of the
facie evidence of conversion. However, the presumption is Office of the Deputy Ombudsmans Resolution of February
merely prima facie and a rebuttable one. The accountable 18, 1997 and for Re-investigation of the case against her
officer may overcome the presumption by proof to the on the ground that the entire amount subject thereof
contrary. If he adduces evidence showing that, in fact, he should not be charged solely to her but also to Delza Bas,
has not put said funds or property to personal use, then the acting collection officer during her absence. The
that presumption is at an end and the prima facie case motion was denied.
destroyed. RTC: The trial court found petitioner guilty beyond
In the case at bar, Quizo successfully overthrew the reasonable doubt of the crime charged, with the mitigating
presumption of guilt. He satisfactorily proved that not a circumstance that she had no intention to commit so grave
single centavo of the missing funds was used by him for a wrong as that committed.
his own personal interest, a fact conceded by the Sandiganbayan: The Sandiganbayan affirmed the
Tanodbayan. The bulk of the reported shortage actually conviction of petitioner. It found, however, that the amount
referred to the items disallowed by the Audit Team totally remitted wasP150,547.82, not P150,000.00 as
representing cash advances extended to co-employees. In found by the trial court, hence, it held petitioner to be liable
fact, evidence disclosed that the itemized list of the cash for the unremitted balance of P182,812.18.
advances was verified and found to be correct by an
Auditing Examiner. Quizo explained that the granting of ISSUE: Whether Pamela Chan is guilty of Malversation of
the cash advances was done in good faith, with no intent Public Funds under Article 217 RPC. Yes.
to gain and borne out of goodwill considering that it was a
practice tolerated in the office. Such being the case, HELD:
negligence evidentiary of malice or intent to defraud the Article 217 of the RPC states:
government cannot be imputed to him. Also to be The failure of a public officer to have duly
considered is the circumstance that the actual cash forthcoming any public funds or property with
shortage was only P1.74 which, together with the which he is chargeable, upon demand by any
disallowed items, was fully restituted within a reasonable duly authorized officer, shall be prima
time from date of audit. facie evidence that he has put such missing
funds or property to personal use.

PAMELA CHAN vs. SANDIGANBAYAN While Chan could have shown that she was not
remiss in her supervision of Bas, by way of rebutting the
FACTS: In November 1989, Chan was hired as Accounting disputable presumption in Article 217, she failed to do so.
Clerk II and assigned at the Regional Office of the NBI in Not only did she omit to report the shortages of Bas to the
Cebu City. Petitioner went on leave from December 7 to proper authority upon her discovery thereof; she even

244
CRIMINAL LAW REVIEW DIGESTS
JUSTICE ROMEO CALLEJO NOTE: = Callejo Ponente

practically admitted to having assisted Bas in covering up China Banking Check was payable to the
such shortages. Chan was thus not merely lax in Municipal Treasurer of Pasig and was drawn by one "D.
supervising Bas; she actively assisted her in concealing Noble". The check bears ENRIQUEZs indorsement (NBI
her shortages to the extent of lending her public funds for found this to be forged) at the back and was accompanied
that purpose. Significantly, petitioner acknowledged the by a statement of checks also bearing the initials of
illegality of her own act. ENRIQUEZ. The subject check was transmitted from the
To make matters worse, petitioner did not only lend Pasig Treasury to the Quezon City Treasury as the official
Bas those amounts given on November 7, 9, and 15, district treasury for municipal deposits. According to Benito
1995. She admittedly extended vales to her in the Buenviaje, a casual janitor of the Pasig Treasury, on,
amount of P112,089.18, and to others, also out of public ENRIQUEZ instructed him to get the bundled checks from
,
funds. In Meneses v. Sandiganbayan the granting of vales his table and to deliver them to the Quezon City Treasury.
has been held to be contrary to law, however:The grant of Benito Buenviaje was issued two official receipts, one of
loans through the "vale" system is a clear case of an which included the amount of the dishonored check.
accountable officer consenting to the improper or Several days after, the Quezon City Treasury informed the
unauthorized use of public funds by other persons, which Pasig Treasury of the dishonor of CBC Check. The then
is punishable by the law. To tolerate such practice is to Mayor Mario Raymundo of Pasig sent a letter-request to
give a license to every disbursing officer to conduct a the NBI to conduct an investigation of the alleged
lending operation with the use of public funds. shortage.
There is no law or regulation allowing accountable It appears that less than a month before the
officers to extend loans to anyone against "vales" or chits dishonor of the subject check, ESPINOSA herself had
given in exchange by the borrowers. gone to the Quezon City Treasury to make a deposit of
The alleged acquiescence of petitioners superior, checks and statement of checks. Felisa Cervantes,
even if true, is not a valid defense. As Ilogon v. Computer Operator of the Quezon City Treasury,
Sandiganbayan teaches: The fact that petitioner did not accepted the checks issued an OR in the amount
personally use the missing funds is not a valid defense of P3,583,084.18. ESPINOSA later returned to Felisa
and will not exculpate him from his criminal liability. And as Cervantes and requested her to cross-out the first figure
aptly found by respondent Sandiganbayan, "the fact that "3" on the official receipt to conform with the actual
(the) immediate superiors of the accused (petitioner amount of P583,084.18 deposited therein.
herein) have acquiesced to the practice of giving out cash In an Information, ENRIQUEZ together with
advances for convenience did not legalize the ESPINOSA were charged with Malversation of Public
disbursements". (Underscoring supplied) Funds. After trial, the Sandiganbayan convicted
ENRIQUEZ and ESPINOSA. The Sandiganbayan found
that ENRIQUEZ and ESPINOSA in conspiracy with each
FRANCISCO ENRIQUEZ V. CRUZ other misappropriated public funds in their custody and
(Sorry guys, super long and magulo case) sought to cover up the shortages already existing in the
municipal treasurys collections by depositing the subject
Facts: ENRIQUEZ was Municipal Treasurer, while China Banking Corporation check in the amount
ESPINOSA was Administrative Officer and acting of P3,178,777.41.
Municipal Cashier of the Pasig Treasury. By virtue of Local
Government Audit Order, an audit team headed by Issue: WON ENRIQUEZ and ESPINOSA are guilty of
Carmencita Antasuda as team leader conducted an audit malversation; WON they had incurred a shortage in their
examination of the cash and accounts of the Pasig accounts as Municipal Treasurer and Administrative
Treasury covering the period from May 4, 1987 to Officer/designated as Acting Cashier, respectively, which
November 30, 1987. The audit disclosed, among other they had attempted to conceal through a bad check. NO!
things, accused Enriquezs accounts contained a shortage Both acquitted.
amounting to P3,178,777.41, which shortage was mainly
due to a dishonored China Banking Check in the amount Ratio: The elements of malversation under the above
of P3,267,911.10. Said check was deposited with the penal provision are: (a) That the offender is a public
Quezon City Treasury as part of the collections of the officer, (b) That he has the custody or control of funds or
Pasig Treasury. A letter of demand was sent to property by reason of the duties of his office, (c) That
ENRIQUEZ by the COA to restitute the value of the those funds or property are public funds or property for
dishonored check. In a reply, ENRIQUEZ denied which he is accountable, (d) That he appropriated, took,
responsibility for the shortage and pointed to ESPINOSA misappropriated or consented or, through abandonment or
as the one to whom the letter of demand should be negligence, permitted another person to take them.
addressed as the custodian of said check

245
CRIMINAL LAW REVIEW DIGESTS
JUSTICE ROMEO CALLEJO NOTE: = Callejo Ponente

Verily, the first two elements are present in this endorsed and signed by ENRIQUEZ. Moreover, the
case, the last two are not. The prosecution, upon whose general rule is that the law will not consider evidence that
burden was laden the task of establishing proof beyond a person has done a certain act at a particular time as
reasonable doubt that petitioners had committed the probative of a contention that he has done a similar act at
offense charged, failed to discharge this obligation. another time. (res inter alios acta) Said incident could not
First. There is no evidence to prove that the even sufficiently establish a plan or scheme between
Pasig Treasury incurred a cash shortage in the amount ENRIQUEZ and ESPINOSA to cover-up a shortage that
of P3,178,777.41, which amount, incidentally, is even less has never been proven.
than the amount of the dishonored check. It was only the In view of the foregoing, the presumption is that
drawn check, based on the audit examination that brought ENRIQUEZ and ESPINOSA are innocent, and the
about the shortage. Therefore, it was palpable error for the presumption continues up to the moment their guilt is
Sandiganbayan to conclude that the check which the audit proved beyond reasonable doubt.
team had pinpointed as the shortage due to its dishonor
was at the same time, intended and used by ENRIQUEZ
and ESPINOSA to "cover up" shortages in the funds JUDGE DOMINGO SAN JOSE VS ROBERT
allegedly in their custody. Evidence of shortage is CAMURONGAN
necessary before there could be any taking, appropriation,
conversion, or loss of public funds that would amount to Facts: (en banc)
malversation. Even the audit team leader, Carmelita Camurongan is a court aide assigned to the
Antasuda, who conducted the cash count and cash office of the clerk of court (OCC) of the MTC of San
examination of the Pasig Treasury, testified that based on Fernando, Pampanga. Judge San Jose charge
their audit examination, it was only the subject check that Camuronogan with gross misconduct and conduct
brought about the shortage. They could not even identify prejudicial to the best interest of the service.
whether it was cash or check that was lost. Allegedly, Judge San Jose received a report from
The SB relied on the statutory presumption that Juanita Flores, Clerk of Court, regarding the theft incident
the failure of a public officer to have duly forthcoming any allegedly perpetrated by Camurongan. The theft involved
public funds with which he is chargeable, upon demand by jueteng evidence in different cash denominations which
any duly authorized officer, shall be prima facie evidence were confiscated from Camurongan. (Total of P11,983)
that he has put such missing funds or property to personal Camurongan admitted taking the monetary
uses. It must be emphasized that the prima exhibits under the custody of the OCC for his personal
facie presumption arises only if there is no issue as to the use. He explained that he took the money only for
accuracy, correctness, and regularity of the audit findings safekeeping because the OCC was flooded at that time.
and if the fact that funds are missing is indubitably While the money was with him, a family emergency
established. constrained him to use it for his personal benefit. He
Second. There is no evidence that ENRIQUEZ asked the court to understand his plight as the sole
or ESPINOSA had received such an amount which they breadwinner and pleaded for leniency.
could no longer produce or account for at the time of the OCA found Camurongan guilty of gross
audit. The Sandiganbayan merely speculated that it was misconduct and conduct prejudicial to the best interest of
"surreptitiously encashed with the municipal treasury the service. His dismissal with forfeiture of all benefits was
through a revenue collection clerk or someone performing recommended.
collection tasks" or "it was borrowed from the account
holder for the purpose of covering-up missing collections." Issue: Whether Camurongan should be dismissed? YES
Third. There is no showing that the subject check
was received by the Pasig Treasury in an official capacity; Held: (Basically, nag-lecture lang yung court the conduct
that there was a duty to receive or collect the said amount; officers in the judicial service should observe in performing
and that there was an obligation to account for the same. their work)
Indeed, the subject check never formed a portion of the The strictest standards have always been valued
public funds of the municipality for which either in judicial service. Everyone involved in the dispensation
ENRIQUEZ or ESPINOSA are accountable for. of justice, from the presiding judge to the lowliest clerk, is
Fourth. As to the correction of the OR, expected to live up to the strictest norm of competence,
ESPINOSA has explained, and her testimony remains honesty and integrity in the public service. The consti
unrebutted, that she requested that the correction be enshrines this. Further, the Code of Conduct and Ehtical
made because she discovered 15 minutes after she was Standards for Public Officials and Employees provides
issued the official receipt that the checks and the that every public servant shall uphold public interest over
accompanying statements of checks had not been and aboe personal interest at all times.

246
CRIMINAL LAW REVIEW DIGESTS
JUSTICE ROMEO CALLEJO NOTE: = Callejo Ponente

Camurongan admitted taking the monetary to adduce evidence to prove that the true and fair market
exhibits for his personal use. The act of taking monetary value of the property was indeed P5/sqm as stated in the
exhibits without authority from their custodian constitutes tax declaration.
theft. Thievery no matter how petty has no place in the A negotiated purchase may usually entail a
judiciary. higher buying price than one arrived at in the course of
This unlawful act of taking cannot be justified by expropriation proceedings.
an alleged intention to safeguard the money from damage It is true that the documents used in the
that might be caused by the flood. Neither can this Court negotiated sale were falsified (as determined by the trial
sanction the appropriation, even if temporary, of property court). However, the heads of the departments or projects
under custodia legis in order to remedy the financial are not required to painstakingly trace every step from
exigency of ones family. Personal problems cannot justify inception, and investigate motives of every person
the misuse by any court employee of judiciary funds in involved in the transaction before affixing his signature
their custody. therein. That is asking for the impossible. All heads of
Integrity in the judicial service is a necessity. offices have to rely to a reasonable extent on their
Camurongans acts indeed constitute gross misconduct subordinates and on good faith of those who prepare the
and conduct prejudicial to the best interest of the service. bids, purchase supplies, or those who enter into
negotiations.
Lastly, the prosecution failed to prove that a conspiracy
CONSOLIDATED CASE OF ARIAS VS existed between the accused persons.
SANDIGANBAYAN AND DATA VS. SANDIGANBAYAN
ARTICLE 218
FACTS: Data was the District Engineer of Pasig
Engineering District. He was tasked to create a committee CAMPOMANES V. PEOPLE
that will handle the Mangahan Floodway Project which will
ease the perennial floods in Marikina and Pasig. The Case involves irregularities in the disbursement of the Phil.
committee was to conduct surveys and negotiate with the Sports Commission funds made available to Federation
owners for the sale of the properties that will be affected Internationale Des Echecs (FIDE a swiss organization).
by the project. The six accused persons were convicted in The PSC submitted a bid offer to FIDE to host a chess
connection with the overpricing of the land purchased by Olympiad, which bid was accepted.
the Bureau of Public Works for the project. The 19,000 PSC board then issued a resolution appropriating
sqm Riceland in Rosario, Pasig was purchased for for the monthly expenditure of FIDE. PSC also remitted to
P80/sqm even though it had been assessed at P5/sqm FIDE Php 12.876M also in connection with the world
only. The land was acquired through negotiated chess olympiad. FIDEs President, Campomanes,
purchase. received said amount. (it is the 12M that is subject of this
The trial court convicted them in connection with case)
causing undue injury to the government and giving a COA then audited PSCs transactions. COA
private party unwarranted benefits through manifest noticed lack of acknowledgement receipts (proof the FIDE
partiality, evident bad faith, or inexcusable negligence. received amounts) and accounting liquidation (used to
explain how funds were spent) attached to the
SC: Data is Acquitted. The assessors tax valuation of disbursement vouchers.
P5/sqm is completely unrealistic and arbitrary as the basis Thus, an information was filed against
for conviction. Campomanes and Hechanova (PSC Chair). It alleged that
Hechanova and Campomanes conspired to get the 12M
No undue injury was caused to the government and failed to render accounts for its disbursement.
The property was acquired through a negotiated purchase. Sandiganbayan ruling: Hechanova acquitted,
It was, therefore, nothing more than an ordinary contract only Campomanes is guilty.
of sale where the purchase price had to be arrived at by Campomanes now appeals, arguing that he cant
agreement between the parties and could never be left to be made liable under Art 218 (failure to render accounts)
the discretion of one of the contracting parties. in relation to Art 222 (officers included in the preceding
Necessarily, the parties have to negotiate the provision) because he is not required by law to render an
reasonableness of the price, taking into consideration accounting.
such other factors as location, potentials, surroundings
and capabilities. This being the case, the parties have Issue: guilty? No.
arrived at the amount of P80/sqm as the fair and
reasonable price for the property. The prosecution failed Ruling: Elements of Art 218:

247
CRIMINAL LAW REVIEW DIGESTS
JUSTICE ROMEO CALLEJO NOTE: = Callejo Ponente

1. Accused is a public officer accountable for public funds under their administration. It
2. He must be an accountable officer for funds/prop was alleged that a 40K allotment for the payment of the
3. The offender is requires by law/regulation to salary differentials of the 34 teachers was approved by the
render accounts to COA or provincial auditor Department of Budget and Management. But since the
4. He fails to render an account for 2 months after entire amount was not utilized for such purpose because
such accounts should be rendered. out of the 34 teachers, only 6 of them were entitled to the
Campomanes is not a public officer, so the salary differential, the remainder was used to pay terminal
Sandiganbayan applied Art 222 in relation to Art 218. Its leave benefits of 6 casual employees.
elements are: In other words, while in the performance of their
1. Offender is a private individual functions, it was alleged that Abdulla and Aguil conspired
2. He has charge of national, provincial or municipal with Darkis who was also a public officer (administrative
funds, revenues or property or is an administrator officer of said school) in applying for the payment of
or depository of funds, property attached, seized, wages of casuals, which amount was appropriated for the
or deposited by public authority, even if such payment of the salary differentials of secondary school
property belongs to a private individual teachers of the said school, to the damage and prejudice
3. He is required by law to render accounts to the of public service.
COA or to a provincial auditor Aguil and Darkis was acquitted but too bad for
4. He fails to do so for 2 months after such accounts Abdulla, she was convicted by the Sandiganbayan.
should be rendered.
The element lacking is #3. Sandiganbayan failed ISSUE:
to specify any law or regulation requiring Campomanes to W/N presence of criminal intent was proven? NO
render accounts to the COA. Sandigan tried to justify this W/N all the requisites for technical malversation
by saying that as recipient of PSCs money, FIDE should was present? NO
render accounts. Office of the Special Prosecutor cites PD
1445 and the Manual on Certificate of Settlement and HELD:
Balances. Both state that persons entrusted with custody 1. The presumption of criminal intent will not apply to all
of funds under the agency head shall be immediately charges of technical malversation because
responsible to him. Also, Art 222 provides that Art 218 disbursement of public funds for public use is per se
shall apply to private individuals who have charge of not an unlawful act. Here, appellant cannot be said
national, provincial or municipal funds. Finally, COA is to have committed an unlawful act when she paid the
vested with power to examine accounts of ALL persons obligation of the Sulu State College to its employees
with respect to funds they received. (Consti, Art IX-D) in the form of terminal leave benefits such employees
SC said that laws invoked by the Office of the were entitled to under existing civil service laws. In
Special Prosecutor dont apply. With regard to the consti the absence of any presumption of unlawful intent, the
provision, it must be read in conjunction with Art 222. burden of proving by competent evidence that
COAs power is only over entities required by law or the appellants act of paying the terminal leave benefits of
granting institution (in this case, PSC) to submit to an employees of the Sulu State College was done with
audit. Doesnt apply to FIDE. PSC entered into a contract criminal intent rests upon the prosecution. The Court
with FIDE, and it complied with all its undertakings. There notes the odd procedure which the prosecution took
is no claim by the PSC to render an accounting. in discharging its undertaking to prove the guilt of
There being no law or regulation requiring FIDE appellant beyond reasonable doubt. As it is, the
to render accounts, its President, Campomanes cannot be prosecution did not present any single witness at all,
liable under Art 218. not even for the purpose of identifying and proving the
authenticity of the documentary evidence on which it
ARTICLE 220 rested its case. The prosecution definitely failed to
prove unlawful intent on the part of appellant. The
ABDULLA V. PEOPLE Sandiganbayans improper reliance on Sec. 5(b) of
Rule 131 does not save the day for the prosecutions
FACTS: Abdulla, Aguil and Darkis were charged with deficiency in proving the existence of criminal intent
crime of illegal use of public funds defined and penalized nor could it ever tilt the scale from the constitutional
under Article 220 of the Revised Penal Code, or more presumption of innocence to that of guilt. In the
commonly known as technical malversation. It was alleged absence of criminal intent, this Court has no basis to
in the information that Abdulla and Aguil were both public affirm appellants conviction.
officers (President and Cashier of the Sulu State College)
and by reason of their positions and duties, they were

248
CRIMINAL LAW REVIEW DIGESTS
JUSTICE ROMEO CALLEJO NOTE: = Callejo Ponente

2. The second assigned error refers to the failure of the specifically appropriating said amount for payment of
prosecution to prove the existence of all the essential salary differentials only. In fine, the third and fourth
elements of the crime of technical malversation elements of the crime defined in Article 220 of the Revised
defined in Article 220 of the Revised Penal Code, Penal Code are lacking in this case. Acquittal is thus in
which are: order.
a. That the offender is a public officer;
b. That there is public fund or property under his
administration; ESTRADA VS SANDIGANBAYAN
c. That such public fund or property has been
appropriated by law or ordinance; Facts: The Court has found President ERAP, with family
d. That he applies the same to a public use other and associates, GUILTY of plunder amounting to P4B
than that for which such fund or property has under the account name Jose Velarde. Here, ERAP
been appropriated by law or ordinance. seeks to petition that (1) RA 7080 (Anti-Plunder Law) is so
Appellant contends that the prosecution was vague, its unconstitutional. (2) Also, due to the
unable to prove the second and third elements of the vagueness, he must be acquitted due to reasonable
crime charged. She argued that the public funds in doubt and (c) the law, in effect, abolishes the requirement
question, having been established to form part of savings, of mens rea to crimes already punishable under the RPC.
had therefore ceased to be appropriated by law or
ordinance for any specific purpose. The Court finds merit Issue: Whether or not RA 7080 is UNCONSTITUTIONAL
in appellants submission. due to:
As found by the Sandiganbayan no less, the Vagueness
amount of forty thousand pesos (P40,000.00) originally Abolishing the Mens Rea requirement through the
intended to cover the salary differentials of thirty four (34) conversion of the crime from mala inse to mala
secondary school teachers whose employment status prohibitum
were converted to Instructor I, were sourced from the
lump sum appropriation authorized by R.A. 668828 and Held: ALL UNTENABLE. The Court held that RA 7080 is
the current savings under personal services of said constitutional.
school. The Court notes that there is no particular
appropriation for salary differentials of secondary school Ratio:
teachers of the Sulu State College in RA 6688. The third (1) Law is not vague. Any purported inability of
element of the crime of technical malversation which Congress to clearly define the words employed will NOT
requires that the public fund used should have been necessarily void the law for vagueness SO LONG AS the
appropriated by law, is therefore absent. The legislative will is clear, or at least can be gathered from the
authorization given by the Department of Budget and whole act. RA 7080 distinctly expressed its legislative will.
Management for the use of the forty thousand pesos Moreover, legal hermeneutics provide that words in a
(P40,000.00) allotment for payment of salary differentials statute shall be interpreted in their plain and ordinary
of 34 secondary school teachers is not an ordinance or meaning since Congress are also regular people, unless
law contemplated in Article 220 of the Revised Penal the law itself indicates a special legal meaning. Here,
Code. when the Anti-Plunder Law used the words combination"
In the absence of a law or ordinance and "series", it simply means combination and series. For
appropriating the public fund allegedly technically just as the accused is entitled to the presumption of
malversed the use thereof for another public purpose will innocence in the absence of proof beyond reasonable
not make the accused guilty of violation of Article 220 of doubt, so must a law be accorded the presumption of
the Revised Penal Code. constitutionality without the same requisite quantum of
Appellant herein, who used the remainder of the proof. "The constitutionality of laws is presumed. To justify
forty thousand pesos (P40,000.00) released by the DBM nullification of a law, there must be a clear and
for salary differentials, for the payment of the terminal unequivocal breach of the Constitution, not a doubtful or
leave benefits of other school teachers of the Sulu State argumentative implication; a law shall not be declared
College, cannot be held guilty of technical malversation in invalid unless the conflict with the Constitution is clear
the absence, as here, of any provision in RA 6688 beyond a reasonable doubt. 'The presumption is always in
favor of constitutionality . . . To doubt is to sustain.' The
28 AN ACT APPROPRIATING FUNDS FOR THE OPERATION "vagueness" doctrine merely requires a reasonable
OF THE GOVERNMENT OF THE REPUBLIC OF THE degree of certainty not absolute precision or
PHILIPPINES FROM JANUARY ONE TO DECEMBER THIRTY-
ONE, NINETEEN HUNDRED AND EIGHTY-NINE, AND FOR
mathematical exactitude, as ERAP seems to suggest.
OTHER PURPOSES

249
CRIMINAL LAW REVIEW DIGESTS
JUSTICE ROMEO CALLEJO NOTE: = Callejo Ponente

(2) Mens Rea. The majority agreed with Justice effect, the law seeks to penalize the accused only on the
Mendoza who stated that Plunder is mala inse. Precisely, basis of a proven scheme or conspiracy. In other words,
his constitutive crimes (series or combinations of acts) RA 7080 circumvents the obligation of the prosecution to
must be proven. Here, the information stated that the prove beyond reasonable doubt every fact necessary to
crime was committed willfully, unlawfully and criminally, constitute the crime of plunder, because here, the law only
thus alleging guilty knowledge of the petitioner. Clearly, requires merely proof of a pattern.
the stealing of P4B from the people must be treated with
the same gravity as arson, murder, etc and not as lightly
as BP 22 offenses and jaywalking, which are mala
prohibita crimes.

Dissents:
9. On vagueness
Dissent of Kapunan, J., citing Fr. Bernas
The Constitution guarantees both substantive
and procedural due process as well as the right of
the accused to be informed of the nature and
cause of the accusation against him. Fr. Bernas
poses the question: "How can you have a 'series'
of criminal acts if the elements that are supposed
to constitute the series are not proved to be
criminal?" The meanings of "combination" and
"series" as used in R.A. No. 7080 are not clear if
the elements that are supposed to constitute the
series are not proved to be criminal?" Because of
this, it is easier to convict for plunder and
sentence the accused to death than to convict him
for each of the component crimes otherwise
punishable under the RPC which are bailable.
This absurdity then, violates due process and
equal protection.

10. On Mens Rea


Dissent of Kapunan, J.,
The component acts of plunder, a heinous
crime, are patently mala in se being inherently
wrongful and immoral, even if punished by SPL.
Thus, criminal intent must clearly be established
together with the other elements of the crime;
otherwise, no crime is committed. Here, when RA
7080 eliminated mens rea, it no longer requires
the prosecution to prove beyond reasonable doubt
the component acts constituting plunder, thus
imposing a lesser burden of proof, thus paving the
way for the imposition of reclusion perpetua or
death, violating due process and equal protection
clauses.

Dissent of Ynares-Santiago, J.,


In malversation or bribery under the RPC, criminal intent is
an important element. Here, it is enough that the acts are
committed and does away with the requirement that each
and every component of the criminal act of plunder be
proved... instead limiting to proving only a pattern of overt
acts indicative of the unlawful scheme or conspiracy. In

250

Você também pode gostar